Review of Surgery for ABSITE and Boards [3 ed.] 0323870546, 9780323870542

Now thoroughly up to date with new topics, new questions, and new contributors, Review of Surgery for ABSITE and Boards,

354 51 20MB

English Pages 496 [493] Year 2023

Report DMCA / Copyright

DOWNLOAD PDF FILE

Table of contents :
Cover
IFC
Half Title
Title
Copyright
Dedication
Contributors
Foreword
Preface
Acknowledgments
Contents
1 - Abdomen—General
ABSITE 99th Percentile High-Yields
QUESTIONS
ANSWERS
2 - Abdomen—Hernia
ABSITE 99th Percentile High-Yields
QUESTIONS
ANSWERS
3 - Abdomen—Biliary
ABSITE 99th Percentile High-Yields
Questions
Answers
4 - Abdomen—Liver
ABSITE 99th Percentile High-Yields
Questions
Answers
5 - Abdomen—Pancreas
ABSITE 99th Percentile High-Yields
Questions
Answers
6 - Abdomen—Spleen
ABSITE 99th Percentile High-Yields
Questions
Answers
7 - Alimentary Tract—Esophagus
ABSITE 99th Percentile High-Yields
Questions
Answers
8 - Alimentary Tract—Stomach
ABSITE 99th Percentile High-Yields
Questions
Answers
9 - Alimentary Tract—Small Bowel
ABSITE 99th Percentile High-Yields
Questions
Answers
10 - Alimentary Tract—Large Intestine
ABSITE 99th Percentile High-Yields
Questions
Answers
11 - Alimentary Tract—Anorectal
ABSITE 99th Percentile High-Yields
Questions
Answers
12 - Breast
ABSITE 99th Percentile High-Yields
Questions
Answers
13 - Endocrine Surgery
ABSITE 99th Percentile High-Yields
Questions
Answers
14 - Skin and Soft Tissue
ABSITE 99th Percentile High-Yields
Questions
Answers
15 - Surgical Critical Care
ABSITE 99 Percentile High-Yields
Questions
Answers
16 - Trauma
ABSITE 99th Percentile High-Yields
Questions
Answers
17 - Vascular—Arterial
ABSITE 99th Percentile High-Yields
QUESTIONS
ANSWERS
18 - Vascular—Venous
ABSITE 99th Percentile High-Yields
Questions
Answers
19 - Vascular—Access
ABSITE 99th Percentile High-Yields
Questions
Answers
20 - Transplant
ABSITE 99th Percentile High-Yields
Questions
Answers
21 - Thoracic Surgery
ABSITE 99th Percentile High-Yields
Questions
Answers
22 - Pediatric Surgery
ABSITE 99th Percentile High-Yields
Questions
Answers
23 - Plastic Surgery
ABSITE 99th Percentile High-Yields
Questions
Answers
24 - Genitourinary
ABSITE 99th Percentile High-Yields
Questions
Answers
25 - Gynecology
ABSITE 99th Percentile High-Yields
Questions
Answers
26 - Head and Neck
ABSITE 99th Percentile High-Yields
Questions
Answers
27 - Nervous System
ABSITE 99th Percentile High-Yields
Questions
Answers
28 - Anesthesia
ABSITE 99th Percentile High-Yields
QUESTIONS
ANSWERS
29 - Fluids, Electrolytes, and Acid-Base Balance
ABSITE 99th Percentile High-Yields
QUESTIONS
ANSWERS
30 - Immunology
ABSITE 99th Percentile High-Yields
Questions
Answers
31 - Infection and \\nAntimicrobial Therapy
ABSITE 99th Percentile High-Yields
Questions
Answers
32 - Nutrition and Metabolism
ABSITE 99th Percentile High-Yields
Questions
Answers
33 - Oncology and Tumor Biology
ABSITE 99th Percentile High-Yields
Questions
Answers
34 - Pharmacology
ABSITE 99th Percentile High-Yields
Questions
Answers
35 - Preoperative Evaluation and Perioperative Care
ABSITE 99th Percentile High-Yields
Questions
Answers
36 - Transfusion and Disorders of Coagulation
ABSITE 99th Percentile High-Yields
Questions
Answers
37 - Wound Healing
ABSITE 99th Percentile High-Yields
Questions
Answers
IBC
Recommend Papers

Review of Surgery for ABSITE and Boards [3 ed.]
 0323870546, 9780323870542

  • 0 0 0
  • Like this paper and download? You can publish your own PDF file online for free in a few minutes! Sign Up
File loading please wait...
Citation preview

Any screen. Any time. Anywhere. Activate the eBook version of this title at no additional charge.

Elsevier eBooks+ gives you the power to browse, search, and customize your content, make notes and highlights, and have content read aloud.

Unlock your eBook today. 1. Visit http://ebooks.health.elsevier.com/ 2. Log in or Sign up 3. Scratch box below to reveal your code 4. Type your access code into the “Redeem Access Code” box 5. Click “Redeem”

It’s that easy!

Place Peel Off Sticker Here

For technical assistance: email [email protected] call 1-800-545-2522 (inside the US) call +44 1 865 844 640 (outside the US) Use of the current edition of the electronic version of this book (eBook) is subject to the terms of the nontransferable, limited license granted on http://ebooks.health.elsevier.com/. Access to the eBook is limited to the first individual who redeems the PIN, located on the inside cover of this book, at http://ebooks.health.elsevier.com/ and may not be transferred to another party by resale, lending, or other means. 2022v1.0

Review of Surgery for

ABSITE

AND

BOARDS

Review of Surgery for

ABSITE

AND

BOARDS

THIRD EDITION

EDITORS

Christian de Virgilio, MD, FACS

Areg Grigorian, MD

Chair Department of Surgery Harbor-UCLA Meical Center Torrance, California; Co-Chair College of Applie Anatomy; Professor of Surgery UCLA School of Meicine Los Angeles, California

Assistant Clinical Professor of Surgery Department of Surgery Division of Trauma, Burns an Critical Care University of California, Irvine Orange, California

ASSOCIATE EDITORS

Amanda C. Purdy, MD

Eric O. Yeates, MD

Naveen Balan, MD

Surgical Resient Physician Department of Surgery Harbor-UCLA Meical Center Torrance, California

Resient Physician Department of Surgery University of California, Irvine Orange, California

Surgical Resient Department of Surgery Harbor-UCLA Meical Center Torrance, California

ILLUSTRATOR

Stephanie Cohen, MD Surgical Resient Beth Israel Deaconess Meical Center Boston, Massachusetts

1600 John F. Kenney Blv. Ste 1800 Philaelphia, PA 19103-899

REVIEW OF SURGERY FOR ABSITE AND BOARDS, THIRD EDITION

ISBN: 978-0-33-87054-

Copyright © 2023 by Elsevier Inc. All rights reserved. No part of this publication may be reprouce or transmitte in any form or by any means, electronic or mechanical, incluing photocopying, recoring, or any information storage an retrieval system, without permission in writing from the publisher. Details on how to seek permission, further information about the Publisher’s permissions policies, an our arrangements with organizations such as the Copyright Clearance Center an the Copyright Licensing Agency, can be foun at our website: www.elsevier.com/permissions This book an the iniviual contributions containe in it are protecte uner copyright by the Publisher (other than as may be note herein).

Notice Practitioners an researchers must always rely on their own experience an knowlege in evaluating an using any information, methos, compouns, or experiments escribe herein.Because of rapi avances in the meical sciences, in particular, inepenent verication of iagnoses an rug osages shoul be mae. To the fullest extent of the law, no responsibility is assume by Elsevier, authors, eitors, or contributors for any injury an/or amage to persons or property as a matter of proucts liability, negligence or otherwise, or from any use or operation of any methos, proucts, instructions, or ieas containe in the material herein.

Previous eitions copyrighte 018 an 010.

Content Strategist: Jessica McCool Content Development Specialist: Shweta Pant Publishing Services Manager: Shereen Jameel Project Manager: Beula Christopher Design Direction: Ryan Cook

Printe in the Unite States of America. Last igit is the print number: 9 8 7

6 5 4 3

 1

To my family, who always support me, and to all students of surgery, who motivate and inspire me to always keep learning the art and science of medicine. —Christian de Virgilio

I would not be where I am today if it wasn't for my mentors. Dr. de Virgilio—you are the reason I love surgical education.Dr. Demetriades, you taught me trauma surgery but more importantly, you taught me how to be an effective and inspiring teacher. Dr. Inaba, you have taught me how to be an effective leader both inside and outside the operating room. Dr.Nahmias, you have taught me how to be an academician and researcher. And to my loving wife, Rebecca Grigorian—a superhero mom and physician! Thank you all! —Areg Grigorian

Contributors Mark Archie, MD Assistant Clinical Professor of Surgery Department of Surgery Harbor-UCLA Medical Center David Geffen School of Medicine at UCLA Los Angeles, California Naveen Balan, MD Surgical Resident Department of Surgery Harbor-UCLA Medical Center Torrance, California Jeremy M. Blumberg, MD Chief of Urology Harbor-UCLA Medical Center; Associate Professor of Urology David Geffen School of Medicine at UCLA Los Angeles, California Nina M. Bowens, MD Assistant Professor Department of Surgery David Geffen School of Medicine at UCLA; Associate Program Director, Vascular Surgery Program Division of Vascular and Endovascular Surgery Harbor-UCLA Medical Center Torrance, California Caitlyn Braschi, MD Resident Physician Department of Surgery Harbor-UCLA Medical Center Torrance, California Formosa Chen, MD, MPH Health Sciences Assistant Clinical Professor Department of Surgery David Geffen School of Medicine at UCLA Los Angeles, California Kathryn T. Chen, MD Assistant Professor Department of Surgery Harbor-UCLA Medical Center Torrance, California Christine Dauphine, MD, FACS Vice Chair, Education Department of Surgery Harbor-UCLA Medical Center Torrance, California; Professor of Surgery David Geffen School of Medicine at UCLA Los Angeles, California

Christian de Virgilio, MD, FACS Chair Department of Surgery Harbor-UCLA Medical Center Torrance, California; Co-Chair College of Applied Anatomy; Professor of Surgery UCLA School of Medicine Los Angeles, California Benjamin DiPardo, MD Resident Department of Surgery UCLA Los Angeles, California Richard Everson, MD Assistant Clinical Professor of Surgery Department of Surgery Harbor-UCLA Medical Center David Geffen School of Medicine at UCLA Los Angeles, California Mytien Goldberg, MD Assistant Clinical Professor of Surgery Department of Surgery Harbor-UCLA Medical Center David Geffen School of Medicine at UCLA Los Angeles, California Areg Grigorian, MD Assistant Clinical Professor of Surgery Department of Surgery Division of Trauma, Burns and Critical Care University of California, Irvine Orange, California Joseph Hadaya, MD, PhD Resident Physician Department of Surgery David Geffen School of Medicine at UCLA Los Angeles, California Danielle M. Hari, MD, FACS Division Chief, Surgical Oncology Department of Surgery Harbor-UCLA Medical Center; Associate Professor Department of Surgery David Geffen School of Medicine at UCLA Los Angeles, California

vii

viii

Contributors

Dennis Kim, MD Trauma Medical Director Island Health Trauma Services Victoria, British Columbia, Canada Catherine M. Kuza, MD, FASA Assistant Professor Department of Anesthesiology, Division of Critical Care Keck School of Medicine of the University of Southern California Los Angeles, California Steven L. Lee, MD, MBA Professor and Chief Pediatric Surgery UCLA Mattel Children’s Hospital Los Angeles, California John McCallum, MD, MPH Assistant Professor Department of Surgery Harbor-UCLA Medical Center Torrance, California Michael A. Mederos, MD Resident Physician Department of Surgery UCLA Los Angeles, California Alexandra Moore, MD Surgery Resident Department of Surgery UCLA Los Angeles, California Jeffry Nahmias, MD, MHPE Associate Professor Department of Surgery University of California, Irvine Orange, California Kristofer E. Nava, MD Department of General Surgery Western Michigan University Homer D. Stryker School of Medicine Kalamazoo, Michigan Junko Ozao-Choy, MD, FACS Vice Chair, Research Department of Surgery Harbor-UCLA Medical Center Torrance, California; Associate Professor of Surgery David Geffen School of Medicine at UCLA Los Angeles, California Joon Y. Park, MD Surgery Resident Department of Surgery David Geffen School of Medicine at UCLA Los Angeles, California

Beverley A. Petrie, MD, FACS, FASCRS Professor of Surgery Department of Surgery David Geffen School of Medicine at UCLA Los Angeles, California; Assistant Chief Division of Colon and Rectal Surgery, Department of Surgery Harbor-UCLA Medical Center Torrance, California Amanda C. Purdy, MD Surgical Resident Physician Department of Surgery Harbor-UCLA Medical Center Torrance, California Shonda L. Revels, MD, MS Assistant Professor Department of Surgery UCLA Los Angeles, California Jordan M. Rook, MD Resident Physician Department of Surgery David Geffen School of Medicine at UCLA Los Angeles, California Saad Shebrain, MBBCh, MMM, FACS Program Director, Associate Professor of Surgery Department of Surgery Western Michigan University Homer Stryker M.D. School ofMedicine Kalamazoo, Michigan Eric R. Simms, MD Chief Division of General, Bariatric and Minimally Invasive Surgery, Assistant Program Director of Surgery Residency Department of Surgery Harbor-UCLA Medical Center Torrance, California Veronica Sullins, MD Assistant Clinical Professor of Surgery Department of Pediatric Surgery David Geffen School of Medicine at UCLA Los Angeles, California Maria G. Valadez, MD General Surgery Resident Department of Surgery Harbor-UCLA Medical Center Torrance, California Luis Felipe Cabrera Vargas, MD, MACC, FACS, MACCVA, MFELAC President of the Future Surgeons Chapter of the Colombian Surgery Association, Professor of the Universidad Javeriana and Universidad El Bosque, Fellow of Vascular Surgery of the Universidad Militar Nueva Granada Bogotá, Colombia

Contributors Zachary N. Weitzner, MD Resident Physician Department of Surgery UCLA Los Angeles, California

Eric O. Yeates, MD Resident Physician Department of Surgery University of California, Irvine Orange, California

James Wu, MD Assistant Clinical Professor of Surgery Department of Surgery UCLA Medical Center David Geffen School of Medicine at UCLA Los Angeles, California

Amy Kim Yetasook, MD MIS and Bariatric Surgeon Department of Surgery Harbor-UCLA Medical Center Torrance, California; Assistant Professor Department of Surgery David Geffen School of Medicine at UCLA Los Angeles, California

Tajnoos Yazdany, MD Vice Chair of Education Program Director, Obstetrics and Gynecology Residency, Chief and Program Director, Female Pelvic Medicine and Reconstructive Surgery, Associate Professor David Geffen School of Medicine at UCLA Harbor-UCLA Medical Center Torrance, California

ix

Foreword It is an honor to write the forewor to the thir eition of Review of Surgery for ABSITE and Boards by one of the foremost surgical eucators of our time, Dr. Christian e Virgilio. This book grew out of his initial informal attempts to improve ABSITE scores among his own resients at Harbor-UCLA. Over the years, this effort has grown an expane, incluing collaborators from multiple institutions, to prouce a book that has become an essential tool in the surgical resient’s armamentarium. The most valuable aspect of this book, in my humble opinion, is that in aition to questions testing pure “iactic” knowlege—factois the resient is expecte to learn by rote an memorize—there are many clinical questions that require an avance level of cognitive effort. Here, the learner is expecte to synthesize anatomic an physiologic knowlege within a clinical context an exercise surgical jugment base on probabilities of ifferent outcomes. Too often, books specically targete at passing multiple-choice examinations ten to skip the latter, in favor of questions that have easy answers—hence the common surgical aphorism that there are more exam questions on the clinical presentation of MEN- synrome than patients with this isease! Writing questions that test esoteric minutiae is easy; writing questions that promote further reaing an stuy of complex surgical scenarios is much harer. I applau Dr. e Virgilio an his colleagues for reaching this higher goal, while still incluing the “knowlege-regurgitation” questions that are an inevitable part of the stanarize exam process. Each question is followe by a thoughtful explanation of the right answer, with accompanying

references, to provie a brief summary of essential relevant knowlege. The newest eition also inclues a summary of “high-yiel” principles at the beginning of each chapter, which will further enhance the goal of rapi issemination of essential information on a given topic. In aition to serving as a valuable training tool for the in-service examination, it is our hope that this book will also inspire the resient to augment their learning by elving into relevant sections of textbooks an online resources, incluing vieos an pocasts—all part an parcel of the total eucational package freely available to moern surgical trainees. The breath an epth of multimeia eucation available toay is enormous, compare to what I ha as a resient; conversely, the volume of knowlege an technical skills new surgeons are expecte to learn an master has also increase signicantly. The oubling of scientic knowlege, in meicine an surgery, is now occurring at an exponential pace, an we nee all the help we can get to keep up! I am grateful to Dr. e Virgilio an his colleagues for continuing to invest the effort necessary to upate this wonerful book, so it can continue to serve as a vital resource for present an future surgeons. Sharmila Dissanaike, MD, FACS, FCCM Peter C. Canizaro Chair, University Distinguishe Professor of Surgery, Texas Tech University Health Sciences Center Lubbock, Texas

xi

Preface We are thrille about this thir eition of Review of Surgery for ABSITE and Boards, create to help stuents of surgery prepare for the American Boar of Surgery In-Training (ABSITE) an the American Boar of Surgery (ABS) Qualifying (written) Examination. The original inspiration for the book stemme from a surgery review program we evelope at Harbor-UCLA Meical Center, esigne to stimulate the resients to rea, improve performance on the ABSITE, an enhance their likelihoo of passing the ABS examinations on the rst try. We were inspire to hear that the rst two eitions prove to be a valuable resource. With that in min, we have strive to make the 3r eition even better with some exciting upates an changes. Areg Grigorian an I have ae three new Assistant Eitors to our team, Drs. Amana Pury, Eric Yeates, an Naveen Balan. All are surgical resients; Drs. Pury an Balan at Harbor-UCLA an Dr. Yeates at UC Irvine. We hanpicke them because of their outstaning recor of accomplishment in test taking an question writing an their emonstrate strong interest in surgical eucation. We have also ae numerous resients an surgical eucators from aroun the country (an even one from Colombia) as contributing authors. Another important new feature is that we ae a summary of high-yiel information at the beginning of each chapter. We feel this will serve as a rapi-re way to brush up on key points. We have also ae new, high-yiel questions to remain up-to-ate with the ever-changing an ynamic el of surgery.

Finally, we have ae illustrations from an increibly talente surgical illustrator, Dr. Stephanie Cohen, who is a surgical resient at Beth Israel Deaconess. We love her work so much that we aske her to make a rawing for the cover! The cover illustration, which combines elements of art, music, an anatomy, remins us that Surgery is both an art an a science. To master the arts requires tremenous eication. Excellent surgical knowlege is one characteristic that is paramount to becoming an outstaning surgeon. This requires a lifelong commitment to reaing an then testing your knowlege. We believe that the ieal way to acquire knowlege is to create a year-roun reaing program. Strive to rea aily, even if just for 15 minutes. As with the original version, we believe that the greatest value of our book lies in the esign of the questions an the robust responses. The questions are intene to make you think (try not to get frustrate if you miss many of them!). We provie in-epth explanations for why we feel the correct answer is right an why the incorrect answers are wrong. Please be aware that no textbook or review book has all the answers. Some questions an answers may be controversial. If you isagree with a question or think you foun an error, we woul love to hear back from you (our emails are cevirgilio@lunquist.org an [email protected]u). We sincerely hope you n our review book useful. Christian de Virgilio and Areg Grigorian

xiii

Acknowledgments We would like to acknowledge the efforts of Elsevier for the timely preparation and publication of this review book, in particular Jessica McCool, Content Strategist, who helped with the development of this book and supported it throughout production, and the contributions made by Shweta Pant, Senior Content Development

Specialist, Beula Christopher, Senior Project Manager, and Ryan Cook, Book Designer. In addition, we would like to thank the surgery faculty and residents at HarborUCLA and UC Irvine Medical Centers who assisted in the production and inspiration of this project.

xv

Contents

PArt i: PATIENT CARE 1.

Abdomen—General, 1 Naveen Balan, Areg Grigorian, and Christian de Virgilio

2.

Abdomen—Hernia, 9 Amanda C. Purdy and Amy Kim Yetasook

3.

Abdomen—Biliary, 19 Amanda C. Purdy and Danielle M. Hari

4.

Abdomen—Liver, 33 Naveen Balan, Kathryn T. Chen, and Danielle M. Hari

5.

Abdomen—Pancreas, 47 Joon Y. Park and Danielle M. Hari

6.

Abdomen—Spleen, 65 Maria G. Valadez, Benjamin DiPardo, and Eric R. Simms

7.

Alimentary Tract—Esophagus, 75 Amanda C. Purdy and Eric R. Simms

8.

Alimentary Tract—Stomach, 87 Naveen Balan, Amy Kim Yetasook, and Kathryn T. Chen

9.

Alimentary Tract—Small Bowel, 105 Zachary N. Weitzner, Formosa Chen, and Beverley A. Petrie

10. Alimentary Tract—Large Intestine, 123 Joseph Hadaya, Formosa Chen, and Beverley A. Petrie

11. Alimentary Tract—Anorectal, 143 Michael A. Mederos, Formosa Chen, and Beverley A. Petrie

12. Breast, 155 Naveen Balan, Junko Ozao-Choy, and Christine Dauphine

13. Endocrine Surgery, 175 Michael A. Mederos and James Wu

14. Skin and Soft Tissue, 197 Eric O. Yeates, Areg Grigorian, and Christian de Virgilio

15. Surgical Critical Care, 205 Eric O. Yeates and Dennis Kim

16. Trauma, 219 Naveen Balan, Caitlyn Braschi, and Dennis Kim

xvii

xviii

ContEnts

17. Vascular—Arterial, 245 Amanda C. Purdy and Nina M. Bowens

18. Vascular—Venous, 265 Amanda C. Purdy and John McCallum

19. Vascular—Access, 275 Luis Felipe Cabrera Vargas, Mark Archie, and Christian de Virgilio

20. Transplant, 283 Joseph Hadaya, Areg Grigorian, and Christian de Virgilio

21. Thoracic Surgery, 291 Jordan M. Rook and Shonda L. Revels

22. Pediatric Surgery, 305 Alexandra Moore, Veronica Sullins, and Steven L. Lee

23. Plastic Surgery, 321 Amanda C. Purdy and Mytien Goldberg

24. Genitourinary, 327 Amanda C. Purdy and Jeremy M. Blumberg

25. Gynecology, 339 Amanda C. Purdy and Tajnoos Yazdany

26. Head and Neck, 347 Zachary N. Weitzner and James Wu

27. Nervous System, 357 Eric O. Yeates and Richard Everson

PArt ii: MEDICAL KNOWLEDGE 28. Anesthesia, 365 Eric O. Yeates and Catherine M. Kuza

29. Fluids, Electrolytes, and Acid-Base Balance, 379 Jordan M. Rook, Areg Grigorian, and Christian de Virgilio

30. Immunology, 391 Kristofer E. Nava and Saad Shebrain

31. Infection and Antimicrobial Therapy, 397 Eric O. Yeates and Jeffry Nahmias

32. Nutrition and Metabolism, 409 Eric O. Yeates, Areg Grigorian, and Christian de Virgilio

33. Oncology and Tumor Biology, 417 Alexandra Moore, Areg Grigorian, and Christian de Virgilio

34. Pharmacology, 427 Eric O. Yeates, Areg Grigorian, and Christian de Virgilio

35. Preoperative Evaluation and Perioperative Care, 437 Naveen Balan, Areg Grigorian, and Christian de Virgilio

36. Transfusion and Disorders of Coagulation, 451 Caitlyn Braschi, Joon Y. Park, and Eric R. Simms

37. Wound Healing, 465 Eric O. Yeates, Areg Grigorian, and Christian de Virgilio

PART I

PATIENT CARE

Abdomen—General NAVEEN BALAN, AREG GRIGORIAN, AND CHRISTIAN DE VIRGILIO

1

ABSITE 99th Percentile High-Yields I. Enhance recovery after surgery (ERAS) – associate with a lower overall complication rate, although there is no ifference in surgical complications or mortality A. Preoperative optimization 1. Inclues preamission patient eucation on analgesia management after OR, control of meical comorbiities, smoking cessation, prehabilitation, nutritional care, an correction of anemia . Ieal patient is ASA 1 or , ambulatory, goo nutritional status; absolute contrainication is urgent surgery, ASA 4–6, severely malnourishe, or immobile B. Intraoperative management 1. Stanar anesthesia protocol, minimizing intraoperative uis, preventing intraoperative hypothermia, maintain normal serum glucose, minimally invasive approach (when feasible), avoi routine use of rains C. Postoperative care 1. Avoi routine use of nasogastric (NG) tubes, multimoal analgesia to minimize opioi use, use of epiurals in laparotomy cases, use of TAP (transversus abominis plane) blocks, early urinary catheter iscontinuation, an early mobilization

QUESTIONS 1. A 56-year-ol male unergoes laparoscopic peritoneal ialysis (PD) catheter placement. Several months later the patient comes to the emergency epartment reporting problems with his PD catheter. He reports that he can instill ialysate without ifculty but is unable to withraw ui through the catheter. His abomen is istene an he has mil abominal pain. He is afebrile an not tachycaric. What is the next best step? A. Prompt removal of PD catheter B. Abominal x-ray C. Instill tPA through the catheter D. Intraperitoneal antibiotics E. Intravenous antibiotics

2. A 4-year-ol male unergoes laparotomy for an anterior abominal stab woun with peritoneal violation. A small perforation of the transverse colon is repaire primarily. While examining the small bowel, an antimesenteric iverticulum is foun 10 cm proximal from the ileocecal junction. It is 3 cm in iameter, 3 cm in height, an there is a brous ban extening from the iverticulum to the abominal wall. There is no palpable abnormality ajacent to the iverticulum an no evience or history of GI bleeing. What is the appropriate management of the iverticulum? A. Obtain aitional imaging postoperatively B. Diverticulectomy C. Biopsy D. Observation E. Segmental resection 1

2

PArt i Patient Care

3. Which of the following is true about intraabominal hypertension (IAH) an abominal compartment synrome (ACS)? A. Diagnosis of ACS is establishe when intraabominal pressure is greater than 0 mmHg B. Intraabominal hypertension is ene as intraabominal pressure >1 mmHg C. Neuromuscular blockae reuces mortality in patients with ACS D. Paracentesis is contrainicate in patients with IAH E. Cerebral perfusion is increase in ACS 4. Which of the following is true regaring omental torsion? A. Seconary torsion is more common than primary B. If surgery is necessary, management consists of etorsion an omentopexy C. Treatment is usually observation with pain control D. The pain is usually in the left lower quarant of the abomen E. It typically prouces purulent-appearing peritoneal ui 5. The most common organism isolate from the infecte peritoneal ui of a patient with a PD catheter is: A. Beta-hemolytic streptococcus B. Enterococcus C. Escherichia coli D. Coagulase-negative staphylococcus E. Coagulase-positive staphylococcus 6. A 70-year-ol woman presents with progressive abominal pain an abominal istention with nonshifting ullness. A CT scan emonstrates loculate collections of ui an scalloping of the intraabominal organs. At surgery, several liters of yellowish-gray mucoi material are present on the omentum an peritoneal surfaces. Which of the following is true about this conition? A. There is no role for surgical resection B. It is most commonly of ovarian origin C. There is a strong genetic inuence D. It is more common in males E. Cytoreuctive surgery may be of benet 7. The most common cause of a retroperitoneal abscess is: A. Diverticulitis B. Appenicitis C. Renal infection D. Tuberculosis of the spine E. Hematogenous sprea from a remote location

8. A 50-year-ol male with cirrhotic ascites seconary to hepatitis C presents with fever, elevate white bloo cell count, an abominal pain. He has a history of esophageal varices. He has been on the liver transplant list for 6 months. Paracentesis was performe an cultures were sent. A single organism grows from the culture. Which of the following is true regaring this conition? A. It is most likely ue to appenicitis B. Prophylactic use of uoroquinolone can be use to prevent this conition C. In aults, nephrotic synrome is the most common risk factor D. In chilren, E. coli is the most common isolate E. He will likely nee an exploratory laparotomy 9. A 74-year-ol male presents to clinic hoping to have his reucible umbilical hernia repaire seconary to increasing but intermittent pain an iscomfort. Two ays before his clinic visit, he ha been ischarge from the hospital for unstable angina, for which he unerwent balloon angioplasty with placement of a bare metal coronary artery stent (BMS). When shoul his surgery be scheule? A.  weeks B. 1 month C.  months D. 6 months E. 1 year 10. Which of the following is true regaring abominal incisions an the prevention of incisional hernias? A. A 4:1 suture:woun length is the current recommene closure length B. There is no ifference in hernia occurrence between a running closure an an interrupte closure C. A permanent monolament suture is preferre in the closure of the fascia in a running fashion D. Prophylactic use of mesh after open aortic aneurysm surgery is not efcacious E. A 1-cm bite between each stitch is the recommene istance uring abominal closure

CHAPtEr 1 Abdomen—General

11. A 55-year-ol obese male presents to the hospital for his bariatric sleeve gastrectomy proceure. His comorbiities inclue iabetes an hypertension, an he states he was iagnose with “walking pneumonia”  weeks ago an place on antibiotics, which he has nishe. Which of the following woul not be benecial if the SCIP measures for preoperative an postoperative care are followe? A. Placing the patient on an insulin sliing scale to keep glucose levels between 80 an 10mg/L B. Clipping the patient’s abominal hair with an electric shaver before operating C. Aministering anticoagulation on postoperative ay 1 D. Aministering antibiotics within 1 hour of surgery E. Discontinuing antibiotics by postoperative ay 1 12. A 3-year-ol female who is 4 weeks pregnant presents to the emergency epartment with acute onset of abominal pain, fever, an vomiting. She states that the pain woke her up in the mile of the night with suen onset of epigastric pain that is now iffuse. She has no vaginal bleeing an fetal monitoring emonstrates normal vitals for the fetus. Upon physical exam, the patient has iffuse tenerness with guaring throughout the abomen, worse in the epigastric region. Pelvic examination is normal. She has a leukocytosis of 15,000 cells/L. Abominal x-ray series shows some ilate bowel loops but no other nings. What is your next step in management of this patient? A. Abominal ultrasoun B. CT scan of the abomen/pelvis with contrast C. Amit an observe with serial abominal exams D. Exploratory laparotomy E. Diagnostic laparoscopy 13. Which of the following is true regaring a rectus sheath hematoma? A. If locate above the umbilicus, it is more likely to resemble an acute intraabominal process B. If locate below the umbilicus, it is more likely to cause severe bleeing C. The majority are associate with a history of trauma D. Operative rainage is the treatment of choice in most cases E. Angiographic embolization is not useful

3

14. A woman presents with a rm, enlarging mass on her abominal wall. After appropriate workup, she is iagnose with a esmoi tumor. Which of the following is true about this conition? A. There is a high rate of metastasis without proper treatment B. The chance of local recurrence is low after appropriate intervention C. These tumors ten to enlarge uring menopause D. They occur most commonly in women after chilbirth E. These tumors arise from proliferative chonroblastic cells 15. Which of the following is true regaring retroperitoneal sarcomas? A. They are best manage by enucleation B. Prognosis is best etermine by histologic grae C. Fibrosarcomas are the most common type D. Lymph noe metastasis is common E. Raiation therapy is often curative for small sarcomas 16. A 75-year-ol female with recently iagnose atrial brillation, for which she was given an anticoagulant, presents with suen onset abominal pain unrelate to oral intake. Surgical history is remarkable for a total hip arthroplasty 3 years ago. Her physical exam is signicant for a tener, palpable abominal wall mass above the umbilicus that persists uring exion of abominal wall muscles. The mass is most likely relate to which of the following? A. A malignancy B. Bleeing from the superior epigastric artery C. Occult trauma D. An intraabominal infection E. Bleeing from the inferior epigastric artery

4

PArt i Patient Care

ANSWERS 1. B. PD catheters can become malpositione postopera-

3. B. IAH is ene as an intraabominal pressure

merman D. Fluoroscopic manipulation of peritoneal ialysis catheters: outcomes an factors associate with successful manipulation. Clin J Am Soc Nephrol. 01;7(5):795–800.

much oes ecompressive laparotomy reuce the mortality rate in primary abominal compartment synrome?: a singlecenter prospective stuy on 66 patients. Medicine (Baltimore). 017;96(5):e6006.

2. B. This patient has a Meckel iverticulum. This is a

4. A. It is important to be aware of omental torsion because

ticulum in aults: surgical concerns.Front Surg. 018;5:55.

omental torsion in chilren. J Pediatr Surg. 1995;30(6):816–817.

tively espite intraoperative conrmation of proper placement. Instilling ialysate in the peritoneal cavity without the ability to remove it may lea to abominal istention an mil pain. The rst step for a suspecte malpositione PD catheter that may have been ippe or kinke is to obtain a KUB. If the catheter appears malpositione, then a reasonable next step woul be to return to the OR for iagnostic laparoscopy to reposition the catheter. For catheters that are clogge (resistance to instilling ialysate through the catheter or inability to instill ui), tPA can be use (C). Omentopexy or omentectomy can also be helpful in cases of a malfunctioning catheter ue to obstruction. Peritonitis is a common complication of PD an accounts for 50% of technical failures. This complication presents with abominal pain, fever, an clouy ialysate. The initial management involves intraperitoneal antibiotics, most commonly vancomycin, which cures 75% of cases without iscontinuation of PD (D). Patients who continue to become increasingly septic may require intravenous (IV) antibiotics as well (E). Any fungal infection of PD requires prompt removal of the catheter (A). Reference: Miller M, McCormick B, Lavoie S, Biyani M, Zim-

true intestinal iverticulum that results from the failure of the vitelline uct to obliterate uring the fth week of fetal evelopment. It is the most common congenital anomaly of the GI tract. Pancreatic heterotopia is foun in a minority of cases. The most common heterotopic tissue foun in resecte specimens is gastric mucosa, which can lea to ulcer formation an GI bleeing. Meckel with gastric mucosa is locate at the antimesenteric borer; however, ulceration occurs in the opposite mesenteric borer of the ileum. Symptomatic cases require surgical intervention. The management of an incientally iscovere asymptomatic Meckel iverticulum uring abominal exploration is a controversial topic. Recently, it has been suggeste to selectively intervene on patients with risk factors, namely age  cm in iameter, presence of heterotopic tissue, palpation of abnormal noules, or presence of brous bans. This patient has three inications for removal incluing age 1 mmHg. This is assesse by measuring the blaer pressure while the patient is paralyze. ACS is ene by IAH >0 mmHg AND evience of en-organ malperfusion (i.e., oliguria) (A). Patients who are mechanically ventilate often have high peak pressures. Primary ACS occurs most commonly after surgical proceures associate with massive resuscitation an tense fascial closure. Seconary ACS is ue to meical conitions such as ascites or conitions requiring resuscitation without an abominal proceure (i.e., signicant burn injury). Nasogastric ecompression an neuromuscular blockae are conservative measures to treat IAH but neither has been proven to signicantly reuce mortality (C). Reucing IAH with paracentesis shoul be performe rst in seconary ACS ue to ascites (D). In refractory cases an all other cases of ACS, ecompressive laparotomy shoul be performe expeitiously to lower mortality. The pathophysiology of ACS involves compression of the IVC, which can lea to elevate SVC pressures, an in turn increase intracranial pressures resulting in ecrease cerebral perfusion pressures (E). Reference: Muresan M, Muresan S, Brinzaniuc K, et al. How

it reaily mimics an intraabominal perforation. Because it is typically very ifcult to iagnose preoperatively, the iagnosis is most often mae at surgery. Torsion of the omentum escribes a twisting of the omentum aroun its vascular peicle along the long axis. Primary torsion, in which case there is no unerlying pathology, is extremely rare. Seconary torsion is much more common, an the torsion is usually precipitate by a xe point such as a tumor, an ahesion, a hernia sac, or an area of intraabominal inammation. Omental torsion is much more common in aults in their fourth or fth ecae of life. Chilren with torsion are typically obese, likely contributing to a fatty omentum that preisposes to twisting. Other factors that preispose a patient to torsion inclue a bi omentum an a narrowe omental peicle. In primary omental torsion, the twiste omentum tens to be localize to the right sie; thus, it is most commonly confuse with acute appenicitis, acute cholecystitis, an pelvic inammatory isease (D). Complicating the iagnosis is the fact that the omentum itself tens to migrate an envelop areas of inammation. Laparoscopy is ieal for establishing the iagnosis an excluing other etiologies. Treatment is to resect the twiste omentum, which can often be infarcte at the time of surgery, an to correct any other relate conition that may be ientie (B, C). The ning of purulent ui woul suggest another iagnosis because it is not consistent with omental torsion. The ui usually seen is serosanguinous (E). References: Chew DK, Holgersen LO, Frieman D. Primary

CHAPtEr 1 Abdomen—General Sánchez J, Rosao R, Ramírez D, Meina P, Mezquita S, Gallaro S. Torsion of the greater omentum: treatment by laparoscopy. Surg Laparosc Endosc Percutan Tech. 00;1(6):443–445. Young TH, Lee HS, Tang HS. Primary torsion of the greater omentum. Int Surg. 004;89():7–75.

5. D. Coagulase-negative staphylococci (Staphylococcus epi-

dermidis) is by far the most common cause of peritoneal catheter–relate infections (A–C). Staphylococcus aureus is coagulase positive (E). Another ening feature of S. aureus is that it is catalase positive. The iagnosis is mae by a combination of abominal pain, evelopment of clouy peritoneal ui, an an elevate peritoneal ui white bloo cell count greater than 100/mm3. Initial treatment consists of intraperitoneal antibiotics, which seem to be more effective than IV antibiotics for a total of  weeks. If the infection fails to clear base on abominal examination, clinical picture, or persistent peritoneal ui leukocytosis, then the catheter nees to be remove an a temporary hemoialysis catheter will nee to be inserte. S. aureus an gram-negative organism infections are less likely to respon to antibiotic management alone.

6. E. Pseuomyxoma peritonei is a rare process in which

the peritoneum becomes covere with semisoli mucus an large loculate cystic masses. There is no familial preisposition (C). A useful classication erive from a large series uses two categories: isseminate peritoneal aenomucinosis (DPAM) an peritoneal mucinous carcinomatosis (PMCA). DPAM is histologically a benign process an is most often ue to a rupture appenix. In one large series, appeniceal mucinous aenoma was associate with approximately 60% of patients with DPAM. In patients classie as PMCA, the origin was either a well-ifferentiate appeniceal or intestinal mucinous aenocarcinoma (B). Pseuomyxoma peritonei is most common in women age 50 to 70 years (D). It is often asymptomatic until late in its course. Symptoms are often nonspecic, but the most common symptom is increase abominal girth. Physical examination may emonstrate a istene abomen with nonshifting ullness. Management is surgical, with cytoreuction of the primary an seconary implants, incluing peritonectomy an omentectomy (A). If there is a clear origin at the appenix, a right colectomy shoul also be performe. If the origin appears to be the ovary, total abominal hysterectomy with bilateral salpingo-oophorectomy is recommene. The recurrence rate is very high (76% in one series). References: Gough D, Donohue J, Schutt AJ, et al. Pseuomyxoma peritonei: long-term patient survival with an aggressive regional approach. Ann Surg. 1994;19():11–119. Hinson FL, Ambrose NS. Pseuomyxoma peritonei. Br J Surg. 1998;85(10):133–1339. Ronnett BM, Zahn CM, Kurman RJ, Kass ME, Sugarbaker PH, Schmookler BM. Disseminate peritoneal aenomucinosis an peritoneal mucinous carcinomatosis: a clinicopathologic analysis of 109 cases with emphasis on istinguishing pathologic features, site of origin, prognosis, an relationship to “pseuomyxoma peritonei.” Am J Surg Pathol. 1995;19(1):1390–1408.

7. C. Primary retroperitoneal abscesses are seconary to

hematogenous sprea while seconary retroperitoneal abscesses are relate to an infection in an ajacent organ. The most common source of retroperitoneal abscesses is seconary, with renal infections accounting for nearly 50% of all

5

cases. Hematogenous sprea is not a signicant contributing factor for seconary retroperitoneal abscesses (E). Other common causes inclue retrocecal appenicitis (B), perforate uoenal ulcers, pancreatitis, an iverticulitis (A). In rare cases, patients may have Pott isease, which is a isseminate form relate to tuberculosis (D). Patients typically present with back, pelvic, ank, or thigh pain with associate fever an leukocytosis. Flank erythema may be present. Kiney infections often have gram-negative ros such as Proteus an E. coli. Treatment consists of broa-spectrum antibiotics an rainage, an ientication of the source. If the abscess is simple an unilocular, then CT-guie rainage is the treatment of choice. Operative rainage may be require for complex abscesses.

8. B. Spontaneous (primary) bacterial peritonitis (SBP) is

ene as bacterial infection of ascitic ui in the absence of any surgically treatable intraabominal infection. Patients usually present with fever, iarrhea, an abominal pain, but if severe enough, they will also have altere mental status, hypotension, hypothermia, an a paralytic ileus. However, 13% of patients will be completely asymptomatic. Treatment is with antibiotics alone. Prophylactic antibiotics (with uoroquinolones) to prevent SBP shoul be consiere in high-risk patients with cirrhosis, ascites, an history of gastrointestinal bleeing (as in the present case). Patients with cirrhosis who have low ascitic ui protein (10 ras) exposure early in pregnancy (within the rst 4 weeks) can lea to fetal emise. However, such a high exposure excees the ose of typical imaging (abominal x-ray is 00 mra while abominal an pelvic CT is about 3–4 ras). Between 8 an 15 weeks’ gestation, high-ose (>10 ras) raiation can lea to intrauterine growth retaration an central nervous efects. Beyon 15 weeks (as in the present case), there o not appear to be any eterministic effects (ose-epenent events such as fetal loss, congenital efects) on the fetus. Stochastic effects (those that are not ose epenent), such as the subsequent risk of cancer or leukemia, are increase with exposure of 1 ra or more. The risk is about 1 cancer for every 500 exposures. Conversely, if the pregnant patient with an acute abomen progresses to peritonitis an bowel perforation, the risk of fetal emise is very high. Thus, the risk of fetal miscarriage is higher with visceral perforation than with raiation exposure, an therefore all measures shoul be taken for an accurate iagnosis. Magnetic resonance imaging (MRI) is consiere a goo imaging option in pregnancy; however, its use in the emergent setting may be limite by its availability. Ultrasoun is also useful but woul be more useful if the patient presente with right upper quarant pain (suspecte biliary isease) or right lower quarant pain (suspecte appenicitis).

CHAPtEr 1 Abdomen—General Reference: Khanelwal A, Fasih N, Kielar A. Imaging of acute abomen in pregnancy. Radiol Clin North Am. 013;51(6):1005–10.

13. B. Rectus sheath hematomas are clinically signicant

because of the fact that they can easily be mistaken for an intraabominal inammatory process. The etiology is an injury to an epigastric artery within the rectus sheath. In most cases, there is no clear history of trauma (C). Particularly in the elerly who are taking oral anticoagulants, these typically occur spontaneously. Patients frequently escribe a suen onset of unilateral abominal pain, sometimes precee by a coughing t. In one series, 11 of 1 patients were women, an in another series, all 8 were women, with an average age in the sixth ecae. Below the arcuate line, there is no aponeurotic posterior covering to the rectus muscle. Therefore, hematomas below this line can cross the miline, causing a larger hematoma to form, an then cause bilateral lower quarant pain resembling a perforate viscus. On physical examination, a mass is often palpable. The Fothergill sign is the ning of a palpable abominal mass that remains unchange with contraction of the rectus muscles. This helps istinguish it from an intraabominal abscess, which woul not be palpable with rectus contraction. The iagnosis is best establishe with a CT scan, which will emonstrate a ui collection in the rectus muscle. The hematocrit shoul be closely monitore. Once the iagnosis is establishe, management is primarily nonoperative an consists of resuscitation, monitoring of serial hemoglobin/hematocrit levels, an reversal of anticoagulation (D). However, one shoul be cautious with reversal of anticoagulation, as stable patients may benet from continue anticoagulation (e.g., recent mechanical valve). On rare occasions, angiographic embolization may be necessary (E). Surgical management, while rarely necessary, woul involve ligation of the bleeing vessel an evacuation of the hematoma. References: Berná JD, Zuazu I, Marigal M, García-Meina V, Fernánez C, Guirao F. Conservative treatment of large rectus sheath hematoma in patients unergoing anticoagulant therapy. Abdom Imaging. 000;5(3):30–34. Zainea GG, Joran F. Rectus sheath hematomas: their pathogenesis, iagnosis, an management. Am Surg. 1988;54(10):630–633.

14. D. Desmoi tumors are unusual soft-tissue neoplasms

that arise from fascial or bro-aponeurotic tissue. They are proliferations of benign-appearing broblastic cells with abunant collagen an few mitoses (E). Desmoi tumors o not metastasize (A); however, they are locally aggressive an have a very high local recurrence rate reaching almost 50% (B). They have been associate with Garner synrome (intestinal polyposis, osteomas, bromas, an epiermal or sebaceous cysts) an familial aenomatous polyposis (FAP), which is why patients shoul be scheule for a colonoscopy soon after iagnosis. In sporaic cases, surgical trauma appears to be an important cause. Desmoi tumors may evelop within or ajacent to surgical scars. Patients with FAP have a 1000-fol increase risk of the evelopment of esmoi tumors. Desmois are more common in women of chilbearing age, ten to occur after chilbirth, an may be linke to estrogen. Oral contraceptive pills (OCP) have also been foun to be associate with the occurrence of these tumors, whereas antiestrogen meications may lea to shrinkage. They’ve been reporte to shrink after menopause (C).

7

Patients are typically in their thir or fourth ecae of life an present with pain, a mass, or both. They are classie as either extra abominal (extremities, shouler), abominal wall, or intraabominal (mesenteric an pelvic). There are no typical raiographic nings, but MRI may elineate muscle or soft-tissue inltration an is require in larger tumors to elineate anatomic relations before surgical intervention. Core neele biopsy often reveals collagen with iffuse spinle cells an abunant brous stroma, which may suggest a low-grae brosarcoma; however, the cells lack mitotic activity. An open incisional biopsy of lesions larger than 3 to 4 cm is often necessary. Wie local excision with negative margins is inicate for symptomatic esmoi tumors. Nonresectable or incientally foun, asymptomatic, intraabominal esmoi tumors (even if resectable) shoul be treate with nonsteroial antiinammatory agents (e.g., sulinac) an antiestrogens, which have met with objective response rates of 50%. In regar to ajuvant therapy, recent retrospective reviews have seen signicant reuctions in recurrence with raiation combine with surgery an even with raiation alone. More research is necessary for the use of chemotherapy agents, but it has been seen that when cytotoxic chemotherapy agents are use in inoperable esmoi tumors, there is a 0% to 40% positive response. The aggressive nature of these tumors an high rate of occurrence make esmoi tumors the secon most common cause of eath in patients with FAP, after colorectal carcinoma. References: Ballo MT, Zagars GK, Pollack A, Pisters PW, Pollack RA. Desmoi tumor: prognostic factors an outcome after surgery, raiation therapy, or combine surgery an raiation therapy. J Clin Oncol. 1999;17(1):158–167. Hansmann A, Aolph C, Vogel T. High ose tamoxifen an sulinac as rst-line treatment for esmoi tumors. Cancer. 004;100(3):61–60. Janinis J, Patriki M, Vini L, Aravantinos G, Whelan JS. The pharmacological treatment of aggressive bromatosis: a systematic review. Ann Oncol. 003;14():181–190. Nuyttens JJ, Rust PF, Thomas CR Jr, Turrisi AT 3r. Surgery versus raiation therapy for patients with aggressive bromatosis or esmoi tumors: a comparative review of  articles. Cancer. 000;88(7):1517–153.

15. B. Most retroperitoneal tumors are malignant an com-

prise approximately half of all soft-tissue sarcomas. The most common sarcomas occurring in the retroperitoneum are liposarcomas, malignant brous histiocytomas, an leiomyosarcomas (C). Approximately 50% of patients will have a local recurrence an 0% to 30% will en up having istant metastases. Lymph noe metastases are rare (D). Retroperitoneal sarcomas present as large masses because they o not typically prouce symptoms until their mass effect creates compression or invasion of ajacent structures. Symptoms may inclue gastrointestinal hemorrhage, early satiety, nausea, vomiting, an lower extremity swelling. Retroperitoneal sarcomas have a worse prognosis than nonretroperitoneal sarcomas. The best chance for long-term survival is achieve with an en bloc, margin-negative resection. Tumor stage at presentation, high histologic grae, unresectability, an grossly positive resection margins are strongly associate with increase mortality rates. Tumor grae is the most signicant preictor of outcome. Complete surgical resection is the most effective treatment for primary or recurrent retroperitoneal sarcomas (A, E). Surgical cure

8

PArt i Patient Care

can be limite because the margins are often compromise by anatomic constraints. There is no ifference in survival between those who ha a resection with a grossly positive margin an those with inoperable tumors. Unlike extremity sarcomas, external beam raiation therapy is limite for retroperitoneal malignancies because there is a low tolerance for raiation to surrouning structures. Postoperative an intraoperative raiation therapy have been shown to reuce local recurrence, but further stuies are neee to etermine if this leas to improve survival. Reference: Lewis JJ, Leung D, Wooruff JM, Brennan MF. Retroperitoneal soft-tissue sarcoma: analysis of 500 patients treate an followe at a single institution. Ann Surg. 1998;8(3):355–365.

16. B. This patient was recently iagnose with atrial

brillation an starte on oral anticoagulants. One shoul suspect a rectus sheath hematoma in oler patients taking anticoagulants who present with the clinical tria of acute abominal pain, an abominal wall mass, an anemia. The

mass is palpable even uring exion of abominal wall muscles, helping to ifferentiate this from an intraperitoneal process (Fothergill sign) (D). In a review of 16 patients by Mayo Clinic, anticoagulation was associate with 70%. Above the arcuate line, the etiology is often relate to a lesion to the superior epigastric artery within the rectus sheath (E). In most cases, there is no clear history of trauma (C). In particular, in the elerly who are taking oral anticoagulants, they typically occur spontaneously. The most common treatment for patients with rectus sheath hematomas is rest, analgesics, an bloo transfusions as necessary. In general, coagulopathies are correcte; however, continuing anticoagulation may be pruent in select patients (e.g., biomechanical valve, recent sale embolus). In extreme cases, angioembolization may be require. References: Alla VM, Karnam SM, Kaushik M, Porter J. Spontaneous rectus sheath hematoma. West J Emerg Med. 010;11(1):76–79. Cherry WB, Mueller PS. Rectus sheath hematoma: review of 16 cases at a single institution. Medicine (Baltimore). 006;85():105–110.

Abdomen—Hernia AMANDA C. PURDY AND AMY KIM YETASOOK

2

ABSITE 99th Percentile High-Yields I. Abominal Wall Hernia a. From skin to peritoneum: skin → fascia of Camper → fascia of Scarpa → external oblique → internal oblique → transversus abominis → transversalis fascia → preperitoneal fat → peritoneum b. Superior to arcuate line: 1. Anterior sheath comprise of aponeurosis of external oblique an the anterior half of the aponeurosis of internal oblique . Posterior sheath comprise of aponeurosis of transversus abominis an aponeurosis of the posterior half of internal oblique; posterior sheath not present inferior to arcuate line c. Ten to 15% of all incisions will evelop into ventral (incisional) hernia; woun infection after surgery oubles risk of a hernia evelopment . Miline epigastrium is a physiologic area of weakness in the abomen where patients can evelop iastasis recti an/or epigastric hernia; risk factors inclue pregnancy an weight gain e. Diastasis recti: attenuation of linea alba causing rectus muscle separation; when the rectus contract, a bulge appears in the upper miline abomen; no fascial efect, not a hernia II. Umbilical Hernias a. Peiatric umbilical hernias 1. Congenital efect, repair by age 5 or sooner if symptomatic b. Umbilical hernias in aults 1. Acquire efect, increase intraabominal pressure causes weakening of transversalis fascia an of the umbilical ring . Women are 3 times more likely to evelop umbilical hernias than men (ue to pregnancy), an up to 90% of women evelop an umbilical hernia uring pregnancy; incarceration occurs more in men 3. Cirrhotic patients with uncomplicate umbilical hernias shoul be meically optimize before unergoing elective surgical repair; this inclues free water restriction, iuretics, an large volume paracentesis (with infusion of albumin); mesh can be use III. Inguinal Hernias a. Cremaster muscle bers arise from internal oblique muscle, inguinal ligament from external oblique muscle; the internal oblique an transversalis fascia form the internal ring of the inguinal canal; the conjoint tenon is the lower common aponeurosis of the internal oblique an the transversus abominis b. Though wiely believe as true, there is little evience to support physical activity with inguinal hernia evelopment; inguinal hernias have increase type 3 collagen an ecrease type 1 collagen c. During issection, an inirect hernia sac is foun on the anteromeial aspect of spermatic cor . The genital branch of the genitofemoral nerve supplies sensation to the mons an labia majora in women, an in men it supplies motor to the cremaster an sensation to the scrotum; it runs within the spermatic cor an exits via the eep inguinal ring 9

10

PArt i Patient Care

e. The iliohypogastric nerve arises from the rst lumbar branch an travels between the transversus abominis an the internal oblique muscles f. The ilioinguinal nerve runs anterior to the spermatic cor in men or roun ligament in women an passes through the supercial inguinal ring; supplies sensation to the upper meial thigh g. Peiatric inguinal hernias (ue to a congenital failure of the processus vaginalis to close): 1. Repair only requires high ligation of the hernia sac (ligation at the internal ring) IV. Hernia Repair a. Open repair i. Open repair with mesh (5%) no matter which metho is chosen 10. One hour after laparoscopic repair of a left inguinal hernia, the patient complains of severe burning groin pain. Which of the following is the most appropriate recommenation? A. Immeiate return to the OR for laparoscopy B. Nonsteroial antiinammatory rugs C. Neurontin D. Opioi analgesia E. Inject groin region with local anesthetic 11. Ischemic orchitis after inguinal hernia repair is most often ue to: A. Too tight a reconstruction of the inguinal ring B. Preexisting testicular pathology C. Inavertent ligation of the testicular artery D. Completely excising a large scrotal hernia sac E. Anomalous bloo supply to the testicle 12. A 45-year-ol man presents with an asymptomatic right inguinal hernia. It is easily reuce with gentle pressure. Which of the following is true about this conition? A. The likelihoo of strangulation eveloping is high without surgery B. Without surgery, intractable pain will most likely evelop C. Waiting until symptoms evelop is a reasonable alternative to surgery D. Laparoscopic repair is the best option E. If the hernia is small, there is a lower chance of incarceration

14

PArt i Patient Care

13. A 5-month-ol previously full-term male infant presents with a tener left groin mass that has been present for the past several hours. There is slight erythema over the skin. He is afebrile an his labs are normal. Which of the following is the best next step? A. Attempt manual reuction, an if successful, scheule surgical repair when infant reaches 1 year of age B. Attempt manual reuction, an if successful, immeiately take to the operating room for surgical repair C. Attempt manual reuction, an if successful, scheule repair in  ays D. Attempt manual reuction, an if successful, scheule left-sie surgical repair with contralateral groin exploration in  ays E. Take immeiately to the operating room for operative repair 14. Which of the following best escribes umbilical hernias in chilren? A. They have a signicant risk of incarceration. B. Repair is inicate once an umbilical hernia is iagnose C. Repair shoul be performe if the hernia persists beyon 6 months of age D. Most close spontaneously E. Repair shoul be performe only if the chil is symptomatic 15. Which of the following is true regaring umbilical hernias in aults? A. Most are congenital B. Repair is contrainicate in patients with cirrhosis C. Strangulation is less common than in chilren

D. Small, asymptomatic hernias can be clinically observe E. Primary closure has recurrence rates similar to those of mesh repair 16. Which of the following is true regaring femoral hernias? A. They are the most common hernia in females B. The Cooper ligament is consiere the anterior borer of the femoral canal C. They are lateral to the femoral vein D. Repair involves approximating the iliopubic tract to the Cooper ligament E. A Bassini operation is consiere an appropriate surgical option 17. A 55-year-ol woman presents with a painless abominal wall bulge. She reports a successful iet an exercise program an has lost almost 40 kg over the past  years. However, she is worrie because yesteray when she was sitting up in be, she notice an upper miline abominal bulge that looks like a large rige between her rib cage an belly button. On physical exam the bulge becomes visible when she lifts her hea off the be. Which of the following is true regaring her conition? A. Surgical repair shoul be one immeiately before signs of incarceration evelop B. There are both congenital an acquire etiologies C. A strict regimen of abominal wall exercises usually results in complete resolution D. The efect is limite to the transversalis fascia E. Typically these efects contain only preperitoneal fat

ANSWERS 1. D. This patient has a symptomatic ventral incisional her-

nia. The best option for repair in this patient with multiple risk factors for perioperative infection (iabetes an obesity) is laparoscopic hernia repair with mesh. Compare to open incisional hernia repair, laparoscopic repair has a lower incience of surgical site infection an is the best option for patients at risk for postoperative infection (C–D). Open an laparoscopic ventral hernia repairs with mesh have similar recurrence rate. Component separation is a technique where the anterior rectus sheath is incise  cm lateral to the semilunar line in orer to primarily close large efects while

minimizing tension. This is unnecessary in this case, as the efect is only 3 cm wie, an a minimally invasive technique is more appropriate (E). Abominal wall strengthening exercises are the primary repair for rectus iastasis, which is an attenuation of the linea alba in the superior abominal wall without a true hernia. This patient has a hernia, as evience by fascial efect on physical exam (A). Reference: Guielines for laparoscopic ventral hernia repair. SAGES. Publishe June 7, 016. https://www.sages.org/publications/ guielines/guielines-for-laparoscopic-ventral-hernia-repair

CHAPtEr 2 Abdomen—Hernia

2. C. This patient has a parastomal hernia. Although the inci-

ence of parastomal hernias is higher with en ostomies than with loop ostomies, this may simply be ue to loop ostomies getting reverse more often, an sooner than en ostomies that are more often permanent. The majority of parastomal hernias are asymptomatic an o not require intervention. However, this patient is experiencing symptoms with intermittent bowel obstruction an shoul unergo repair (E). The best option for management of a symptomatic parastomal hernia is to take the ostomy own if appropriate. Unfortunately, this is not an option for this patient with a prior abominoperineal resection (APR). The next best option is repair of the hernia with synthetic mesh using the Sugarbaker technique, where intraperitoneal mesh covers the entire efect, an the bowel leaing to the ostomy enters laterally between the mesh an abominal wall. Biologic mesh is associate with higher recurrence rates compare to prosthetic mesh (D). It may be consiere for patients with signicant contamination. Primary repair of parastomal hernias has been largely abanone ue to unacceptable recurrence rates of up to 70% (A). Ostomy relocation solves the problem at han (the current symptomatic parastomal hernia); however, it is inferior to repair with mesh as there is a high risk of eveloping another parastomal hernia at the new ostomy site (B). Reference: Hansson BM, Slater NJ, van er Velen AS, et al. Surgical techniques for parastomal hernia repair: a systematic review of the literature. Ann Surg. 01;55(4):685–695.

3. B. After vertical miline abominal incision, approx-

imately 10% to 0% of patients evelop incisional hernias. Ranomize controlle trials have shown that small (5 mm) fascial bites 5 mm apart have a signicantly lower rate of eveloping incisional hernia than large (1 cm) bites 1 cm apart (A, C). Also, a suture to woun length ratio of at least 4:1 is associate with less tension an a ecrease incience of incisional hernia evelopment (D, E). References: Deerenberg EB, Harlaar JJ, Steyerberg EW, et al. Small bites versus large bites for closure of abominal miline incisions (STITCH): a ouble-blin, multicentre, ranomise controlle trial. Lancet (London, England). 015;386(10000):154–160. Millbourn D, Cengiz Y, Israelsson LA. Effect of stitch length on woun complications after closure of miline incisions: a ranomize controlle trial. Arch Surg. 009;144(11):1056–1059.

4. D. Conitions that increase intraabominal pressure

(cystic brosis, chronic lung isease, ventriculoperitoneal shunts, constipation, an peritoneal ialysis) are associate with higher risk for eveloping an inguinal hernia. Patients with small asymptomatic hernias are at risk for eveloping symptoms as their hernias enlarge uring peritoneal ialysis. Therefore, everyone unergoing peritoneal ialysis shoul be examine for presence of abominal hernias preoperatively. If a hernia is foun, the patient shoul unergo concurrent herniorrhaphy at the time of peritoneal ialysis catheter placement (A–C). Hernia repair shoul be one with mesh, as mesh is associate with ecrease recurrence rates an are safe in patients unergoing peritoneal ialysis (E). Reference: Chi Q, Shi Z, Zhang Z, Lin C, Liu G, Weng S. Inguinal hernias in patients on continuous ambulatory peritoneal ialysis: is tension-free mesh repair feasible? BMC Surg. 00;0(1):310.

5. D. The genitofemoral nerve arises from the L1-L level.

The genital branch innervates the cremaster muscle an

15

sensation to the sie of the scrotum an the labia. It is responsible for the cremasteric reex. In women, it accompanies the roun ligament of the uterus. The genital branch of the genitofemoral nerve is part of the cor structures. It lies on the iliopubic tract an accompanies the cremaster vessels (B). The ilioinguinal nerve lies on top of the spermatic cor (A). It innervates the internal oblique muscle an is sensory to the upper meial thigh ajacent to the genitalia. The nerve can sometimes splay out over the cor, making issection ifcult. The iliohypogastric an ilioinguinal nerves arise from the T1-L1 level an intermingle. They provie sensation to the skin of the groin, the base of the penis, an the upper meial thigh. The iliohypogastric nerve lies on the internal oblique muscle (C), provies sensory innervation from the skin overlying the pubis, an oes not intermingle with the genitofemoral nerve because they cross ifferent paths (E). Reference: Wantz GE. Testicular atrophy an chronic resiual neuralgia as risks of inguinal hernioplasty. Surg Clin North Am. 1993;73(3):571–581.

6. A. Poupart ligament is another name for the inguinal lig-

ament. The inguinal ligament is forme from the anteroinferior portion of the external oblique aponeurosis foling back on itself. It extens from the anterosuperior iliac spine to the pubic tubercle, turning posteriorly to form a shelving ege. The cremaster muscle bers arise from the internal oblique muscle an surroun the spermatic cor (B). The genital branch of the genitofemoral nerve passes through the eep ring (C), whereas the ilioinguinal nerve passes through the supercial ring. The genital branch innervates the cremaster muscle, whereas the femoral branch controls sensation to the upper lateral thigh (D). Inirect hernias arise lateral to the inferior epigastric vessels, whereas irect hernias arise meial to the inferior epigastric vessels. The lateral borer of the rectus muscle, inferior inguinal ligament, an inferior epigastric artery ene the borers of Hesselbach triangle an ene the location of a irect hernia (E).

7. C. The arcuate line is locate below the umbilicus, typ-

ically one-thir the istance to the pubic crest (A). Between the costal margin an the arcuate line, the anterior rectus sheath is mae up of a combination of the aponeurosis of the external an internal oblique muscles. The posterior sheath is mae up of a combination of the aponeuroses of the internal oblique an transverse abominal muscles. Below the arcuate line, the anterior sheath is mae up of the aponeuroses of all three abominal muscles (E). The internal oblique aponeurosis splits above the arcuate line to envelop the rectus abominis muscle (B). There is no posterior sheath below the arcuate line (D), an the transversalis fascia therefore makes up the posterior aspect of the rectus abominis muscle.

8. E. This patient has bilateral inguinal hernias, one of

which is recurrent an shoul be offere a laparoscopic repair. The avantages of this inclue the ability to visualize both sies through a single incision an a potentially easier surgery in the setting of recurrence. It also protects the patient from eveloping a femoral hernia since the femoral canal is covere by the mesh. Of note, femoral hernias are known to evelop after open inguinal hernia repair. They evelop on average sooner than a typical recurrence, suggesting that the original hernia was in fact a femoral one an

16

PArt i Patient Care

was misse at the original surgery. The two laparoscopic approaches inclue TEP an TAPP. TEP involves issecting a plane in the preperitoneal space, which may actually be avantageous when compare to TAPP because intraabominal ahesions are avoie (A). This oes not hol true for prior pelvic surgery as the preperitoneal space may be obliterate in these patients, necessitating a TAPP. If the peritoneum is violate uring TEP, it is important to repair the efect to prevent ahesion formation postoperatively, but it is not manatory to convert to a ifferent technique (C). Though there are few absolute contrainications to laparoscopic hernia surgery, bowel ischemia with perforation or sepsis preclues the use of mesh, which is require in both TEP an TAPP. Tacking of the mesh in either laparoscopic approach can reuce mesh migration but shoul be avoie lateral to the epigastric vessels an inferior to the iliopubic tract to avoi placement in the “triangle of oom” or the “triangle of pain,” which contains the external iliac vessels an several nerves (lateral femoral cutaneous an femoral branch of genitofemoral, respectively) (B). Injury to these nerves is relatively specic to laparoscopic repairs (D). Reference: Fischer JE. Fischer’s mastery of surgery. Wolters Kluwer Health/Lippincott Williams & Wilkins; Chicago, IL, 01.

9. C. The preferre initial approach for an uncomplicate

inguinal hernia is still actively ebate within the surgical community. The LEVEL-trial specically compare TEP repair versus open mesh repair an emonstrate reuce pain in the immeiate postoperative perio an earlier return to work. However, this came at the expense of longer operating room times an higher intraoperative complication rates (B, D). This seems to be consistent with the results of a New England Journal of Medicine (NEJM) stuy from 004 comparing open mesh repair to all methos of laparoscopic mesh repair. However, they iverge on reporte recurrence rates, with the NEJM stuy favoring open repair (recurrence of 4% versus 10.1%) while the LEVEL-Trial showe equivalent recurrence rates (3.0% for open an 3.8% for TEP) (E). The LEVEL-Trial also inicate an equivalent prevalence of chronic pain, which was not one of the outcomes in the NEJM article (A). References: Langevel HR, van’t Riet M, Weiema WF, et al. Total extraperitoneal inguinal hernia repair compare with Lichtenstein (the LEVEL-Trial): a ranomize controlle trial. Ann Surg. 010;51(5):819–84. Neumayer L, Giobbie-Hurer A, Jonasson O, et al. Open mesh versus laparoscopic mesh repair of inguinal hernia. N Engl J Med. 004;350(18):1819–187.

10. A. Severe groin pain eveloping in the recovery room

following laparoscopic hernia repair is most likely ue to a stapling/tacking injury to a nerve. If this complication is suspecte, the patient shoul return to the operating room to remove the offening tack. Acute groin pain is most likely from injury to the ilioinguinal nerve. However, the most commonly injure nerve uring laparoscopic hernia repair is the lateral femoral cutaneous nerve (provies sensation to the lateral thigh). Injecting the groin with local anesthetic may not relieve the pain an if it works, it will only be a temporary measure (E). Meical therapy is not appropriate if the suspecte etiology is irritation of the nerve seconary to stapling/tacking (B–D). Chronic groin pain may occur in 10% to 5% of patients 1 year after surgery. The etiology is

thought to be entrapment of the nerve uring surgery or postoperative scarring. Chronic groin pain is best worke up with MRI. If conservative management oes not resolve the pain, operative exploration an ivision of the nerve(s) have met with success. The ieal approach in the setting of hernia reoperation after open repair is to enter a space in which the tissue planes have not been violate. The preferre management is a laparoscopic retroperitoneal triple neurectomy, which allows a single stage approach to access the ilioinguinal, iliohypogastric, an genitofemoral nerves.

11. D. Ischemic orchitis is thought to evelop as a result of

thrombosis of veins of the pampiniform plexus, leaing to testicular venous congestion. It has thus been terme congestive orchitis. The precise etiology of ischemic orchitis is unclear. The most commonly ientie risk factor is extensive issection of the spermatic cor. This occurs particularly when a patient has a large hernia sac, an the entire istal sac is issecte an excise. As such, it is recommene that the sac instea is ivie an the istal sac left in situ. In aition, the cor shoul never be issecte past the pubic tubercle. The presentation is that of a swollen, tener testicle, usually  to 5 ays after surgery. The testicle is often high riing. This may eventually progress to testicular atrophy. Scrotal uplex ultrasonography has been shown to be useful in evaluating the perfusion of the testicle after hernia repair. However, it oes not change the management of ischemic orchitis. Management is expectant. In the past, attempts to reexplore the groin were unertaken to try to loosen the inguinal ring, but this was not successful (A). The bloo supply to the testicle is via the testicular artery, but there are rich collaterals incluing the external spermatic artery an the artery to the vas. Thus, inavertent ligation of the testicular artery oes not typically lea to this complication (C). Preexisting testicular pathology (B) or anomalous bloo supply (E) to the testicle is not thought to contribute to ischemic orchitis following inguinal hernia repair. However, ischemic orchitis can occur more frequently in recurrent inguinal hernia surgery using the anterior approach; thus, the laparoscopic approach shoul be consiere for recurrent hernias. References: Holloway B, Belcher HE, Letourneau JG, Kunberger LE. Scrotal sonography: a valuable tool in the evaluation of complications following inguinal hernia repair. J Clin Ultrasound. 1998;6(7):341–344. Wantz GE. Testicular atrophy an chronic resiual neuralgia as risks of inguinal hernioplasty. Surg Clin North Am. 1993;73(3):571–581.

12. C. A large prospective ranomize stuy in men

emonstrate that watchful waiting for patients with asymptomatic or minimally symptomatic inguinal hernias is an acceptable option for surgery (D). The patients were followe for as long as 9 years. Acute hernia incarceration without strangulation evelope in only one (0.3%) patient, an acute incarceration with bowel obstruction evelope in only one (A). Approximately one-fourth of the watchful waiting group eventually crosse over to receive surgical repair ue to increase hernia-relate pain (B). Smaller hernias ten to have a smaller neck, placing them at higher risk for eveloping incarceration (E). Reference: Fitzgibbons RJ Jr, Giobbie-Hurer A, Gibbs JO, etal. Watchful waiting vs repair of inguinal hernia in minimally symptomatic men: a ranomize clinical trial. JAMA. 006;95(3):85–9.

CHAPtEr 2 Abdomen—Hernia

13. C. The vast majority of inguinal hernias in chilren are

the inirect type ue to a persistent patent processus vaginalis. Approximately 1% to 5% of chilren can evelop an inguinal hernia. However, the incience increases in preterm infants an those with a low birth weight. Right-sie hernias are more common, an 10% of hernias iagnose at birth are bilateral. Incarceration is a more serious problem in peiatric patients than in aults. Emergent operation on an infant with an incarcerate hernia can be very challenging. Thus, it is preferable to try to reuce the hernia, which is successful in 75% to 80% of cases, allow the inammation to subsie over several ays, an then perform the repair semielectively. The routine use of contralateral groin exploration is not wiely supporte (D). For elective cases, one option is to perform laparoscopy via the hernia sac to look for a contralateral hernia an, if foun, procee to repair. If there are any signs of strangulation (e.g., leukocytosis, fever, elevate lactate), then manual reuction shoul be avoie, an the patient shoul be taken immeiately to the operating room for surgical intervention (E). In the patient escribe, though the skin is erythematous, there are no signs of systemic toxicity. Methos to achieve reuction inclue the use of intravenous (IV) seation, Trenelenburg positioning, ice packs, an gentle irect pressure. Reuction without subsequent surgery is not appropriate. That being sai, infants with anemia an history of prematurity are at signicantly increase risk of postoperative apnea an woul require overnight monitoring. Reference: Özemir T, Arıkan, A. Postoperative apnea after inguinal hernia repair in formerly premature infants: impacts of gestational age, postconceptional age an comorbiities. Pediatr Surg Int. 013;9(8):801–804.

14. D. In chilren, umbilical hernias are congenital. They

are forme by a failure of the umbilical ring to close, causing a central efect in the linea alba. Most umbilical hernias in chilren are small an will close by  years of age, particularly if the efect is less than 1 cm in size. As such, repair is not always inicate at the time of iagnosis (B). Aitionally, the ecision to perform an elective repair is not solely etermine by the presence of symptoms (E). If closure oes not occur by age 4 or 5 years, elective repair is then consiere a reasonable option (C), even if the patient is asymptomatic. If the hernia efect is large (> cm) or the family is bothere by the cosmetic appearance, repair shoul be consiere. Although umbilical hernias in chilren can incarcerate, this is very rare (A). If the chil presents with abominal pain, bilious emesis, an a tener, har mass protruing from the umbilicus, immeiate exploration an hernia repair are inicate.

15. D. Unlike in chilren, umbilical hernias in aults are

usually acquire (A). Risk factors are any conitions that increase intraabominal pressure, such as pregnancy, obesity, an ascites. Overall strangulation of umbilical hernias in aults is uncommon, but it occurs more often than in chilren (C). Small, barely palpable an asymptomatic hernias can be followe clinically. Larger or symptomatic hernias shoul be repaire. In patients with cirrhosis an ascites, the markely increase pressure causes the skin overlying the hernia to become thin an eventually ischemic. One of the most catastrophic complications in this setting is rupture of

17

the hernia through the ischemic skin, leaing to peritonitis an eath. Thus, patients with cirrhosis an ascites shoul unergo repair if there is evience that the skin overlying the hernia is thinning or becoming ischemic (B). However, repair shoul be elaye until after meical management of the ascites. If meical management fails an the skin over the hernia is thinne an tense, then a transjugular portosystemic shunt shoul be consiere before repair. Alternatively, if the patient is a caniate for liver transplant, the hernia can be repaire uring the transplantation. Umbilical hernias have historically all been repaire by primary closure. Borrowing from the low recurrence rates using mesh for inguinal hernias, umbilical hernias are now more frequently being repaire using mesh, particularly those with large efects. A recent prospective, ranomize stuy compare primary closure with mesh repair. The early complication rates such as seroma, hematoma, an woun infection were similar in the two groups. However, the hernia recurrence rate was signicantly higher after primary suture repair (11%) than after mesh repair (1%) (E). Some authors are now avocating for the routine use of mesh for all ault umbilical hernias in the absence of bowel strangulation. References: Arroyo A, García P, Pérez F, Anreu J, Canela F, Calpena R. Ranomize clinical trial comparing suture an mesh repair of umbilical hernia in aults. Br J Surg. 001;88(10):131–133. Belghiti J, Duran F. Abominal wall hernias in the setting of cirrhosis. Semin Liver Dis. 1997;17(3):19–6.

16. D. Femoral hernias occur more commonly in females

an have a high risk of incarceration. However, the most common overall hernia in females is an inirect inguinal hernia (A). Bowel entering a femoral hernia passes own a narrow femoral canal. This is because the femoral ring, which serves as the entrance for the femoral canal, is very rigi an unyieling. Thus, the xe neck of a femoral hernia is prone to pinching off the bowel, putting the patient at risk for incarceration. The borers of the femoral canal are as follows: inguinal ligament (anterior) (B), Cooper ligament (posterior), femoral vein (lateral), an Poupart ligament (meial). Femoral hernias occur most commonly lateral to the lymphatics an meial to the femoral vein, within the empty space (C). It is important to recognize that femoral hernias pass eep (posterior) to the inguinal ligament. As such, repairs to the inguinal ligament (such as a Bassini operation an stanar mesh repair) will not obliterate the efect (E). The femoral hernia can be xe either through a stanar inguinal approach or irectly over the bulge using an infrainguinal incision. The essential elements of femoral hernia repair inclue issection an removal of the hernia sac an obliteration of the efect in the femoral canal. This can be accomplishe by either approximation of the iliopubic tract to the Cooper ligament or by placement of prosthetic mesh. Reference: e Virgilio C, Frank PN, Grigorian A, es. Surgery: a case based clinical review. Springer; 015.

17. B. It is important to unerstan the ifference between

epigastric hernias an iastasis recti because the former is a true hernia, which shoul be repaire, an the latter is a benign conition. Diastasis recti is cause by increase separation of the rectus abominis muscles an a relative thinning of the linea alba, which can mimic a hernia. The

18

PArt i Patient Care

conition can be acquire, such as in multiparous women where the repeate stretching of the abominal wall causes the rectus muscles to separate, or congenital, seconary to more lateral attachment of the rectus muscles at birth. Classically, patients present after recent weight loss because this allows for the lesion to be visible. There is no risk for strangulation in iastasis recti because all of the facial layers are intact (A, D). Though several methos of surgical repair have been escribe, these are mainly cosmetic. In general, all that is require is reassurance an abominal wall exercises to help strengthen the musculature—though complete resolution in aults is unlikely (C). In contrast, epigastric hernias are true hernias an represent a true efect in the linea alba. They are generally small an contain either preperitoneal fat or part of the falciform ligament (E). They arise from efects in the fascia in locations where neurovascular bunles

perforate through. Though small, they can cause signicant pain because of compression of the nerves traveling through the efect. There is some evience to suggest that iastasis rectus may increase the risk for evelopment of an epigastric hernia an will make primary repair of epigastric hernias more challenging. Of note, patients with iastasis recti are at increase risk of abominal aortic aneurysms. References: Brunicari FC, Anersen DK, Schwartz SI. Schwartz’s principles of surgery. 10th e. McGraw-Hill Eucation. Köhler G, Luketina RR, Emmanuel K. Suture repair of primary small umbilical an epigastric hernias: concomitant rectus iastasis is a signicant risk factor for recurrence. World J Surg. 015;39(1):11–16. Townsen CM, Jr, Beauchamp RD, Evers BM, Mattox KL, es. Sabiston textbook of surgery: the biological basis of modern surgical practice. 17th e. Philaelphia, PA: W.B. Sauners; 004.

Abdomen—Biliary AMANDA C. PURDY AND DANIELLE M. HARI

3

ABSITE 99th Percentile High-Yields I. Physiology A. Bile consists of water, bile salts, phospholipis, an cholesterol B. Primary bile acis (cholic & chenoeoxycholic acis) become seconary bile acis when ehyroxylate by gut bacteria (lithocholate an eoxycholate acis) C. Mechanism of bile concentration in the gallblaer: active transport of NaCl into gallblaer mucosal cells, passive absorption of water II. Cholecystitis, Choleocholithiasis, an Cholangitis A. Acute calculous cholecystitis: gallblaer inammation ue to gallstone impacte in the neck; management is NPO, IV antibiotics, resuscitation, laparoscopic cholecystectomy B. Acute acalculous cholecystitis: usually seen in critically ill patients in the ICU; US emonstrates gallblaer wall thickening, pericholecystic ui, with no stones; HIDA if US is equivocal, tx is IV antibiotics, resuscitation, an percutaneous cholecystostomy tube if critically ill versus laparoscopic cholecystectomy if stable for surgery C. Choleocholithiasis: may have obstructive jaunice; elevate irect bilirubin, may have transaminitis; US emonstrates gallstones, ilate CBD, +/− stone in the CBD (sensitivity only 50%); tx is ERCP followe up by laparoscopic cholecystectomy 1. If intraoperative cholangiogram is positive for choleocholithiasis: rst attempt to ush the stone with saline; if it oesn’t work try ushing after giving 1 mg IV glucagon (relaxes sphincter of Oi); if stones on’t clear, options inclue: a) Postoperative ERCP b) Transcystic CBD exploration—best for small stones, large cystic uct, or small CBD; generally preferre over transuctal CBD exploration because it avois a CBD incision c) Transuctal CBD exploration—best for large stones (>8–10 mm), large CBD, proximal stones (above cystic uct), choleochotomy mae anterior to avoi vasculature laterally D. Acute ascening cholangitis: Charcot tria (RUQ pain, fever, jaunice) presents in about 0%; Reynols penta as hypotension an confusion; RUQ US: +/− gallstones, ilate CBD, elevate irect BR; tx is IV antibiotics, ui resuscitation, pressors if in septic shock, followe by ERCP or percutaneous transhepatic cholangiography (PTC) tube placement after resuscitation; laparoscopic cholecystectomy uring same amission if ue to gallstones E. Mirizzi synrome: large stone in the gallblaer neck compresses the common hepatic uct (CHD), can cause CHD stricture or stula between the gallblaer an CHD; usually presents similar to cholecystitis an iagnose uring cholecystectomy; manage with cholecystectomy III. Choleochal cysts A. Due to an anomalous pancreaticobiliary junction, with a fuse, long common pancreaticobiliary channel allowing pancreatic enzymes to reux into the biliary tree leaing to inammation an cystic egeneration 19

20

PArt i Patient Care

B. More common in females an those of Asian escent, 60% iagnose before age 10 C. First step in workup is US but MRCP is best for iagnosis an preop planning D. Associate with cholangiocarcinoma an gallblaer cancer; type III has very low risk of malignancy; management for all types besies type III is surgical to ecrease subsequent malignancy risk 1. Management is base on location (escribe by Toani Classication): . Type I (fusiform ilation, most common): cyst excision, Roux-en-Y hepaticojejunostomy, cholecystectomy 3. Type II: cyst excision, primary closure, cholecystectomy 4. Type III: enoscopic sphincterotomy an cyst unroong 5. Type IVa: cyst excision, partial hepatectomy, Roux-en-Y hepaticojejunostomy, cholecystectomy 6. Type IVb: cyst excision, Roux-en-Y hepaticojejunostomy, cholecystectomy 7. Type V (Caroli isease): if only in one lobe of the liver—hepatic resection an cholecystectomy; if bilobar or unresectable—liver transplant IV. Gallblaer Polyps A. Polypoi lesions of the gallblaer: cholesterolosis (most common, cholesterol-laen macrophages in the lamina propria), aenomatous polyp (risk for gallblaer cancer) B. Inications for cholecystectomy for gallblaer polyps: symptomatic, polyp >10 mm, primary sclerosing cholangitis, an polyp of any size C. If cholecystectomy is not inicate, shoul follow patient with serial US in 6 to 1 months V. Gallblaer Aenocarcinoma (most common biliary malignancy) A. Risk factors: gallstones, gallblaer polyp >10 mm, porcelain gallblaer with selective mucosal calcication (as oppose to transmural calcication) B. May present similarly to cholecystitis, often iagnose on pathology after cholecystectomy C. Management: 1. T1a (into lamina propria) → cholecystectomy . >T1b (into muscularis) OR >N1 → cholecystectomy, segment IVb & V hepatectomy, portal lymphaenectomy (port site resection not inicate); followe by ajuvant chemotherapy (gemcitabine an cisplatin) 3. If positive cystic uct margin, nee extrahepatic bile uct resection an hepaticojejunostomy VI. Cholangiocarcinoma A. Risk factors: primary sclerosing cholangitis, ulcerative colitis, choleochal cyst, liver uke infection B. Can present with painless jaunice; suspect in patient with focal bile uct stenosis without history of biliary surgery or pancreatitis; best imaging is MRCP C. Unresectable if istant metastasis, which inclues multifocal hepatic isease an lymph noe mets beyon the porta hepatis D. For potentially resectable cholangiocarcinoma, start with iagnostic laparoscopy; goal of surgery is negative margins; all surgery inclues portal lymphaenectomy; management epens on location:

Location/Classięcation

Management

Lower 1/3 of extrahepatic bile duct

Whipple

Middle 1/3 of extrahepatic bile duct

Resection, hepaticojejunostomy

Upper 1/3 of bile duct AKA Klatskin tumor Further classięed with the Bismuth classięcation:

Type I

CHD (not to the conĚuence)

Type II

CHD to the conĚuence

Type IIIa

CHD + RHD

Type IIIb

CHD + LHD

Type IV

CHD + RHD + LHD

If localized to one side—hemihepatectomy, extrahepatic bile duct excision, Roux-en-Y hepaticojejunostomy If unresectable hilar tumor ≤3cm without nodal disease or distant mets—evaluate for transplant

CHAPtEr 3 Abdomen—Biliary

21

VII. Bile uct injuries (incience 0.3%–0.8%, most commonly ue to cystic uct stump leak) A. Risk of bile uct injury higher with laparoscopic cases an elective (not emergent/urgent cases) B. Principles of management: control sepsis, rain bile collections, an establish secure biliary rainage C. Marke laboratory abnormalities are not typical; bilirubin may be elevate ue to systemic resorption; US is initial imaging stuy, +/− HIDA D. In immeiate postop perio, treat with IV antibiotics, ui resuscitation, percutaneous rainage an ERCP with stent placement an/or sphincterotomy as this is sufcient for majority of cases; if not, percutaneous transhepatic catheter require; if leak has not heale in 6 to 8 weeks, biliary reconstruction is consiere with Roux-en-Y hepaticojejunostomy E. If iscovere intraoperatively, repair only inicate if aequate hepatobiliary surgical experience is available; otherwise, wie rainage an referral to higher level of care

Fig. 3.1 Biliary Cysts.

22

PArt i Patient Care

Questions 1. A 10-year-ol boy with sickle cell isease presents with right upper quarant pain, nausea, vomiting, fever, an yellowing of the eyes for the past ay. He enies ark urine or light stool. On exam, he is febrile, hemoynamically stable, an has a positive Murphy sign. He has leukocytosis, elevate alkaline phosphatase, an elevate unconjugate bilirubin. On ultrasoun, there are gallstones, pericholecystic ui, an gallblaer wall thickening, an CBD iameter is 4 mm. After starting IV ui resuscitation an IV antibiotics, what is the next step? A. MRCP B. ERCP C. Percutaneous transhepatic cholangiography D. Laparoscopic cholecystectomy E. Cholecystostomy tube 2. A 5-year-ol woman is unergoing elective laparoscopic cholecystectomy for symptomatic cholelithiasis. When removing the gallblaer from the fossa, a -mm tubular structure is completely transecte an is leaking bile. The structure appears to come from the liver fossa an enter irectly into the gallblaer. What is the most appropriate management? A. Laparoscopic clip placement B. Repair over a T-tube C. Roux-en-Y hepaticojejunostomy D. Immeiate transfer to a hospital with a hepatobiliary surgeon E. Complete cholecystectomy an plan for postoperative ERCP 3. A 45-year-ol male presents with hematemesis two weeks after a motor vehicle accient in which he suffere a liver injury that was manage nonoperatively. Laboratory values are signicant for an elevate total bilirubin an alkaline phosphatase, as well as signicant anemia. This patient is most likely to have which of the following? A. Arterioportal vein stula B. Arteriohepatic vein stula C. Arterial pseuoaneurysm D. Portal venous pseuoaneurysm E. Cavernous hemangioma

4. Which of the following patients shoul be offere a cholecystectomy? A. A 40-year-ol woman with an incientally iscovere 6-mm gallblaer polyp B. A 30-year-ol man with asymptomatic gallstones unergoing gastric bypass C. A 65-year-ol woman with asymptomatic gallstones an an incientally iscovere porcelain gallblaer with selective mucosal calcication D. A 50-year-ol man with a history of iabetes an asymptomatic gallstones E. A 1-year-ol boy with sickle cell isease an asymptomatic gallstones 5. Which of the following is true regaring bile an gallstones? A. The primary bile acis are eoxycholic an lithocholic aci B. The primary phospholipi in bile is lecithin C. Cholecystectomy ecreases bile salt secretion D. Brown pigmente gallstones are more likely to be foun in the gallblaer versus the CBD E. Bile consists of an equal part of bile salts, phospholipis, an cholesterol 6. Which of the following is true regaring the gallblaer? A. It passively absorbs soium an chlorie B. In the setting of cholelithiasis, cholecystokinin (CCK) can cause gallblaer pain that waxes an wanes C. It harbors an alkaline environment D. Glucagon can help empty the gallblaer E. Its contraction is inhibite by vagal stimulation

CHAPtEr 3 Abdomen—Biliary

7. A 75-year-ol woman presents to the emergency epartment with a -ay history of nausea, feculent vomiting, an obstipation. Her bloo pressure on amission is 80/60 mm-Hg, an her heart rate is 10 beats per minute, both of which normalize after uis. Plain lms reveal istene loops of small bowel with air–ui levels an air in the biliary tree. Which of the following is the best management option? A. Small bowel enterotomy with removal of the gallstone plus B. Small bowel enterotomy with removal of the gallstone C. Small bowel enterotomy with removal of the gallstone followe 8 weeks later by cholecystectomy an takeown of stula D. Small bowel resection to inclue area of impacte gallstone E. Small bowel resection to inclue area of impacte gallstone plus cholecystectomy an takeown of the stula 8. Jaunice with absent urine urobilinogen is most consistent with: A. Hepatitis B. Cirrhosis C. Hemolysis D. Biliary obstruction E. Sepsis 9. Which of the following is true regaring bile an gallblaer isease? A. Primary bile acis are forme by econjugation B. Bile acis are passively absorbe in the terminal ileum C. Bile acis are responsible for the yellow color of bile D. Bile uct stones occurring 1 year after cholecystectomy are consiere primary common uct stones E. In between meals, gallblaer emptying is stimulate by motilin 10. Which of the following is true regaring biliary anatomy? A. The right hepatic uct tens to be longer than the left an more prone to ilation B. Venous return from the gallblaer is most often via a cystic vein to the portal vein C. Heister valves have an important role in the gallblaer’s function as a bile reservoir D. The CBD an pancreatic uct typically unite outsie the uoenal wall E. The arterial supply to the CBD erives primarily from the left hepatic an right gastric arteries

23

11. Ultrasonography of the gallblaer reveals a polypoi lesion. This most likely represents: A. a cholesterol polyp B. aenomyomatosis C. a benign aenoma D. aenocarcinoma E. an inammatory polyp 12. Which of the following is the correct pairing of anatomic structure an irection for retraction uring a laparoscopic cholecystectomy? A. Gallblaer funus laterally B. Gallblaer infunibulum laterally C. Gallblaer boy laterally D. Gallblaer infunibulum cephala E. Gallblaer funus meially 13. Hyrops of the gallblaer: A. Poses a signicantly increase risk of malignancy B. Is ue to a stone impacte in the cystic uct C. Typically starts with an enteric bacterial infection D. Is associate with marke right upper quarant tenerness E. Results in the gallblaer getting lle with bile-staine ui 14. During a laparoscopic cholecystectomy for symptomatic cholelithiasis, the surgeon inavertently transects the CBD. An experience hepatobiliary surgeon is available. The best choice for operative repair is: A. En-to-en CBD anastomosis B. Choleochouoenostomy C. Choleochojejunostomy D. Hepaticouoenostomy E. Hepaticojejunostomy 15. The most common cause of benign bile uct stricture is: A. Ischemia from operative injury B. Chronic pancreatitis C. Common uct stones D. Acute cholangitis E. Sclerosing cholangitis

24

PArt i Patient Care

16. A 45-year-ol man has a 50% total boy surface area thir-egree burn. Fever, marke leukocytosis, an right upper quarant pain evelop on hospital ay 7. His bloo pressure is 130/80 mm-Hg, an his heart rate is 110 beats per minute. Ultrasonography shows a istene gallblaer with gallblaer wall thickening an sluge. However, it is negative for gallstones. Antibiotics are initiate. The next step in management woul consist of: A. Laparoscopic cholecystectomy B. Compute tomography C. Hepatobiliary iminoiacetic aci (HIDA) scan D. Percutaneous cholecystostomy E. Upper enoscopy 17. During laparoscopic cholecystectomy, bile appears to be emanating near the junction of the CBD an cystic uct. Upon conversion to open cholecystectomy, the injury is note to be a 3-mm longituinal tear in the anterolateral istal common hepatic uct. The uct itself measures 7 mm in iameter. Management consists of: A. Primary repair of the injury without a T tube B. Primary repair of the injury over a T tube C. Primary repair of the injury with a T tube place through a separate choleochotomy D. Hepaticojejunostomy E. Choleochouoenostomy 18. Which of the following statements is true regaring the use of intraoperative cholangiography (IOC) uring laparoscopic cholecystectomy? A. It helps prevent inavertent incision of the common bile uct (CBD) B. It is the best way to ientify clinically signicant common uct stones C. Routine use is justie because of its ability to ientify anatomic anomalies of the hepatic ucts D. Routine use is helpful to ensure complete removal of the gallblaer an cystic uct E. Routine use is unnecessary

19. An 80-year-ol patient presents with nausea, fever, an right upper quarant pain an tenerness. Ultrasonography reveals gallstones as well as air in the wall of the gallblaer. His temperature is 103.5°F an bloo pressure is 70/40 mm-Hg. Meical therapy is initiate, an pressors are neee to maintain bloo pressure. Which of the following is true regaring this conition? A. Metroniazole is an important antibiotic choice B. Emergent cholecystectomy is inicate C. Urgent percutaneous rainage is preferre over cholecystectomy D. The most common organism is an anaerobic gram-negative ro E. Perforation of the gallblaer is rare 20. Which of the following best escribes the role of preoperative biliary rainage before a Whipple proceure in a patient with obstructive jaunice? A. It has been shown to ecrease the rate of cholangitis B. It has been shown to increase the rate of woun infections C. It shoul be performe routinely if the bilirubin level is greater than 8 mg/L D. It has been shown to shorten the hospital stay E. It has been shown to ecrease the mortality rate 21. A 35-year-ol Chinese man presents with a fever of 103.5°F, right upper quarant pain, an jaunice. Laboratory values are signicant for a white bloo cell count of 15,000 cells/L, an alkaline phosphatase level of 400 U/L, an a serum bilirubin level of 3.8 mg/L. Magnetic resonance cholangiopancreatography (MRCP) emonstrates a markely ilate CBD, markely ilate intrahepatic ucts with several intrahepatic uctal strictures, an multiple stones throughout the uctal system. Which of the following is true regaring this conition? A. It is associate with close contact with ogs an sheep B. It is more commonly associate with black pigment stones versus brown pigment stones C. It more commonly affects males D. Metroniazole is able to resolve the majority of cases E. Initial treatment is with enoscopic retrograe cholangiopancreatography an transhepatic cholangiography

CHAPtEr 3 Abdomen—Biliary

22. A 65-year-ol woman presents with symptoms an signs of acute cholecystitis an unergoes an uneventful laparoscopic cholecystectomy. On postoperative ay 7, the pathology report inicates a supercial gallblaer carcinoma that invaes the perimuscular connective tissue. There is no evience of istant metastasis on subsequent imaging. Which of the following woul be the best management? A. Raiation an chemotherapy B. Observation C. Reoperation with resection of liver segments IVB an V D. Reoperation with resection of liver segments IVB an V an regional lymph noe issection E. Reoperation with resection of liver segments IVB an V, regional lymph noe issection, an resection of all port sites 23. A 4-year-ol male presents with acholic stools an cola-colore urine. Alkaline phosphatase is 000 IU/L, AST is 78 IU/L, ALT is 88 IU/L, an total bilirubin is .1 mg/L. Liver biopsy emonstrates periuctal concentric brosis aroun macroscopic bile ucts. He is positive for perinuclear antineutrophil cytoplasmic antiboy (p-ANCA). Which of the following is true about this conition? A. It is more commonly associate with Crohn isease than it is with ulcerative colitis B. Cancer antigen (CA) 19-9 levels shoul be etermine C. Enoscopic retrograe cholangiopancreatography (ERCP) will preominantly emonstrate irregular narrowing of the intrahepatic biliary tree D. Symptoms are often well controlle with meical management E. It is more common in females 24. Which of the following is a feature of gallblaer cancer? A. Speckle cholesterol eposits are foun on the gallblaer wall B. There are thickene noules of mucosa an muscle C. Gallblaer cancer is more common in males D. It is more likely to be accompanie by large gallstones compare with smaller ones E. Cancer invaing muscularis layer is manage with cholecystectomy alone

25

25. Choleochal cyst isease is thought to be cause by an abnormality of the: A. Bile uct smooth muscle B. Bile composition C. Bile uct aventitia D. Pancreaticobiliary uct junction E. Bile uct mucosa 26. On CT scan, a type I choleochal cyst appears to be aherent to the posterior wall of the portal vein. Management consists of: A. Partial excision of the cyst, leaving posterior wall behin, an cholecystectomy with Rouxen-Y hepaticojejunostomy B. Complete excision of the cyst, cholecystectomy, an hepaticojejunostomy C. Partial excision of the cyst, fulguration of posterior cyst mucosa, an cholecystectomy with Roux-en-Y hepaticojejunostomy D. Observation E. Roux-en-Y cyst jejunostomy 27. Which of the following is the best management of a localize Klatskin tumor? A. Pancreaticouoenectomy (Whipple proceure) B. Resection of the entire extrahepatic biliary tree with hepatic resection if necessary C. Resection of the mile thir of the biliary tree with hepaticojejunostomy D. Chemotherapy E. Raiation followe by chemotherapy 28. Which of the following is true regaring cholangiocarcinoma? A. The majority are intrahepatic B. Bismuth-Corlette type I cholangiocarcinoma occurs above the conuence of the right an left hepatic ucts C. Most patients benet from ajuvant chemoraiation after surgical intervention D. It arises from malignant transformation in hepatocytes E. Resection with biliary-enteric bypass is consiere appropriate management in patients with early isease

26

PArt i Patient Care

Answers 1. D. This patient with sickle cell isease has acute calculous cholecystitis an shoul unergo laparoscopic cholecystectomy after ui resuscitation an initiation of antibiotics. Signs that point to acute cholecystitis in this case inclue: fever, positive Murphy sign, leukocytosis, an ultrasoun nings of gallstones, gallblaer wall thickening, an pericholecystic ui. MRCP is reasonable if there is concern for possible choleocholithiasis. However, it is important to istinguish obstructive jaunice from jaunice from hemolytic anemia (as seen in this patient) (A). Although this patient has jaunice, his labs show an increase unconjugated bilirubin. He also oes not have ark urine or acholic stools, an CBD iameter is normal. This is more consistent with hemolytic anemia than with obstructive jaunice (in which you woul expect conjugate bilirubinemia, ark urine, acholic stools, an CBD ilation). This young patient with sickle cell isease has chronic hemolysis, which likely le to evelopment of pigmente gallstones, an now cholecystitis. Sepsis can trigger increase hemolysis in patients with sickle cell isease an is responsible for his perceive increase jaunice since symptom onset. ERCP woul be an appropriate choice if there is a very high suspicion for choleocholithiasis or ascening cholangitis; however, there is no evience of biliary obstruction in this case (B). Percutaneous transhepatic cholangiography can also be use to ecompress the biliary tree, which is not inicate in this case (C). Cholecystostomy tube can be consiere in patients with cholecystitis that are too unstable to unergo cholecystectomy, which is not true in this case (E). 2. A. Ducts of Luschka are small ucts that originate in the gallblaer fossa an rain irectly into the gallblaer, as escribe in this question. When transecte, they can cause bile leaks. When iscovere intraoperatively, the uct shoul be clippe or oversewn. More commonly these are iagnose postoperatively as a ui collection at the gallblaer fossa (biloma) an shoul be raine percutaneously an an ERCP with sphincterotomy an stent placement shoul be performe to encourage bile ow into the uoenum (E). Primary repair over a T-tube an Roux-en-Y hepaticojejunostomy are the appropriate treatment for common bile uct injuries (with 50% luminal injury, respectively), which is not what is escribe in this case (B, C). If a common uct injury occurs at a hospital without a surgeon who is experience in biliary reconstruction, the surgeon shoul place wie rains an then arrange transfer to a referral center. However, that is not necessary in this case (D). References: Mercao MA, Domínguez I. Classication an management of bile uct injuries. World J Gastrointest Surg. 011;3(4):43–48. Spanos CP, Syrakos T. Bile leaks from the uct of Luschka (subvesical uct): a review. Langenbecks Arch Surg. 006;391(5):441–447.

3. C. Hemobilia is a rare conition an presents with a classic (Quinke) tria of upper gastrointestinal bleeing (hematemesis), combine with jaunice an right-sie upper abominal pain. It is most often a result of iatrogenic injury of the right hepatic artery (more common if there is an aberrant right hepatic artery off the superior mesenteric

artery) uring laparoscopic cholecystectomy but may also occur following blunt an penetrating traumatic injuries. The unerlying lesion is typically an arterial pseuoaneurysm that has a connection with the biliary tree (hence the jaunice). It can also occur in association with gallstones, tumors, inammatory isorers, an vascular isorers. Treatment in most instances involves angiographic embolization of the artery (thus angiography is most likely to be the therapeutic stuy of choice). Enoscopy may show bloo coming from the ampulla of Vater but will not typically be therapeutic (because the bleeing is coming from a hepatic artery pseuoaneurysm). The remaining answer choices are not thought to play a role in hemobilia (A, B, D, E). References: Ahrent SA, Pitt HA. Biliary tract. In: Townsen CM, Jr, Beauchamp RD, Evers BM, Mattox KL, es. Sabiston textbook of surgery: the biological basis of modern surgical practice. 17th e. Philaelphia: W.B. Sauners; 004:1597–164. Bloechle C, Izbicki JR, Rashe MY, et al. (1994). Hemobilia: presentation, iagnosis, an management. Am J Gastroenterol. 1994;89(9):1537–1540. Croce MA, Fabian TC, Spiers JP, Kusk KA. Traumatic hepatic artery pseuoaneurysm with hemobilia. Am J Surg. 1994;168(3):35–38. Nicholson T, Travis S, Ettles D, etal. Hepatic artery angiography an embolization for hemobilia following laparoscopic cholecystectomy. Cardiovasc Radiol. 1999;(1):0–4.

4. C. Asymptomatic patients who are incientally iscovere to have gallstones usually o not require surgery because the lifetime risk of eveloping symptoms is 70 years). The most specic stuy to help conrm iagnosis is a CT scan showing air in the biliary tree. It usually results from a large gallstone (>.5 cm) that has eroe through the gallblaer into the

27

ajacent uoenum an causing air in the biliary tree, creating a cholecystouoenal stula (the most common type of biliary stula). Less commonly, the stula can be between the gallblaer an the colon (hepatic exure) or the stomach. The stone typically loges in the narrowest portion of the gastrointestinal tract—the istal ileum, near the ileocecal valve. The iagnosis of gallstone ileus is mae preoperatively in only approximately half of cases because a history of biliary isease may be absent, pneumobilia may not be seen, the gallstone may not be visualize, or the abominal raiographic nings may be nonspecic. Because many of these patients are elerly, have other major comorbiities, an are often markely ehyrate, initial surgical management shoul focus on relieving the obstruction. This is best accomplishe by a transverse enterotomy proximal to the palpable stone an stone removal (C–E). It is also important to run the small bowel because a signicant portion of patients will have more than one gallstone. Leaving the stula oes not seem to lea to signicant morbiity on long-term follow-up. Most surgeons woul not recommen taking the patient back at a later time for stula takeown. A resection of the small bowel is usually not necessary. References: Ahrent SA, Pitt HA. Biliary tract. In: Townsen CM, Jr, Beauchamp RD, Evers BM, Mattox KL, es. Sabiston textbook of surgery: the biological basis of modern surgical practice. 17th e. Philaelphia: W.B. Sauners; 004:1597–164. Roríguez-Sanjuán JC, Casao F, Fernánez MJ, Morales DJ, Naranjo A. Cholecystectomy an stula closure versus enterolithotomy alone in gallstone ileus. Br J Surg. 1997;84(5):634-637. Tan YM, Wong WK, Ooi LLPJ. A comparison of two surgical strategies for the emergency treatment of gallstone ileus. Singapore Med J. 004;45():69–7. Halabi WJ, Kang CY, Ketana N, Lafaro KJ, Nguyen VQ, Stamos MJ, Imagawa DK, Demirjian AN. Surgery for gallstone ileus: a nationwie comparison of trens an outcomes. Ann Surg. 014;59():39–35.

8. D. Bilirubin is the result of the breakown of ol re bloo cells into heme. Heme is broken own into biliverin an then bilirubin. Bilirubin is boun to albumin in the circulation, but as it reaches the liver, it is conjugate an eventually enters the gastrointestinal tract. In the gastrointestinal tract, it is econjugate into urobilinogen by bacteria. Some urobilinogen gets reabsorbe in the gut, returns to the liver, an is excrete in the urine, where it is eventually converte to urobilin, giving urine its yellow appearance. The remaining urobilin is oxiize to stercobilin in the intestines, giving stool its brown appearance. In the presence of biliary obstruction, less bilirubin enters the gut, less urobilinogen is mae, an therefore less appears in the urine. Less stercobilin is mae an therefore the stools turn pale. Hemolysis woul generate an increase in bilirubin an a corresponing increase in urobilinogen in the gut an in the urine (C). The remaining answer choices o not play a signicant role in bilirubin metabolism (A, B, E). Reference: Ahrent SA, Pitt HA. Biliary tract. In: Townsen CM, Jr, Beauchamp RD, Evers BM, Mattox KL, es. Sabiston textbook of surgery: the biological basis of modern surgical practice. 17th e. Philaelphia: W.B. Sauners; 004:1597–164.

9. E. Cholesterol that has been conjugate with taurine or glycine is consiere a primary bile (cholic an chenoeoxycholic aci). Seconary bile acis are a result of bacterial

28

PArt i Patient Care

econjugation in the gastrointestinal tract (A). Although bile acis are passively absorbe along the entirety of the small intestine, they are actively absorbe only in the terminal ileum (B). Bile acis are colorless, an the yellow hue of bile is a result of the pigmente biliverin (breakown prouct of bilirubin) that is also foun in bile (C). Bile uct stones occurring after  years are consiere primary common uct stones an are often pigmente (D). During the fasting state, gallblaer emptying is stimulate by motilin. Reference: Luiking YC, Peeters TL, Stolk MF, et al. Motilin inuces gall blaer emptying an antral contractions in the faste state in humans. Gut. 1998;4(6):830–835.

10. D. The left hepatic uct is longer than the right an is more likely to be ilate in the presence of istal obstruction (A). The spiral Heister valves within the cystic uct o not have any true valvular function (C). In approximately threefourths of iniviuals, the CBD an the main pancreatic uct unite outsie the uoenal wall an traverse the uoenal wall as a single uct. The bloo supply to the CBD runs along the lateral an meial walls at the 3 an 9 o’clock positions an comes from the right hepatic artery an retrouoenal artery (off gastrouoenal artery) (E). Thus, a transverse hemitransection of the uct will likely interrupt the bloo supply an rener a repair prone to ischemia an stricture. Venous return of the gallblaer is typically raine irectly to the parenchyma of the liver (B). Reference: Osottir, M, Hunter, JG. Gallblaer. In: Brunicari FC, Anersen DK, Billiar TR, etal., es. Schwartz’s principles of surgery. 8th e. New York: McGraw-Hill; 005:1187–100.

11. A. Most polypoi lesions of the gallblaer are benign, an of these, cholesterol polyps are the most common. They are usually small (10 mcU/mL with elevate C-peptie (≥.5 ng/mL),

CHAPtEr 5 Abdomen—Pancreas

iii.

iv.

v.

vi.

49

fasting insulin to glucose ratio >0.4, no sulfonylurea or meglitinie etecte; less likely to be etecte with octreotie scan; if can’t localize with CT, can try 18-F-DOPA PET scan 1. Management: b. Manage symptoms with small, frequent meals c. If  mm from the pancreatic uct: enucleate . If > cm OR 1000 (while off PPI ×7 hours at least), gastric pH  cm OR 1000; most in istal pancreas so present late without obstructive jaunice, management: formal resection (inclue splenectomy if oing istal pancreatectomy because of malignancy risk) VIpoma: presents with high-volume watery iarrhea, ehyration, muscle cramping, cutaneous ushing; most are malignant; labs that support the iagnosis: high VIP levels, hypokalemia, achlorhyria, metabolic aciosis, hypercalcemia, hyperglycemia; management: formal resection (inclue splenectomy if oing istal pancreatectomy) Somatostatinoma: from elta cells; mostly malignant (90%); can be associate with neurobromatosis 1; presents with steatorrhea, iabetes, gallstones, hypochlorhyria; iagnose with high fasting somatostatin levels; management: formal resection

V. Pancreatic aenocarcinoma a. Risk factors: increase age, smoking, obesity, new-onset iabetes in elerly b. Workup an staging: CT pancreas protocol, CA19-9, CT chest/abomen/pelvis c. Consier iagnostic laparoscopy to assess for M1 isease prior to resection or neoajuvant therapy . Pancreatic aenocarcinomas with istant metastases (M1) are consiere unresectable an treate with systemic therapy; those not associate with istant metastases are further classie below:

Classięcation

Arterial Contact

Venous Contact

Initial Management

Resectable

None

SMV/PV: ≤180° AND no contour irregularity IVC: no contact

Surgery*

Borderline Resectable

SMA/Celiac: ≤180°

SMV/PV: >180° and/or contour irregularity, appears reconstructable IVC: contact

Neoadjuvant therapy, restage, surgery if appropriate

Locally Advanced

SMA/Celiac: >180° (encasement)

Unreconstructable SMV/PV

Neoadjuvant therapy, restage, surgery if appropriate

SMV = superior mesenteric vein, PV = portal vein, IVC = inferior vena cava, SMA = superior mesenteric artery. *Can also consider neoadjuvant therapy especially for high-risk masses, but most common ABSITE answer is still surgery for resectable disease.

50

PArt i Patient Care

e. If patient has biliary obstruction on presentation an cannot procee irectly to surgery, can have ERCP an stent placement; stent associate with increase risk of perioperative infection; shoul obtain a new CA19-9 level after biliary ecompression f. Generally, if a patient has a symptomatic pancreatic hea mass, you can procee with Whipple without a biopsy g. If planning on neoajuvant therapy, neeEUS-guie biopsy prior to treatment; after completion of neoajuvant therapy, restage with CT an CA19-9 an resect if appropriate h. All pancreatic aenocarcinoma gets ajuvant therapy i. Benets of chemotherapy versus chemoraiation are not clear; either are appropriate

Fig. 5.1 Geography of Pancreatic Neuroenocrine Tumors.

CHAPtEr 5 Abdomen—Pancreas

Fig. 5.2 The Puestow Proceure.

Fig. 5.3 The Beger Proceure.

51

52

PArt i Patient Care

Fig. 5.4 The Frey Proceure.

CHAPtEr 5 Abdomen—Pancreas

53

Questions 1. A 50-year-ol man with a history of Roux-en-Y gastric bypass presents with epigastric pain an fullness two months after an episoe of acute pancreatitis. CT scan reveals an 8-cm pancreatic pseuocyst that abuts the gastric funus. What is the most appropriate management? A. Enoscopic cystogastrostomy via the gastric remnant B. Percutaneous rainage C. Surgical cystogastrostomy via the gastric remnant D. Surgical Roux-en-Y cyst-jejunostomy E. Repeat imaging in 4 weeks 2. A 55-year-ol woman with a history of coronary artery isease is iagnose with a resectable 3-cm insulinoma in the tail of the pancreas. She ha a percutaneous angioplasty with rug-eluting stent placement three weeks ago an is on aspirin an clopiogrel. Despite eating small, frequent meals, she continues to have signicant, intermittent light-heaeness, palpitations, an iaphoresis aily. What is the most appropriate next step in management? A. Octreotie B. Diazoxie C. Neoajuvant chemotherapy D. Continue aspirin an clopiogrel, procee with enucleation E. Hol clopiogrel, continue aspirin, procee with istal pancreatectomy 3. A 60-year-ol man with chronic pancreatitis is presenting for follow-up. Despite alcohol an smoking cessation, oral analgesic meication, celiac axis nerve block, an ERCP with stent placement, he continues to have severe pain an foul-smelling iarrhea. Imaging reveals pancreatic calcication, an enlarge pancreatic hea, an pancreatic uct iameter is 5 mm. What is the most appropriate management to help resolve his symptoms? A. Puestow proceure B. Frey proceure C. Beger proceure D. Minor papilla sphincterotomy E. Whipple

4. Which of the following is true regaring the role of enoscopic retrograe cholangiopancreatography (ERCP) an/or timing of surgery for acute biliary pancreatitis? A. In mil pancreatitis, laparoscopic cholecystectomy can be safely performe within 48 hours of amission B. ERCP with sphincterotomy shoul be use routinely before surgery C. If the total bilirubin fails to normalize, ERCP with sphincterotomy shoul be performe preoperatively D. In severe pancreatitis, early cholecystectomy reuces morbiity an mortality E. There is minimal risk of worsening the pancreatitis with the performance of ERCP 5. Which of the following is true regaring pancreatic cysts? A. Serous cystaenoma has malignant potential B. Asymptomatic patients with mixe-type intrauctal papillary mucinous neoplasm (IPMN) shoul unergo conservative management C. Weight loss in patients with IPMN is mostly attribute to an elevate level of TNF-alpha D. Mucinous cystaenoma usually occurs in women an in the boy or tail of the pancreas E. Asymptomatic mucinous cystaenoma can be manage with repeat imaging in 6 months 6. A 55-year-ol man presents with a 1-hour history of epigastric pain, nausea, an vomiting. He has iffuse mil abominal tenerness to palpation. Laboratory values are signicant for serum amylase of 800 U/L, serum glucose of 130mg/L, chlorie of 104 mEq/L, white bloo cell count of 1,000 cells/μL, serum soium of 15mEq/L, an triglycerie levels of 1800mg/L. The most likely explanation for the hyponatremia is: A. Excessive ui loss B. Inappropriate antiiuretic hormone response C. Excessive free water replacement D. Pseuohyponatremia E. Arenal insufciency

54

PArt i Patient Care

7. Management of pancreatic lymphoma is by: A. Pancreaticouoenectomy B. Chemotherapy C. Pancreaticouoenectomy with postoperative chemotherapy D. Raiation therapy E. Preoperative chemoraiation followe by pancreaticouoenectomy 8. Which of the following is true regaring pancreas ivisum? A. The uct of Santorini ens in a blin pouch B. The inferior portion of the pancreatic hea rains through the uct of Santorini C. The majority of the pancreas rains through the uct of Santorini D. The uct of Wirsung rains through the minor papilla E. The ucts of Wirsung an Santorini fail to evelop 9. The preferre enitive treatment for recurrent acute pancreatitis ue to pancreas ivisum is: A. Lateral pancreaticojejunostomy (Puestow proceure) B. Pancreaticouoenectomy (Whipple proceure) C. Minor papilla sphincterotomy D. Major papilla sphincterotomy an pancreatic uctal septotomy E. Distal pancreatectomy 10. A 50-year-ol male with chronic pancreatitis has faile meical management an is being consiere for more invasive treatment. Which of the following is true regaring potential interventions? A. Pancreaticouoenectomy (Whipple proceure) is inappropriate for chronic pancreatitis B. Enoscopic proceures have been shown to be superior to surgical treatment C. Lateral pancreaticojejunostomy (Puestow proceure) is appropriate if the pancreatic uct is larger than 6 mm D. The most common inication for invasive intervention in chronic pancreatitis is poor exocrine an enocrine function E. Long-term pain control is similar to either the Puestow, Beger, or Frey proceure 11. Aenocarcinoma of the pancreas arises most often from which anatomic site? A. Main pancreatic uct B. Branch pancreatic uct C. Pancreatic acinus D. Ampulla of Vater E. Pancreatic islet

12. A 60-year-ol man presents with obstructive jaunice, acholic stools, an weight loss. An abominal ultrasoun scan emonstrates a ilate biliary tree an no gallstones. A ynamic contrast-enhance CT scan emonstrates a soli mass localize to the hea of the pancreas without evience of istant metastasis, or aenopathy. Vascular involvement can’t be exclue. The patient is otherwise in goo health. Laboratory values are normal. Which of the following is the next step in the management? A. Exploratory laparotomy B. Diagnostic laparoscopy C. MRI D. Enoscopic ultrasoun E. Positron emission tomography (PET) scan 13. Which of the following is true regaring alcohol an its relation to the pancreas an/or pancreatitis? A. It inuces spasm of the sphincter of Oi B. It ecreases pancreatic secretion C. A single episoe of binge rinking cannot lea to pancreatitis D. The type of alcohol consume is an important risk eterminant E. It inhibits chymotrypsin 14. A 48-year-ol male presents with vague abominal pain of  weeks uration. He was recently ischarge for an episoe of alcohol-relate pancreatitis. Laboratory exam is remarkable for a milly elevate serum amylase. A compute tomography (CT) scan emonstrates a 4-cm wellcircumscribe peripancreatic ui collection with homogenously low attenuation. The borers of the collection appear to be ill-ene. The patient is afebrile an hemoynamically stable. What is the most appropriate next step? A. Intravenous (IV) antibiotics an uis B. Amit an place the patient on nothing by mouth (NPO) C. Percutaneous aspirate for carcinoembryonic antigen (CEA) level D. Exploratory laparotomy E. Observe 15. Which of the following is the least favorable management option for a chronic large pancreatic pseuocyst? A. Enoscopic transpapillary rainage using a stent B. Laparoscopic cystogastrostomy C. CT-guie rainage with a pigtail catheter D. Open Roux-en-Y cystojejunostomy E. Enoscopic transgastric cystogastrostomy

CHAPtEr 5 Abdomen—Pancreas

55

16. A 65-year-ol man presents with a persistent skin rash of the lower abomen an perineum, accompanie by intermittent vague left upper quarant pain an recent weight loss. A chemistry panel reveals serum glucose to be 160 mg/L, but results are otherwise unremarkable. CT reveals a large mass in the pancreas. Which of the following is true regaring the most likely conition? A. This patient is at higher risk for venous thromboembolic isease B. The mass is most commonly in the hea of the pancreas C. The secretory peptie responsible for the symptoms also stimulates exocrine pancreatic ow D. Patients often have associate hypokalemia E. These are often benign lesions

20. A 41-year-ol female presents with palpitations, trembling, iaphoresis, an confusion. Serum glucose is 48 mg/L an C-peptie level is elevate. Her symptoms resolve with the aministration of a carbohyrate loa. Which of the following is true regaring the most likely conition? A. Elevate C-peptie an hypoglycemia rule out an exogenous source B. Patients will often have a mass in the neck of the pancreas C. The most sensitive stuy for localization is a high-resolution CT scan D. Recurrent lesions can be manage with streptozocin an 5-FU E. It is the least common functional pancreatic enocrine neoplasm

17. The most common cause of chronic pancreatitis worlwie is: A. Gallstones B. Alcohol abuse C. Hereitary D. Hypertriglyceriemia E. Infectious

21. Octreotie scanning is most useful for localization of which of the following tumors? A. VIPoma B. Glucagonoma C. Pancreatic polypeptie-secreting tumor D. Gastrinoma E. Insulinoma

18. A 35-year-ol cachectic woman presents with episoic severe watery iarrhea that has le to multiple hospital amissions for replacement of uis an electrolytes over the course of several months. Stool cultures are repeately negative an she has no history of travel abroa. On examination, a mass is palpate in the epigastrium/right upper quarant. CT reveals a large, bulky pancreatic mass with extension into the superior mesenteric vein an ajacent organs. The best palliative management option for this patient’s symptoms is: A. Octreotie B. Streptozotocin C. Embolization D. Chemotherapy E. Raiation therapy

22. Which of the following is true regaring pancreatogenic (type 3) iabetes? A. Ketoaciosis is common B. The iabetes is easily controlle C. Peripheral insulin sensitivity is ecrease D. Glucagon an pancreatic polypeptie (PP) levels are low E. Hyperglycemia is usually severe

19. A 65-year-ol male presents for evaluation of yellowing skin. Review of systems is signicant for loose-tting clothes, fatigue, an night sweats. Laboratory evaluation is remarkable for elevate total bilirubin. CT scan reveals a pancreatic mass. Which of the following is least likely to contribute to this conition? A. History of cholecystectomy B. Diabetes C. Smoking D. BRCA E. Coffee consumption

23. A 30-year-ol nurse presents with intermittent iaphoresis, trembling, an palpitations. Her fasting bloo sugar is 50 mg/L. Her insulinto-C peptie ratio is greater than 1. Which of the following is the next step in management? A. CT scan of the abomen B. Psychiatric counseling to iscuss sulfonylurea abuse C. Psychiatric counseling to iscuss exogenous insulin abuse D. Octreotie scan E. Magnetic resonance imaging

56

PArt i Patient Care

24. A 60-year-ol alcoholic man presents with chronic, vague abominal pain. He enies a history of pancreatitis an is otherwise in goo health. CT reveals a 6-cm multiloculate, septate cyst at the tail of the pancreas. FNA of the cyst is noniagnostic. Flui amylase an CEA are in the high normal range. Management consists of: A. Distal pancreatectomy with possible splenectomy B. CT-guie rainage of the cyst C. Enoscopic cystogastrostomy D. Roux-en-Y cystojejunostomy E. Repeat imaging in 6 months

28. Which of the following is true regaring anatomy or the embryologic evelopment of the pancreas? A. The most commonly injure vessel uring issection behin the neck of the pancreas is the celiac vein B. The pancreas receives its arterial supply from only the celiac artery C. The ventral pancreas constitutes the hea an part of the boy of the pancreas D. Venous rainage of the pancreas is to the inferior vena cava E. The uncinate process is orsal to the portal vein an superior mesenteric artery

25. After a motor vehicle accient, persistent ascites evelops in a 55-year-ol man. Other than the ascites, CT nings are unremarkable. Paracentesis reveals clear ui with an amylase level of 5000 U/L. The patient fails an attempt at bowel rest, parenteral nutrition, an paracentesis. Denitive management woul consist of: A. Distal pancreatectomy B. Placement of pigtail catheter C. Roux-en-Y pancreaticojejunostomy D. Pancreaticouoenectomy E. Placement of a transuoenal pancreatic uct stent

29. A 35-year-ol man presents with severe abominal pain an iffuse abominal tenerness. CT scan with IV contrast emonstrates areas of hypoattenuation in the pancreas. His vitals are stable. His temperature is 38.4°C. Which of the following is true regaring his conition? A. Fine-neele aspiration (FNA) for culture shoul be performe B. Early IV antibiotics have emonstrate improve survival C. Early necrosectomy ecreases morbiity an mortality when compare with elaye intervention D. The patient shoul be observe with repeat imaging if he eteriorates clinically E. Percutaneous rainage shoul be performe

26. A 60-year-ol man presents with chronic epigastric abominal pain an jaunice. CT reveals iffuse swelling of the pancreas with compression of the intrapancreatic common uct. Neele biopsy of the pancreas reveals iffuse brosis an a plasma an lymphocytic inltrate. Serum IgG levels are increase. Primary management consists of: A. Whipple proceure B. Sterois C. Chemotherapy D. Hepaticojejunostomy E. ERCP with stenting 27. A 61-year-ol female unergoes a pancreaticouoenectomy (Whipple) operation. On postoperative ay ve she becomes hypotensive, tachycaric, an has severe abominal pain. Nasogastric tube emonstrates bilious output. She receives  L of uis an BP improves to 110 mmHg. A CT scan reveals a signicant amount of free (with HU [hounsel units] of 5). The next step in her management is: A. Angiography with embolization B. Immeiate take back to the OR C. IV octreotie rip D. Transfuse bloo an transfer to ICU E. Upper enoscopy

30. A 60-year-ol woman presents with gallstone pancreatitis. Which of the following is the best preictor of a resiual gallstone persisting in the common bile uct? A. Persistent elevation of the total bilirubin level B. A ilate common bile uct on amission C. Persistent elevation of the alkaline phosphatase level D. Persistent elevation of the serum amylase level E. Persistent abominal pain 31. Which of the following pancreatic cystic lesions is almost exclusively foun in a young female? A. Serous cystic aenoma B. Mucinous cystic neoplasm C. Sie-uct IPMN D. Main-uct IPMN E. Soli pseuopapillary epithelial neoplasm

CHAPtEr 5 Abdomen—Pancreas

57

Answers 1. C. This patient has a symptomatic, large pancreatic

pseuocyst. Since it has been at least 6 weeks after his episoe of acute pancreatitis, an the pseuocyst is >6 cm, he shoul be offere enitive treatment (E). The majority of pancreatic pseuocysts are manage with enoscopic cystogastrostomy as it is minimally invasive an has a high success rate. In orer to perform enoscopic cystogastrostomy, the pseuocyst must abut the gastric wall. However, the above patient has a history of Roux-en-Y gastric bypass. The gastric funus is part of the remnant stomach an is not easily accessible enoscopically. As such, an enoscopic cystogastrostomy woul not be routinely offere as this woul require the expertise of a highly skille enoscopist using ouble-push balloon enoscopy techniques (A). Percutaneous rainage is not an ieal option because there is a high rate of pancreaticocutaneous stula formation an shoul be reserve for infecte pancreatic pseuocysts in patients too unstable for enoscopy or surgery (B). Surgical options inclue cystogastrostomy or Roux-en-Y cyst-jejunostomy. In this case, the patient alreay has a Roux-en-Y bypass an cystogastrostomy is the more appropriate option to rain the pseuocyst without signicantly altering the anatomy (D). Reference: Nealon WH, Walser E. Surgical management of complications associate with percutaneous an/or enoscopic management of pseuocyst of the pancreas. Ann Surg. 005;41(6):948–957.

2. B. This patient is experiencing symptoms of hypoglyce-

mia seconary to her insulinoma espite ahering to eating frequent, small meals. The treatment of choice for insulinomas is surgical removal. If the insulinoma is small ( mm away from the pancreatic uct, enucleation can be performe. Choice (D) is incorrect for two reasons: (1) it woul be inappropriate to procee with major pancreatic surgery while on ual antiplatelet therapy; an () because this patient’s tumor is 3 cm, the treatment of choice is resection with istal pancreatectomy an not enucleation (D). However, because of her history of percutaneous coronary intervention with rug-eluting stent placement less than 1 month ago, she shoul continue ual antiplatelet therapy (E). After rug-eluting stent placement, ual antiplatelet therapy shoul ieally be continue for 6 months to minimize stent thrombosis. If urgent surgery is neee, clopiogrel can be temporarily hel prior to the 6-month mark but shoul not be hel within the rst 4 to 6 weeks when stent thrombosis risk is the highest. Therefore, this patient’s treatment shoul be focuse on symptom management until she is reay for surgery. Diazoxie is the initial meication of choice to control symptoms in patients with insulinomas. It works by inhibiting the release of insulin from beta islet cells. While octreotie is a goo option to control symptoms from VIPomas an glucagonomas, it oes not work reliably for insulinomas as insulinomas o not always contain somatostatin receptors. Octreotie shoul only be consiere as symptom management for insulinomas if octreotie scanning is positive (inicating that the tumor contains somatostatin receptors). Otherwise, octreotie will inhibit glucagon an actually worsen hypoglycemia (A). Chemotherapy is generally only

consiere for metastatic insulinomas an is inappropriate in this case as the majority of insulinomas are benign (C). References: Valgimigli M, Bueno H, Byrne RA, et al. 017 ESC focuse upate on ual antiplatelet therapy in coronary artery isease evelope in collaboration with EACTS: The Task Force for ual antiplatelet therapy in coronary artery isease of the European Society of Cariology (ESC) an of the European Association for Cario-Thoracic Surgery (EACTS). Eur Heart J. 018;39(3):13–60. Gill GV, Rauf O, MacFarlane IA. Diazoxie treatment for insulinoma: a national UK survey. Postgrad Med J. 1997;73(864):640–641.

3. C. This patient has chronic pancreatitis with persistent

pain espite meical management with celiac axis nerve block an is therefore a surgical caniate. When etermining which proceure to perform, there are two main factors to consier: (1) if the pancreatic uct is ilate (≥6 mm), an () if the pancreatic hea is involve. In the case of an enlarge pancreatic hea an a normal-size main pancreatic uct (4 mg/L). Otherwise, an intraoperative cholangiogram shoul be performe, an if a common bile uct stone is etecte, either a laparoscopic common uct exploration or a postoperative ERCP shoul be performe (C).

58

PArt i Patient Care

References: Chang L, Lo S, Stabile BE, Lewis RJ, Toosie K, e Virgilio C. Preoperative versus postoperative enoscopic retrograe cholangiopancreatography in mil to moerate gallstone pancreatitis: a prospective ranomize trial. Ann Surg. 000;31(1):8–87. Kelly TR, Wagner DS. Gallstone pancreatitis: a prospective ranomize trial of the timing of surgery. Surgery. 1988;104(4):600–605. Rosing DK, e Virgilio C, Yaghoubian A, et al. Early cholecystectomy for mil to moerate gallstone pancreatitis shortens hospital stay. J Am Coll Surg. 007;05(6):76–766.

5. D. Serous cystaenoma is a benign true cyst that most

commonly occurs in women an in the pancreatic hea. It is often asymptomatic, but large cysts (>4 cm) may cause vague abominal pain. They o not nee to be resecte unless they are symptomatic (A). Mucinous cystaenoma is consiere premalignant, has a female preominance, occurs commonly in the boy or tail of the pancreas, an shoul always unergo resection (E). IPMN is ivie into three types base on pancreatic uct involvement: main-uct, sie-branch, an mixe-type. Main-uct IPMN carries up to a 50% risk of harboring malignant cells an shoul always be resecte in surgically appropriate caniates. Mixe-type IPMN also has a higher risk an shoul be remove (B). Sie-branch IPMN has a lower risk of malignancy an can be observe unless it is symptomatic, larger than 3 cm, or associate with mural noules. The weight loss in patients with IPMN is mostly attribute to exocrine insufciency from uct blockage an not TNF-alpha cachexia (C).

6. D. Severe hypertriglyceriemia leas to a falsely low

soium level. Water is isplace in the serum by lipis, resulting in an error in measurement. The anger is that the clinician who is unaware may try to correct the hyponatremia with hypertonic saline, leaing to severe hypernatremia. Similarly, a signicantly elevate level of serum glucose can also result in pseuohyponatremia. Excess volume loss seconary to emesis can lea to a hypovolemic hyponatremia but is accompanie by a hypochloremic metabolic alkalosis (A). Patients with gastrointestinal (GI) losses can have hyponatremia exacerbate by excessive free water replacement (C). Arenal insufciency may lea to hyponatremia seconary to the loss of action of alosterone at the istal convolute renal tubules but is accompanie by severe refractory hypotension an marke hyperkalemia (E). Reference: Howar J, Ree J. Pseuohyponatremia in acute hyperlipemic pancreatitis: a potential pitfall in therapy. Arch Surg. 1985;10(9):1053–1055.

7. B. Primary pancreatic lymphoma is extremely rare. Thus,

the management approach is base on case series an experience with lymphoma at other sites. Patients with pancreatic lymphoma may present with symptoms an CT nings suggestive of pancreatic aenocarcinoma, an as such, it may be ifcult to iagnose preoperatively. However, suspicion of lymphoma shoul be raise in the presence of a large bulky pancreatic tumor or with more iffuse pancreatic involvement. This is one situation in which CT-guie neele biopsy of the mass is inicate because the majority of stuies inicate that pancreatic lymphoma respons to chemotherapy as the primary moality. Surgery or raiation is not typically use in the management of pancreatic lymphoma (A, C–E).

References: Arcari A, Anselmi E, Bernuzzi P. Primary pancreatic lymphoma: a report of ve cases. Haematologica. 005;90(1), ECR09. Bouvet M, Staerkel GA, Spitz FR, et al. Primary pancreatic lymphoma. Surgery. 1998;13(4):38–390. Grimison P, Chin M, Harrison M. Primary pancreatic lymphoma-pancreatic tumors that are potentially curable without resection: a retrospective review of four cases. BMC Cancer. 006;6.

8. C. In pancreatic ivisum, the ucts of Wirsung an San-

torini fail to fuse (E). The result is that the majority of the pancreas rains through the uct of Santorini an through the lesser papilla. The inferior portion of the pancreatic hea an uncinate process rains through the uct of Wirsung an the major papilla (B, D). It is consiere a normal anatomic variant an is seen in 10% of iniviuals. It is thought to lea to an increase risk of pancreatitis because the minor papilla sometimes cannot hanle the higher ow of pancreatic juices. In another more common variant, the uct of Santorini ens in a blin pouch but still fuses with the Wirsung uct (A).

9. C. Pancreas ivisum can lea to recurrent episoes of

acute pancreatitis as well as chronic pancreatitis with intractable pain. Unlike other forms of chronic pancreatitis, however, marke ilation of the orsal uct is unusual. As such, surgical ecompressive proceures are not successful (A, B). For patients with recurrent attacks of acute pancreatitis, the best option is sphincterotomy of the minor papilla because the uct of Santorini is proviing the primary rainage to the pancreas. A stuy from Marseille foun a ecrease rate of acute pancreatitis in 4 patients after minor papilla sphincterotomy an orsal uct stenting. The complication rate was lower with sphincterotomy than with stent insertion. Major papilla sphincterotomy woul not likely be helpful because it rains a minority of the pancreas in pancreatic ivisum (D). Distal pancreatectomy is typically not neee (E). Reference: Heyries L, Barthet M, Delvasto C, Zamora C, Bernar JP, Sahel J. Long-term results of enoscopic management of pancreas ivisum with recurrent acute pancreatitis. Gastrointest Endosc. 00;55(3):376–381.

10. C. The most common inication for surgical interven-

tion in patients with chronic pancreatitis is chronic pain (D). Surgical rainage of a ilate pancreatic uct with istal obstruction is more effective than enoscopic approaches in patients with chronic pancreatitis (B). The Puestow proceure involves cutting open the length of the main pancreatic uct an anastomosing a Roux limb of jejunum to the uct but requires a ilate uct (>6 mm). Both the Whipple proceure (for inammation limite to the pancreatic hea) an total pancreatectomy are options for the treatment of intractable chronic pancreatitis, although they are associate with greater morbiity than a rainage proceure (A). The Beger proceure is another option, which resects the pancreatic hea but spares the uoenum, stomach, an bile uct, but this is a technically challenging proceure. The Frey proceure is similar to Beger but easier to perform since it avois the transection of the pancreatic neck over the superior mesenteric vessels. The best long-term pain control is achieve with longituinal pancreaticojejunostomy with limite resection of the hea of the pancreas, which Beger an Frey both satisfy, with Frey being the preferre option (E). However, Frey requires a ilate uct an pancreatic hea.

CHAPtEr 5 Abdomen—Pancreas References: Cahen DL, Gouma DJ, Nio Y, et al. Enoscopic versus surgical rainage of the pancreatic uct in chronic pancreatitis. N Engl J Med. 007;356(7):676–684. DiMagno MJ, DiMagno EP. Chronic pancreatitis. Curr Opin Gastroenterol. 01;8(5):53–531. Jawa ZAR, Kyriakies C, Pai M, et al. Surgery remains the best option for the management of pain in patients with chronic pancreatitis: a systematic review an meta-analysis. Asian J Surg. 017;40(3):179–185. Roch A, Teysseou J, Mutter D, Marescaux J, Pessaux P. Chronic pancreatitis: a surgical isease? Role of the Frey proceure. World J Gastrointest Surg. 014;6(7):19–135.

11. A. The majority of aenocarcinomas of the pancreas

arise from the main pancreatic uct. Approximately 66% of pancreatic aenocarcinomas evelop within the hea or uncinate process of the pancreas. The remaining answer choices can lea to pancreatic aenocarcinoma, but it occurs less frequently (B, C, E). Carcinoma at the ampulla of Vater is most commonly uoenal aenocarcinoma (D). Reference: Albores-Saavera J, Schwartz AM, Batich K, Henson DE. Cancers of the ampulla of Vater: emographics, morphology, an survival base on 5,65 cases from the SEER program: Cancer of the Ampulla of Vater. J Surg Oncol. 009;100(7):598–605.

12. D. In a patient with obstructive jaunice, the rst stuy

to perform is an abominal ultrasoun scan. In the absence of abominal pain an in the presence of weight loss, it is highly likely that the iagnosis is malignancy. A ynamic, contrast-enhance CT scan is highly effective in etermining the resectability of the mass. In cases where vascular involvement is not clear, enoscopic ultrasonography has aie in etermining resectability. Pancreatic cancer is consiere unresectable if the tumor is encasing or occluing the superior mesenteric vein or portal vein an causing vein contour irregularity, as this is consiere unreconstructable. Aitionally, pancreatic cancer is consiere unresectable if the tumor is abutting or encasing the superior mesenteric artery, hepatic artery, or celiac trunk by more than 180°. More frequently, enoscopic guie biopsy is being performe. The avantage of this approach is that there is no risk of tumor seeing because the area through which the neele is passe becomes part of the Whipple specimen. That being sai, in the situation in which the mass appears to be resectable, percutaneous or enoscopic ultrasonography–guie biopsy is not consiere necessary. Neele biopsy is prone to sampling error; therefore, a negative biopsy ning woul not alter the plan to perform a Whipple proceure (A). Likewise, a positive biopsy ning woul not alter the operative ecision. Operative morbiity an mortality after the Whipple proceure are sufciently low that one woul accept the low likelihoo (∼5%) that the lesion is benign. Biopsy shoul be reserve for situations in which the lesion appears to be unresectable because it may guie chemotherapy. It is also inicate in situations in which the appearance of the mass suggests other less common pathologies such as pancreatic lymphoma. Diagnostic laparoscopy is often one before proceeing with a Whipple to conrm there are no obvious hepatic or peritoneal lesions (B). Suspecte lesions are sent for a frozen sample. MRI may be a useful ajunct in patients with equivocal nings on CT or in cases where hepatic metastasis is suspecte (C). The role of PET in cancer workup continues to evelop but as of now it is unclear if PET as

59

any aitional information beyon what is provie with CT (E). References: Small W, Hayes JP, Suh WW. ACR appropriateness criteria [r] borerline an unresectable pancreas cancer. Oncology. 016;30(7):619–619. Tummala P, Junaii O, Agarwal B. Imaging of pancreatic cancer: an overview. J Gastrointest Oncol. 011;(3):168–174. Wang WL, Ye S, Yan S, et al. Pancreaticouoenectomy with portal vein/superior mesenteric vein resection for patients with pancreatic cancer with venous invasion. Hepatobiliary Pancreat Dis Int. 015;14(4):49–435.

13. A. The exact mechanism by which alcohol inuces pan-

creatitis is unclear. Ethanol inuces spasm of the sphincter of Oi, an this may lea to an increase in uctal pressure with a simultaneous brief stimulation of pancreatic secretion (B). It also increases pancreatic uct permeability, ecreases pancreatic bloo ow, an inappropriately activates chymotrypsin (E). Most patients with alcohol-relate pancreatitis have a longstaning history of heavy rinking. The type of alcohol consume is not important but rather the quantity an uration (D). The mean amount consume in patients in whom pancreatitis evelops is 100 to 175 g/ay, although it can rarely evelop after just one binge (C). Aitionally, the risk of pancreatitis seems to be higher in patients who have a iet high in protein an fat.

14. E. The history of recent pancreatitis combine with the

history of vague abominal pain, elevate serum amylase, an CT scan emonstrating a peripancreatic ui collection most likely represents pancreatic pseuocyst. Most patients with pseuocyst o not nee amission an can continue to eat, although a low-fat iet is recommene. Amission an total parenteral nutrition (TPN) woul only be recommene if they were unable to tolerate an oral iet (B). There is no reason to start IV antibiotics because he is not presenting with an infecte pseuocyst (A). Initial management of pseuocysts is conservative via observation because most spontaneously resolve. Pancreatic cyst CEA level is consiere the most accurate tumor marker for iagnosing a mucinous pancreatic cystic lesion. However, in the present setting, given the high suspicion for a pseuocyst, it woul not be neee (C). Invasive interventions are inappropriate because most pseuocysts resolve spontaneously (D). Preictors of failure for conservative management inclue pancreatic pseuocysts larger than 6 cm or those that have persiste for more than 6 weeks. CT or ultrasoun can be use to characterize interval changes in pancreatic pseuocysts.

15. C. Internal rainage is usually preferre to external

rainage for a symptomatic pancreatic pseuocyst that has faile to resolve with conservative therapy. External rainage is associate with a higher rate of complications, incluing infection an pancreaticocutaneous stula. The only inication for percutaneous rainage is in a patient with a ocumente or clinically apparent infected pancreatic pseuocyst that is unstable for a surgical or enoscopic proceure. Pseuocysts communicate with the pancreatic uctal system in 80% of cases. Internal rainage can be achieve enoscopically via a transmural approach or a transpapillary approach. This is gaining popularity making it the new rst-line treatment for pancreatic pseuocyst. If there

60

PArt i Patient Care

is portal hypertension (e.g., splenic vein thrombosis, unerlying cirrhosis, esophageal or gastric varices), then surgical open internal rainage may be more appropriate. Options inclue a cystogastrostomy, a Roux-en-Y cystojejunostomy, an a cyst uoenostomy (A–B, D–E). Cystogastrostomy can be performe enoscopically, laparoscopically, or with a combine approach. Failure of the enoscopic approach can be preicte by the ning of major uctal isruption or stenosis on enoscopic retrograe cholangiopancreatography (ERCP) or magnetic resonance cholangiopancreatography. Regarless of the approach, biopsies of the cyst wall must be one to rule out malignancy. References: Cantasemir M, Kara B, Kantarci F, Mihmanli I, Numan F, Erguney S. Percutaneous rainage for treatment of infecte pancreatic pseuocysts. South Med J. 003;96():136–140. Nealon WH, Walser E. Surgical management of complications associate with percutaneous an/or enoscopic management of pseuocyst of the pancreas. Ann Surg. 005;41(6):948–957. Yusuf TE, Baron TH. Enoscopic transmural rainage of pancreatic pseuocysts: results of a national an an international survey of ASGE members. Gastrointest Endosc. 006;63():3–7.

16. A. Glucagonoma can be remembere by the 4 Ds: ia-

betes, ermatitis, eep vein thrombosis, an epression. The rash is terme necrolytic migratory erythema an tens to manifest on the lower abomen or perineum. The mass characteristically appears in the tail of the pancreas along with VIPoma (a neuroenocrine tumor that secretes vasoactive intestinal polypeptie [VIP]). The responsible hormone, glucagon, inhibits exocrine pancreatic ow (C). The iagnosis of glucagonoma is conrme by measuring fasting glucagon levels. Because the tumors are in the istal pancreas, the patient oes not usually present with jaunice; as such, the iagnosis is often mae late when the tumor is large. Because glucagonoma is most commonly malignant, it shoul be remove with enucleation (if 0.3) an an elevate C peptie level. However, the avent of newer antiiabetic meications such as sulfonylureas can also present with a similar biochemical prole (A). Localization is achieve by CT scan an enoscopic ultrasonography. On occasion, they cannot be localize preoperatively, in which case, intraoperative ultrasonography is useful an is consiere the most sensitive imaging stuy. In contrast to the other functional enocrine pancreatic neoplasms, an octreotie scan is poor at localizing insulinoma owing to the fact that these lesions may not express sufcient somatostatin receptors (C). They are evenly istribute throughout the hea, boy, an tail of the pancreas. There is no pancreatic tumor that characteristically appears in the neck of the pancreas (B). The majority of insulinomas are benign (90%). They can be treate with enucleation. Diazoxie inhibits insulin release an is

CHAPtEr 5 Abdomen—Pancreas occasionally use for preoperative control of symptoms relate to hypoglycemia symptoms. For patients with recurrent or metastatic malignant insulinoma, tumor ebulking may be benecial as is the use of streptozocin an 5-FU. References: Dewitt CR, Hear K, Waksman JC. Insulin an C-peptie levels in sulfonylurea-inuce hypoglycemia: a systemic review. J Med Toxicol. 007;3(3):107–118. Halfanarson TR, Rubin J, Farnell MB, Grant CS, Petersen GM. Pancreatic enocrine neoplasms: epiemiology an prognosis of pancreatic enocrine tumors. Endocr Relat Cancer. 008;15():409–47.

21. D. Many pancreatic enocrine tumors have high con-

centrations of somatostatin receptors an can therefore be image with a raiolabele form of the somatostatin analogue octreotie (inium-111 pentetreotie). Octreotie scanning has the avantage of whole-boy scanning, which is useful in gastrinomas because they can present in a wie area. Use in combination with enoscopic ultrasonography, it etects more than 90% of gastrinomas. It is also useful for localizing carcinoi tumors. As many as 90% of gastrinomas are foun in the Passaro triangle, an area ene by the junction of the cystic uct an common bile uct, the secon an thir portions of the uoenum, an the neck an boy of the pancreas. Although a CT scan is also useful, an octreotie scan is particularly helpful in localizing gastrinomas smaller than 1 cm. Somatostatinoma an VIPoma ten to be large bulky tumors an are thus reaily seen by CT (A). Glucagonoma may present with a mass seen in the pancreatic tail (B). Octreotie scanning will miss as many as 40% of insulinomas because they may not express sufcient somatostatin receptors (E). Pancreatic polypeptie (PP) seems to have an important role in glucose metabolism. PP regulates the expression of the hepatic insulin receptor gene. PP-secreting tumor is rare an often asymptomatic but can be establishe by the presence of an enhancing solitary pancreatic hea tumor on CT imaging with elevate fasting PP level (C). Reference: e Herer WW, Kwekkeboom DJ, Valkema R, et al.

61

to insulin is proinsulin. Proinsulin is package in the pancreatic B cell, where it is cleave to insulin an C peptie, which are then release into the circulation at an equal ratio. Insulin is cleare by the liver, whereas C peptie is cleare by the kiney an is cleare more slowly than insulin, such that the normal insulin-to-C peptie ratio is less than 1 uring fasting. With a true insulinoma, both insulin an C peptie levels woul be elevate; however, the ratio woul still be less than 1. Factitious hypoglycemia will present with an insulin-to-C peptie ratio greater than 1 only if the patient is using exogenous insulin. In contrast, sulfonylurea abuse will have a ratio of less than 1 since it stimulates proinsulin release from the pancreas (B). Factitious hypoglycemia has been reporte more frequently in health-care workers an is associate with a higher incience of suicie, epression, an personality isorers. Thus, the patient shoul be referre for psychiatric counseling. Octreotie scan (D) is not useful in the workup for insulinoma but CT, MRI, or enoscopic ultrasoun may emonstrate a pancreatic mass (A, E). References: Lebowitz M, Blumenthal S. The molar ratio of insulin to C-peptie: an ai to the iagnosis of hypoglycemia ue to surreptitious (or inavertent) insulin aministration. Arch Intern Med. 1993;153(5):650–655. Waickus CM, e Bustros A, Shakil A. Recognizing factitious hypoglycemia in the family practice setting. J Am Board Fam Pract. 1999;1():133–136.

24. A. It is important to be aware that not all ui-lle pan-

pancreatic resection is terme type 3 diabetes. It iffers from type 1 an  iabetes in that it is associate with ecrease glucagon an PP levels an insulin ue to pancreatic loss or estruction. Because all three of these hormones regulate glucose levels, the ensuing iabetes is consiere to be ifcult to control (B). Furthermore, peripheral insulin sensitivity is increase, whereas hepatic insulin sensitivity is ecrease (C). The result is that patients are prone to the evelopment of hypoglycemia, but ketoaciosis an marke hyperglycemia are rare (A, E). For iabetes to evelop as a result of pancreatitis, extensive estruction of the pancreas must occur. In fact, resections involving up to 80% of an otherwise normal glan can be one without enocrine insufciency. This may help explain why not all post-Whipple patients evelop poor glucose control.

creatic abnormalities in a patient with a history of rinking represent pseuocysts (B–E). Some of these lesions may represent cystic neoplasms of the pancreas. Suspicion of a cystic neoplasm shoul be particularly increase in the absence of a history of pancreatitis, as in this patient. A cystic neoplasm shoul also be suspecte when the CT scan emonstrates a soli component (septation) in the cystic lesion. The ifferential iagnosis inclues serous cystaenoma, mucinous cystic neoplasm, intrauctal papillary-mucinous aenoma, an soli pseuopapillary neoplasm. On a CT scan, a central scar is characteristic of a serous cystaenoma (although present in only 0%), whereas the ning of peripheral eggshell calcications, although rare, is iagnostic of mucinous cystic neoplasm an highly suggestive of cancer. In the patient presente, the proceure of choice is surgical resection with istal pancreatectomy an splenectomy. This is base on several factors: the patient is having symptoms; he is a goo caniate for surgery; the lesion is reaily amenable to resection; an the lesion is large, has septations, an has multiple loculations. If, conversely, a patient has an incientally iscovere pancreatic cyst without symptoms, surgery is generally recommene if the risk of surgery is low. Before surgery, further stuies are recommene to attempt to etermine the malignant potential. The workup may inclue MRI, enoscopic ultrasonography to better elineate the mass, an CT-guie aspiration of the ui for amylase level an tumor markers (carcinoembryonic antigen, CA 19–9, CA 15, CA 7–4, CA 15–3).

23. C. Although the patient has symptomatic hypoglycemia,

25. E. After surgery, trauma, or bouts of pancreatitis, per-

Neuroenocrine tumors an somatostatin: imaging techniques. J Endocrinol Invest. 005;8(11 Suppl International):13–136.

22. D. Diabetes in the setting of chronic pancreatitis or after

seemingly consistent with an insulinoma, her insulin-to-C peptie ratio is greater than 1. This combination, particularly in a health-care worker, is highly suggestive of factitious hypoglycemia with exogenous insulin abuse. The precursor

sistent ascites or pleural effusions can evelop. These are generally cause by a isruption of the pancreatic uct, with free extravasation of pancreatic ui, leaing to the evelopment of an internal pancreatic stula, which is rare.

62

PArt i Patient Care

More commonly, the extravasate ui leas to the formation of a containe ui collection known as a pseuocyst. Management of pancreatic ascites or effusion rst requires establishing the iagnosis by obtaining a sample of the ui an emonstrating a markely elevate amylase level an a protein level greater than 5 g/L. Serum amylase may be elevate from reassertion across the peritoneal membrane. The recommene management is a stepwise progression, rst with conservative management with bowel rest, parenteral nutrition, placing the patient NPO, an paracentesis to completely rain the ui. If this fails to resolve the internal stula, ERCP with pancreatic stenting is recommene. If this fails, surgery is inicate an shoul be tailore to the location of the uctal injury (B). For istal uct isruptions, a istal pancreatectomy is recommene (A), whereas for isruption of the boy, a Roux-en-Y pancreaticojejunostomy is performe (C). Whipple proceure (pancreaticouoenectomy) is not neee (D). Conservative therapy incluing somatostatin is successful in only approximately 50%, so nearly one-half will require an invasive proceure. References: Gómez-Cerezo J, Barbao Cano A, Suárez I, Soto A, Rios JJ, Vazquez JJ. Pancreatic ascites: Stuy of therapeutic options by analysis of case reports an case series between the years 1975 an 000. Am J Gastroenterol. 003;98(3):568–577. O’Toole D, Vullierme MP, Ponsot P, et al. Diagnosis an management of pancreatic stulae resulting in pancreatic ascites or pleural effusions in the era of helical CT an magnetic resonance imaging. Gastroenterol Clin Biol. 007;31(8–9 Pt 1):686–693.

26. B. Autoimmune pancreatitis is a form of chronic pan-

creatitis that is increasingly being recognize an can be confuse with pancreatic lymphoma or pancreatic cancer. It presents most often as a iffusely enlarge hypoechoic pancreas. A CT scan often shows iffuse narrowing of the main pancreatic uct without the typical calcications seen with chronic alcoholic pancreatitis. Pathology reveals a plasma cell an lymphocytic inltrate. Laboratory values reveal increase levels of IgG an often iabetes. Antiboies against lactoferrin an carbonic anhyrase have been reporte, but they are not a specic ning. The treatment of choice is steroi therapy, an the isease respons well to this management. Chemotherapy or invasive surgical/enoscopic proceures are not necessary (A, C–E). References: Ketikoglou I, Moulakakis A. Autoimmune pancreatitis. Dig Liver Dis. 005;37(3):11–15. Okazaki K. Autoimmune-relate pancreatitis. Curr Treat Options Gastroenterol. 001;4(5):369–375.

27. A. This presentation is concerning for elaye blee-

ing following a pancreaticouoenectomy (Whipple) proceure. This is most often ue to a gastrouoenal artery stump leak. Flui with HU >5 is most consistent with bloo. CT may show a pseuoaneurysm, but this may not always be present. On hospital ay 5, the tissue planes are often fragile, making it ifcult to control bleeing in the operating room (B). After resuscitation with bloo proucts, the most appropriate next step involves performing an angiography with embolization. A bleeing ulcer is also in the ifferential, but less likely in the absence of blooy nasogastric tube output an with CT nings, so upper enoscopy is not likely to be helpful (E). Esophagogastrouoenoscopy (EGD) nees to be selectively performe this early after surgery because the scope may compromise the freshly mae

gastrojejunostomy anastomosis if the afferent/efferent limbs are to be evaluate. Transfusion of bloo is appropriate but transport interventional suite shoul be next, as the patient may have a heral blee followe by exsanguination (D). Octreotie has no role in the management of gastrouoenal artery stump bleeing (C). One stuy emonstrate that wrapping the gastrouoenal artery stump using the falciform ligament uring surgery may ecrease the risk of this complication. References: Xu C, Yang X, Luo X. Wrapping the gastrouoenal artery stump uring pancreatouoenectomy reuce the stump hemorrhage incience after operation. Chin J Cancer. 014;6(3):99–308. Han GJ, Kim S, Lee NK, et al. Preiction of late postoperative hemorrhage after Whipple proceure using compute tomography performe uring early postoperative perio. Korean J Radiol. 018;19():84–91.

28. E. The ventral pancreas constitutes the uncinate

process an inferior portion of the hea of the pancreas, leaving the remainer the embryologic remnant of the orsal pancreas (C). The uncinate process lies ventral to the aorta but orsal to the portal vein an superior mesenteric artery. The most commonly injure vessel uring issection behin the neck of the pancreas is the superior mesenteric vein (A). The pancreas receives bloo supply from two sources: the celiac axis (superior pancreaticouoenal artery) an superior mesenteric artery (inferior pancreaticouoenal artery) (B). Venous rainage of the pancreas is to the portal system (D).

29. D. CT scan with IV contrast emonstrating areas of

hypoattenuation (nonperfuse) in the pancreas in a patient with this presentation is concerning ue to necrotizing pancreatitis. It is important to note that the necrotic pancreas is not usually infecte initially. Thus, initial management of necrotizing pancreatitis is conservative with the avoiance of early invasive interventions. FNA with culture might be consiere later (because infecte necrosis typically evelops weeks later) in the course of the hospitalization if the patient were to manifest evience of sepsis such as leukocytosis, tachycaria, refractory abominal pain, bacteremia, an/or persistent fevers (A). Prophylactic antibiotics for severe pancreatitis shoul not be routinely aministere (B). In patients with proven (via neele aspiration) infecte necrosis, minimally invasive percutaneous or enoscopic interventions (step-up approach) followe by vieo-assiste retroperitoneal ebriement with the goal of postponing or obviating the nee for open surgery is preferre (E). Furthermore, early necrosectomy has been shown to increase morbiity an mortality when compare with elaye intervention (C). In a patient that oes not appear to have an infecte necrotizing pancreatitis, it is appropriate to approach management conservatively with meical optimization an repeat CT scan if there is a eterioration in clinical status. It is best to allow the patient to manifest the severity of the isease before invasive interventions. References: Bugiantella W, Ronelli F, Boni M, et al. Necrotizing pancreatitis: a review of the interventions. Int J Surg. 016;8 Suppl 1:S163–S171. Mier J, León EL, Castillo A, Robleo F, Blanco R. Early versus late necrosectomy in severe necrotizing pancreatitis. Am J Surg. 1997;173():71–75.

CHAPtEr 5 Abdomen—Pancreas

63

30. A. Although elevation of alkaline phosphatase can be

31. E. Soli pseuopapillary epithelial neoplasm is A

rubin is a useful preictor of persisting common bile uct stone in gallstone pancreatitis. Am Surg. 008;74(10):977–980. Chang L, Lo SK, Stabile BE, Lewis RJ, e Virgilio C. Gallstone pancreatitis: a prospective stuy on the incience of cholangitis an clinical preictors of retaine common bile uct stones. Am J Gastroenterol. 1998;93(4):57–531.

lary epithelial neoplasm—A rare but curable pancreatic tumour in young women. S Afr J Surg. 011;49():78–81.

seen with a resiual common bile uct stone, the best preictor is a persistent elevation of the total bilirubin (C). Amylase is not typically elevate in this patient population (D). Because the pathophysiology of gallstone pancreatitis is transient obstruction of the ampulla of Vater by a gallstone, a signicant number of patients will have some egree of common bile uct ilation on amission; as such, common bile uct ilation is not a specic ning (B). This iffers from patients with symptomatic cholelithiasis, in which uctal ilation is frequently associate with common uct stones. Persistent abominal pain can occur as a result of multiple etiologies an shoul be appropriately worke up with history an physical, laboratory stuies, an/or imaging, if necessary (E). References: Chan T, Yaghoubian A, Rosing D, et al. Total bili-

rare tumor occurring almost exclusively in young women. It has low malignant potential an for the majority of patients, the tumor can be resecte with curative intent regarless of the size. Metastasis an recurrence are uncommon. Serous cystic aenoma also occurs most commonly in women, but this has no malignant potential an oes not nee to be resecte unless it is causing mass effect (A). Mucinous cystic neoplasm is consiere a premalignant lesion, has a female preominance, occurs commonly in the boy or tail of the pancreas, an shoul always unergo resection (B). Main-uct IPMN has a high risk of harboring malignant cells an shoul be resecte (D). Sie-uct IPMN can be manage conservatively unless it is symptomatic, larger than 3 cm, or associate with mural noules (C). Reference: Frost M, Krige JE, Bornman PC. Soli pseuopapil-

Abdomen—Spleen MARIA G. VALADEZ, BENJAMIN DIPARDO, AND ERIC R. SIMMS

6

ABSITE 99th Percentile High-Yields I. Anatomy an Physiology A. White pulp contains macrophages an both B an T lymphocytes B. Re pulp is responsible for removing eforme or abnormal RBCs an nuclear remnants foun in RBCs C. Splenocolic, gastrosplenic, splenorenal, an phrenicosplenic ligaments 1. Short gastric arteries are foun in the gastrosplenic ligament an can be a source of postoperative hemorrhage . Splenic artery lies anterior an superior to the splenic vein 3. Lack of normal peritoneal attachments results in a wanering spleen D. Accessory spleen 1. Suspecte if peripheral bloo smear not consistent with asplenia after splenectomy or if recurrence of primary pathology; splenic hilum is the most common location followe by tail of pancreas II. Splenic masses A. Most common benign splenic tumor: hemangioma B. Most common primary splenic tumor: non-Hogkin lymphoma C. Parasitic cysts (most common worlwie but rare in the Unite States): 1. Majority are hyati cysts seconary to echinococcus; treate with partial or total splenectomy ue to risk of rupture D. Nonparasitic cysts: 1. Cysts can be true cysts or pseuocysts, but this ifferentiation is ifcult to make preop; true cysts or primary cysts have epithelial lining an are congenital; pseuocysts or seconary cysts lack an epithelial lining an typically result from traumatic hematoma formation . If asymptomatic, can be observe with serial imaging regarless of size; if symptomatic, treate with partial or total splenectomy E. Splenic abscesses 1. Etiology: bacteremia, trauma, hemoglobinopathies, splenic artery embolization, following acute pancreatitis, immunosuppression, or trauma . Most common organism: Streptococcus pneumoniae 3. Treatment: IV antibiotics followe by splenectomy (gol stanar) 4. If poor surgical caniate with thick-walle unilocular abscess, treat with percutaneous rainage III. Splenectomy A. Vaccinations 1. Vaccinate against encapsulate organisms: Streptococcus pneumoniae, Neisseria meningitidis, an Haemophilus inuenzae; ieally  weeks before surgery;  weeks after if emergent

65

66

PArt i Patient Care

. Pneumococcal (PPSV3) vaccine shoulbegiven at least 8 weeks after the PCV13 vaccine with revaccination at 5 years; meningococcal (MenACWY) vaccination shoul be given 8 weeks after initial ose with revaccination every 5 years; also require yearly inuenza an COVID vaccination B. Postsplenectomy consierations 1. Overwhelming postsplenectomy sepsis (OPSI) (5 cm) an causing mass effect, resection recommene b) Enoscopic esophagoiverticulostomy: uses stapler or energy evices to ivie the cricopharyngeus enoscopically; ONLY for large iverticula >3 cm (if iverticulum too small, the cricopharyngeal myotomy will be incomplete) c) Open surgery has lower recurrence rates an slightly higher complication rates than enoscopic management V. Esophageal Tumors A. Leiomyoma (hypoechoic mass on enoscopic ultrasoun, more common in males) 1. Most common benign tumor of the esophagus, arise from the smooth muscle cells (mesenchymal); if 5 cm, surgically enucleate (via VATS or laparoscopy)

CHAPtEr 7 Alimentary Tract—Esophagus

77

B. Esophageal cancer 1. Aenocarcinoma an squamous; aenocarcinoma more common in the Unite States . Use enoscopic ultrasoun to etermine T stage, CT for N stage 3. Localize: limite to the mucosa or invaing lamina propria (an N0, M0) a) Enoscopic resection for T1a = within the mucosa (to the lamina propria or muscularis mucosa) b) Esophagectomy for T1b = within the submucosa c) Neoajuvant chemoraiation rst if >T = to the muscularis propria 4. Regional: (noal isease but M0) a) Neoajuvant chemoraiation rst, followe by esophagectomy b) Nee 15 noes for proper oncologic staging 5. Distant (palliative care) 6. Esophagectomy, transhiatal approach (left cervical an abominal incisions) or transthoracic approach (right thoracotomy an abominal incision); similar outcomes, transhiatal may be associate with shorter length of hospital stay 7. Conuit choices after esophagectomy inclue the stomach (most common, nee to preserve the right gastroepiploic artery for perfusion), the colon, an the jejunum 8. Special circumstances a) Upper esophageal cancer (within 5 cm of the upper esophageal sphincter); chemoraiation as primary treatment moality b) Distal esophageal cancer (within 5 cm of GE junction) (1) Some may originate in esophagus an others in stomach () Depening on location, may sprea to meiastinal or abominal noes (3) Most receive neoajuvant or perioperative chemoraiation prior to surgery VI. Esophageal Perforation A. Etiologies: iatrogenic by instrumentation (most common at cricopharyngeus), spontaneous (Boerhaave, occurs 5 cm above GE junction on left), trauma, caustic ingestion B. Workup: CXR (may see left-sie effusion), esophagram with gastrogran → if negative or inconclusive: esophagram with thin barium 1. Gastrogran major side effect: pneumonitis if aspirated . Barium major side effect: severe peritonitis/mediastinitis C. Management: start with NPO, IV ui resuscitation, broa-spectrum antibiotics incluing antifungals D. Nonoperative management: if patient hemoynamically stable, not septic, with mil symptoms, an a containe perforation (minimal meiastinal contamination) E. Surgical approach: right posterolateral thoracotomy for proximal or mile one-thir, left posterolateral thoracotomy for lower one-thir F. Management epens on etiology of perforation 1. Malignant obstruction a) Early cancer: perform esophagectomy b) Avance cancer: esophageal stenting . Benign obstruction a) If ue to achalasia, perform myotomy on contralateral sie 3. Normal esophagus G. Repair of perforation a) Exten myotomy to expose full length of mucosal injury (musical injury often longer than muscle injury) b) Debrie all nonviable tissue c) Two-layer closure (mucosa with absorbable suture an muscle with nonabsorbable suture) ) Reinforce repair with intercostal ap

78

PArt i Patient Care

Fig. 7.1

CHAPtEr 7 Alimentary Tract—Esophagus

79

Questions 1. A 40-year-ol woman with a history of Raynau isease presents for evaluation of ysphagia. Barium esophagram shows reux but no structural abnormalities, an upper enoscopy shows reux esophagitis. Manometry shows absent peristalsis of the istal esophagus an ecrease tone of the lower esophageal sphincter. Initial management consists of: A. Laparoscopic Heller myotomy with partial funoplication B. Proton pump inhibitor an metoclopramie C. Pneumatic ilation D. Calcium channel blocker E. Nissen funoplication 2. A 63-year-ol man with a history of GERD presents for progressive ysphagia over the past 4months. Esophagram shows an irregular lesion in the istal esophagus. Enoscopy with biopsy an enoscopic ultrasoun conrms a  cm irregular mass with invasion into the submucosa. Imaging oes not show any istant masses or abnormal lymph noes. What is the rst step in treatment? A. Chemotherapy B. Chemoraiation C. Enoscopic resection D. Enucleation E. Esophagectomy 3. A 0-year-ol man presents to your ofce after being hospitalize for lye ingestion. Esophagoscopy reveale a high-grae esophageal caustic injury, an the patient was treate with supportive care. The patient asks about the longterm complications after caustic injury to the esophagus. Which of the following is true? A. He is at increase risk for esophageal aenocarcinoma B. He shoul unergo enoscopic surveillance beginning 6 weeks after ingestion C. The most common complication is an esophageal stricture D. Ault caustic ingestion is less severe than peiatric ingestion E. Early use of a neutralizing agent ecreases the risk of subsequent stricture formation

4. A 35-year-ol woman is in the recovery room after enoscopic ilation of a peptic stricture in the mi-thoracic esophagus. She begins to complain of chest pain an oynophagia. She is hemoynamically stable an chest raiograph oes not show any free air or pleural effusion. Esophagram with water-soluble contrast is normal. What is the most appropriate next step in management? A. CT scan of the chest, abomen, an pelvis with IV an oral contrast B. Esophagram with thin barium C. Nasogastric tube placement D. Left posterolateral thoracotomy E. Right posterolateral thoracotomy 5. Which of the following is true regaring the surgical approach, anatomy, or bloo supply to the esophagus? A. Outer longituinal muscle is an extension of the cricopharyngeus muscle B. Cervical esophagus is supplie by the inferior thyroi artery C. The narrowest point in the esophagus is at the aortic arch D. Branches off the intercostal arteries are the major bloo supply to the thoracic esophagus E. The stanar surgical approach to the miesophagus is a left thoracotomy 6. Which of the following statements is true about Mallory-Weiss synrome? A. The chief pathologic ning is spontaneous perforation of the esophagus B. It typically occurs on the right sie C. It is usually associate with air in the meiastinum D. Enoscopy shoul be performe to conrm the iagnosis E. Esophageal balloon tamponae is an appropriate option in cases of persistent bleeing

80

PArt i Patient Care

7. A 40-year-ol female has been using a proton pump inhibitor (PPI) to control gastroesophageal reux isease (GERD) for the past 7 years. She is otherwise healthy. She was seen in clinic an eeme a suitable caniate for enitive surgical intervention. During the operation, after the phrenoesophageal ligament is mobilize, her istal esophagus is inspecte, an it appears shortene. Preoperative upper gastrointestinal stuy i not ientify a hiatal hernia. Which of the following will most likely nee to be one? A. Procee with a stanar Nissen funoplication B. Procee with a Dor funoplication C. Perform Collis gastroplasty D. Abort the operation an initiate management with sterois E. Take several biopsies before aborting the operation

11. Esophageal manometry performe in a patient with a true paraesophageal hernia will emonstrate that the LES is: A. Above the normal position B. At the normal position C. Hypertensive D. Hypotensive E. Short

8. A 51-year-ol male has been unergoing yearly enoscopy with biopsy for Barrett esophagus (BE). His most recent biopsy emonstrates highgrae ysplasia without noules. Which of the following is the best next step in management? A. Esophagectomy with reconstruction B. Repeat enoscopy with biopsy in 3 months C. Enoscopic raiofrequency ablation D. Antireux operation E. Oncology referral for consieration of neoajuvant chemotherapy

13. Which of the following will preispose a patient to the evelopment of esophageal isease? A. LES length of 3 cm B. Resting LES pressure of 8 mm Hg C. Resting upper esophageal sphincter (UES) pressure of 70 mm Hg D. Abominal length less than 1 cm E. Relaxation of LES with swallowing

9. Barrett esophagus: A. Is a congenital abnormality B. Occurs more frequently in women C. When iagnose, shoul be treate with an antireux proceure to prevent cancer D. Diagnosis requires replacement of a 3-cm segment of the squamous cells by columnar epithelium E. Features the presence of goblet cells 10. Which of the following is true regaring Barrett esophagus? A. PPIs are consiere a more effective treatment option than H blockers B. Dietary restrictions such as those use for patients with GERD are not useful C. Patients with short- an long-segment Barrett esophagus have a similar risk of high-grae ysplasia D. Use of high-ose PPIs with aspirin is contrainicate E. Photofrin is a useful treatment moality

12. Which of the following statements about a paraesophageal hernia is true? A. It is associate with anemia B. It oes not pose a risk for incarceration an strangulation C. Diagnosis is not reaily mae with upper enoscopy D. It is usually cause by a traumatic injury E. It rarely requires operative repair

14. A 5-year-ol male with cirrhosis an known esophageal varices presents with a large amount of hematemesis. Which of the following statements is true? A. Beta blockae is ineffective for preventing rebleeing B. The most important next step is enoscopy for both iagnostic an therapeutic interventions C. Prophylactic antibiotics o not improve survival D. Early aministration of vasoactive rugs oes not improve outcomes E. Enoscopic ban ligation has been emonstrate to be superior to enoscopic sclerotherapy

CHAPtEr 7 Alimentary Tract—Esophagus

15. A 59-year-ol iabetic male with a history of chronic obstructive pulmonary isease (COPD) an prior congestive heart failure presents with a -year history of progressively ifcult swallowing. Esophagram emonstrates a ilate proximal esophagus with abrupt tapering istally. Manometry shows high pressure in the lower esophageal sphincter (LES) at rest an failure of the LES to relax after swallowing. Upper enoscopy is negative. Which of the following is true regaring this patient? A. The unerlying conition is characterize by high-amplitue peristaltic waves of the esophagus B. Laparoscopic esophagomyotomy with complete funoplication is the treatment of choice C. A trial of calcium channel blockers shoul be starte D. Esophageal pneumatic ilation is the next step in management E. Peroral enoscopic myotomy (POEM) is the treatment of choice 16. During the course of an upper enoscopy for manometry conrme achalasia, the enoscopist thinks he may have cause an inavertent perforation of the left lower istal esophagus. The patient is stable an shows no signs of sepsis. Esophagogram conrms a markely ilate esophagus with a istal-free perforation. Management consists of: A. Intravenous (IV) antibiotics, placing patient NPO (nothing by mouth), an close observation B. Left thoracotomy, primary repair, longituinal myotomy on the contralateral sie C. Laparoscopic primary repair an longituinal myotomy on the ipsilateral sie D. Esophagectomy with immeiate reconstruction E. Esophageal stent placement 17. A 36-year-ol male presents for consultation regaring an inciental esophageal mass seen on compute tomography (CT) scan. This was performe after he was involve in a motor vehicle collision (MVC). He ha no serious injuries an was ischarge the same ay. Barium swallow emonstrates a smooth, crescent-shape lling efect. Which of the following is true regaring this mass? A. Resection with a 1-cm margin is the treatment of choice B. They most commonly present with satellite tumors C. They have no risk of malignant egeneration D. Esophageal ultrasonography may be useful E. A preoperative enoscopic biopsy shoul be performe

81

18. Which of the following statements is true about Zenker iverticulum? A. It is a true iverticulum B. It is best iagnose with esophagoscopy C. It is unlikely to cause aspiration D. It is a pulsion iverticulum E. Small iverticula (5 cm). Otherwise, they are incientally iscovere uring the course of other stuies. They have a characteristic appearance on barium swallow of a smooth, crescent-shape lling efect that encroaches on the lumen. On enoscopy, the mucosa is usually intact, an the tumor moves up an own with swallowing. If it has the characteristic appearance, the tumor shoul not unergo biopsy because of an increase risk of mucosal perforation. This can create scarring that may affect later efforts at resection (E). Esophageal ultrasonography is very useful in the iagnosis of leiomyomas because it will emonstrate a homogeneous region of hypoechogenicity. Treatment is to enucleate the mass, which can be one via a vieoscopic approach with intraoperative esophagoscopy (A). The cell of origin of these tumors is mesenchymal. The average age at presentation is 38 years, an they are twice as common in males an most commonly locate in the lower two-thirs of the esophagus. Leiomyomas are usually solitary, but multiple tumors are seen in as many as 10% of patients (B). Reference: Aurea P, Grazia M, Petrella F, Bazzocchi R. Giant leiomyoma of the esophagus. Eur J Cardiothorac Surg. 00;(6):1008–1010.

18. D. A Zenker iverticulum is a false esophageal iver-

ticulum that oes not contain all layers of the esophagus; it is also a type of pulsion iverticulum (A). A pulsion iverticulum forms at a point of weakness an is ue to alterations in luminal pressure. Conversely, a traction iverticulum is from external pulling on the esophageal wall, such as from iname lymph noes with tuberculosis. Zenker iverticulum is the most common type of esophageal iverticulum. It usually presents in oler patients (>60 years). It characteristically arises at a point of weakness, most commonly at the Killian triangle, which is forme by the inferior bers of the inferior constrictor muscle an the superior borer of the cricopharyngeus muscle. Patients typically present with ysphagia, regurgitation of unigeste foo, halitosis, episoes

86

PArt i Patient Care

of aspiration, an salivation (C). With the characteristic history, the rst iagnostic stuy is a barium swallow. In the absence of other pathology (such as an irregular mucosa), enoscopy is not neee (B). Treatment is surgical by either open or enoscopic techniques. The open technique involves cervical esophagomyotomy with stapling an amputation of the iverticulum. The enoscopic technique involves ivision of the common wall between the iverticulum an the esophagus. Stuies have shown that results with the enoscopic technique are better with larger iverticula (E). Diverticula smaller than 3 cm are too short to accommoate one cartrige of staples an to allow complete ivision of the sphincter; therefore, this size is consiere a contrainication to this technique. References: Bonavina L, Bona D, Abraham M, Saino G, Abate E. Long-term results of enosurgical an open surgical approach for Zenker iverticulum. World J Gastroenterol. 007;13(18):586–589. Collar JM, Otte JB, Kestens PJ. Enoscopic stapling technique of esophagoiverticulostomy for Zenker’s iverticulum. Ann Thorac Surg. 1993;56(3):573–576. Narne S, Cutrone C, Bonavina L, Chella B, Peracchia A. Enoscopic iverticulotomy for the treatment of Zenker’s iverticulum: results in 10 patients with staple-assiste enoscopy. Ann Otol Rhinol Laryngol. 1999;108(8):810–815.

19. A. Cricopharyngeal ysfunction has multiple causes,

incluing such neurogenic an myogenic etiologies as stroke, multiple sclerosis, peripheral neuropathy, Parkinson isease, an ermatomyositis. The exact cause is unknown, but the primary theory is that the cricopharyngeus muscle, which is normally in a state of tonic contraction, fails to relax an allow the passage of foo into the cervical esophagus. This prouces a Zenker iverticulum, which is consiere a false iverticulum (only involves an outpouching of the mucosa an submucosa) an can be conrme with a barium swallow (C). Enoscopic evaluation of a suspecte Zenker iverticulum is iscourage as it can lea to an iatrogenic perforation. Patients escribe ifculty swallowing foo, which worsens throughout the ay as the iverticulum increasingly gets lle with foo. Another key element of the iagnosis is the classic history of an inability to hanle saliva secretion, such that the patient escribes expectoration of saliva. Patients also report hoarseness. Diverticulectomy is often performe uring surgery for a Zenker iverticulum. However, the most important aspect of management is cricopharyngeal myotomy, which is necessary to correct the unerlying pathology (B). Weight loss results from a ecrease caloric intake. Although one shoul always be suspicious of carcinoma in a patient with ifculty swallowing an weight loss, the long uration of symptoms makes carcinoma unlikely (D, E). References: Cameron JL, Cameron AM. The management of Barrett’s esophagus. In: Cameron JL, Cameron AM, es. Current surgical therapy. 11th e. Philaelphia: W.B. Sauners; 014. Cameron JL, Cameron AM. The management of pharyngeal esophageal (Zenker) iverticula. 11th e. Philaelphia, PA: W.B. Sauners; 014.

20. B. Patients with newly iagnose esophageal cancer

frequently present with poor nutritional status, which only worsens after starting neoajuvant therapy. As such, although nutritional optimization is an important component in the management of esophageal cancer, the optimal approach remains unene. Percutaneous gastrostomy, however, shoul be iscourage because it may compromise the gastric conuit neee uring esophageal reconstruction an will elay chemotherapy for an aitional  to 4weeks. The role for parenteral nutrition is limite because of its high cost an high rate of complications (A). Nasogastric tube insertion can lea to migration of the tube an aspiration (D). Esophageal stents are frequently offere because they can signicantly improve the ysphagia associate with esophageal cancer. Unfortunately, its role in improving nutritional status has ha inconsistent results in the literature (C). Stent migration an chest iscomfort are common an lea to the frequent removal of the stents (E). Aitional stuies are neee to etermine the best approach for nutritional optimization in this patient population. References: Jones CM, Grifths EA. Shoul oesophageal stents be place before neo-ajuvant therapy to treat ysphagia in patients awaiting oesophagectomy? Best evience topic (BET). Int J Surg. 014;1(11):117–1180. Mão-e-Ferro S, Serrano M, Ferreira S, et al. Stents in patients with esophageal cancer before chemoraiotherapy: high risk of complications an no impact on the nutritional status. Eur J Clin Nutr. 016;70(3):409–410. Naharaja V, Cox MR, Eslick GD. Safety an efcacy of esophageal stents preceing or uring neoajuvant chemotherapy for esophageal cancer: a systemic review an meta-analysis. J Gastrointest Oncol. 014;5():119–16.

21. C. Surgical intervention in esophageal cancer is an

area of active research. The three stanar approaches inclue TTE, THE, an a combination of the two using a three-incision esophagectomy. TTH was initially escribe as a two-stage proceure by Dr. Ivor Lewis in which he performe mobilization of the stomach using an upper miline laparotomy incision followe by resection of the esophagus using a right thoracotomy incision several ays later (A). A large multicenter prospective stuy comparing THE an TTE faile to emonstrate any ifference in overall mortality an morbiity between the two approaches (D). However, THE has been shown in several stuies to be associate with a lower total hospital length of stay (B). THE is performe with a left cervical incision an miline laparotomy (E). It is often performe for patients with istal esophageal cancer. References: D’Amico TA. Outcomes after surgery for esophageal cancer. Gastrointest Cancer Res. 007;1(5):188–196. Hulscher JB, Tijssen JG, Obertop H, van Lanschot JJ. Transthoracic versus transhiatal resection for carcinoma of the esophagus: a meta-analysis. Ann Thorac Surg. 001;7(1):306–313. Litle VR, Buenaventura PO, Luketich JD. Minimally invasive resection for esophageal cancer. Surg Clin North Am. 00;8(4):711–78. Rentz J, Bull D, Harpole D, et al. Transthoracic versus transhiatal esophagectomy: a prospective stuy of 945 patients. J Thorac Cardiovasc Surg. 003;15(5):1114–110.

Alimentary Tract—Stomach NAVEEN BALAN, AMY KIM YETASOOK, AND KATHRYN T. CHEN

8

ABSITE 99th Percentile High-Yields I. Ulcers A. Peptic ulcer isease (PUD): imbalance of pepsin/aci an mucosal protection 1. Almost always cause by Helicobacter pylori (gram-negative spirochete) an NSAID overuse . Triple therapy: PPI, clarithromycin, amoxicillin, or metroniazole ×14 ays 3. Daintree Johnson classication for types of gastric ulcers a) Type I: along lesser curvature in the antrum, solitary, not aci associate b) Type II: prepyloric, solitary, aci associate c) Type III: prepyloric an uoenal, one in each location, aci associate ) Type IV: proximal stomach/caria, solitary, not aci associate e) Type V: anywhere in the stomach, usually multiple, NSAID associate 4. Biopsy all gastric ulcers: higher risk of cancer

Operation

Procedure

Result

Complication

Antrectomy

Resection of distal stomach (need reconstruction with Billroth I or II)

Removal of antral G cells

Incomplete antrectomy can lead to continued ulcer disease

Truncal vagotomy (TV)

Ligation of anterior and posterior vagal trunks 4 cm proximal to GEJ

Removal of vagal stimulation to gastric body parietal cells, decreasing acid secretion

Loss of parasympathetic innervation to pylorus which prevents gastric emptying (primarily to solid foods)

Selective vagotomy (SV)

Ligation of anterior and posterior vagal trunks distal to hepatic/ celiac branches

Lower rate of delayed gastric emptying but higher risk of recurrent ulcer disease

Highly selective vagotomy (HSV)

Similar to SV except preserves crow’s foot ębers innervating pylorus and antrum

Lowest rate of delayed gastric emptying but highest risk of recurrent ulcer disease

Pyloroplasty

Widen the pyloric channel, multiple approaches

Improved gastric emptying (usually with TV/SV)

Can lead to rapid gastric emptying

B. Surgical inications: surgery for bleeing PUD entails oversewing of the ulcer (to ligate the bleeing artery) with consieration for truncal vagotomy (to ecrease aci secretion); more time-consuming proceures (selective vagotomy) reserve for elective ulcer surgery, as are antrectomy an Billroth II (when obstruction complicates PUD)

87

88

PArt i Patient Care

1. Truncal vagotomy an selective vagotomy always nee rainage proceure (e.g., pyloroplasty) whereas highly selective vagotomy oes not

Duodenal Ulcer* Problem

Treatment

Bleeding

Oversew bleeder

Perforation

Graham patch

*For gastric ulcer, ęrst-line treatment is same; however, ęrst biopsy ulcer to rule out malignancy.

II. Upper Gastrointestinal Blee A. Meical management: hyration, PPI bi (noninferior to continuous PPI) ×7 hours B. Proceural interventions: enoscopic hemostasis attempts ×, angioembolization(usually GDA); n look EGD only for high-risk lesion (visible vessel or aherent clot) C. Surgery inications: multiple faile enoscopies, hemoynamic instability, associate perforation III. Bariatric Surgery A. Inications: 1. BMI >40 (morbi obesity) or BMI 35 to 40 with ≥1 weight-relate comorbiity (iabetes, hypertension, hyperlipiemia, GERD, obstructive sleep apnea, pseuotumor cerebri, severe lifelimiting arthritis) B. Sleeve gastrectomy: most commonly performe, lower excess boy weight (EBW) loss an incience of vitamin eciency; postoperative complications inclue staple line leak, bleeing, e novo reux, incisura stricture, twisting/kinking of sleeve, weight regain C. Roux-en-Y gastric bypass: higher EBW loss an incience of vitamin eciency; postoperative complications inclue anastomotic leak, internal hernia, marginal ulcer, perforate marginal ulcer, bile reux, jejunojejunostomy, intussusception, roux stasis synrome, gastrogastric stula, umping synrome, an weight regain D. Pulmonary embolus is leaing cause of eath, occurring within a few weeks after ischarge E. Common postoperative complications:

Complication

Symptoms

Etiology

Management

Alkaline reĚux (requires surgery more than any other postgastrectomy syndrome)

Bilious emesis does NOT relieve epigastric pain

Bile reĚux into the stomach usually with Billroth II anatomy

Workup with EGD (pathology: foveolar hyperplasia with mucosal congestion and edema); conęrm with HIDA demonstrating biliary secretion into stomach/esophagus; treatment: convert to Roux-en-Y

Small intestinal bacterial overgrowth (SIBO)

Bloating, chronic watery diarrhea

Bacterial overgrowth in blind loop due to immotility

Labs for B12 deęciency, macrocytic anemia, conęrm with lactulose breath test and treat with antibiotics (rifaximin)

Early dumping syndrome

Nausea, vomiting, diarrhea 15–30 min postprandially

Hyperosmolar load and release of serotonin, neurotensin, bradykinins, and enteroglucagon

Eat smaller, more frequent meals with low carbs, high protein, and fat throughout the day, separate solids from liquids, octreotide is most eěective

Late dumping syndrome

Dizziness, fatigue, diaphoresis 1–3 h postprandially

High carbohydrate load causing hyperinsulinemia leading to reactive hypoglycemia

Similar to above; if refractory, convert to Roux-en-Y

Marginal ulcer

Nausea, pain, and perforation/bleeding at anastomosis

NSAIDs, smoking, H. pylori infection, acid exposure to small bowel

Discontinue NSAIDs, treat H. pylori, give acid suppression therapy, bleeding or perforation may need surgery

CHAPtEr 8 Alimentary Tract—Stomach

89

IV. Gastric Cancer A. Aenocarcinoma: risk factors inclue H. pylori infection, iet (high salt, nitrosamine-containing foos), obesity, smoking, ETOH, hereitary iffuse gastric cancer (CDH1 mutations) 1. Staging: EGD for biopsy, EUS for T/N stage, CT chest/abomen/pelvis, +/− PET-CT, iagnostic laparoscopy to evaluate for peritoneal isease . Early gastric cancer: cTis or T1a-enoscopic or surgical resection; T1b-surgical resection with 5 cm gross margins to achieve R0 resection 3. Locoregional isease: T+ or noal involvement nee neoajuvant chemotherapy followe by surgical resection an ajuvant chemotherapy 4. Surgical options: istal versus subtotal versus total gastrectomy with R0 goal, nee 16 lymph noes; CDH1 nees total gastrectomy, also acceptable option for palliation in stage IV isease 5. Siewert-Stein classication: a) Type I: epicenter within 1 to 5 cm above GE junction, treat as esophageal cancer b) Type II: epicenter within 1 cm above an  cm below GE junction, treat as esophageal cancer c) Type III: epicenter within  to 5 cm below GE junction, treat as stomach cancer 6. Incience of PROXIMAL gastric cancer has been rising the fastest (ue to obesity an reux); proximal gastric cancer is two times more common in men an white race 7. H. pylori is PROTECTIVE against proximal gastric cancer but associate with increase risk of istal gastric cancer B. Gastrointestinal stromal tumor (GIST) 1. Enoscopy: submucosal mass with central umbilication an ulceration . Essential features: majority ue to mutations in protooncogene KIT (most common) or PDGFRA; spinle cell histology; can occur anywhere along GI tract, but stomach is most common (60%) followe by jejunum/ileum 3. High-risk features for recurrence: tumor size >5 cm or >5 mitoses/hpf 4. Imatinib: tyrosine kinase inhibitor, highest response rate with exon 11 KIT mutation a) Neoajuvant therapy: use when surgical resection will result in signicant morbiity b) Ajuvant therapy: high-risk features for recurrence an tumor rupture uring surgery C. MALT lymphoma: B-cell lymphoma, arises in the setting of chronic H. pylori infection 1. Histology: lymphoepithelial lesions are pathognomonic . Treatment: triple or quaruple antibiotic therapy will eraicate cancer in majority of early cases

90

PArt i Patient Care

Fig. 8.1

CHAPtEr 8 Alimentary Tract—Stomach

Fig. 8.2

91

92

PArt i Patient Care

Questions 1. A 56-year-ol male with fatigue unergoes upper enoscopy after initial workup shows a microcytic anemia. He is foun to have an ulcer with irregular borers with biopsy showing a ense lymphoi inltrate with prominent lymphoepithelial lesions. Which of the following is true for this malignancy? A. Tumors with chromosomal translocation t(11;18) respon poorly to antibiotics B. After successful treatment, yearly enoscopy is use for surveillance C. It is most commonly a result of a grampositive ro D. There is no role for surgical resection E. Early-stage isease requires chemoraiation as rst-line treatment 2. A 46-year-ol female was incientally foun to have a peunculate mass along the greater curvature of the stomach on CT imaging following a motor vehicle collision. Further workup with enoscopy shows a submucosal mass with central umbilication an ulceration that is foun to be CD117-positive after biopsy. Which of the following is true regaring the management of this lesion? A. It is consiere a raiosensitive tumor B. The highest response rate to therapy involves mutations in the KIT proto-oncogene at exon 11 C. It arises from an enoermal-erive component D. Most patients become symptomatic early in the course of their isease E. Early tumors can be treate with enoscopic mucosal resection

3. A 41-year-ol female presents to the emergency epartment with acute severe abominal pain an nausea but no vomiting. She reports a history of uncomplicate Roux-en-Y gastric bypass years ago but enies other abominal surgeries. Abominal exam reveals mil tenerness without guaring or reboun tenerness. A CT scan shows ilate proximal small bowel but no intraabominal free air or ui. What is the next best step in the management of this patient? A. Avise the patient to eat smaller, more frequent meals B. Exploratory laparotomy C. Nasogastric tube ecompression followe by 4-hour water-soluble contrast challenge D. Antibiotic therapy E. Upper enoscopy 4. A 40-year-ol male with severe epigastric pain is foun to have multiple uoenal ulcers on EGD. What is the normal location of the cells that secrete the majority of the hormone that is being overprouce in this patient? A. Stomach boy B. Stomach antrum C. Pancreas D. Duoenum E. Jejunum 5. Three years after a laparoscopic Roux-en-Y gastric bypass (LRYGB), a 45-year-ol male presents with symptoms an signs of a small bowel obstruction (SBO). He reports a 150-lb weight loss. Which of the following is the most likely etiology? A. An internal hernia B. Ahesions C. Roux compression ue to mesocolon scarring D. Kinking of the jejunojejunostomy E. Incarcerate abominal wall hernia

CHAPtEr 8 Alimentary Tract—Stomach

6. A 79-year-ol male with chronic back pain an chronic obstructive pulmonary isease (COPD) requiring supplemental oxygen presents to the emergency epartment (ED) with epigastric abominal pain that starte suenly  ays ago. His abominal examination is signicant for epigastric tenerness but is otherwise unremarkable. A compute tomography (CT) scan emonstrates a small amount of free air uner the right hemiiaphragm but no contrast extravasation. An upper gastrointestinal (GI) water-soluble contrast stuy emonstrates a uoenal ulcer but no extravasation. Which of the following is the best management? A. Nasogastric tube ecompression, intravenous (IV) antibiotics, an proton pump inhibitor (PPI) B. Exploratory laparotomy C. Diagnostic laparoscopy D. Oral antibiotics, clear liqui iet for  weeks, an follow-up in clinic E. Serial abominal exam in the ED for 6 to 8hours an, if improving, he may be ischarge with oral antibiotics 7. Which of the following is true regaring the management of obesity? A. Inications for bariatric surgery inclue a boy mass inex (BMI) greater than 30 with weight-relate comorbiities or BMI greater than 35 B. Sibutramine acts by inhibiting pancreatic lipase C. Roux-en-Y gastric bypass (RYGB) oes not have a restrictive component D. RYGB has a lower 30-ay mortality compare with biliopancreatic iversion (BPD) E. Patients with obesity-relate comorbiities o not nee to attempt nonoperative management before obesity surgery 8. Which of the following is the gol stanar for the iagnosis of gastroparesis? A. Upper enoscopy B. Plain abominal x-rays C. Nuclear meicine scan D. CT E. It is consiere a clinical iagnosis.

93

9. Which of the following is true regaring gallstone isease after weight loss surgery? A. The rate of postoperative cholecystectomy is the same regarless of the type of weight loss surgery B. Prophylactic cholecystectomy shoul be performe at the time of surgery in most patients C. Ursoiol is recommene for 6 months after gastric bypass surgery D. Decrease secretion of calcium an mucin contributes to gallstone formation after weight loss surgery E. Acute cholecystitis after weight loss surgery is uncommon 10. Which of the following is the rst manifestation of gastric leak following Roux-en-Y gastric bypass? A. Abominal pain B. Tachycaria C. Nausea D. Increase serum glucose E. Tachypnea 11. A 45-year-ol male with a history of laparoscopic gastric baning 5 years ago presents to the ED with complaints of pain at his port site. He rst notice it several ays ago after he got his gastric ban ajuste in clinic. On exam, the port site appears erythematous, warm, an is tener to palpation. He is afebrile an normotensive. Which of the following is the best next step? A. CT of the abomen B. Amit to the hospital, start IV antibiotics an ui resuscitation C. EGD D. Incision an rainage E. Discharge with oral antibiotics

94

PArt i Patient Care

12. A 60-year-ol man presents with a 1-hour history of worsening epigastric pain. He has a history of uoenal ulcer, an the results of a recent biopsy  weeks earlier were negative for Helicobacter pylori. Upright chest raiograph emonstrates free air uner the iaphragm. The patient is hemoynamically stable. At surgery, a perforate uoenal ulcer is foun with mil peritoneal contamination. Which of the following is the best management option? A. Graham patch of uoenal ulcer B. Graham patch of uoenal ulcer with truncal vagotomy an pyloroplasty C. Truncal vagotomy an antrectomy with Billroth I reconstruction D. Truncal vagotomy an antrectomy with Billroth II reconstruction E. Graham patch of uoenal ulcer with a highly selective vagotomy 13. Which of the following is true regaring postvagotomy iarrhea? A. It is effectively treate with octreotie B. It oes not improve with oral cholestyramine C. Cariovascular manifestations are common D. Most patients require the creation of a reverse jejunal segment E. Diarrhea may improve with the aministration of coeine 14. A 45-year-ol woman is unergoing an exploratory laparotomy for Zollinger-Ellison synrome (ZES). Preoperative localization stuies faile to emonstrate the location of the tumor. At surgery, no obvious tumor is seen espite an extensive Kocher maneuver an careful inspection. An intraoperative ultrasoun scan is negative. The next step in the management woul be: A. Closing the abomen B. Distal pancreatectomy an splenectomy C. Proximal pancreaticouoenectomy D. Blin proximal uoenotomy E. Blin istal uoenotomy

15. Which of the following is true regaring TNM (tumor, noes, an metastases) staging for gastric aenocarcinoma? A. Compute tomography scan is the most accurate means of etermining T an N staging B. The accuracy of enoscopic ultrasoun (EUS) is higher for N stage than T stage C. Fifteen lymph noes are require for an oncologic resection to appropriately stage the patient D. Magnetic resonance imaging (MRI) with gaolinium shoul be routinely performe E. T3 invaes the subserosa 16. Which of the following is associate with hypergastrinemia? A. Diabetes B. Hypothyroiism C. Hyperparathyroiism D. Chronic gastritis E. D-cell hyperplasia 17. A 46-year-ol male unergoes a istal gastrectomy for a tumor in the gastric antrum that was biopsy proven to be aenocarcinoma. The specimen is sent for pathology. Pathology reveals microscopic evience of tumor at the margins. Which of the following most accurately escribes this resection? A. D1 resection B. D resection C. R0 resection D. R1 resection E. R resection 18. Which of the following is consiere to be a risk factor for gastric cancer? A. Pernicious anemia B. Bloo group O C. Carbonate aciic soa D. Female sex E. Diabetes 19. Which of the following is true regaring the types of gastric ulcers? A. Type II ulcers are the most common B. Type IV ulcers occur near the gastroesophageal junction C. Type I ulcers usually have increase aci secretion D. Type III ulcers are associate with ecrease aci secretion E. Type I gastric ulcers are prepyloric

CHAPtEr 8 Alimentary Tract—Stomach

95

20. Which of the following is true regaring gastrointestinal stromal tumor (GIST)? A. The extent of the tumor is best etermine preoperatively by enoscopy B. They arise from smooth muscle cells C. Malignant potential is reaily etermine by histologic features D. They can be manage by laparoscopic wege resection E. They rarely present with GI bleeing

26. Which of the following is true regaring ZES? A. Symptoms ecrease with fasting B. Ulcers are most often locate in the istal uoenum C. It is most commonly familial D. It is the most common functional neuroenocrine tumor E. Treatment with proton pump inhibitors (PPIs) can control symptoms in the majority of patients

21. Which of the following is true regaring postgastrectomy bile reux? A. It is more likely to occur after a Billroth I than a Billroth II reconstruction B. Symptoms usually correlate with the amount of bile entering the stomach C. In symptomatic patients, meical management is generally effective D. Creation of a Roux-en-Y gastrojejunostomy is an effective surgical option E. Most patients with bile reux into the stomach will evelop symptoms

27. A 70-year-ol man presents with an 8-hour history of acute abominal pain an a history of melena. On examination, the patient is febrile to 101°F, with a bloo pressure of 105/70 mmHg an a heart rate of 130 beats per minute an has iffuse abominal tenerness with reboun an guaring. The rectal examination is guaiac positive. Laboratory values are signicant for a white bloo cell count of 16,000 cells/μL an a hematocrit of 6%. CT emonstrates extravasation of oral contrast in the proximal uoenum. After resuscitation, management consists of: A. Closure of the perforation with omental patch plus an HSV B. Closure of the perforation an omental patch via the open approach C. Perform uoenotomy over perforation, oversew posterior ulcer, close uoenotomy, an place omental patch D. Vagotomy an antrectomy with oversewing of the posterior ulcer an omental patch E. Closure of the perforation an omental patch via laparoscopic approach

22. The best test for localization of a gastrinoma is: A. MRI B. CT C. Abominal ultrasoun D. Octreotie scan E. Selective angiography 23. The best test to conrm eraication of H. pylori after treatment is: A. H. pylori serology B. Urea breath test C. Rapi urease test D. Histologic biopsy E. Antral mucosal biopsy with culture 24. Which of the following is true regaring a highly selective vagotomy (HSV)? A. The anterior an posterior vagal trunks are ivie B. The nerve of Grassi is spare C. The anterior Latarjet nerve is ivie D. The crow’s feet to the antrum are spare E. The celiac branch is ivie 25. The most common metabolic isorer after gastric resection is a eciency of: A. Iron B. Vitamin B1 C. Folate D. Calcium E. Vitamin D

28. A 50-year-ol woman with a history of iabetes presents with symptoms of early satiety, nausea, vomiting, an epigastric pain. Upper enoscopy reveals a large mass of unigeste foo particles in the stomach that is partially obstructing the pylorus. Which of the following is true regaring this conition? A. Most patients require surgery B. It can be treate with oral aministration of cellulase C. Psychiatric treatment is critical in long-term management D. The patient likely has patchy areas of alopecia E. Peptic ulcer isease is a risk factor

96

PArt i Patient Care

29. A 70-year-ol man presents to the ED with suen onset of severe epigastric pain associate with retching but with little vomitus. His bloo pressure is 140/90 mmHg an his heart rate is 90beats per minute. Attempts by the ED physician to place a nasogastric tube are unsuccessful. An upright chest raiograph reveals a large gas bubble just above the left iaphragm. Which of the following is true regaring this conition? A. The stomach is likely twiste along the axis, transecting the lesser an greater curvature B. In chilren it is largely ue to a paraesophageal hernia C. It is associate with Bergman’s tria D. Percutaneous gastrostomy tube for enitive management is acceptable in select patients E. It is initially manage conservatively for the majority of patients 30. Which of the following escribes the association between Sister Mary Joseph noule an gastric cancer? A. A metastatic left axillary lymph noe B. A metastatic left supraclavicular lymph noe C. An ovarian mass from gastric metastasis D. Umbilical metastasis suggesting carcinomatosis E. An anterior noule palpable on rectal examination suggesting rop metastasis 31. A 68-year-ol woman presents with an upper GI hemorrhage. She has a history of ulcer isease an has recently complete a treatment for H. pylori. Upper enoscopy reveals brisk arterial bleeing from a uoenal ulcer locate on the posterior wall. Despite numerous attempts to control the bleeing enoscopically, the ulcer continues to blee. The patient has receive 4 units of bloo. Her hematocrit is 5%, her bloo pressure is 110/60 mmHg, an her heart rate is 10 beats per minute. Which of the following is the best management option? A. Duoenotomy, oversewing the ulcer, truncal vagotomy, an pyloroplasty B. Duoenotomy an oversewing the ulcer C. Truncal vagotomy an antrectomy with Billroth I reconstruction D. Truncal vagotomy an antrectomy with Billroth II reconstruction E. Highly selective vagotomy

32. A 4-year-ol alcoholic male with recurrent episoes of pancreatitis presents to the ED with one episoe of hematemesis in the morning. He oes not appear to have any active bleeing currently. CT scan emonstrates splenic artery thrombosis. Lipase an liver function tests are normal. EGD emonstrates isolate gastric varices that are not currently bleeing an one -cm ulcer at the angularis. Which of the following enoscopic features confers the lowest risk of rebleeing? A. Oozing ulcer B. Nonbleeing ulcer with overlying clot C. Nonbleeing visible vessel D. Visible ulcer base E. Flat pigmente spot 33. Which of the following is true regaring gastric polyps? A. Funic gastric polyps have the highest risk of harboring malignant cells B. Aenomatous gastric polyps are the most common type C. Hamartomatous polyps are associate with H.pylori infection D. Heterotopic polyps most commonly present with gastrointestinal bleeing E. Inammatory polyps o not have a risk of malignancy 34. Bleeing from a Dieulafoy gastric lesion is ue to: A. Antral vascular ectasia B. Abnormal gastric rugal fols C. Ingeste foreign material D. An abnormal submucosal vessel E. A premalignant lesion 35. The most sensitive an specic iagnostic test for gastrinoma is: A. Basal an stimulate gastric aci outputs B. Octreotie scan C. Fasting serum gastrin D. Calcium stimulation test E. Secretin stimulation test

CHAPtEr 8 Alimentary Tract—Stomach

97

Answers 1. A. Lymphoepithelial tissue on biopsy is virtually pathog-

nomonic for gastric MALT lymphoma, an inolent malignancy primarily thought to arise from chronic H. pylori infection (gram-negative spirochete) (C). Treatment with triple therapy or quaruple therapy antibiotics to eraicate H. pylori is the rst-line treatment for patients with early-stage 1 or  isease. Most cases take 1 year to achieve remission; however, although rare, it can take up to 3 years. Refractory cases, as well as stage 3 an 4 isease, require chemoraiation using CHOP (cyclophosphamie, oxorubicin, vincristine, an prenisone) (E). Surgical resection is reserve for cases complicate by perforation, bleeing, or obstruction (D). Tumor biology affects the response to antibiotics, namely the t(11;18) chromosomal translocation that has a 5 cm or >5 mitoses/50 hpf or gastric tumors >10 cm or >5 mitoses/50 hpf, or patients with tumor rupture. The most common averse effect of imatinib is eema. While small Tis or T1a gastric aenocarcinomas can be treate with enoscopic resection, complete resection of gastric GISTs typically requires at least a wege resection (E).

3. B. One of the more feare postoperative complications

following Roux-en-Y gastric bypass is acute afferent loop synrome with small bowel obstruction (SBO) of the biliopancreatic limb. A high inex of suspicion is neee to iagnose this complication, usually base on patient symptoms an evience of proximal SBO on imaging. Unlike most

ahesive SBOs, which can be ecompresse with vomiting, obstruction of the afferent loop via an internal hernia causes a close loop obstruction with a high risk of perforation with urgent surgical exploration inicate to relieve the obstruction. Eating smaller, more frequent meals is the rst-line treatment of early umping synrome postgastrectomy (A). Nasogastric tube ecompression with water-soluble contrast challenge is the conservative management for ahesive SBO an is not appropriate in this patient (C). Antibiotic therapy is a treatment for small intestine bacterial overgrowth (SIBO) but not for bowel obstruction (D). While upper enoscopy woul be useful in the iagnosis of reux gastritis or marginal ulcer, it woul be of low utility in this patient (E).

4. B. This patient with multiple uoenal ulcers likely has

Zollinger-Ellison synrome seconary to hypersecretion of gastrin from a gastrinoma. Gastrin is usually prouce by antral G cells an acts on parietal cells to prouce hyrochloric aci an chief cells to prouce pepsinogen, both cell types of which are most preominant in the stomach boy (A). The pancreas is the site of the secretion of many hormones incluing somatostatin from D cells, insulin from beta cells, an glucagon from alpha cells (C). The uoenum an jejunum are the sites of the secretion of cholecystokinin (CCK) from I cells an secretin from K cells (D, E). The uoenum is the most common site for gastrinomas (50%–88%), followe by the pancreas (5%).

5. A. The most common etiology of small bowel obstruction

in the Unite States is ahesions from previous abominal surgery. However, this oes not hol true for patients that have previously ha an LRYGB. In this proceure, a potential hernia site (Petersen space hernia) is create, increasing the risk for the evelopment of an internal hernia, which is the most common cause of SBO in this patient population with an incience of 1% to 5%. This potential space results from herniation of intestinal loops through a efect in the mesentery an between small bowel limbs, transverse mesocolon, an the retroperitoneum. Aitionally, when compare to its open counterpart, the laparoscopic approach further facilitates a Petersen hernia because of the ecrease frequency of postoperative ahesions, which seemingly have a physiologic role of preventing bowel mobility, an thus, internal herniation. Risk of SBO is signicantly higher with a retrocolic versus an antecolic approach. Roux compression ue to mesocolon scarring is the secon most common etiology for SBO in patients with LRYGB followe by ahesions (B, C). Kinking of the jejunojejunostomy an incarcerate abominal wall hernia occur less frequently (D, E). Reference: Champion JK, Williams M, Husain S, Johnson AR. Small-bowel obstruction after laparoscopic Roux-en-Y gastric bypass: etiology, iagnosis, an management. Arch Surg. 003;13(4):988–993.

6. A. Nonoperative management for perforate peptic

ulcer isease is gaining popularity an is now accepte as an appropriate rst-line management for poor surgical caniates (e.g., COPD using home oxygen) who are stable, have

98

PArt i Patient Care

no evience of peritonitis, an have no contrast extravasation. Conservative management is also more favorable if the uration of symptoms has laste more than 4 hours. By this time the perforation has typically been seale. Self-sealing of the perforation is achieve by either ahesion formation to the cauate lobe, the greater omentum, the gallblaer, or the falciform ligament. In one stuy, only 3 out of 109 patients manage nonoperatively evelope an intraabominal abscess (which can be manage with antibiotics an percutaneous rainage). This may speak to the intrinsic immune function of the omentum an the fact that the upper GI tract has a low bacterial loa. Eighty percent of nonoperative cases respon favorably, an morbiity is not signicantly increase. Patients eeme appropriate caniates for nonoperative management shoul be amitte, place NPO (nothing by mouth), an given IV ui resuscitation, IV antibiotics covering gram-negative an anaerobic organisms, an PPIs. Nasogastric tube insertion is critical to help ecompress the stomach an allow the perforation to heal. CT scan may be consiere for patients who fail to improve or those who eteriorate clinically. Surgery is the next step for patients who fail conservative management (B, C). Outpatient follow-up is not appropriate because nonoperative management shoul be performe in a monitore setting with frequent abominal exams an follow-up esophagogastrouoenoscopy (EGD) to ensure that the perforation has seale (D, E). References: Nusree R. Conservative management of perforate peptic ulcer. Thai J Surg. 005;6:5–8. Hanumanthappa MB, Gopinathan S, Guruprasa R. A nonoperative treatment of perforate peptic ulcer: a prospective stuy with 50 cases. J Clin Diagnostic Res. 01;41:4161.

7. D. Obesity has been linke to multiple comorbiities,

incluing hypertension an iabetes, an is on the rise. As such, many clinicians have turne to meical management an/or bariatric surgery to help ght this epiemic in cases where iet an exercise fail. Two FDA-approve meications to help treat obesity inclue sibutramine an orlistat. Sibutramine blocks the presynaptic uptake of serotonin, thereby potentiating its anorexic effects in the CNS. Orlistat inhibits pancreatic lipase, which ecreases ietary fat absorption an results in weight loss (B). A signicant complication limiting its use for most patients is severe atulence. Inications for weight loss surgery inclue BMI >35 with associate obesityrelate comorbiities (e.g., hypertension, iabetes) or BMI >40 (A). Aitionally, all patients will nee to emonstrate that they have successfully attempte an faile nonoperative weight loss management such as iet an exercise programs (E). Patients will also nee to be evaluate by a physiatrist an eeme suitable for the proceure. The four stanar approaches in the Unite States inclue laparoscopic gastric baning, sleeve gastrectomy, BPD, an RYGB. Laparoscopic gastric baning an sleeve gastrectomy are consiere restrictive proceures as they physically limit the intake of foo. BPD is consiere a malabsorptive proceure as it involves constructing an alimentary channel istally to the GI tract an thereby preventing the absorption of caloric intake. RYGB is consiere a combine approach an involves creating a small restricte gastric remnant (restrictive component) an a roux-limb from the stomach to the istal jejunum (malabsorptive component) (C). These

proceures result in up to 50% resolution of weight-relate comorbiities an up to 50% excess weight loss. Compare to BPD, RYGB has a lower 30-ay mortality an is slightly favore by surgeons as it is technically easier to perform. RYGB has a slightly higher mean excess weight loss at  years compare to sleeve gastrectomy, but sleeve gastrectomy has a higher perioperative leak rate. Both proceures are equally effective in eliminating type  iabetes mellitus. References: Duarte MIX e T, Bassitt DP, Azeveo OC e, Waisberg J, Yamaguchi N, Pinto Junior PE. Impact on quality of life, weight loss an comorbiities: a stuy comparing the biliopancreatic iversion with uoenal switch an the bane Roux-en-Y gastric bypass. Arq Gastroenterol. 014;51(4):30–37. Santry HP, Gillen DL, Lauerale DS. Trens in bariatric surgical proceures. JAMA. 005;94(15):1909–1917. O’Brien P. Surgical treatment of obesity. Endotext. 016;19:9–46. Zingmon DS, McGory ML, Ko CY. Hospitalization before an after gastric bypass surgery. JAMA. 005;94(15):1918–194.

8. C. Gastroparesis is ene as elaye gastric emptying

without a mechanical cause for obstruction. Although iabetes is the most common known cause of gastroparesis (9%), iiopathic gastroparesis occurs more frequently (36%). The most common symptoms are nausea, early satiety, an abominal bloating. Most patients o not have abominal pain. Although symptoms alone can be suggestive of this conition, it nees to be conrme by imaging (E). Gastric emptying scintigraphy (elaye gastric emptying stuy) is the gol stanar in iagnosing gastroparesis. This involves asking the patient to eat a small meal along with a raioactive tracer. The rate of emptying is measure 1, , 3, an 4 hours after the meal is ingeste. If more than 10% of the meal remains in the stomach after 4 hours, the stuy is consiere consistent with gastroparesis (A, B, D).

9. E. Gallstone formation occurs in 30% to 5% of patients

unergoing weight loss surgery, but only 7% to 15% are symptomatic. Among those 7% to 15% who o become symptomatic, acute cholecystitis is uncommon. Rapi weight loss is a known risk factor for cholelithiasis. In fact, excess weight loss greater than 5% is consiere the strongest preictor of postoperative cholecystectomy an occurs more commonly in patients who have ha a gastric bypass versus laparoscopic baning or sleeve gastrectomy (A). Several mechanisms have been shown to contribute to gallstone formation uring weight loss incluing increase secretion of calcium an mucin into bile, increase concentrations of arachionic aci erivatives, an bile stasis seconary to stringent ietary restrictions postoperatively (D). Prophylactic cholecystectomy at the time of weight loss surgery has been a point of ebate in the surgical community. Proponents argue that it helps prevent the morbiity of symptomatic biliary isease an avois the nee for treatments such as enoscopic retrograe cholangiopancreatography (ERCP), which can be particularly challenging in this patient population (e.g., RYGB). However, it has been shown in several large stuies that the rate of postoperative cholecystectomy remains uner 15%; therefore, the routine removal of the gallblaer uring weight loss surgery is not currently supporte by the American Society of Metabolic an Bariatric Surgery (B). In contrast, symptomatic patients may unergo concomitant cholecystectomy safely. Ursoiol after gastric bypass can signicantly ecrease the rate of gallstone

CHAPtEr 8 Alimentary Tract—Stomach formation, but because it has not been shown to be cost effective an lea to improve outcomes, it is not routinely recommene (C). References: D’Hont M, Sergeant G, Deylgat B, Devrient D, Van Rooy F, Vansteenkiste F. Prophylactic cholecystectomy, a manatory step in morbily obese patients unergoing laparoscopic Roux-en-Y gastric bypass? J Gastrointest Surg. 011;15(9):153–1536. Shiffman ML, Shamburek RD, Schwartz CC, Sugerman HJ, Kellum JM, Moore EW. Gallblaer mucin, arachionic aci, an bile lipis in patients who evelop gallstones uring weight reuction. Gastroenterology. 1993;105(4):100–108. Sugerman HJ, Brewer WH, Shiffman ML, et al. A multicenter, placebo-controlle, ranomize, ouble-blin, prospective trial of prophylactic ursoiol for the prevention of gallstone formation following gastric-bypass-inuce rapi weight loss. Am J Surg. 1995;169(1):91–96. Tucker ON, Fajnwaks P, Szomstein S, Rosenthal RJ. Is concomitant cholecystectomy necessary in obese patients unergoing laparoscopic gastric bypass surgery? Surg Endosc. 008;(11):450–454. Villegas L, Schneier B, Provost D, et al. Is routine cholecystectomy require uring laparoscopic gastric bypass? Obes Surg. 004;14():06–11.

10. B. The rate of obesity is rising in the Unite States an

an increasing number of patients are unergoing weight loss surgery. Gastric leak in the early postoperative perio may be an inication to go back to the operating room, so early recognition of this complication is important. The rst manifestations of a gastric leak are tachycaria an fever (A, C–E). This may also be accompanie by tachypnea, abominal pain, chest pain, oliguria, an/or hypotension. Reference: Bekehit M, Katri K, Nabil W. Earliest signs an management of leakage after bariatric surgeries: single institute experience. Alexandria J Med. 013;49(1):9–33.

11. A. Laparoscopic gastric baning involves placing an

inatable balloon aroun the proximal stomach at the angle of His. A properly place lap ban will have an approximately 45° upwar angle from the horizontal plane on a plain lm of the abomen. The proceure was very popular when it rst appeare but lost traction after subsequent stuies emonstrate that it was far inferior to gastric bypass. Aitionally, patients with laparoscopic bans were more likely to require revisions for complications associate with the gastric ban. One such complication is ban erosion (BE) into the stomach an/or ajacent organs. This may present as port site erythema (inammation tracking own the tube), fooling the clinician into thinking the patient may only have an overlying skin infection. In fact, most patients with BE presenting with port site erythema o not have a subfascial port infection. BE can occur many years after surgery, an one propose mechanism involves overtightening of the ban (e.g., after clinic visit). CT of the abomen shoul be performe in patients suspecte of having BE an, if foun, the port site shoul be completely eate an the patient shoul be scheule for laparoscopic removal of the ban. EGD may emonstrate BE if it has completely eroe into the gastric lumen but may miss partial BE (C). Incision an rainage are not inicate because there is no abscess (D). The patient shoul be monitore for the evelopment of a subsequent port site infection, but the rst step is to get a CT scan (B–E). References: Dilorenzo N, Lorenzo M, Furbetta F. Intragastric gastric ban migration an erosion: an analysis of multicenter experience on 177 patients. Surg Endosc. 013;7(4):1151–1157.

99

Naef M, Naef U, Mouton WG, Wagner HE. Outcome an complications after laparoscopic Sweish ajustable gastric baning: 5-year results of a prospective clinical trial. Obes Surg. 007;17():195–01. Stroh C, Hohmann U, Will U, etal. (008).

12. E. In the majority of patients with a perforate uo-

enal ulcer, simple closure of the ulcer with an omental (Graham) patch is all that is necessary (A). This is then followe by treatment of H. pylori. In aition, a Graham patch alone shoul be use if the patient is unstable, if there is extensive exuative peritonitis, or if the perforation is long staning (>4 hours). However, in the setting of a patient with a known ulcer iathesis who has either alreay been treate for H. pylori or is H. pylori negative, an ulcer surgery shoul be ae to the operation, provie the patient is a goo operative risk, is hemoynamically stable, an oes not have extensive peritonitis. The options are either to perform a highly selective vagotomy (HSV) or a vagotomy an pyloroplasty (B). An HSV is the preferre approach in the stable goo-risk patient, provie the surgeon is comfortable with the proceure. Pyloroplasty is typically performe along with a vagotomy because the wiene outlet from the stomach to the uoenum helps circumvent any unwante effects of the ecrease gastric peristalsis an overall change in gastric emptying patterns that occur following vagotomy. The entire proceure can be performe laparoscopically in select patients. Truncal vagotomy an antrectomy (C, D) is generally not recommene in the setting of perforation because of the high associate morbiity an mortality rates. References: Caiere GB, Bruyns J, Himpens J, Van Alphen P, Verturyen M. Laparoscopic highly selective vagotomy. Hepatogastroenterology. 1999;46(7):1500–1506. Joran PH Jr, Thornby J. Perforate pylorouoenal ulcers: longterm results with omental patch closure an parietal cell vagotomy. Ann Surg. 1995;1(5):486–488. Siu WT, Leong HT, Law BKB, et al. Laparoscopic repair for perforate peptic ulcer: a ranomize controlle trial. Ann Surg. 00;35(3):313–319.

13. E. Postvagotomy synromes inclue iarrhea, gas-

tric atony, an incomplete vagotomy (leaing to recurrent ulceration). Diarrhea follows truncal vagotomy an may be confuse with umping synrome. The iarrhea associate with vagotomy occurs more frequently an is not associate with the other cariovascular manifestations seen with umping synrome (C). The initial treatment is similar to that for umping synrome, with ietary moications such as frequent small meals with ecrease ui intake an an increase in ber. A propose mechanism of the iarrhea is an increase in stool bile salts. Oral cholestyramine is often helpful because it bins bile salts (B). Loperamie an coeine have also been shown to elay intestinal transit time an improve symptoms. In the very rare patient who oes not respon to meical management, reversal of a segment of jejunum is effective in slowing transit time an improving iarrhea (D). Octreotie is not effective for postvagotomy iarrhea an may make the situation worse by ecreasing pancreatic secretions an thus increasing steatorrhea (A). References: Duncombe V, Bolin T, Davis A. Double-blin trial of cholestyramine in post-vagotomy iarrhea. Gut. 1977;18(7):531–535. O’Brien JG, Thompson DG, Mcintyre A. Effect of coeine an loperamie on upper intestinal transit an absorption in normal subjects an patients with postvagotomy iarrhea. Gut. 1988;9(3):31–318.

100

PArt i Patient Care

14. D. More than 80% of gastrinomas are localize preoper-

atively. For those that cannot be localize, surgical exploration is still inicate because excision of the primary tumor leas to a ecrease rate of liver metastasis. When exploring, it is important to be aware that 80% of gastrinomas are foun within the gastrinoma (Passaro) triangle, an area ene by the junction of the cystic uct an common bile uct, the secon an thir portions of the uoenum, an the neck an boy of the pancreas. As many as 60% of gastrinomas are within the wall of the uoenum, primarily in the rst an secon portions an can be very small. Thus, the next maneuver woul be to perform a blin proximal uoenotomy to manually palpate the uoenal wall for tumors. Closing the abomen (A) woul be inappropriate. Blin istal pancreatectomy an splenectomy (these share bloo supply) (B) or istal uoenotomy (E) woul have very low yiels. A pancreaticouoenectomy (Whipple proceure) (C) woul not be inicate in this setting. It is potentially inicate for multiple uoenal or proximal pancreatic hea tumors that coul not be enucleate.

15. C. Achieving an aequate lymphaenectomy with a

≥15 lymph noe harvest uring an oncologic resection of gastric cancer is important in accurately staging the patient an reucing the noal false negative rate. Staging of gastric cancer involves epth of invasion (T1 invaes lamina propria; T, muscularis propria or subserosa; T3, serosa; T4, ajacent structures), noes, an istant metastasis (E). EUS is the best moality for assessing tumor epth of invasion an noal status. It is approximately 80% accurate in etermining whether the tumor is transmural (invaing serosa, T3) but only 50% accurate in assessing whether pathologically enlarge lymph noes are present (B). EUS seems to be more accurate with avance isease than early isease. CT scanning is the preferre metho for etermining istant metastases, but it is not as useful for T an N staging (A). The routine use of MRI an positron emission tomography scanning for staging of gastric cancer has not as yet been establishe (D). N1 isease inclues 1 to 6 regional noes; N, 7 to 15 regional noes; an N3, more than 15 regional noes. References: Puli SR, Batapati Krishna Rey J, Bechtol ML, Antillon MR, Ibah JA. How goo is enoscopic ultrasoun for TNM staging of gastric cancers? A meta-analysis an systematic review. World J Gastroenterol. 008;14(5):4011–4019. Willis S, Truong S, Gribnitz S, Fass J, Schumpelick V. Enoscopic ultrasonography in the preoperative staging of gastric cancer: accuracy an impact on surgical therapy. Surg Endosc. 000;14(10):951–954. Xi W, Zhao C, Ren G. Enoscopic ultrasonography in preoperative staging of gastric cancer: etermination of tumor invasion epth, noal involvement an surgical respectability. World J Gastroenterol. 003;9():54–57.

16. D. When consiering gastrinoma, it is important to

be aware of the ifferential iagnosis of an elevate gastrin level. Causes of hypergastrinemia with increase aci prouction inclue gastrinoma, G-cell hyperplasia (not D-cell) (E), retaine antrum after istal gastrectomy, renal failure, an gastric outlet obstruction. Hypergastrinemia with normal or low aci prouction inclues pernicious anemia, postvagotomy states, use of aci-suppressive meication, an chronic gastritis. Hypothyroiism is associate with a low gastrin level, whereas hyperthyroiism increases gastrin levels (B). Diabetes (A) an hyperparathyroiism (C) o not affect gastrin levels.

References: Seino Y, Matsukura S, Inoue Y, Kaowaki S, Mori K, Imura H. Hypogastrinemia in hypothyroiism. Am J Dig Dis. 1978;3():189–191. Korman MG, Laver MC, Hansky J. Hypergastrinemia in chronic renal failure. BMJ. 197;1(5794):09–10.

17. D. R0 resection is resection of all gross an microscopic

tumors (C). R1 inicates removal of all macroscopic isease but microscopic margins are positive for isease. An R resection inicates that gross resiual isease is left behin (E). A D1 resection (A) refers to removal of perigastric lymph noes; D (B) refers to the aitional resection of lymph noes along the name vessels aroun the stomach. A D3 resection is a D resection plus removal of para-aortic lymph noes.

18. A. Risk factors for gastric cancer inclue ietary factors

such as a large consumption of smoke meats, pickle foos, high nitrates, an high salt, whereas a iet high in fruits an vegetables may be protective (D, E). Other risk factors inclue smoking, low socioeconomic status, Black race, H. pylori infection, chronic atrophic gastritis, bloo type A, previous partial gastrectomy, achlorhyria, pernicious anemia, polyps (aenomatous an hyperplastic), male sex, an certain familial synromes such as hereitary nonpolyposis colorectal cancer, Li-Fraumeni synrome, familial aenomatous polyposis, an Peutz-Jeghers synrome (B, D). Peutz-Jeghers synrome is associate with a markely increase risk of cancer in the esophagus, stomach, small bowel, colon, pancreas, breast, lung, uterus, an ovary, with a cumulative 93% risk of cancer. Carbonate aciic soa has not been shown to increase the risk for cancer (C). Gastric cancer has been categorize by Lauren into intestinal an iffuse types base on histology. The intestinal type is thought to be more relate to environmental factors, is associate with chronic gastritis, an is well ifferentiate. The iffuse type is usually poorly ifferentiate an associate with signet rings an occurs in younger patients an in association with familial isorers an with type A bloo. The iffuse type has a worse prognosis. References: Bernt H, Wilner GP, Klein K. Regional an social ifferences in cancer incience of the igestive tract in the German Democratic Republic. Neoplasma. 1968;15(5):501–515. Giariello FM, Brensinger JD, Tersmette AC, et al. Very high risk of cancer in familial Peutz-Jeghers synrome. Gastroenterology. 000;119(6):1447–1453. Wyner EL, Kmet J, Dungal N, Segi M. An epiemiological investigation of gastric cancer. Cancer. 1963;16(11):1461–1496.

19. B. Gastric ulcers have been categorize into ve types.

The most common is the type I lesion (≈60%) (A), which is locate near the angularis incisura at the borer between the antrum an the funus, usually along the lesser curve. These patients usually have normal or ecrease aci secretion. Type II gastric ulcers are locate in the funus an are associate with a concomitant uoenal ulcer. Type III gastric ulcers are prepyloric. Both types II an III gastric ulcers are usually associate with increase gastric aci secretion. Type III ulcers are thought to behave like uoenal ulcers. Type IV gastric ulcers are locate near the gastroesophageal junction. Like type I ulcers, type IV gastric ulcers have normal or low aci prouction an are associate with impaire mucosal efense. Type V gastric ulcers are consiere a iffuse process an are associate with NSAID use.

CHAPtEr 8 Alimentary Tract—Stomach

20. D. GISTs were previously calle leiomyomas or leiomyo-

sarcomas because they were thought to arise from smooth muscle cells, but they in fact originate from mesenchymal components (from Cajal cells) (B). They stain positive for CD117 (c-kit). They are most commonly foun in the stomach an, although rare, they are the most common mesenchymal tumors of the intestinal tract. Because they are not epithelial tumors an grow in the wall of the stomach, they ten to be large at the time of presentation. They cause mucosal ulceration an frequently present with GI bleeing (E). Large tumors may also prouce symptoms of weight loss, abominal pain, an fullness an early satiety. An abominal mass may be palpable. An enoscopic biopsy specimen may be negative in as many as one-half of cases ue to sampling error because most of the tumor is submucosal (A). A CT scan provies a better assessment of the extent of the tumor. Determining whether a GIST is malignant is not straightforwar because there are no iscriminating cellular features (C). The malignant potential is etermine by mitotic activity (>5 mitoses/50 high power el) with 1 cm. Lymph noe issection is not necessary because tumors sprea hematogenously an lymph noe metastasis is extremely rare. Wege resection with 1 cm margins is aequate treatment in most cases. This can be performe laparoscopically. However, microscopically positive margins have not been emonstrate to affect survival. References: Dempsey DT. Stomach. In: Brunicari FC, Anersen DK, Billiar TR, etal., es. Schwartz’s principles of surgery. 8th e. New York: McGraw-Hill; 005:933–996. Mercer DW, Robinson EK. Stomach. In: Townsen CM, Jr, Beauchamp RD, Evers BM, Mattox KL, es. Sabiston textbook of surgery: the biological basis of moern surgical practice. 17th e. Philaelphia: W.B. Sauners; 004:165–13. Novitsky YW, Kercher KW, Sing RF, Henifor BT. Long-term outcomes of laparoscopic resection of gastric gastrointestinal stromal tumors. Ann Surg. 006;43(6):738–745. Sexton JA, Pierce RA, Halpin VJ, et al. Laparoscopic gastric resection for gastrointestinal stromal tumors. Surg Endosc. 008;(1):583–587. Malangoni MA, etal. Stomach. In: Cameron JL, e. Current surgical therapy. 8th e. Philaelphia: Mosby; 004:67–100.

21. D. Bile reux into the stomach can occur without previ-

ous surgery, but in most instances it follows ablation of the pylorus, such as after gastric resection or pyloroplasty. After such proceures, most patients will have bile in the stomach on enoscopic examination, along with some egree of gross or microscopic gastric inammation. However, only a small fraction of patients will have a signicant egree of symptoms such as nausea, epigastric pain, an bilious vomiting consistent with alkaline (bile) reux gastritis (B, E). Symptoms often evelop months or years after the inex operation. The ifferential iagnosis inclues afferent or efferent loop obstruction, gastric stasis, an small bowel obstruction. These other iagnoses can be rule out using a combination of abominal raiographs, upper enoscopy, an abominal CT scan. A hepatoiminoiacetic aci (HIDA) scan is particularly helpful for emonstrating bile reux. Bile reux an gastritis are more likely to occur after Billroth II reconstruction (A) than after Billroth I an least likely after vagotomy an pyloroplasty. Meical management of symptomatic patients is not particularly effective (C). The surgical proceure of choice is to convert the Billroth II into a Roux-en-Y gastrojejunostomy with a lengthene jejunal limb (at least 45 cm).

101

22. D. More than 90% of gastrinomas have receptors for

somatostatin. Octreotie scanning (somatostatin receptor scintigraphy) has been shown to be the most sensitive test for localization of gastrinomas. However, successful localization epens on size an location. Somatostatin receptor scintigraphy is poor for very small tumors (cm). Even when bleeing recurs after having been controlle enoscopically, enoscopic treatment can again be attempte with a high rate of success, thus avoiing surgery. The bleeing is usually from a posterior ulcer that has eroe into the gastrouoenal artery (remember anterior ulcers cause a free perforation an peritonitis, posterior ulcers penetrate an blee). Surgical management ecisions shoul be base on the hemoynamic stability of the patient, the patient’s overall meical conition, an whether the patient has a history of ulcer isease that has been treate for H. pylori. In the patient who is actively bleeing, the uoenum shoul be opene across the pylorus as is use in a pyloroplasty. The ulcer be shoul be oversewn with multiple gure-of-eight sutures. If the patient has a history of ulcers that have been treate for H. pylori an is stable in the operating room, an ulcer operation shoul be performe. The best option in this type of emergent setting is to perform a truncal vagotomy an to close the longituinal uoenotomy in a transverse fashion as with a pyloroplasty. If the patient is a high surgical risk an unstable, another option woul be to simply perform a smaller uoenotomy, oversew the ulcer, simply close the uoenotomy, an treat postoperatively for H. pylori (B). Although vagotomy an antrectomy are another option, they woul selom be use in the emergent setting because of the higher associate morbiity rate (D, E). An HSV (C) woul not aress the actively bleeing ulcer. References: Brullet E, Calvet X, Campo R, Rue M, Catot L, Donoso L. Factors preicting failure of enoscopic injection therapy in bleeing uoenal ulcer. Gastrointest Endosc. 1996;43():111–116. Lau JY, Sung JJ, Lam YH, et al. Enoscopic retreatment compare with surgery in patients with recurrent bleeing after initial enoscopic control of bleeing ulcers. N Engl J Med. 1999;340(10):751–756.

32. D. Recurrent episoes of acute pancreatitis preispose

patients to eveloping splenic vein thrombosis, which can result in isolate gastric varices. Historically, patients were offere a splenectomy as a prophylactic measure to prevent severe upper GI bleeing. However, with improve imaging we are better able to ientify splenic vein thrombosis, an we now know that only 4% of patients will have clinically signicant gastric variceal bleeing, so routine splenectomy has fallen out of favor. This patient also has a concomitant ulcer, which coul have been contributing to hematemesis. The Forrest classication graes peptic ulcers base on enoscopic features an allows the clinician to etermine risk of rebleeing. The risk ecreases in the following orer: active spurting bleeing (17%–100%), active oozing bleeing (17%–100%), nonbleeing visible vessel (0%–81%), aherent clot (14%–36%), at pigment spot (0%–13%), an clean visible ulcer base (0%–10%) (A–C, E). Although patients with high-risk peptic ulcers (active bleeing/oozing, nonbleeing visible vessel) may benet from a secon-look enoscopy, current guielines recommen against routine secon-look enoscopy. References: Forrest JH, Finlayson NDC, Shearman DJC. Enoscopy in gastrointestinal bleeing. Lancet. 1974;304(7877):394–397. Heier TR, Azeem S, Galanko JA, Behrns KE. The natural history of pancreatitis-inuce splenic vein thrombosis. Ann Surg. 004;39(6):876–88. Laine L, Jensen DM. Management of patients with ulcer bleeing. Am J Gastroenterol. 01;107(3):345–360.

104

PArt i Patient Care

33. E. Hyperplastic polyps are by far the most common

gastric polyps (70%–90%) (B). Other types inclue aenomatous, hamartomatous, inammatory (pseuopolyps), funic glan, an heterotopic. Hyperplastic polyps are seen in association with chronic atrophic gastritis, which is ue to H. pylori infection (C). Hyperplastic polyps are further classie into polypoi foveolar hyperplasia an typical hyperplastic polyps. Polypoi foveolar hyperplasia oes not seem to have malignant potential, whereas the typical hyperplastic polyp has an approximately % chance of eveloping malignancy. Aenomatous polyps have the highest risk of malignancy (10%–0%), an the risk of malignancy seems to be relate to size an histology (greater risk for villous than tubular) (A). Funic gastric polyps are associate with long-term PPI use, an the risk of cancer is negligible. Aitionally, hamartomatous, inammatory, an heterotopic polyps o not seem to have a risk of malignancy. Heterotopic polyps are usually the result of ectopic pancreatic tissue an are typically benign lesions without clinical signicance (D). However, large heterotopic polyps can lea to obstruction an intussusception. Treatment for most polyps is simply enoscopic polypectomy. Aitional surgical resection is recommene for polyps that are sessile an larger than  cm, those with areas of invasive tumor, an those that cause symptoms (bleeing or pain). References: Orlowska J, Jarosz D, Pachlewski J, Butruk E. Malignant transformation of benign epithelial gastric polyps. Am J Gastroenterol. 1995;90(1):15–159. Jalving M, Koornstra JJ, Wesseling J, Boezen HM, DE Jong S, Kleibeuker JH. Increase risk of funic glan polyps uring long-term proton pump inhibitor therapy. Aliment Pharmacol Ther. 006;4(9):1341–1348.

34. D. A Dieulafoy lesion is a congenital malformation in

the stomach (typically on the lesser curvature) characterize by a submucosal artery that is abnormally large an tortuous. As a result of its relatively supercial location, it may eroe through the mucosa an become expose to gastric secretions, leaing to massive upper GI hemorrhage. On enoscopy, the mucosa of the stomach appears normal, an the only ning is a pinpoint area of mucosal efect with brisk arterial bleeing. The lesion may easily be misse if the bleeing is not active. Dieulafoy lesion is not premalignant (E) an is not associate with the ingestion of foreign material (C). Treatment is enoscopic, via electrocautery, heater probe, or injection with a sclerosing agent. Surgery, which consists of a wege resection, is reserve for the rare patient

who is not controlle enoscopically. Antral vascular ectasia (A) is seen in a conition known as watermelon stomach an can lea to signicant acute or chronic GI bloo loss. Dilate mucosal bloo vessels containing thrombus, mucosal bromuscular ysplasia, an hyalinization are prominent features. It erives its name from the mucosal vessels that create parallel lines in the mucosal fols (B). The stomach is typically not enlarge. It is seen preominantly in elerly women with autoimmune isease or elerly males with cirrhosis. Reference: Selinger CP, Ang YS. Gastric antral vascular ectasia (GAVE): an upate on clinical presentation, pathophysiology, an treatment. Digestion. 008;77():131–137.

35. E. The most sensitive an specic test for gastrinoma

(ZES) is the secretin stimulation test. An IV bolus of secretin is aministere, an gastrin levels are checke before an after injection. An increase in serum gastrin of 10 pg/ mL or greater has the highest sensitivity an specicity for gastrinoma. There are numerous other causes of hypergastrinemia. They can be ivie into those associate with an increase aci prouction an those with a ecrease aci prouction (A). In the latter situation, the hypergastrinemia is reactive ue to hypo- or achlorhyria. In aition to ZES, G-cell hyperplasia, gastric outlet obstruction, an retaine antrum after Billroth II reconstruction are associate with increase aci prouction. Reactive hypergastrinemia is seen with atrophic gastritis, pernicious anemia, an gastric cancer; in patients receiving H-receptor antagonists an PPIs; an after vagotomy. Hypergastrinemia is also seen in chronic renal failure ue to ecrease catabolism. Given this broa ifferential, fasting serum gastrin levels (C) are not sufciently specic to establish the iagnosis of ZES in the majority of patients unless gastrin levels are extremely high (>1000pg/mL). The secretin stimulation test has higher sensitivity an specicity than the calcium stimulation test (D). The calcium stimulation test is use if the secretin test result is negative an there is a high suspicion for ZES in the presence of hypergastrinemia. Once the iagnosis of ZES is establishe, a nuclear octreotie scan (B) seems to be the most sensitive test to localize the tumor. Reference: Berna M, Hoffmann K, Long S. Serum gastrin in Zollinger-Ellison synrome: II. Prospective stuy of gastrin provocative testing in 93 patients from the National Institutes of Health an comparison with 537 cases from the literature: evaluation of iagnostic criteria, proposal of new criteria, an correlations with clinical an tumoral features. Medicine. 006;(6):331–364.

Alimentary Tract— SmallBowel ZACHARY N. WEITZNER, FORMOSA CHEN, AND BEVERLEY A. PETRIE

9

ABSITE 99th Percentile High-Yields I. Duoenum A. Releases alkaline mucus from Brunner glans to neutralize gastric aci B. S cells release secretin (inhibits gastric aci, stimulates pancreatic bicarb, increases bile prouction in liver) an I cells release cholecystokinin (CCK) (inhibits gastric emptying, stimulates pancreatic enzyme prouction, increases bile, gallblaer contraction, satiety) C. Leafy appearing villi, absorb iron, eroe in celiac isease D. Bulb: 1st segment; ulcerations ue to Helicobacter pylori, Zollinger–Ellison synrome E. Descening: n segment; pancreatic an common bile uct empty F. Transverse: 3r segment; anterior to inferior vena cava, aorta, vertebral column G. Ascening: 4th segment; joins jejunum; ens at ligament of Treitz H. Bloo supply: celiac axis via gastrouoenal artery an SMA axis via pancreaticouoenal arteries II. Jejunum A. Does not have Brunner glans; thus marginal ulcers are more likely with Billroth II than with Billroth I B. Begins at ligament of Treitz C. Ientiable by long vasa recta from SMA an plicae circularis D. Dense villi for absorption of water, lipis, NaCl, glucose, amino acis III. Ileum A. Three meters long, short vasa recta from SMA, atter mucosa with Peyer patches lymphoi tissue B. Terminal ileum (TI) is chief site of absorption of B1, folate, an bile salts (conjugate in TI, unconjugate elsewhere in ileum) IV. Absorptive Defects A. Diagnostic tests 1. Suan re stain: etects presence of fat in stool . Schilling test: etects B1 eciency; oral labele B1 given with unlabele IM B1 to force urinary excretion; if raiolabele B1 in urine, able to absorb B1; if none, repeate with PO intrinsic factor; if labele B1 in urine → pernicious anemia; if not, primary small bowel (SB) malabsorption 3. D-xylose test: PO D-xylose given, if not excrete in urine, then primary SB absorption ecit B. Short Gut Synrome (SGS) 1. Dene by malabsorption ue to loss of intestinal length; iagnosis base on symptoms . Due to both loss of absorption an ecrease transit time 3. Risk of SGS with less than 180 cm of small bowel; lower risk with competent ileocecal valve a) Having 50% to 100% of functional large intestine is equivalent to 50 cm small bowel b) Ileum can aapt after resection by ilating, elongating, increasing size of villi to increase absorption, more important than jejunum in SGS 105

106

PArt i Patient Care

4. Complications: total parenteral nutrition (TPN) associate liver isease, cholelithiasis, calcium oxalate nephrolithiasis, coagulopathy, bacterial overgrowth, sepsis 5. Tx: prevention of sepsis, slow transit (loperamie, etc.), reuce GI secretion with octreotie an PPI, restrict oxalate in iet 6. Teuglutie: GLP- agonist to promote intestinal absorption an health 7. Surgical tx: slow transit by interposing colon, reversing intestinal segments, lengthening proceures (STEP), intestinal transplantation V. Structural Disease A. Duoenal iverticula 1. Most common small bowel iverticula; most are asymptomatic . Operate only if symptomatic with small bowel obstruction (SBO), biliary obstruction, concern for malignancy VI. Mechanical Disease A. Small bowel obstruction 1. Most commonly ue to ahesions in rst worl; hernia elsewhere . Early small bowel follow-through (SBFT) to etermine likelihoo of resolution with conservative management 3. Close-loop obstruction may have normal lactate 4. Avoi nitrous oxie uring anesthesia as gas thir spaces 5. Law of LaPlace: P=T/r 6. Malignancy is suspecte for SBO in a virgin abomen; always check for a hernia B. Stricture 1. Heineke-Mikulicz: stricture 15 cm or multiple consecutive strictures, bowel ivie at mipoint of stricture, stenotic areas overlappe, longituinal enterotomies along stricture, then anastomose in sie-to-sie isoperistaltic fashion VII. Small Bowel Neoplasms A. Carcinoi: most common SB neoplasm 1. Slow-growing tumors of Kulchitsky enterochromafn cells, usually nonfunctional, carcinoi synrome when functional . Most commonly locate in ileum followe by rectum 3. Strong esmoplastic reaction an mesenteric brosis surrouning can obstruct 4. Carcinoi synrome: secretes serotonin, histamine, VIP, braykinin, prostaglanins which cause ushing, iarrhea, bronchoconstriction, an right-sie valvular heart isease (most commonly tricuspi insufciency); tx=octreotie 5. Elevate chromogranin-A, pancreatic polypeptie, NSE, 5-HIAA 6. Resect if possible; can ebulk liver mets 7. Gastric carcinoi (types I an II have hypergastrinemia; type III oes not) a) Type I: autoimmune etiology associate with atrophic gastritis; associate with hypergastrinemia an in setting of chronic atrophic gastritis or pernicious anemia, rarely malignant, usually women, small an multicentric; if less than 1 cm without concerning risk factors, can remove enoscopically b) Type II: associate with the Zollinger-Ellison synrome (hypergastrinemia) an MEN-1; grows slowly but more likely to metastasize to lymph noes an istant sites; somatostatin analogs may be initiate an result in tumor regression; nees gastric resection c) Type III: usually poorly ifferentiate neuroenocrine cell, normal gastrin level, high rate of cancer, an usually nees partial gastrectomy an lymph noe issection

CHAPtEr 9 Alimentary Tract—SmallBowel

107

8. Appeniceal carcinoi: a)  cm or at base of appenix: right hemicolectomy 9. Bronchial carcinoi: a) Carcinoi tumors may be iagnose by bronchoscopy, appearing as pink or purple friable enobronchial masses covere by intact epithelium b) Tx: Complete surgical resection with meiastinal lymph noe sampling or issection, regarless of the presence of noal involvement 10. Diagnosis a) Octreotie scan: best for localizing; inium 111-labele somatostatin scintigraphy b) Ga-68 DOTATATE PET/CT: better than octreotie scan for localizing NETs c) Chromogranin A level most sensitive for etection B. Aenocarcinoma 1. Secon most common primary SB tumor . Risk factors: smoking, EtOH, peptic ulcer isease, celiac isease, Crohn isease, FAP, HNPCC, PJS 3. Goal is R0 resection with 10 mesenteric lymph noes, may nee Whipple C. Lymphoma 1. From lymphoi tissues such as Peyer patches, thus more common in jejunum an ileum . B-cell NHL most common, better prognosis than T-cell lymphomas 3. Enteropathy-associate T-cell lymphoma has poor prognosis, associate with celiac 4. Tx is surgical resection with ajuvant CHOP or R-CHOP chemotherapy D. Metastases 1. Melanoma most common; colon, breast, lung, kiney also seen VIII. Fistulas an Ileostomies A. High output stula >500 cc/ay, unlikely to spontaneously resolve B. Check for FRIENDS: foreign boies, raiation, inammation/inammatory bowel isease, neoplasm, istal obstruction, sterois/sepsis C. High ileostomy output consiere >100 cc/ay an manage with antimotility agents, maintain hyration D. Loss of bicarb leas to uric aci nephrolithiasis in high output stulae an ostomies IX. Crohn Disease A. Abominal pain, iarrhea +/– hematochezia, ileitis, obstruction, perianal isease, lateral ssures B. Aphthous ulcers, anemia, vitamin eciencies, malnutrition, cholesterol gallstones, oxalate kiney stones, blin loop synrome, stulae, megaloblastic anemia (ecrease B1) C. Extraintestinal sx that resolve with resection: uveitis, erythema noosum D. Extraintestinal sx that o not resolve with resection: ankylosing sponylitis, pyoerma gangrenosum E. Tx: inuction therapy with sterois (sometimes iniximab), maintenance therapy with biologics an mesalamine/sulfasalazine F. Skip lesions, rare rectal involvement but common perianal involvement, fat creeping, eep transmural ulcers that are serpiginous, common stenotic lesions, noncaseating granulomas on histology G. Surgery to treat obstruction, maintain intestinal length; o not perform stulotomies

108

PArt i Patient Care

Fig. 9.1 Heineke-Mikulicz Stricturoplasty an Finney Stricturoplasty.

CHAPtEr 9 Alimentary Tract—SmallBowel

109

Questions 1. A 55-year-ol woman with a history of total abominal colectomy with en ileostomy for refractory Crohn isease presents with a bulge ajacent to her ileostomy. The bulge has been present for months an has always been easily reucible. However, she frequently evelops bloating, obstipation, an low ostomy output requiring manual reuction of the bulge. What is the best treatment option for this patient? A. Primary parastomal hernia repair B. Relocation of her ileostomy with mesh closure of the previous efect C. Parastomal hernia repair with mesh D. Ileal pouch-anal anastomosis E. Observation 2. A 6-year-ol female presents to clinic for her -week follow-up appointment after unergoing a low anterior resection with iverting loop ileostomy for rectal cancer. Her incisions are healing well. She states her ostomy has put out 1.9 L per ay an she has ouble her aily ui intake ue to increase thirst. Her serum creatinine remains normal an she has no electrolyte abnormalities. What is the best treatment option for this patient? A. Amission to the hospital for IV hyration B. Oral loperamie an close outpatient monitoring C. Revision to a more istal ostomy D. Methylnaltrexone an close outpatient monitoring E. Observation 3. A 55-year-ol man with a history of heavy nonsteroial antiinammatory rugs (NSAID) use is amitte with a perforate antral ulcer. He unergoes a Billroth II reconstruction. On postoperative ay 4, he evelops acute abominal pain an hemoynamic instability. Which of the following complications is most likely causing this presentation? A. Anastomotic leak of gastrojejunostomy B. Efferent limb synrome C. Duoenal stump blowout D. Marginal ulcer E. Internal hernia

4. Two weeks after an open aortic aneurysm repair, the patient presents with marke abominal istention without nausea or vomiting. There is no tenerness on abominal examination. Plain lms are unremarkable. CT scan reveals a large amount of ascites but is otherwise unremarkable. Paracentesis reveals turbi ui that is culture negative. Flui analysis reveals a triglycerie level of 400 mg/L. The white bloo cell count is 600 cells/μL with a preominance of lymphocytes. Which of the following is true about this conition? A. The patient shoul be place on total parental nutrition (TPN) an NPO B. Octreotie is not useful C. The patient shoul immeiately be reexplore D. Interventional raiology (IR) embolization is rst-line treatment E. Most patients respon to a high-protein, lowfat iet with meium-chain triglyceries 5. Which of the following is true regaring short bowel synrome in aults? A. The presence of an intact ileocecal valve reuces malabsorption B. It is ene as less than 300 cm of the resiual small bowel C. Resection of the ileum is better tolerate than resection of the jejunum D. The presence of an intact colon oes not alter the severity E. It is most commonly cause by multiple operations requiring small bowel resection 6. Which of the following is true regaring the management of short bowel synrome? A. Glutamine shoul be avoie B. Octreotie is the cornerstone of management C. Coeine is contrainicate D. Early enteral feeing is inicate E. Patients who require TPN after 6 months will require permanent TPN

110

PArt i Patient Care

7. A 6-year-ol boy has short bowel synrome cause by migut volvulus that evelope uring infancy an has since been epenent on TPN, which he has tolerate well. He has approximately 8 cm of small bowel remaining with an intact colon. The small bowel is markely ilate without evience of small bowel obstruction. Which of the following is the best option? A. Serial transverse enteroplasty proceure (STEP) B. Continue with TPN C. Small bowel transplantation D. Small bowel tapering proceure E. Tapering an lengthening proceure (Bianchi) 8. Which of the following is true regaring small bowel neoplasms? A. Aenocarcinoma is the most common type B. Small bowel lymphoma most commonly occurs in the uoenum C. The incience of primary small intestinal cancers is increasing D. Five-year survival is higher for aenocarcinoma compare with carcinoi tumors E. Small bowel lymphoma is primarily treate by chemotherapy 9. A 68-year-ol woman presents with an exacerbation of congestive heart failure an acute abominal pain. Physical examination of the abomen is signicant for mil iffuse abominal tenerness but no reboun or guaring. CT arteriography of the abomen emonstrates iffuse narrowing of the superior mesenteric artery (SMA) an its branches but no vascular occlusion, pneumatosis, free air, or portal venous gas. Which of the following is an appropriate management option? A. IV heparin rip B. Exploratory laparotomy C. Aggressive ui resuscitation D. Intraarterial papaverine E. Increase cariac output 10. Which of the following is true regaring carcinoi? A. The majority of carcinoi synrome is from appeniceal tumors that have metastasize B. The most common symptom of carcinoi synrome is iarrhea C. Chromogranin A will not be elevate in nonfunctioning tumors D. Patients are at an increase risk for glossitis E. Urinary 5-hyroxyinoleacetic aci (5-HIAA) is not sensitive for etecting metastatic carcinoi

11. Which of the following is the best test for prognosis an monitoring treatment response in carcinoi tumors? A. Platelet serotonin levels B. 4-hour urinary 5-HIAA test C. Serum chromogranin A levels D. Serum serotonin levels E. Neuron-specic enolase 12. Which of the following is true regaring small bowel obstruction? A. The most common worlwie etiology is ahesions from prior surgery B. It is more frequent with upper intestinal than lower intestinal surgery C. In a complete close-loop obstruction, serum lactate can be normal D. Partial obstruction symptoms typically improve within 4 hours with conservative management E. Abominal pain isproportionate to exam nings occurs early in the setting of obstruction 13. Which of the following is true regaring uoenal iverticula? A. They ten to occur on the antimesenteric sie of the bowel B. Most are ientie in young patients C. Treatment with enoscopic interventions is contrainicate D. Malabsorption ue to bacterial overgrowth within the iverticula manates surgery E. When iscovere incientally at surgery, they shoul be left alone 14. Which of the following is the most common cause of obscure GI bleeing in aults? A. Small intestine angioysplasia B. Meckel iverticulum C. Crohn isease D. Infectious enteritis E. Vasculitis 15. Which of the following is true regaring GISTs of the small bowel? A. Most patients are symptomatic with GI bleeing B. They stain positive for CD134 C. Patients eeme caniates for chemotherapy shoul receive it for 1 year D. A patient with a 6-cm tumor shoul receive ajuvant chemotherapy E. Malignancy is primarily etermine by evience of local invasion

CHAPtEr 9 Alimentary Tract—SmallBowel

16. A hernia sac containing a Meckel iverticulum is known as: A. Petit hernia B. Littre hernia C. Spigelian hernia D. Richter hernia E. Grynfeltt hernia 17. Superior mesenteric artery (SMA) (Wilkie) synrome: A. Involves the secon portion of the uoenum B. Causes venous outow obstruction from the left kiney C. Is best iagnose with arteriography D. Shoul initially be manage with a high caloric intake iet E. Is best manage by gastrojejunostomy 18. A 45-year-ol woman with a history of laparotomy an 5000 cGy of abominal an pelvic irraiation for ovarian cancer 10 years ago presents with symptoms an signs of an acute bowel obstruction. CT scan shows a complete small bowel obstruction at the level of the mi jejunum with no evience of any masses. Which of the following is true about this conition? A. If a stricture is present, it is best manage by strictureplasty B. Sterois shoul be aministere C. Acute raiation enteritis is ue to an obliterative arteritis D. The risk of this complication increases in the setting of iabetes E. The egree of raiation amage is not affecte by whether the patient receive chemotherapy 19. A 75-year-ol male with a history of chronic obstructive pulmonary isease (COPD) presents to the ED with a 1-ay history of abominal istention an nausea. He enies abominal pain. Abominal examination is benign. Laboratory values are normal. CT scan emonstrates free air uner the iaphragm an thin-walle, airlle cysts within the bowel wall. Which of the following is true regaring this conition? A. Laparotomy is inicate B. The primary form occurs more commonly than the seconary form C. It is unlikely to be relate to the patient’s COPD D. It is most commonly seen in the ileum E. It is associate with steroi use

111

20. Which of the following is true regaring PeutzJeghers synrome? A. Patients shoul begin breast an cervical cancer screening at age 5 B. It is autosomal recessive C. Small bowel obstruction is uncommon D. Prophylactic colectomy is recommene to most patients starting at age 0 E. These patients are not at increase risk for small bowel cancer 21. Which of the following is correct with regars to Crohn isease? A. Mesenteric fat wrapping is consiere pathognomonic B. Symptoms of ankylosing sponylitis improve with resection of isease bowel C. The majority of patients with an initial presentation of terminal ileitis progress to Crohn isease on long-term follow-up D. Exaggerate skin injury after minor trauma (pathergy) is a commonly associate conition E. Pyoerma gangrenosum is commonly foun on the initial presentation of Crohn isease 22. Which intestinal cells have been implicate in the formation of gastrointestinal stromal tumors (GISTs)? A. Goblet cells B. Interstitial cells of Cajal C. Enteroenocrine cells D. Paneth cells E. Absorptive enterocytes 23. A 46-year-ol woman is about to unergo hepatic resection for a metastatic carcinoi tumor. During anesthesia inuction, her bloo pressure ecreases to 80 mmHg systolic an her heart rate increases to 110 beats per minute. Her entire boy appears ushe. Her temperature is normal, as is en-tial CO. Management consists of: A. Corticosterois B. Antihistamine C. Octreotie D. Abort operation E. Dantrolene

112

PArt i Patient Care

24. A 70-year-ol woman presents with vague abominal pain, iarrhea, steatorrhea, an anemia with an elevate mean corpuscular volume. Her meical an surgical history is unremarkable. A CT scan of the abomen an pelvis is negative. An upper GI series an small bowel follow-through are signicant only for a large jejunal iverticulum. Which of the following is true regaring this patient? A. It is typically cause by an autoimmune etiology B. A long-chain triglycerie iet may be helpful C. The iverticulum shoul be resecte D. Broa-spectrum antibiotics are inicate only if the patient presents with a fever an leukocytosis E. Vitamin B1 is inicate 25. A 57-year-ol male with no past surgical history presents with  ays of abominal pain, nausea, an vomiting. On exam he is istene an tympanic an is milly tener to palpation without reboun or guaring. He has no groin hernias. Compute tomography (CT) scan emonstrates multiple ilate loops of small bowel with a transition point in the istal small bowel, with some ajacent mesenteric fat straning. He has a mil leukocytosis. His last bowel movement was 1 ay ago. He has not passe atus for over a ay. He has not ha any similar symptoms previously. A nasogastric (NG) tube is place, intravenous (IV) uis are aministere, an the patient is place NPO (nothing by mouth). Which of the following is the best next step in management? A. A 4-hour trial of NG tube suction, then exploratory laparotomy if high output continues B. Exploratory laparotomy C. Water-soluble oral contrast challenge D. Water-soluble oral an rectal contrast challenge E. Diagnostic laparoscopy 26. Which of the following has been shown to be the most efcacious means of reucing postoperative ileus in patients unergoing bowel resection? A. Early ambulation B. Gum chewing C. Alvimopan D. Ketorolac combine with reuction in opioi use E. Nasogastric intubation

27. Which of the following is true regaring Crohn isease? A. It is more common in iniviuals of high socioeconomic status B. The most common inication for surgery is perforation C. It has a unimoal istribution D. It is more prevalent in females E. The most common initial presentation is an acute onset of abominal pain an iarrhea 28. The earliest lesion characteristic of Crohn isease is: A. Aphthous ulcer B. Caseating granuloma C. Noncaseating granuloma D. Cobblestone mucosa E. Serosal thickening 29. Which of the following is the best therapeutic option for mil active Crohn isease? A. Sulfasalazine B. Prenisone C. Buesonie D. Metroniazole E. Iniximab 30. Which of the following is true regaring the principles of operative management of the small bowel in Crohn isease? A. The optimal margin is at least 4 cm beyon grossly visible isease B. Frozen section shoul be obtaine to conrm the absence of active isease in at least one margin C. A 3-cm stricture segment of uoenum is best manage by resection D. A 10-cm stricture segment of jejunum can be manage by a Heineke-Mikulicz strictureplasty rather than by resection E. Strictures longer than 10 cm are best manage by resection

CHAPtEr 9 Alimentary Tract—SmallBowel

113

Answers 1. C. This patient presents with a chronic, reucible para-

stomal hernia. The enitive treatment for parastomal hernia repair is ostomy reversal; however, given this patient’s Crohn isease, ostomy takeown with ileoanal anastomosis or ileal pouch-anal anastomosis is contrainicate (D). Given the recurrent iscomfort an intermittent obstruction from her hernia, simple observation woul not be appropriate (E). Once taught as the surgical treatment of choice for parastomal hernia repair, ostomy relocation is no longer avise as it creates the potential for a new parastomal hernia an for hernias from prior ostomy site an laparotomy incisions. SAGES now recommens against ostomy relocation as the treatment of parastomal hernias (B). Primary parastomal hernia repair results in high tension with a high rate of hernia recurrence (A). The most effective treatment for parastomal hernias ajacent to ostomies that are unable to be reverse is mesh repairs (C). Stanar approaches inclue the Sugarbaker repair, in which an unerlay mesh is place on the efect with the stoma exiting the peritoneum at the sie of the mesh, an the keyhole approach, when the stoma is brought through a hole create in the mesh. Reference: Gillern S, Bleier JIS. Parastomal hernia repair an reinforcement: the role of biologic an synthetic materials. Clin Colon Rectal Surg. 014;7(4):16–171.

2. B. This patient presents with high ostomy output, usu-

ally ene as over 1. L per ay. High ostomy output can result in ehyration, loss of bicarbonate resulting in uric aci nephrolithiasis, an skin breakown. In the absence of ehyration or electrolyte abnormalities requiring inpatient amission, high ostomy output can be manage as an outpatient with close follow-up an titration of oral loperamie or Lomotil. When high ostomy output results in symptomatic ehyration or electrolyte isturbances, amission for inpatient hyration an monitoring is recommene (A). Ostomy revision is not typically require for high ostomy output an iverting loop ileostomies are usually istal enough to aequately absorb ui (C). Methylnaltrexone is an opioi antagonist use to treat opioi-inuce ileus an is not absorbe enterally (D). Observation is not appropriate for high ileostomy output (E).

3. C. The most serious complication after a Billroth II recon-

struction is a uoenal stump blowout. In a Billroth II, the pylorus an antrum are resecte, an the rst portion of the uoenum is oversewn. The biliary system rains into the uoenum, which then rejoins path of foo at a surgically constructe gastrojejunostomy. The inammation cause by a perforate ulcer results in inammation of the uoenum, which may leak after oversewing. Aitional causes of uoenal stump blowout inclue afferent limb obstruction an pancreatitis. Efferent limb synrome is the result of obstruction of the efferent limb istal to the gastrojejunostomy, resulting in bile reux an bilious emesis (B). Anastomotic leak of the gastrojejunostomy is possible but less common an catastrophic than uoenal stump blowout. A marginal ulcer is more likely to occur after a Billroth II reconstruction,

as compare to after a Billroth I, because the alkaline environment secrete by Brunner glans in the uoenum is not present in the jejunum. However, marginal ulcers, which are present on the jejunal sie of the anastomosis, present with abominal pain an possible upper GI bleeing (D). Any proceure that can result in ahesions has the potential to cause an internal hernia, though the highest chance is after a gastric bypass. The presentation of internal hernia is typically inolent with vague abominal pain, nausea, an vomiting an can be confuse for gastroenteritis or peptic ulcer isease. A high inex of suspicion is require, an the iagnosis often requires iagnostic laparoscopy for conrmation (E).

4. E. The patient has chylous ascites. In Western countries,

chylous ascites is most often ue to malignancy an cirrhosis, whereas infectious etiologies such as tuberculosis an lariasis preominate in Eastern an eveloping countries. Other causes inclue postlaparotomy inammatory isorers, trauma, raiation therapy, congenital lymphatic abnormalities, an pancreatitis. The operations most associate with this complication inclue aortic aneurysm repair, retroperitoneal lymph noe issection, inferior vena cava surgery, an liver transplantation, because these are operations in which retroperitoneal lymphatics are most likely to be interrupte. The mechanisms thought to lea to the evelopment of chylous ascites inclue exuation of chyle ue to obstruction of the cisterna chyli, irect leakage of chyle through a lymphoperitoneal stula, an exuation through ilate retroperitoneal vessels. The iagnosis of chylous ascites is best establishe by analysis of the ui. Chyle typically has a turbi appearance; however, it may be clear in fasting patients. Elevate triglycerie levels in the ui are consiere iagnostic, usually above 00 mg/L, although some use a threshol above 110 mg/L. In aition, the white bloo cell count is greater than 500, with a preominance of lymphocytes. The total protein level is between .5 an 7.0g/L. Cultures are negative, except for cases of tuberculosis, in which aenosine eaminase is also positive in the ui. The initial treatment of chylous ascites is to aminister a high-protein, low-fat iet with meium-chain triglyceries. This iet minimizes chyle prouction an ow. Meium-chain triglyceries are absorbe by the intestinal epithelium an are transporte to the liver through the portal vein an o not contribute to chylomicron formation. Conversely, long-chain triglyceries are converte to monoglyceries an free fatty acis, which are then transporte to the intestinal lymph vessels as chylomicrons. If this iet regimen fails, placing the patient NPO an on TPN with octreotie has been shown to be useful in patients with postoperative chylous ascites (A, B). If these meical approaches fail, then lymphoscintigraphy is often useful to localize lymph leaks an the site of obstruction. In some instances, IR can percutaneously inject glue to stop leak. Surgical reexploration with localization an closure of the lymphatic leak shoul be performe if leak persists beyon  weeks (C). Alternatively, in facilities with capabilities to perform

114

PArt i Patient Care

percutaneous lymphangiography, embolization of lymphatics may be attempte after faile ietary management (D). This latter complication may be ue to a high plasminogen concentration in the ascitic ui. Reference: Cárenas A, Chopra S. Chylous ascites. Am J Gastro-

Intestinal aaptation occurs over a perio of 1 to  years in most aults. Thus, the nal etermination of whether permanent TPN will be necessary is not etermine until after this perio (E).

enterol. 00;97(8):1896–1900.

7. A. Many patients with short bowel synrome can even-

5. A. The total length of small bowel is approximately 0

feet (each foot is equal to ≈30 cm), or approximately 600 cm (6 m). Short bowel synrome is ene as the presence of less than 180 cm of resiual an functional small bowel in ault patients (B). Thus, resection of less than 50% of the small intestine is generally well tolerate. In approximately 75% of cases, short bowel synrome results from one massive small bowel resection, as oppose to multiple sequential resections (E). In aults, the most common etiologies inclue acute mesenteric ischemia, malignancy, an Crohn isease. In peiatric patients, the most common etiologies inclue intestinal atresia, migut volvulus, an necrotizing enterocolitis. Resection of the jejunum is better tolerate than resection of the ileum because the absorption of bile salts an vitamin B1 occurs in the ileum (C). An intact ileocecal valve is thought to reuce malabsorption because it increases the resience time of the chyme in the small intestine. Likewise, an intact colon is important because it has a tremenous water-reabsorbing capacity an electrolytes an can also absorb fatty acis (D). With an intact colon, a shorter small bowel remnant is tolerate. The key to preventing short bowel synrome is avoiance of excessive small bowel resection. In Crohn patients, the use of strictureplasty as oppose to resection is recommene when possible. Also, one shoul resect only obviously ea bowel in acute mesenteric ischemia, leaving marginal bowel in situ an performing a secon-look proceure.

6. D. In the early phase of short bowel synrome, treat-

ment is irecte at slowing intestinal transit; reucing GI secretions; an maintaining nutrition, ui, an electrolyte balance. Transit time is slowe by the aministration of narcotics such as coeine an iphenoxylate, as well as with the antimotility agents Lomotil (iphenoxylate an atropine) an loperamie (C). Massive small bowel resection is associate with hypergastrinemia an aci hypersecretion. The increase aciity in the small bowel results in the inhibition of igestive enzymes. This can be controlle with H-receptor antagonists or proton pump inhibitors such as omeprazole an thus shoul be starte in all patients with short gut synrome (B). Nutrition is achieve with the institution of TPN. In aition, enteral feeing shoul be institute as soon as postoperative ileus has resolve. Enteral feeing assists in the process of intestinal aaptation an prevents the evelopment of villous atrophy associate with being NPO for a prolonge perio of time. Glutamine is helpful because it serves as a trophic factor for the gut an is consiere the principal fuel of the small intestine (A). Cholestyramine is also useful in controlling iarrhea ue to unabsorbe bile salts. The role of octreotie is controversial. Short-term use leas to a reuction in iarrhea, but long-term use may lea to steatorrhea, gallstones, an an inhibition of intestinal aaptation. More recently, a high-carbohyrate, low-fat enteral iet rich in glutamine combine with growth hormone aministration has shown promise in improving intestinal absorptive capacity.

tually iscontinue TPN, particularly if the bowel length is more than 10 cm in aults or more than 60 cm in chilren. Treatment options for short bowel synrome epen on the length of small bowel remaining, whether the remnant small bowel is markely ilate, whether the patient remains TPN epenent, an whether multiple complications of TPN have evelope such as catheter-relate infections, vena cava thrombosis, an liver amage (B). A short remnant (5 cm or >5 mitoses per 50 high-power el (E). The ajuvant treatment of GISTs inclues chemotherapy with imatinib (Gleevec), a tyrosine kinase inhibitor. In one stuy, imatinib controlle tumor growth in as many as 85% of avance GISTs. Currently, imatinib is recommene for unresectable, metastatic, or recurrent lesions. Ajuvant therapy shoul continue for a total of 3 years (C). Patients that harbor an exon 9 KIT mutation will require a higher ose of imatinib (800 mg aily versus 400 mg). The most useful inicators of survival an the risk of metastasis inclue the size of the tumor at presentation, the mitotic inex, location within the GI tract, an the absence of tumor rupture. References: Blay JY, Bonvalot S, Casali P, et al. Consensus meeting for the management of gastrointestinal stromal tumors. Report of the GIST Consensus Conference of 0-1 March 004, uner the auspices of ESMO. Ann Oncol. 005;16(4):566–578. Crosby JA, Catton CN, Davis A, et al. Malignant gastrointestinal stromal tumors of the small intestine: a review of 50 cases from a prospective atabase. Ann Surg Oncol. 001;8(1):50–59. Dematteo RP, Ballman KV, Antonescu CR, et al. Ajuvant imatinib mesylate after resection of localise, primary gastrointestinal stromal tumour: a ranomise, ouble-blin, placebo-controlle trial. Lancet. 009;373(9669):1097–1104. Joensuu H, Eriksson M, Sunby Hall K, et al. One vs three years of ajuvant imatinib for operable gastrointestinal stromal tumor: a ranomize trial: A ranomize trial. JAMA. 01;307(1):165–17.

16. B. A hernia sac containing a Meckel iverticulum is

calle a Littre hernia. Lumbar hernias can be either congenital or acquire an occur in the lumbar region of the posterior abominal wall. Hernias through the superior lumbar

117

triangle (Grynfeltt triangle) (E) are more common than those through the inferior lumbar triangle (the Petit triangle) (A). The Petit triangle is boune by the external oblique muscle, latissimus orsi muscle, an iliac crest. The Grynfeltt triangle is boune by the quaratus lumborum muscle, the 1th rib, an the internal oblique muscle. A spigelian hernia occurs through the spigelian fascia, which is compose of the aponeurotic layer between the rectus muscle meially an the semilunar line laterally (C). Nearly all spigelian hernias occur in the spigelian belt locate below the umbilicus but above the epigastric vessels. The absence of posterior rectus fascia may contribute to an inherent weakness in this area. A Richter hernia occurs when only the antimesenteric borer of the bowel herniates through the fascial efect (D). It involves only a portion of the circumference of the bowel. As such, incarceration an strangulation may occur in the absence of any evience of bowel obstruction. Reference: Skanalakis PN, Zoras O, Skanalakis JE, Mirilas P. Spigelian hernia: surgical anatomy, embryology, an technique of repair. Am Surg. 006;7(1):4–48.

17. D. The SMA leaves the aorta at a ownwar an acute

angle. SMA synrome or Wilkie synrome is a rare conition characterize by compression of the thir portion of the uoenum by the SMA as it passes over this portion of the uoenum (A). It occurs most often in the setting of profoun weight loss. Factors that preispose to the conition inclue supine immobilization, scoliosis, placement of a boy cast, an eating isorers. Symptoms inclue profoun nausea an vomiting, abominal istention, weight loss, an postpranial epigastric pain, which varies from intermittent to constant, epening on the severity of the uoenal obstruction. Weight loss usually occurs before the onset of symptoms. It is believe to occur more commonly in women, likely seconary to the increase prevalence of anorexia. However, a recent stuy of SMA synrome among intellectually isable chilren showe that it preominantly affects males. The iagnosis can be mae by a CT scan, which emonstrates a ecrease aortomesenteric angle an a ecrease istance between the aorta an the SMA, as well as evience of obstruction of the uoenum (C). It can also be iagnose by a barium upper GI series or hypotonic uoenography, emonstrating abrupt or near-total cessation of ow of barium from the uoenum to the jejunum. Conservative measures that are trie initially are primarily focuse on weight gain to increase the mesenteric root fat pa. The operative treatment is uoenojejunostomy (E). Nutcracker synrome is characterize by compression of the left renal vein by the aorta, superior to the uoenum (B). References: Ason DE, Mitchell JE, Trenkner SW. The superior mesenteric artery synrome an acute gastric ilatation in eating isorers: a report of two cases an a review of the literature. Int J Eat Disord. 1997;1():103–114. Agrawal GA, Johnson PT, Fishman EK. Multietector row CT of superior mesenteric artery synrome. J Clin Gastroenterol. 007;41(1):6–65. Geskey JM, Erman HJ, Bramley HP, Williams RJ, Shaffer ML. Superior mesenteric artery synrome in intellectually isable chilren. Pediatr Emerg Care. 01;8(4):351–353.

18. D. The small-intestinal epithelium is acutely susceptible

to raiation injury because raiation has its greatest impact on rapily proliferating cells. Raiation-inuce injury to the

118

PArt i Patient Care

bowel can present with acute or chronic enteritis. Approximately 75% of patients unergoing raiation therapy for abominal an pelvic cancers evelop acute raiation enteritis transiently. Chronic raiation enteritis results from an obliterative arteritis in the submucosal vessels, while acute raiation enteritis is a transient perio of nausea, vomiting, iarrhea, an abominal pain that occurs aroun 3 weeks after treatment (C). This leas to progressive submucosal brosis an stricture formation. Not infrequently, patients with raiation-inuce injury may evelop a small bowel obstruction. The risk of raiation enteritis correlates with the amount of raiation receive. It is uncommon if the total raiation ose is less than 4000 cGy. The risk of raiation amage increases if the patient receive chemotherapy or has unerlying vascular isease or iabetes (E). Early symptoms of raiation amage inclue iarrhea, abominal pain, an malabsorption an are usually self-limite. The treatment of acute raiation enteritis inclues antispasmoic agents, analgesic agents, an antiiarrheal agents. Sterois are not use in the management of raiation enteritis (B). Only a small group of patients with chronic raiation enteritis will require surgery for either SBO from stricture formation or stulas. Unlike Crohn isease, for which strictureplasty is use, it is not recommene for raiation enteritis because there is a high risk of tissue breakown (A). The extent of macroscopic raiation injury is ifcult to etermine on gross inspection. Extensive lysis of ahesions shoul be avoie because this creates a risk of an enterotomy an subsequent stula formation as well. The two main surgical proceures are primary resection with reanastomosis or bypass. If the source of obstruction is a loop of bowel stuck in the pelvis, it is best treate with a bypass rather than an attempt to take own the ahesions an risk injury. References: Gallan RB, Spencer J. Natural history an surgical management of raiation enteritis. Br J Surg. 1987;74(8):74–747. Tavakkoli A, Ashley SW, Zinner MJ. Small Intestine. In: Brunicari F, Anersen DK, Billiar TR, Dunn DL, Hunter JG, Matthews JB, Pollock RE, es. Schwartz’s principles of surgery. 10th e. New York: McGraw-Hill Eucation; 015:1146–1151.

19. E. Pneumatosis intestinalis is a raiographic ning

an not a isease unto itself. Its iscovery on imaging is vexing because it can be a completely benign ning, or it can be associate with life-threatening bowel ischemia. It has been ivie into primary an seconary pneumatosis intestinalis. The primary form is less common an is terme pneumatosis cystoides intestinalis (B). It consists of thin-walle, air-lle cysts within the bowel wall, usually in the colon, but it can occur anywhere in the GI tract (D). It is an inciental ning, an the iagnosis is reaily mae on plain raiograph or CT scan. The gas can appear as linear, curvilinear, bubbly, or cystic. There is no specic treatment (A). Seconary pneumatosis intestinalis occurs when there is an unerlying isease process. The exact cause of pneumatosis intestinalis is unclear, but there seem to be several pathways that allow gas to enter the bowel wall. Immunoecient an inammatory bowel states lea to a loss of mucosal barrier function that may permit air to enter the bowel wall. Bowel obstruction leas to gas formation uner pressure. Alterations in bacteria ora, with invasion of the bowel wall, likewise lea to gas formation. In aults, seconary pneumatosis intestinalis is most often associate with COPD (C). It is also

seen with collagen vascular isease, celiac sprue, Crohn isease, use of sterois, an in immunoecient states. More ominously, it is also associate with ischemic bowel. Thus, it is important to recognize that not all cases of pneumatosis are benign. In neonates, it is most commonly associate with necrotizing enterocolitis. The ning of pneumatosis intestinalis in association with necrotizing enterocolitis oes not manate surgical exploration. It is also seen with pyloric stenosis, Hirschsprung isease, an other causes of bowel obstruction. Pneumoperitoneum can rarely be the result of a benign case of pneumatosis intestinalis because the air-lle cysts are thin-walle an can burst. References: Mularski RA, Ciccolo ML, Rappaport WD. Nonsurgical causes of pneumoperitoneum. West J Med. 1999;170(1):41–46. Peter S, Abbas M, Kelly K. The spectrum of pneumatosis intestinalis. Arch Surg. 003;138(1):68–75. Hsueh KC, Tsou SS, Tan KT. Pneumatosis intestinalis an pneumoperitoneum on compute tomography: beware of non-therapeutic laparotomy. World J Gastrointest Surg. 011;3(6):86–88.

20. A. Peutz-Jeghers synrome features mucocutaneous

melanotic pigmentation an hamartomatous polyps (not aenomatous) of the small intestine. It is an autosomal ominant inherite synrome (B). The skin lesions are foun in the circumoral region of the face, buccal mucosa, forearms, palms, soles, igits, an perianal area, whereas the hamartomas are usually in the jejunum an ileum. The most common symptom is recurrent colicky abominal pain (C). Symptoms of a bowel obstruction evelop in as many as 50% of patients, which is usually ue to intussusception or obstruction by the polyp itself. Hemorrhage or chronic anemia can also occur as a result of the polyps. The polyps can also unergo aenomatous change. Patients are at signicantly increase risk of eveloping cancer in the GI tract (esophagus, stomach, small intestine, colon, an pancreas) an extraintestinal cancer (testis, breast, uterus, ovary). Female patients shoul begin breast an cervical cancer screening starting at age 5. Over the long term, cancer evelops in as many as 90% of patients. Compare with the general population, they are at 500 times increase risk of the evelopment of small intestine cancer (E). Operative intervention is only inicate in the presence of symptoms (D). References: Boarman LA, Pittelkow MR, Couch FJ, et al. Association of Peutz-Jeghers-like mucocutaneous pigmentation with breast an gynecologic carcinomas in women. Medicine (Baltimore). 000;79(5):93–98. Giariello FM, Brensinger JD, Tersmette AC, et al. Very high risk of cancer in familial Peutz-Jeghers synrome. Gastroenterology. 000;119(6):1447–1453. Wu YK, Tsai CH, Yang JC. Gastrouoenal intussusception ue to Peutz-Jeghers synrome: a case report. Hepatogastroenterology. 1994;41():134–136. van Lier MGF, Wagner A, Mathus-Vliegen EMH, Kuipers EJ, Steyerberg EW, van Leeram ME. High cancer risk in Peutz-Jeghers synrome: a systematic review an surveillance recommenations. Am J Gastroenterol. 010;105(6):158–164.

21. A. The ning of “creeping fat” or mesenteric fat wrap-

ping is a gross feature of Crohn isease that is consiere pathognomonic. It inicates the encroachment of mesenteric fat onto the serosal surface of the bowel. The presence of fat wrapping correlates well with the presence of unerlying acute an chronic inammation. A recent stuy suggests that

CHAPtEr 9 Alimentary Tract—SmallBowel aiponectin, an aipocyte-specic protein with antiinammatory properties foun in mesenteric aipose tissue, may play an important role in the inammation seen in Crohn isease. Terminal ileitis refers to any acute inammation of the istal ileum ajacent to the ileocecal valve an is therefore not pathognomonic. Terminal ileitis is associate with numerous infectious causes incluing Yersinia enterocolitica an pseuotuberculosis, Mycobacterium, cytomegalovirus (in acquire immunoeciency synrome), Salmonella, Campylobacter, an Shigella, among others. The ning of terminal ileitis oes not warrant bowel resection. Overall, a minority of patients (10% in one stuy) who present with terminal ileitis progress to Crohn isease on long-term follow-up (C). The majority of extraintestinal manifestations in inammatory bowel isease improve with bowel resection but ankylosing sponylitis an primary sclerosing cholangitis o not (B). Pyoerma gangrenosum is rarely the initial presentation of Crohn isease (E). These patients present with small papules often on the lower extremities that resemble a “cat’s paw” appearance an can progress to larger ulcerations with necrotic centers. Rarely, patients evelop pathergy, a conition in which minor trauma leas to the evelopment of large an ifcult-to-heal ulcers (D). Debriement of these lesions shoul be avoie because this worsens the lesion. Iniximab or another tumor necrosis factor-alpha inhibitor shoul be use. References: Menachem Y, Gotsman I. Clinical manifestations of pyoerma gangrenosum associate with inammatory bowel isease. Isr Med Assoc J. 004;6():88–90. Yamamoto K, Kiyohara T, Murayama Y, et al. Prouction of aiponectin, an anti-inammatory protein, in mesenteric aipose tissue in Crohn’s isease. Gut. 005;54(6):789–796. Hatemi I, Hatemi G, Celik AF, et al. Frequency of pathergy phenomenon an other features of Behçet’s synrome among patients with inammatory bowel isease. Clin Exp Rheumatol. 008;6(4 Suppl 50):S91–S95.

22. B. There are four main cell types in the small intestine:

absorptive enterocytes (E), which make up 95% of intestinal cells; goblet cells (A); Paneth cells (D); an enteroenocrine cells (C). Goblet cells secrete mucus. Paneth cells secrete several substances incluing lysozyme, tumor necrosis factor, an cryptiins, which assist in host mucosal efense. There are more than 10 istinct types of enteroenocrine cells that secrete various gut hormones. The interstitial Cajal cell is a specialize cell of mesoermal origin that seems to regulate peristalsis. It is referre to as an intestinal pacemaker cell. The cells normally express KIT, a tyrosine kinase receptor. These cells have been implicate as the cells of origin of GISTs. References: Miettinen M, Majii M, Lasota J. Pathology an iagnostic criteria of gastrointestinal stromal tumors (GISTs): a review. Eur J Cancer. 00;38:S39–S51. Sircar K, Hewlett BR, Huizinga JD, Chorneyko K, Berezin I, Riell RH. Interstitial cells of Cajal as precursors of gastrointestinal stromal tumors. Am J Surg Pathol. 1999;3(4):377–389.

23. C. The patient has a carcinoi crisis. This has been

escribe after anesthetic inuction as well as after other stressful situations such as biopsies or invasive proceures. Carcinoi crisis is characterize by hypotension, bronchospasms, ushing, an tachycaria. The primary treatment is IV octreotie aministere as a bolus of 50 to 100 μg. Even

119

more rarely, a carcinoi crisis can manifest with hypertension. Octreotie is effective for a hypertensive crisis as well. Ajunctive treatment with antihistamines may also be of benet ue to frequent histamine release from carcinoi tumors (B). If the above measures o not resolve the crisis, then aborting the proceure may be necessary (D). Dantrolene is the preferre choice of management for malignant hyperthermia (E). This iagnosis is supporte by an increase in en-tial CO. Corticosterois are not use in the management of carcinoi crisis (A). References: Bax NDS, Woos HF, Batchelor A, Jennings M. Octreotie therapy in carcinoi isease. Anticancer Drugs. 1996;7(Suppl 1):17–. Warner RR, Mani S, Profeta J, Grunstein E. Octreotie treatment of carcinoi hypertensive crisis. Mt Sinai J Med. 1994;61(4):349–355.

24. E. The patient has a blin loop synrome, which is ue

to bacterial overgrowth (A). Symptoms inclue iarrhea, steatorrhea, megaloblastic anemia, weight loss, abominal pain, an eciencies of fat-soluble vitamins. The megaloblastic anemia is ue to the utilization of vitamin B1 by the bacteria. The unerlying cause may be an intestinal abnormality such as a iverticulum, stula, an intestinal stricture, or it may follow a Billroth II proceure. In the patient presente, the large jejunal iverticulum is likely the etiology. The iagnosis can be conrme by various means. A barium stuy is useful to ene the anatomic abnormality. The d-xylose test involves ingesting xylose, which is metabolize by the bacteria. Excessive CO in the breath conrms the iagnosis. Cultures of the small intestine can be obtaine; however, passing an intestinal tube istal enough to obtain an aequate culture can be challenging. Another useful stuy is the Schilling test. Oral raiolabele vitamin B1 is aministere along with parenteral unlabele vitamin B1. The unlabele vitamin B1 saturates liver receptors. Thus, if the oral raiolabele vitamin B1 is properly absorbe an liver receptors are saturate, the raiolabele vitamin B1 will be excrete in high concentrations in the urine. With pernicious anemia an blin loop synrome, oral absorption will be low, an thus urinary excretion of raiolabele vitamin B1 will be low. When the test is repeate after the aition of intrinsic factor, vitamin B1 excretion will increase, whereas with blin loop synrome, vitamin B1 excretion will remain low. The initial treatment of blin loop synrome consists of broa-spectrum antibiotics incluing metroniazole with tetracycline as well as vitamin B1 supplementation given parenterally. This shoul be given to all patients presenting with blin loop synrome (D). Prokinetic agents o not seem to help. In aition, ietary moications such as a lactose-free iet are useful because patients with blin loop synrome often become lactose intolerant. Meium-chain triglycerie iets are more reaily absorbe than long-chain triglyceries because they o not require igestive enzymes (B). Resection of the iverticulum is not recommene initially (C). Surgery shoul be reserve for patients who fail repeate meical management attempts. References: Ross CB, Richars WO, Sharp KW, Bertram PD, Schaper PW. Diverticular isease of the jejunum an its complications. Am Surg. 1990;56(5):319–34. Woos K, Williams E, Melvin W, Sharp K. Acquire jejunoileal iverticulosis an its complications: a review of the literature. Am Surg. 008;74(9):849–854.

120

PArt i Patient Care

25. C. Surgical ogma has state that a small bowel obstruc-

tion (SBO), in the absence of prior surgery or visible external hernia, requires surgical intervention, as the ifferential invariably are all surgical iseases, such as internal hernia, appenicitis, intussusception, inammatory bowel isease, malignancy, or obstructe Meckel iverticulum. However, in a recent stuy, as many as 40% of SBO in patients without prior history of surgery resolve nonoperatively. Of those that require surgical intervention, the majority were foun to have ahesions espite no prior operations (A). Most centers have transitione to a water-soluble oral contrast challenge to help ecie which patients with ahesive SBOs will require surgical intervention. This is being performe even in patients without prior abominal surgery. This entails performing nasogastric ecompression for two hours followe by aministration of a water-soluble contrast either by mouth or via NG tube. This is followe by plain lms 8 hours later. Patients with plain lms emonstrating contrast in the colon after 8 hours are unlikely to require surgical intervention while those without contrast in the colon after 8 hours are more likely to fail nonoperative management (C). Rectal contrast is not typically use in the workup nor in management of ahesive SBO (D). Reference: Ng YYR, Ngu JCY, Wong ASY. Small bowel obstruction in the virgin abomen: time to challenge surgical ogma with evience: Small bowel obstruction in the virgin abomen. ANZ J Surg. 018;88(1–):91–94.

26. C. Postoperative ileus remains a major source of pro-

longe hospitalization in patients unergoing abominal surgery. The use of early ambulation, early postoperative feeing protocols, an routine nasogastric intubation have not been shown to be associate with earlier resolution of postoperative ileus (A, E). Reucing opioi use in combination with the use of nonsteroial antiinammatory rugs such as ketorolac has been shown to reuce the uration of ileus in most stuies. The mechanism may be a combination of the reuction in opiois an the antiinammatory properties of ketorolac. However, ketorolac has been associate with an increase risk of operative site an gastrointestinal (GI) bleeing as well as ui retention (D). Recently, ketorolac has been also shown to increase the risk of reamission an reinterventions after GI surgery. Another rug that has been investigate is erythromycin, which is useful for gastroparesis because it works by its agonistic effect on the motilin receptor. However, it oes not seem to be useful for ileus an shoul be avoie in cases of obstruction, as woul all promotility agents (B). Metoclopramie is a opaminergic antagonist with antiemetic an prokinetic properties, but it has also not been shown to be useful for ileus. Gum chewing has ha conicting results in the literature, but a recent ranomize controlle trial from New Zealan emonstrate a signicant reuction in postoperative ileus in patients with colorectal cancer unergoing bowel resection (7% versus 48%). The most efcacious agent, however, is alvimopan (Entereg), which has been emonstrate in ranomize stuies to improve postoperative ileus in patients unergoing bowel resection. Alvimopan is an opioi receptor antagonist. It bins μ-opioi receptors in the GI tract an selectively inhibits the opioi effects on GI function an motility while not affecting opioi analgesia. It is the rst US Foo an Drug Aministration–approve rug for postoperative ileus. It is

approve for short-term (maximum 15 oses over 5 ays) in-hospital use only. Patients on long-term narcotics (e.g., for chronic pain) shoul not use Alvimopan because this population has an increase risk of myocarial infarction. References: Luwig K, Enker WE, Delaney CP, et al. Gastrointestinal tract recovery in patients unergoing bowel resection: results of a ranomize trial of alvimopan an placebo with a stanarize accelerate postoperative care pathway. Arch Surg. 008;143(11):1098–1105. Wolff BG, Weese JL, Luwig KA, et al. Postoperative ileus-relate morbiity prole in patients treate with alvimopan after bowel resection. J Am Coll Surg. 007;04(4):609–616. Su’a BU, Hill AG. Perioperative use of chewing gum affects the inammatory response an reuces postoperative ileus following major colorectal surgery. Evid Based Med. 015;0(5):185–186. Kotagal M, Hakkarainen TW, Simianu VV, Beck SJ, AlfonsoCristancho R, Flum DR. Ketorolac use an postoperative complications in gastrointestinal surgery. Ann Surg. 016;63(1):71–75.

27. A. Crohn isease is the most common primary surgi-

cal isease of the small bowel. Acute onset of abominal pain an iarrhea is not the most common presentation for Crohn isease; the majority of patients rst present with an insiious onset of vague abominal iscomfort (E). It has a bimoal istribution, with one large peak in the secon an thir ecaes of life an a secon smaller peak in the sixth ecae (C). Several risk factors for Crohn isease have been ientie, incluing living in northern latitues, Ashkenazi Jewish escent, smoking, an a familial inheritance. The relative risk among rst-egree relatives of patients with Crohn isease is as high as 14 to 15 times greater than in the general population. It is also more common in urban areas an in patients with a high socioeconomic status. Most stuies suggest that Crohn isease is approximately of equal prevalence in females an males (D). Breastfeeing may also be protective against the evelopment of Crohn isease. Although meical management is the rst-line treatment for Crohn isease, about 75% of patients will ultimately nee surgery. The most common reasons for surgery inclue stula, abscess, an obstruction; perforation is quite rare (B). References: Passier JLM, Srivastava N, van Puijenbroek EP. Isotretinoin-inuce inammatory bowel isease. Neth J Med. 006;64():5–54. Strong SA. Surgical management of Crohn’s isease. In: Holzheimer RG, Mannick JA, es. Surgical treatment: evidence-based and problem-oriented. Munich: Zuckschwert; 001:714–75.

28. A. In the early stages of Crohn isease, patients emon-

strate small supercial ulcers in the mucosa known as aphthous ulcers. These supercial ulcers are often surroune by a halo of erythema. The ulcers form as a result of submucosal lymphoi follicle expansion. As the isease progresses, the ulcers coalesce to form larger ulcers, which are stellate shape, as well as eep linear ulcers. Further coalescence of the ulcers leas to a cobblestone appearance (D), which is a hallmark of Crohn isease. Other hallmarks of Crohn isease inclue noncaseating granulomas (C), transmural inammation, serosal thickening (E), an “skip lesions,” meaning that the areas of intestinal inammation are iscontinuous. The noncaseating granulomas are foun in both areas of active isease, an grossly normal-appearing intestine is seen in all layers of the bowel wall an in mesenteric lymph noes (B). Because the inammation is transmural, iname loops

CHAPtEr 9 Alimentary Tract—SmallBowel of bowel become ahere to one another, thereby leaing to brosis, stricture formation, intraabominal abscess, stulas, an, rarely, free perforation. Reference: Levine MS. Crohn’s isease of the upper gastrointestinal tract. Radiol Clin North Am. 1987;5(1):79–91.

29. A. Numerous pharmacologic agents are use to treat

Crohn isease. Treatment options shoul be ivie into those use for maintenance therapy for mil active isease, those use to treat an acute exacerbation, an rugs for maintaining remission. In patients with mil active isease, the most commonly use rug is sulfasalazine, an aminosalicylate that acts as an antiinammatory agent. This is particularly useful in patients with colitis an ileocolitis. Mesalamine is another antiinammatory agent in the same family as sulfasalazine. It seems to have fewer sie effects owing to the fact that it is activate by colonic bacteria, thus limiting its action to the colon. For acute are-ups, the treatment of choice remains corticosterois, prenisone in particular. Prenisone is highly effective in inucing remission (in approximately three-fourths of patients); however, ue to the sie effects of long-term use, it is not recommene for longterm prevention of remission (B). Buesonie (C), a synthetic glucocorticoi, is another option. It has an avantage over prenisone in that it has a markely reuce systemic absorption an thus fewer long-term sie effects. Nevertheless, it can also suppress the arenal glan. If corticosterois are ineffective in inucing remission, the next step woul be to aminister iniximab (E), a monoclonal antiboy that targets tumor necrosis factor-alpha. Care must be use in aministering iniximab. Because it targets tumor necrosis factor-alpha, a cytokine that regulates inammatory reactions, patients who receive iniximab are at increase risk of acquiring opportunistic infections such as tuberculosis an aspergillosis. It is also associate with activation of latent multiple sclerosis, emyelinating central nervous system isorers, an worsening congestive heart failure. Iniximab has also been shown to be effective in healing complex stulas associate with Crohn isease. Rarely, it has been associate with T-cell lymphoma an almost exclusively in young teenage males. Antibiotics have an ajunctive role in the treatment of infectious complications associate with Crohn isease (D). They are use to treat patients with perianal isease, enterocutaneous stulas, an active colon isease an ai in situations in which bacterial overgrowth has occurre. Once remission has been achieve after an acute are-up, it is important to maintain remission. Although corticosterois woul theoretically be useful, the sie effects preclue longterm aministration. Iniximab is use to maintain remission, as are azathioprine an 6-mercaptopurine. These latter

121

rugs act by inhibiting DNA synthesis an thus suppressing the function of T cells an natural killer cells. A secon-line agent for maintenance of remission is methotrexate.

30. D. Approximately three-fourths of patients with Crohn

isease will eventually require surgery. Inications for surgery inclue failure of meical management, intestinal obstruction, stula, abscess, bleeing, an perforation. In chilren, growth retaration is another inication. Because patients with Crohn isease will often require repeat operations, it is important to avoi unnecessary resection of small bowel because this puts the patient at risk of short bowel synrome. As such, several principles of surgical management shoul be followe. Surgical resection shoul be limite to the segment of bowel that is causing the complication. Other areas of active isease shoul be left alone, provie they are not causing obvious complications. Resection margins of  cm beyon grossly visible isease are recommene (A). Resection margins have not been shown to affect recurrence. The presence of microscopic isease in the resection margin also oes not aversely affect outcome or recurrence. Thus, frozen section is unnecessary (B). When the inication for surgery is SBO, strictureplasty has been shown to be equally effective as resection for jejunal an ileal isease while sparing bowel length. Two types of strictureplasty are recommene: the Heineke-Mikulicz pyloroplasty (for strictures 50 years ol who are unergoing screening colonoscopy C. The mean withrawal time is ≥6 minutes in colonoscopies with normal results that are performe in patients with intact anatomy D. Mucosally base peunculate polyps an sessile polyps 50% rop in PTH at 10 minutes post excision (check again 0 minutes post excision if criterion not met) . Location of ectopic superior glans: tracheoesophageal groove & retroesophageal > intrathyroial, caroti sheath, in cervical thymus 3. Ectopic inferior glans (more variable): thymus > tracheoesophageal groove, intrathyroial a) Perform cervical thymectomy if unable to locate inferior glan 4. If unable to locate parathyroi, complete operation an perform postoperative localization stuies 5. Correcting hyperparathyroiism oesn’t improve osteoporosis t-score but it ecreases the rate at which it rops III. Arenal A. Incientaloma 1. Majority are benign, nonfunctioning (0), BMP to check potassium c) Pheochromocytoma: 4-hour urine metanephrines 3. If nonfunctional, rule out malignancy a) Higher malignancy risk: size >4 cm, >30 Hounsel units, heterogeneity, >50% washout b) Role of FNA for arenal mass is very limite (1) If (+) cancer history, consier biopsy to rule out arenal metastasis () Primary arenal cortical carcinoma cannot be iagnose with FNA (3) FNA of unsuspecte pheochromocytoma can trigger catecholamine surge c) Consier arenalectomy for lesions >4 to 5 cm (except myelolipoma) or lesions with suspicious raiographic features regarless of size B. Cushing isease versus primary hypercortisolism 1. Twenty-four-hour urine cortisol, ACTH level, high-ose examethasone suppression test . Patients with subclinical hypercortisolism may have normal 4-hour cortisol levels; if there is high suspicion, procee with low-ose examethasone suppression testing C. Pheochromocytoma 1. Associate synromes: von Hippel-Linau, MEN , neurobromatosis 1 (von Recklinghausen isease) . Alpha-blockae prior to initiating beta-blockers D. Hyperalosteronism (Conn synrome) 1. Hypertension, hypokalemia, alkalosis . Alosterone to renin ratio ≥30 (90% sensitive), plasma alosterone concentration >10 3. Unilateral aenoma > bilateral arenal hyperplasia; aenomas are usually small E. Arenal cortical carcinoma consierations 1. High attenuation (>0 Hounsel units), >4 cm, heterogenous appearance on CT . Often are functional: hypercortisolism, hyperalosteronism, hyperanrogenism 3. Open arenalectomy; laparoscopic arenalectomy is currently contrainicate (higher rate of local recurrence, poorer isease-free survival) 4. Mitotane for positive margins, vascular or capsular invasion, rupture/spillage, unresectable/ recurrent/metastatic isease (most commonly liver & lung) 5. Associate synrome: Li-Fraumeni (p53) F. Arenal metastases 1. Lung, kiney, melanoma, breast most common; often bilateral . Bilateral arenalectomy may benet select patients a) Evaluate for an correct arenal insufciency prior to bilateral arenalectomy to prevent perioperative arenal crisis; 30% of patients have entire glan replace with tumor

CHAPtEr 13 Endocrine Surgery

177

IV. Thyroiitis

Thyroiditis

Hyper-/hypothyroid

Cause & associations

Treatment

Hashimoto (chronic or lymphocytic) thyroiditis

Hypothyroid May have thyrotoxicosis with transient hyperthyroidism due to gland destruction

• Autoimmune, (HLA) -DR3, -DR5, & -B8 • Antithyroid peroxidase (anti-TPO) antibodies (90%); antithyroglobulin antibodies (20%–50%) • Female > male, 30–60 years old, painless goiter • Risk of primary thyroid lymphoma

• Levothyroxine • Thyroidectomy reserved for large goiters with compressive symptoms

Postpartum thyroiditis

Hyperthyroid with decreased uptake on RAI scan due to follicular cell destruction → euthyroid period → transient hypothyroidism → euthyroid (recovery)

• Autoimmune, may have higher antithyroid titers • Painless

• No treatment unless symptoms are severe • Hyperthyroid—betablockers (propranolol if breastfeeding) • Hypothyroid— levothyroxine

Acute (suppurative) thyroiditis

Euthyroid

• Bacterial (Staphylococcus aureus and Streptococcus pyogenes) > fungal > parasitic • Painful

• Ultrasound-guided ęneneedle aspiration with Gram stain and culture to determine etiology, antibiotics

Subacute granulomatous (de Quervain) thyroiditis

Hyperthyroid with decreased uptake on RAI scan due to gland destruction → euthyroid period → transient hypothyroidism → euthyroid (recovery)

• Preceded by viral URI • Female > male, painful goiter, fatigue, weight loss

• Supportive care, NSAIDs, steroids

Riedel (ębrous) thyroiditis

Hypothyroid

• Firm, nontender, extensive ębrosis that may compress adjacent structures • May have elevated serum IgG4

• Glucocorticoids, tamoxifen, levothyroxine • Surgery reserved for severe compressive symptoms

V. Bethesa Classication

Category

Cytopathology

Malignancy rate

Management

I

Nondiagnostic/inadequate

0%–20%

Repeat FNA with ultrasound guidance

II

Benign

0%–3%

Clinical and sonographic followup

III

Atypia of undetermined signięcance/ follicular lesion of undetermined signięcance (FLUS)

5%–15%

Repeat FNA, molecular testing, or lobectomy

IV

Follicular neoplasm/suspicious for follicular neoplasm, e.g., Hürthle cell (oncocytic) type

15%–30%

Molecular testing and lobectomy

V

Suspicious for malignancy

60%–75%

Near-total thyroidectomy or lobectomy

VI

Malignant

97%–99%

Near-total thyroidectomy or lobectomy

178

PArt i Patient Care

IV. Hyperparathyroiism

Type

Cause

Primary

Parathyroid adenoma (85%) or hyperplasia Lithium-induced parathyroid hyperplasia

↑/↓

Secondary

Chronic renal failure, hyperphosphatemia Vitamin D deęciency

↓/↑

Tertiary

Persistent hyperfunctioning parathyroid glands after kidney transplantation

↑/↓



3.5-gland parathyroidectomy

↑↑/↓



Parathyroidectomy with en bloc ipsilateral thyroidectomy + isthmusectomy & ipsilateral central neck dissection (lateral neck dissection only if evidence of gross LN involvement). Repeat neck exploration for recurrent disease.

Parathyroid carcinoma

Serum Ca/Phos

Urine Ca

Treatment

↑ (calcium creatinine clearance ratio >0.02 suggestive)

Adenoma—parathyroidectomy of aěected gland Hyperplasia—3.5-gland excision or 4-gland excision with autotransplantation Phosphate binders & dietary modięcation, calcium supplementation, vitamin D, cinacalcet 3.5-gland parathyroidectomy for calciphylaxis, bone pain, intractable pruritus, persistent anemia, or pathologic fractures

Fig 13.1 Location of Lost Parathyroi Glans.

CHAPtEr 13 Endocrine Surgery

179

Questions 1. A 55-year-ol male has recalcitrant hypertension espite taking three antihypertensive meications. He is subsequently foun to have an alosterone to renin ratio greater than 30. A CT reveals a 1-cm left arenal lesion an a -cm right arenal lesion. What is the next best step in management? A. Right arenalectomy B. Left arenalectomy C. Bilateral arenalectomy D. Selective venous sampling E. 11C-metomiate scan 2. A 5-year-ol obese male who unerwent CT scan  months ago following a car collision was incientally foun to have a 3-cm left arenal mass. Which of the following is the next appropriate step? A. Laparoscopic left arenalectomy B. Surveillance CT performe at 6 months an then annually for 1 to  years C. No further testing necessary D. PET CT E. Overnight low-ose (1 mg) examethasone suppression test 3. A 40-year-ol female presents with incientally iscovere mil elevation in serum calcium. She is otherwise healthy. A PTH level is elevate as well. Both ultrasoun an sestamibi scan of the neck are negative. Which of the following is true about this conition? A. It may represent tertiary hyperparathyroiism B. A 4-hour urine calcium is inicate C. She shoul procee to neck exploration D. It shoul be treate with cinacalcet E. Selective venous sampling is inicate 4. A 40-year-ol female presents with incientally iscovere hypercalcemia to 11.7 mg/L. She is otherwise healthy. A PTH level is elevate as well. Both ultrasoun an sestamibi scan of the neck are negative. Urine calcium is elevate. What is the most appropriate next step? A. Procee to neck exploration B. Treatment with cinacalcet C. Selective venous sampling D. MRI of the neck E. Observation

5. Following total thyroiectomy for follicular cancer, a 65-year-ol female presents to the emergency epartment 4 ays later complaining of circumoral numbness an tingling of her ngers. Phosphate level is normal. Which of the following is true about this conition? A. It likely represents hungry bone synrome (HBS) B. It may lea to a shortene QT on ECG C. The risk can be reuce by routine postoperative calcium an vitamin D supplementation D. Most patients are symptomatic E. It is more common with thyroiectomy for benign lesions 6. A 45-year-ol man with episoic severe hypertension is foun to have an elevate plasma metanephrine level an a serum calcium level of 11.5 mg/L. Which of the following woul be inicate in the workup? A. CT scan of the sella turcica B. Calcitonin level C. Serum gastrin level D. Serum prolactin level E. A 4-hour urine cortisol 7. Which of the following laboratory nings is characteristically associate with primary hyperparathyroiism? A. Elevate serum phosphate B. Increase serum chlorie C. Decrease urinary calcium D. Metabolic alkalosis E. Elevate calcium with a ecrease PTH 8. A 60-year-ol woman presents with fatigue, weakness an confusion. She has history of kiney stones an pathologic fractures. On physical she has a palpable neck mass. Her serum calcium level is 14.8 mg/L. The most likely iagnosis is: A. Parathyroi aenoma B. Parathyroi hyperplasia C. Parathyroi cancer D. Breast cancer with bone metastasis E. Seconary hyperparathyroiism

180

PArt i Patient Care

9. During neck exploration for primary hyperparathyroiism, only three parathyroi glans are ientie, all of which appear normal in size. Which of the following woul be appropriate? A. Perform a transcervical thymectomy B. Remove all three glans an reimplant one in the forearm C. Remove two an a half glans an then close D. Perform meian sternotomy to look for ectopic parathyroi E. Obtain biopsy samples of all three parathyroi glans an then close 10. After total thyroiectomy an postoperative ioine ablation for a 5-cm follicular thyroi cancer, the best test to monitor for recurrent isease is: A. Serum thyroi-stimulating hormone (TSH) B. Serum calcitonin C. Serum thyroglobulin D. 131I scan E. Cross-sectional CT or MRI 11. Which of the following is true regaring arenal cortical carcinoma? A. Associate evience of hormonal excess is common B. The iagnosis is generally mae by CT-guie neele biopsy C. Staging is base on tumor histology D. Because of malignant potential, arenal masses larger than 3 cm shoul be excise E. Laparoscopic arenalectomy is the preferre approach for surgical resection 12. Malignancy within a thyroglossal uct cyst is typically: A. Follicular thyroi B. Papillary thyroi C. Squamous cell D. Anaplastic thyroi E. Hürthle cells 13. After a total thyroiectomy, the right vocal cor is note to be xe in a parameian position. This most likely represents: A. Injury to the recurrent laryngeal nerve (RLN) B. Injury to the internal branch of the superior laryngeal nerve C. Injury to the external branch of the superior laryngeal nerve D. Trauma from enotracheal intubation E. Compression from hematoma

14. The most common pituitary neoplasm associate with MEN 1 secretes: A. ACTH B. Prolactin C. Growth hormone D. Thyroi-stimulating hormone E. Follicle-stimulating hormone 15. Which of the following features of Graves isease oes not improve with antithyroi therapy? A. Tremor B. Anxiety C. Graves ermopathy D. Gastrointestinal isturbance E. Exophthalmos 16. A 56-year-ol male presents with refractory hypertension espite being starte on hyrochlorothiazie an lisinopril by his primary care physician. His bloo pressure is 18/9 mmHg. Laboratory stuies are remarkable for an alosterone-renin ratio of 5 an hypokalemia. Which of the following is the next best step? A. Triamterene B. Amilorie C. Spironolactone D. Phenoxybenzamine E. Eplerenone 17. A 40-year-ol female presents with a 4 cm thyroi noule that is biopsy-proven papillary thyroi carcinoma. The patient is taken to surgery, an nal pathologic evaluation reveals a 4-cm papillary thyroi carcinoma with microscopic invasion of the perithyroial tissue, but no vascular invasion. A 1-cm lymph noe in the lateral neck is positive. Which of the following answer choices correctly pairs this patient’s American Thyroi Association (ATA) risk stratication with her pathologic nings? A. Low risk; no vascular invasion B. Intermeiate risk; tumor 4 cm C. Intermeiate risk; positive lymph noe 1 cm an the contralateral arenal glan is normal on CT. Alternatively, some surgeons recommen routine arenal venous sampling in most patients, especially those oler than 40 years ol as they are more likely to have nonfunctioning arenal aenomas. In the setting of bilateral aenomas on CT, arenal venous sampling shoul be performe (A, B). It woul be inappropriate to perform bilateral

arenalectomy without further attempts at localizing the source (C). Functional nuclear meicine stuies can also ai with lateralization but is typically performe if venous sampling is unsuccessful (E). Further, if this patient’s hyperalosteronism is ue to hyperplasia, it woul be treate meically. Reference: Yeh MW, Livhits MJ, Duh Q. The arenal glans. In: Townsen CM Jr, Beauchamp RD, Evers BM, Mattox KL. Sabiston textbook of surgery: the biological basis of modern surgical practice. 0th e. Elsevier; 016:963–995.

2. E. Incientalomas are iscovere in 1% to 4% of imaging

stuies that are evaluating an unrelate issue. The majority of incientalomas are nonfunctioning aenomas (60%). The remaining tumors in a patient that o not have a history of

186

PArt i Patient Care

malignancy inclue pheochromocytoma, cortisol-proucing aenoma, alosteronoma, arenocortical carcinoma, an myelolipoma. There shoul be a high level of suspicion for arenal metastasis in a patient with a history of malignancy an/or bilateral lesions. All arenal incientalomas shoul unergo biochemical testing to evaluate for subclinical Cushing synrome, pheochromocytoma, an alosteronoma (E). A functional incientaloma is an inication for arenalectomy. For patients with negative biochemical testing an size 5 cm. For patients with 3 to 5 cm nonfunctioning incientalomas, arenalectomy can be consiere for patients with few surgical risk factors an those with concerning raiographic features (irregular borers, central necrosis, high vascularity, an internal calcications) (A). PET CT is not part of the initial workup (D). No further testing woul be incorrect (C). Reference: Yeh MW, Livhits MJ, Duh Q. The arenal glans. In: Townsen CM Jr, Beauchamp RD, Evers BM, Mattox KL. Sabiston textbook of surgery: the biological basis of modern surgical practice. 0th e. Elsevier; 016:963–995.

3. B. Surgery is inicate in asymptomatic patients uner

the age of 50 that are suspecte to have primary hyperparathyroiism. Familial hypocalciuric hypercalcemia (FHH) causes mil increase in serum calcium an can initially be misiagnose as primary hyperparathyroiism. It is a benign conition ue to mutations in CASR, which encoes a calcium receptor. The lack of calcium signal increases the PTH level, which increases renal calcium reabsorption. Thus, part of the workup of primary hyperparathyroiism is to obtain a 4-hour urine calcium. Hypercalciuria with a high PTH level an high serum calcium level conrms primary hyperparathyroiism. A low urine calcium level suggests FHH. Once FHH is rule out, four-glan neck exploration can be performe without the nee for further imaging (C). Tertiary hyperparathyroiism typically occurs in patients with renal failure, most of whom have unergone kiney transplantation (A). Cinacalcet is inicate for patients with seconary hyperparathyroiism (D). Selective venous sampling is an invasive proceure that is inicate in patients with recurrent hyperparathyroiism, when other forms of imaging fail to ientify the abnormal glan (E).

4. A. Noninvasive localization stuies shoul always be

employe before taking a patient to surgery for primary hyperparathyroiism. Inications for parathyroiectomy in the asymptomatic patient inclue serum calcium >1 mg/L above normal, age 33). PTH levels are increase (E). Hypercalcemia typically results in hypercalciuria, with the exception being in patients with familial hypocalciuric hypercalcemia (C).

8. C. Parathyroi carcinoma is extremely rare an accounts

for less than 1% of cases of primary hyperparathyroiism. It shoul be suspecte in the setting of severe symptoms of hypercalcemia, in association with very high serum calcium (usually 14.6–15.0 mg/L) an PTH, history of kiney stones an pathologic fractures, an a palpable neck mass (A, B). Benign causes of hyperparathyroiism very rarely result in a palpable neck mass an are less likely to cause a hypercalcemic crisis. Determination of malignancy is ifcult because, similar to other enocrine malignancies, there are not any classic histologic features that reliably istinguish parathyroi malignancy from benign isease. Thus, one must look for evience of local invasion at the time of surgery as well as enlarge lymph noes. Treatment is surgical an involves en bloc resection of the parathyroi tumor with the ipsilateral thyroi glan, as well as a moie raical lymph noe issection if noal metastasis is present. Recently, cinacalcet was approve by the US Foo an Drug Aministration an is effective in controlling the hypercalcemia associate with parathyroi carcinoma. Breast cancer with bone metastasis may be associate with a paraneoplastic synrome in which a high level of PTH-relate protein is foun. This is unlikely to present with a palpable neck mass (D). Seconary hyperparathyroiism is associate with a low level of serum calcium (E). Reference: Shane E. Parathyroi carcinoma. J Clin Endocrinol Metab. 001;86():485–493. Sharretts JM, Kebebew E, Simons WF. Parathyroi cancer. Semin Oncol. 010;37(6):580–590.

9. A. On occasion, espite careful neck exploration, only

three parathyroi glans will be encountere. A careful search for the ectopic glan shoul be conucte (B, C). The inferior glans are more likely to be ectopic than the superior ones. Most inferior glans are to be foun within  cm of the inferior thyroi pole. If not foun, the next step is to perform a cervical thymectomy an sen the tissue for frozen section. If still glans are not foun, the caroti sheath shoul be opene. Intraoperative ultrasonography shoul then be use to etermine whether there is an intrathyroial parathyroi glan. If ultrasonography is not available, ipsilateral thyroi lobectomy shoul be consiere. Another useful moality in

10. C. Serum thyroglobulin levels are the most useful

preictive value of serum thyroglobulin levels, measure uring the rst year of follow-up after thyroi hormone withrawal, in thyroi cancer patients. J Clin Endocrinol Metab. 003;88(3):1107–1111. Duren M, Siperstein AE, Shen W, et al. Value of stimulate serum thyroglobulin levels for etecting persistent or recurrent ifferentiate thyroi cancer in high- an low-risk patients. Surgery. 1999;16(1):13–19. Lal G, Clark OH. Thyroi, parathyroi an arenal. In: Brunicari FC, Anersen DK, Billiar TR, etal., es. Schwartz’s principles of surgery. 8th e. New York: McGraw-Hill; 005:1395–1470.

11. A. Arenocortical carcinomas are rare. They shoul

be suspecte in the presence of large tumors (>5–6 cm) or if the CT scan shows evience of necrosis, hemorrhage, or local invasion. Approximately 60% of patients with arenocortical carcinoma present with hormonal excess, incluing Cushing synrome an virilization. There are no istinctive histologic or cytologic features that istinguish arenocortical carcinoma from an aenoma (C). Thus, one must rely on evience of local invasion, lymph noe metastasis, or istant metastasis. CT-guie neele biopsy is not recommene (B). The best chance for cure is surgical resection. Open arenalectomy is the stanar of care for surgical resection for arenal cortical carcinoma as the laparoscopic approach is associate with higher local recurrence rates an poorer isease-free survival (E). Arenal masses that are hormonally active shoul be excise. In the absence of hormonal activity an in the absence of CT scan features suggestive of malignancy, resection is recommene for asymptomatic masses if they are larger than 5 to 6 cm (D). Reference: Ng L, Libertino JM. Arenocortical carcinoma: iagnosis, evaluation an treatment. J Urol. Publishe online 003:5–11.

188

PArt i Patient Care

12. B. The frequency of thyroi carcinoma among patients

with a surgically remove thyroglossal uct cyst in one large series was 0.7%. The majority is papillary cancer that is foun incientally after a Sistrunk proceure (performe for the cyst) (A, C–E). If iscovere incientally, the patient shoul subsequently unergo a total thyroiectomy because aitional cancer is usually foun within the thyroi glan as well. Reference: Heshmati HM, Fatourechi V, van Heeren JA, Hay ID, Goellner JR. Thyroglossal uct carcinoma: report of 1 cases. Mayo Clin Proc. 1997;7(4):315–319.

13. A. The RLN innervates the intrinsic muscles of the lar-

ynx, except the cricothyroi muscles, which are innervate by the external branch of the superior laryngeal nerve (C). The internal branch of the superior laryngeal nerve provies sensory input for the pharynx (B). Injury to one RLN leas to paralysis of the ipsilateral vocal cor. The cor becomes xe in either the parameian position or the abucte position. If the cor becomes xe in the parameian position, the patient will have a weak voice, whereas if it becomes xe in the abucte position, the patient will have a hoarse voice an an ineffective cough. If both RLNs are injure, an airway obstruction may evelop acutely in the patient. Trauma from enotracheal intubation or compression from hematoma oes not typically cause vocal cor paralysis (D, E).

14. B. Pituitary tumors are the thir most common tumors

in MEN 1. The majority are prolactinomas (A, C–E). They may cause bitemporal hemianopsia ue to local compression of the optic chiasm resulting in loss of peripheral vision or may lea to amenorrhea an galactorrhea in women or hypogonaism in men. Women are more likely to present early in the course of the isease as they are more likely to have hormonal symptoms. Men typically present later with mass-effect of the tumor (visual changes, heaaches, etc.).

15. E. Graves isease is the most common cause of hyper-

thyroiism in the Unite States an is ue to antiboies targeting thyrotropin receptors, which increase prouction of thyroi hormone. Patients present with anxiety, rapi or irregular heart rate, heat intolerance, weight loss, thinning hair, ecrease libio, iarrhea, thick an shiny skin (Graves ermopathy), an exophthalmos. The preferre therapy is raioactive ioine ablation, but meical therapy with propylthiouracil (PTU) or methimazole is also available. Exophthalmos evelops in about 10% of patients an is the only symptom that is resistant to antithyroi therapy an even worsens after raioactive ioine ablation (A–D). Some stuies suggest that the use of prenisone before antithyroi therapy can help improve exophthalmos. References: Bartalena L, Marcocci C, Bogazzi F, et al. Relation between therapy for hyperthyroiism an the course of Graves’ ophthalmopathy. N Engl J Med. 1998;338():73–78. Shiber S, Stiebel-Kalish H, Shimon I, Grossman A, Robenshtok E. Glucocorticoi regimens for prevention of Graves’ ophthalmopathy progression following raioioine treatment: systematic review an meta-analysis. Thyroid. 014;4(10):1515–153. Stein JD, Chilers D, Gupta S. Risk factors for eveloping thyroi-associate ophthalmopathy among iniviuals with Graves’ isease. JAMA. 015;133(3):90–96.

16. C. Primary hyperalosteronism shoul be suspecte

in patients with hypertension an hypokalemia. Primary

hyperalosteronism results from autonomous alosterone secretion, which, in turn, leas to suppression of renin secretion. The iagnosis is mae by emonstrating a combination of inappropriate potassium excretion in the urine (kaliuresis), low plasma renin, an a high alosterone-to-renin ratio (>0). While it was previously believe that an arenal aenoma (Conn synrome) was the most common cause of primary hyperalosteronism, we now know that nearly 60% of cases are ue to iiopathic bilateral arenal hyperplasia (IBAH). It is important to clearly establish the etiology because the management is ifferent. An arenal aenoma shoul be remove with a unilateral arenalectomy but IBAH is manage with meical therapy alone using a mineralocorticoi replacement such as spironolactone or eplerenone. Amilorie an triamterene are also potassium-sparing iuretics but are less optimal (A, B). A ouble-blin ranomize controlle stuy emonstrate the superiority of spironolactone in controlling hypertension compare with eplerenone (E). Bilateral arenalectomy is consiere in cases of severe refractory hypertension. However, this has a high risk of complications an will subject the patient to lifelong epenence of mineralocorticois (urocortisone) an sterois. Phenoxybenzamine is an alpha-1 receptor antagonist use in the preoperative management of pheochromocytoma (D). References: Kaplan NM. The current epiemic of primary alosteronism: causes an consequences. J Hypertens. 004;(5):863–869. Stowasser M. Upate in primary alosteronism. J Clin Endocrinol Metab. 009;94(10):363–3630. Parthasarathy HK, Ménar J, White WB, et al. A ouble-blin, ranomize stuy comparing the antihypertensive effect of eplerenone an spironolactone in patients with hypertension an evience of primary alosteronism. J Hypertens. 011;9(5):980–990.

17. C. The AJCC/TNM staging system oes not aequately

preict the risk of recurrence in ifferentiate thyroi cancer. Thus, the ATA evelope a 3-tiere clinic-pathologic risk stratication for recurrence in 009 with moications in 015. For papillary thyroi carcinoma, low-risk patients inclue those having intrathyroial tumors without extrathyroial extension, vascular invasion, metastases, aggressive histology, an clinical N0 or ≤5 N1 micrometastases (5 pathologic N1 noes with all involve noes 3 cm in largest imension. This patient has intermeiate isease base on  factors: microinvasion into perithyroial tissue an a metastatic lymph noe 90% of cases). In the complete form, the eciency leas to a ecrease in both cortisol an alosterone. This leas to ambiguous genitalia in females (ue to anrogen excess), salt wasting with hypernatremia, an hypokalemia. The remaining answer choices can also cause congenital arenal hyperplasia but are less commonly foun (A, B, D, E).

54. A. A patient with a history of primary hyperparathy-

roiism, newly enlarging thyroi noule, an elevate calcitonin level likely has multiple enocrine neoplasm-A. These patients are at risk for eveloping meullary thyroi carcinoma (MTC). The characteristics of MTC that affect surgical approach inclue the following: (1) MTC is more aggressive than other thyroi cancers with higher recurrence an mortality rates. () MTC oes not take up raioactive ioine, an raiation therapy an chemotherapy are ineffective (B, E). (3) MTC is multicentric in 90% of MEN  patients. (4) In patients with palpable isease, more than 70% have noal metastases (D). (5) The ability to measure postoperative stimulate calcitonin levels has allowe assessment of the aequacy of surgical extraction. The two main factors affecting survival are stage an age at iagnosis (D). A key factor in survival is early etection via calcitonin screening in at-risk patients. In one large stuy, biochemical cure preicte a survival rate of 97.7% at 10 years. Management of MTC inclues total thyroiectomy with routine central noe issection (A). It shoul be note that MEN A is rare, an in fact, most MTCs

Clark OH. Meullary thyroi carcinoma: clinical characteristics, treatment, prognostic factors, an a comparison of staging systems. Cancer. 000;88(5):1139–1148. Moigliani E, Cohen R, Campos JM, et al. Prognostic factors for survival an for biochemical cure in meullary thyroi carcinoma: results in 899 patients. The GETC Stuy Group. Groupe ’étue es tumeurs à calcitonine. Clin Endocrinol (Oxf). 1998;48(3):65–73.

with signicant suppression in hyperthyroi states. In most states of hyperthyroiism, free T4, total T4, an total T3 are elevate (A–C). Thyroi scan is not use in the initial workup for hyperthyroiism (E).

56. E. The thyroi glan is supplie by paire superior

thyroi arteries from the external caroti arteries an the inferior thyroi arteries from the thyrocervical trunk. The superior thyroi artery is the rst branch of the external caroti artery (B). During thyroiectomy, care must be taken when ligating the superior thyroi arteries to avoi injury to the external branch of the superior laryngeal nerve (D). To avoi injury, ligating the artery an vein separately an close to the thyroi glan is recommene. In approximately 3% of iniviuals, a thyroiea ima artery also provies bloo to the thyroi glan an arises either from the aorta or the innominate artery. When ligating the inferior thyroi arteries, care must be taken to avoi injury to the RLNs (C). The inferior thyroi arteries usually supply the parathyroi glans (A). Ligation of the main trunk of the inferior thyroi arteries uring total thyroiectomy can lea to parathyroi glan ischemia. There are three main pairs of veins raining the thyroi glan: the superior, mile, an inferior thyroi veins. The mile veins are the least constant. The superior an mile veins rain into the internal jugular veins, whereas the inferior veins rain into the brachiocephalic veins.

57. C. The superior laryngeal nerve an RLN arise from

the vagus nerve. The superior laryngeal nerve ivies into two branches an is both motor an sensory to the larynx (D). The internal branch is sensory to the supraglottic larynx, an, although rare, injury uring thyroi surgery woul lea to aspiration (A). The external branch innervates the cricothyroi muscle. Injury to the external superior laryngeal nerve causes an inability to tense the ipsilateral vocal cor. This oes not cause hoarseness, but rather results in voice fatigue, an in singers creates ifculty in hitting high notes. It has been referre to as the nerve of Amelita Galli-Curci or “high note” nerve after the opera singer who unerwent thyroi goiter surgery in the 1930s an lost her ability to sing afterwar. The left RLN loops aroun the aorta at the ligamentum arteriosum. The right RLN loops aroun the right subclavian artery. The RLN innervates the intrinsic muscles of the larynx with the exception of the cricothyroi muscle, which is innervate by the external laryngeal nerve (E).

196

PArt i Patient Care

Injury to one RLN leas to paralysis of the ipsilateral vocal cor, which becomes xe in the parameian or abucte position. Bilateral RLN injury may lea to airway obstruction an complete loss of the voice (B).

58. C. A nonrecurrent laryngeal nerve is rare an occurs

much more commonly on the right (A, B). It branches off the vagus nerve in the neck an heas irectly to the larynx, as oppose to arising from the vagus after passing the subclavian artery (D). The anomalous location, as oppose to its normal position in the tracheoesophageal groove, makes it more prone to injury (E). On the right, a patient can have both a nonrecurrent nerve an a recurrent nerve. Nonrecurrent left laryngeal nerves have been reporte but are extremely rare. The recurrent laryngeal nerve is most vulnerable to injury uring the last  to 3 cm of its course but also can be amage if the surgeon is not alert to the possibility of nerve branches an nonrecurrent nerves, particularly on the right sie.

59. E. PTH increases the bone resorption by stimulating

osteoclasts an inhibiting osteoblasts, leaing to the release of calcium an phosphate into the circulation. At the kiney, PTH limits calcium excretion at the istal convolute tubule via an active transport mechanism an inhibits phosphate an bicarbonate reabsorption, the latter leaing to a mil metabolic aciosis (B, C). PTH also enhances hyroxylation of 5-hyroxyvitamin D to 1,5-hyroxyvitamin D in the kiney, which in turn irectly increases intestinal calcium absorption (not a irect effect of PTH) (D). Cholecalciferol is hyroxylate to 5-hyroxyvitamin D in the liver. This is not regulate by PTH (A).

60. A. Lateral aberrant thyroi is a term use to enote

what appears to be ectopic thyroi tissue foun within the neck. In most instances, it actually represents metastatic thyroi cancer within a lymph noe, most often of the papillary type. It is not typically associate with the remaining answer choices (B–E). Reference: Jong D, Demeter S, Jarosz J. Primary papillary thyroi carcinoma presenting as cervical lymphaenopathy: the operative approach to the “lateral aberrant thyroi. Am Surg. 1993;59:17–176.

61. C. The accepte management of low-risk papillary thy-

roi cancer is either right hemithyroiectomy or total thyroiectomy with or without postoperative 131I. In patients with papillary carcinoma with a history of raiation exposure, there is a higher rate of multicentricity. As such, total thyroiectomy is the recommene proceure (A, B). Postoperative raioactive ioine following total thyroiectomy is inicate for tumors larger than 4 cm, gross extrathyroial extension of the tumor regarless of size, lymph noe metastases, an for high-risk features incluing tall-cell or columnar-cell variant (E). An ae avantage of postoperative raiation is that it allows for the continue monitoring for recurrence with thyroglobulin. Prophylactic central neck noe issection is gaining popularity as well. Moie raical neck issection woul not be inicate unless there were obvious lateral neck noes (D). References: Guerrero MA, Clark OH. Controversies in the management of papillary thyroi cancer revisite. ISRN Oncol. 011;011:30318. Hay ID, Thompson GB, Grant CS, et al. Papillary thyroi carcinoma manage at the Mayo Clinic uring six ecaes (1940–1999): temporal trens in initial therapy an long-term outcome in 444 consecutively treate patients. World J Surg. 00;6(8):879–885.

Skin and Soft Tissue ERIC O. YEATES, AREG GRIGORIAN, AND CHRISTIAN DE VIRGILIO

14

ABSITE 99th Percentile High-Yields I. Most Common Skin Cancers A. Basal cell carcinoma: most common skin cancer an overall cancer 1. Majority foun on hea an neck, more commonly on upper lip . Typically appears as a shiny, pearly skin noule with rolle borers 3. Treatment 1. Excision with 4 to 5 mm margins for low risk, an 1 to  cm for high risk . Low risk is trunk an extremity lesions 5 mm miline shift C. Massive subarachnoi hemorrhage: CTA hea to evaluate for rupture aneurysm or arteriovenous malformation D Start VTE chemoprophylaxis 48 hours from most recent stable CT; low-molecular weight heparin preferre in TBI II. Spinal Cor Injury (SCI) A. Unstable spine injury: isruption of /3 of the longituinal spinal columns (requires surgery) B. High SCI (above T6), concern for neurogenic shock acutely (hypotension, vasoilation with warm skin, braycaria, or inappropriately normal heart rate for trauma setting); long term concern (months later) for autonomic ysreexia with braycaria, iaphoresis, an uncontrolle hypertension

Injury

Location

Mechanism

Management

Hangman’s fracture (bilateral pedicles)

Cervical

Hyperextension of the neck, caused by hanging, diving

Traction and external immobilization (Halo) vs spinal fusion

Dens fracture

C2 (odontoid process)

Hyperextension of the neck, falls in the elderly

Type I is above the base and considered a stable fracture. Type II (which extends to the base of dens) is unstable and need surgical ęxation. Type III fractures extend into the C2 vertebral body—these tend to have a beĴer healing rate than type II and rarely require surgery.

Chance fracture (horizontal disruption of all columns)

Thoracolumbar

Flexion-distraction injury from rapid deceleration during blunt trauma

Orthotic brace for low-grade injury, surgery if neurologic deęcits or ligamentous injury; also higher risk of hollow viscous injury

219

220

PArt i Patient Care

III. Neck Trauma A. Screening for blunt cerebrovascular injury (caroti or vertebral artery): use expane Denver criteria

Risk factors for BCVI

Signs/symptoms of BCVI

LeFort II/III or mandible fracture

Pulsatile bleeding from neck or nasal/oral cavity

Complex skull fracture or basilar skull fracture

Carotid bruit

Traumatic brain injury (GCS  cm segmental loss: esophagectomy, wie rainage, an reconstruction if stable or iversion with esophagostomy (spit stula) if unstable 4. Surgical access: if cervical esophagus use left neck incision, if upper /3 of esophagus use right posterolateral thoracotomy, if istal 1/3 esophagus use left posterolateral thoracotomy C. Tracheal injury: ebrie, repair with primary closure in one layer (two layers coul lea to stenosis) using absorbable suture an buttress with strap muscles D. Surgical airway: cricothyroiotomy preferre over tracheostomy; chilren (age 50% circumference: segmental resection with primary anastomosis for all segments except D C. Colorectal injuries 1. Colon an intraperitoneal rectal: a) Nonestructive (50% circumference): segmental resection with 1° anastomosis if stable, segmental resection left in iscontinuity with planne n look operation if unstable . Extraperitoneal rectum: proximal iversion alone (no presacral rainage or washout) VI. Retroperitoneal hematomas A. Zone I: explore all hematomas B. Zone II: selective exploration for expaning hematoma (blunt or penetrating) C. Zone III: explore penetrating, o not explore blunt VII. Orthopeic an Neurovascular Injury Patterns

Associated neurovascular injury

Complication/deęcit

Anterior (more common) shoulder dislocation

Axillary nerve injury

Weak shoulder abduction

Posterior shoulder dislocation (e.g., seizures)

Axillary artery injury

Humeral shaft fracture

Radial nerve palsy

Wrist-drop

Supracondylar humerus fracture

Brachial artery injury

Forearm compartment syndrome, Volkmann ischemic contracture

Colles fracture (distal radius)

Median nerve compression

Pain, paresthesias in digits 1–3 ½

Fracture/Injury

Scaphoid fracture Posterior (more common) hip dislocation (adducted and internally rotated)

Snuġox tenderness, avascular necrosis; often normal initial XR Sciatic nerve injury (peroneal branch)

Posterior knee dislocation

Popliteal artery injury

Fibula head fracture (or prolonged lithotomy)

Peroneal nerve injury

Foot drop

Exposure/maneuver

Location

Right posterolateral thoracotomy

Mid esophagus

Left posterolateral thoracotomy

Distal esophagus, descending aorta (distal to left subclavian takeoě)

Left anterolateral thoracotomy

Left distal subclavian artery

Median sternotomy

Ascending and arch of aorta, innominate artery, bilateral common carotid artery, superior vena cava, proximal right and left subclavian artery

Left infraclavicular incision

Left mid-subclavian artery

Kocher maneuver

Head of pancreas, SMV, SMA

Left medial visceral rotation (MaĴox maneuver)

Aorta, celiac trunk, SMA, left renal artery, common iliac arteries

Right medial visceral rotation (CaĴell-Braasch maneuver)

Inferior vena cava, right renal vessels, common iliac veins

Pringle maneuver

Control intrahepatic liver hemorrhage, max clamping 30–60 min

222

PArt i Patient Care

VIII. Acute Compartment Synrome (ACS) A. Risk factors: open fractures > close fractures, crush injuries, young, male, long bone fracture (e.g., tibia, raius), high voltage burns, >6 hours ischemia with reperfusion, combine arterial/venous injury B. Diagnosis: high clinical suspicion, compartment pressure >30 mmHg, elta P (compartment pressure/ iastolic pressure)  units of packe re bloo cells [PRBCs]), presence of a pseuoaneurysm or suspecte arteriovenous stula provie that they are hemoynamically stable. Serial abominal exams an trening the hematocrit woul be inappropriate in the presence of active extravasation of contrast (D). If angioembolization is not available, laparoscopic an open splenectomy are both reasonable options in hemoynamically stable patients that meet the above inications for surgery (A, E), whereas in the unstable patient or with iffuse peritonitis, open splenectomy is recommene. Once in the operating room, attempts at splenic preservation via splenorrhaphy are reasonable in hemoynamically stable patients (B). Reference: Stassen NA, Bhullar I, Cheng JD, et al. Selective nonoperative management of blunt splenic injury: an Eastern Association for the Surgery of Trauma practice management guieline. J Trauma Acute Care Surg. 01;73(5 Suppl 4):S94–S300.

30. C. The physical exam nings are concerning for the

presence of a urethral injury. The most common location is at the prostatic urethra. Genitourinary injuries may occur in up to 15% of patients with pelvic fractures. Hea injury is the most common associate injury seen in patients with pelvic fractures. Clinical suspicion of a urethral injury warrants the performance of a RUG to ientify the presence an location of a urethral injury. Blin insertion of a Foley catheter is contrainicate in this patient (A). CT abomen with IV contrast is helpful for ientifying injuries to the kineys an elaye acquisition images may also ai in the ientication of ureteral or blaer injuries (B). A CT cystogram accurately iagnoses both extraperitoneal an intraperitoneal blaer injuries (D). Intravenous pyelogram is use to ientify renal injuries an is rarely performe (E). Management of urethral injuries epens on the location an severity of injury, as well as presence of associate injuries, an surgical expertise. Reference: Johnsen NV, Dmochowski RR, Mock S, Reynols WS, Milam DF, Kaufman MR. Primary enoscopic realignment of urethral isruption injuries—A ouble-ege swor? J Urol. 015;194(4):10–106.

31. C. Although A, B, C (Airway, with cervical spine pre-

cautions; Breathing; Circulation with hemorrhage control) has always been the recommene sequence in trauma patients, recent recommenations are shifting to C, A, B in those with penetrating injuries who are severely hypotensive, as the combination of rapi-sequence intubation an positive pressure ventilation can worsen hypotension an lea to cariac arrest (A). Thus, bloo proucts woul be the preferre rst step, followe by immeiate transport to the

CHAPtEr 16 Trauma operating room. Some meical centers are now proviing initial resuscitation with whole bloo for the trauma patient in shock. Given the location of the injury (zone I of the neck), one shoul have a high suspicion for a right subclavian or innominate artery injury. Once in the operating room (if possible), the patient is preppe an rape prior to intubation. REBOA (E) is utilize for control of vascular injuries below the iaphragm. Proximal control of such an injury on the right via an open approach is best achieve by a meian sternotomy. If the same injury were present on the left, proximal control of the left subclavian artery is best achieve via a left anterolateral thoracotomy. Enovascular balloon occlusion is another option. If bloo is exsanguinating through the bullet hole, manual compression in this area is ineffective. Temporary tamponae can be achieve via insertion an ination of a Foley balloon irectly into the woun, permitting rapi transportation to the operating room. Thoracostomy is inicate for pneumothorax or hemothorax seen on raiograph imaging or after primary survey suggestive of these conitions (B). The above patient has not ha a cariopulmonary arrest, nor oes he meet any inication for ED thoracotomy (D). References: American College of Surgeons Committee on Trauma. Advanced trauma life support program for doctors. 9th e. American College of Surgeons; 01. Demetriaes D, Chahwan S, Gomez H, et al. Penetrating injuries to the subclavian an axillary vessels. J Am Coll Surg. 1999;188(3):90–95.

32. D. With penetrating neck trauma, there is concern that

bleeing may rapily compress the trachea. As such, the rst step in the management algorithm is to establish an airway, particularly in the presence of an expaning hematoma or epresse level of consciousness. If the patient has a “har sign” of a vascular injury, such as a rapily expaning or pulsatile hematoma, visible exsanguination, palpable thrill or auible bruit, or ense neurologic ecit (such as this patient with GCS 8), the patient shoul then be transporte irectly to the OR. If the patient is hemoynamic unstable, without har signs, the presumption shoul be that the patient exsanguinate in the el. Thus, shock is another inication for immeiate surgical exploration (this patient has a low BP as well). Conversely, in the absence of har signs, the next step woul be to obtain CT arteriography of the neck vessels. This historically has been achieve with formal arteriography, because of the ease an rapiity of its use (B, C). In aition, an assessment for injuries to the aeroigestive tract (triple enoscopy an/or esophagography) an cervical spine nees to be performe (E). As a general guie, repairing a caroti artery injury in a patient with a neurologic ecit is recommene as it may result in improve neurologic function, whereas caroti ligation typically oes not. Repair can be achieve by primary suturing, resection with a primary reanastomosis, or interposition graft placement (saphenous vein or polytetrauoroethylene) (A).

33. D. Resuscitative thoracotomy is a potentially lifesaving

proceure. Inications an guielines continue to evolve. There are many articles in the literature on the topic, with variable nings an recommenations. However, several overarching themes consistently permeate these stuies. Outcomes are better for those with SOL than those without, penetrating trauma than blunt, chest trauma than abominal, isolate injury than multiple injuries, without hea injury than with, short uration

239

of CPR than long uration, an stab wouns than GSW (A–C, E). Thus, the best scenario for resuscitative thoracotomy woul be an isolate stab woun to the chest, with SOL (survival from poole ata is 1%). Such a patient is much more likely to have arreste ue to cariac tamponae an therefore has not suffere exsanguinating hemorrhage. Conversely, at the other extreme, for blunt trauma without SOL, survival was only 0.7%. The following are consiere SOL: agonal respirations, cariac electrical activity, palpable pulse, measurable bloo pressure, spontaneous movement, or pupillary reactivity. Thus, the benet of resuscitative thoracotomy for SOL an penetrating chest trauma is clear. Less compelling but still potentially benecial inications woul be penetrating chest trauma without SOL, penetrating extrathoracic injury with or without SOL, an blunt trauma with SOL. There is no benet for blunt trauma with no SOL. For those that survive, a surprising majority survive with favorable neurologic outcomes. References: Burlew CC, Moore EE, Moore FA, et al. Western Trauma Association critical ecisions in trauma: resuscitative thoracotomy: resuscitative thoracotomy. J Trauma Acute Care Surg. 01;73(6):1359–1363. Seamon MJ, Haut ER, Van Arenonk K, et al. An evience-base approach to patient selection for emergency epartment thoracotomy: a practice management guieline from the Eastern Association for the Surgery of Trauma. J Trauma Acute Care Surg. 015;79(1):159–173.

34. E. The preferre access for young chilren an infants

following trauma is via the peripheral percutaneous route (antecubital fossa or saphenous vein at the ankle). After two unsuccessful attempts, consieration shoul be given to IO infusion via a bone marrow neele (18 gauge in infants, 15 gauge in young chilren). IO cannulation of the proximal tibia provies goo short-term access for resuscitation because it targets the noncollapsible veins of the meullary sinus. The optimal site of insertion is the anteromeial tibia  to 3 cm below the tibial tuberosity, ensuring to angle away from the growth plates. This can be performe using a bone marrow neele or an IO vascular access system such as the EZ-IO®. Once the patient has been resuscitate, follow-up attempts at peripheral access shoul be mae. If a patient has obvious eformities in the tibiae (as in this patient), the next location for IO cannulation woul be the istal femur just above the femoral conyles (D). In aults, there has been a shift in recent years, an sternal IO access is now consiere the preferre initial site for cannulation (thinner cortex an abunant re bone marrow) followe by the tibia. The proximal humerus is an aitional option in aults. It is also important to note that serum electrolytes, bloo gases, an type an cross can all be performe using bloo from interosseous access. A istal saphenous vein cutown is another option in chilren ages 1 to 6 years, but in a chil younger than 1 year of age, it woul be challenging an not appropriate in the setting of obvious leg eformity (B). In hypovolemic peiatric patients younger than 6 years of age, percutaneous femoral vein cannulation is another alternative but is associate with an increase risk of venous thrombosis an woul be much more challenging in a chil younger than 1 year (C). Subclavian an internal jugular central lines woul be too ifcult to perform in the trauma setting in such a small chil an woul be associate with an increase risk of iatrogenic injury (A). The interosseous cannula shoul

240

PArt i Patient Care

be remove expeitiously (within 4 hours) because of the potential risk of infectious complications incluing osteomyelitis. Extremity compartment synrome is another potential complication of IO infusion. References: Cullen PM. Intraosseous cannulation in chilren. Anaesth Intensive Care Me, 01; 13:8–30. Pasley J, Miller CHT, DuBose JJ, et al. Intraosseous infusion rates uner high pressure: a caaveric comparison of anatomic sites. J Trauma Acute Care Surg. 015;78():95–99.

35. D. Both bloo volume an re cell volume increase in

the pregnant patient, but bloo volume increases more than re cell volume. Bloo volume increases by approximately 50% as term approaches, whereas re cell volume increases by approximately 30%, resulting in a functional hemoilution an resultant physiologic anemia of pregnancy (A). Thus, pregnant patients are less likely to manifest signs of bloo loss such as tachycaria an hypotension, an if such signs are present, they are inicative of an even more severe bloo loss than in the nonpregnant patient (on the orer of 1500–000 mL of bloo loss). The pregnant patient has an increase tial volume an minute ventilation, esigne to increase oxygen release to the fetus. This results in a mil respiratory alkalosis, with a PCO in the 7 to 3 range (B). Oxygen consumption is increase, an functional resiual capacity is ecrease. In aition, the ,3-iphosphoglycerate level is increase to enhance the release of oxygen to the fetus. However, these physiologic changes result in less pulmonary reserve in an acutely ill pregnant patient. The use of raiographs is thought to be safe for the fetus after the 0th week of gestation (C). The glomerular ltration rate increases, resulting in a ecrease in serum creatinine (E). Other important aspects to be aware of are that the gravi uterus can compress the IVC, resulting in ecrease venous return. Therefore, the pregnant patient shoul be place in the left lateral position at approximately 15 egrees. Pregnant patients are more prone to aspiration, so early NG tube ecompression is important. Finally, the progressive stretching of the peritoneum leas to esensitization so that a pregnant patient is less likely to emonstrate peritoneal signs. Reference: Shah AJ, Kilcline BA. Trauma in pregnancy. Emerg Med Clin North Am. 003;1(3):615–69.

36. D. In the past, the injury escribe woul have been

ealt with by performing a formal lobectomy (A). However, pulmonary tractotomy is now use as a less aggressive alternative. The technique involves using a linear stapling evice to insert irectly into the injure bullet tract. Two hemostatic staple lines are create, an the lung is ivie in between. This allows irect access to the bleeing vessels within the parenchyma as well as any leaking bronchi. Bleeing vessels can then be oversewn with a polypropylene monolament (C). Lobectomy is a better choice for a completely evascularize or estroye lobe. A pneumonectomy is rarely inicate an, in the trauma setting, is associate with an 80% mortality rate (B). Similarly, ligation of a lobar pulmonary artery has a high rate of morbiity (E). References: Cothren C, Moore EE, Bif WL, et al. Lungsparing techniques are associate with improve outcome compare with anatomic resection for severe lung injuries. J Trauma. 00;53(3):483–487.

Kim DY, Coimbra R. Thoracic amage control. In: Di Saverio S, Tugnoli G, Catena F, Ansaloni L, Naioo N, es. Trauma surgery: volume 2: thoracic and abdominal Trauma. Springer Milan; 014:35–46.

37. C. Flail chest occurs when two or more ribs are fracture in at least two locations. Paraoxical movement of this free-oating segment of chest wall is typically not sufcient alone to compromise ventilation (B). Rather, pain an splinting, in conjunction with unerlying pulmonary contusions, may result in hypoxemia an hypercarbia ue to shunting an ineffective ventilation, respectively. Most patients can be manage without intubation (E). Respiratory failure often oes not occur immeiately, an frequent reevaluation is warrante. The initial chest raiograph usually unerestimates the egree of pulmonary contusion, an the lesion tens to evolve with time an with ui resuscitation (A). Intravenous ui aministration shoul be limite as overzealous resuscitation may result in blossoming of pulmonary contusions (D). The most important aspect of treatment of ail chest is pain control. Stanar approaches inclue the use of patient-controlle analgesia an oral pain meications an the placement of continuous epiural catheters. Although the treatment of ail chest has historically been nonoperative, recent literature inicates that internal xation of the chest wall in select patients without pulmonary contusion ecreases intubation time, ecreases mortality, shortens uration of mechanical ventilation as well as hospital stay, ecreases complications, an improves cosmetic an functional results. In the presence of a pulmonary contusion, however, internal xation may not be as benecial. Eastern Association for the Surgery of Trauma (EAST) guielines recommen ORIF in aults with ail chest after blunt trauma. Situations in which internal xation shoul be consiere inclue ail chest in patients who are alreay unergoing thoracotomy for an intrathoracic injury, ail chest without pulmonary contusion, noticeable paraoxical movement of a chest wall segment while a patient is being weane from the respirator, an severe eformity of the chest wall. References: Kasotakis G, Hasenboehler EA, Streib EW, et al. Operative xation of rib fractures after blunt trauma: a practice management guieline from the Eastern Association for the Surgery of Trauma. J Trauma Acute Care Surg. 017;8(3):618–66. Leinicke JA, Elmore L, Freeman BD, Colitz GA. Operative management of rib fractures in the setting of ail chest: a systematic review an meta-analysis. Ann Surg. 013;58(6):914–91. Voggenreiter G, Neueck F, Aufmkolk M, Obertacke U, Schmit-Neuerburg KP. Operative chest wall stabilization in ail chest–outcomes of patients with or without pulmonary contusion. J Am Coll Surg. 1998;187():130–138.

38. A. BCI shoul be suspecte in anyone with severe blunt

chest trauma. Attempts to ientify a BCI an stratify severity on the basis of CK-MB, nuclear scans, an echocariography have not been successful because these moalities lack sensitivity. ECG is the most commonly recommene tool for the initial iagnosis of BCI. The presence of a sternal fracture is not a marker for BCI (D). A normal screening ECG has a negative preictive value of 95% (E). Aition of a normal cariac troponin increases the negative preictive value to 100%. If a stable patient has an abnormal cariac troponin level or ECG, he/she shoul be amitte for observation to a monitore be. However, troponin level oes not correlate with risk of cariac complications in BCI. If the patient is

CHAPtEr 16 Trauma unstable, an emergent echocariogram shoul be performe. If a tamponae is seen, emergent sternotomy shoul be performe for suspecte cariac rupture. Very rarely, BCI can lea to coronary artery thrombosis, valvular isruption, or septal isruption (C). In an unstable patient with BCI without an anatomic abnormality on echocariography, invasive bloo pressure monitoring with pressor support shoul be institute. Most patients with a iagnosis of myocarial contusion have a benign course, with very few eveloping arrhythmias or heart failure (B). References: Clancy K, Velopulos C, Bilaniuk JW, et al. Screening for blunt cariac injury: an Eastern Association for the Surgery of Trauma practice management guieline. J Trauma Acute Care Surg. 01;73(5 Suppl 4):S301–S306. Velmahos GC, Karaiskakis M, Salim A, et al. Normal electrocariography an serum troponin I levels preclue the presence of clinically signicant blunt cariac injury. J Trauma. 003;54(1):45–50.

39. E. The Cattell maneuver involves a right meial visceral

rotation of the cecum an ascening colon. It is achieve by incising the peritoneal reection at the white line of Tolt. It is useful for exposing right retroperitoneal structures, such as the IVC an the right ureter (C). Further cephala, mobilization an meial rotation of the uoenum (Kocher maneuver) aitionally assists in exposing the suprarenal IVC below the liver. The Kocher maneuver is not useful for exposing the celiac axis (D). This is best one by combining a Mattox maneuver with a ivision of the left crus of the iaphragm an iviing the celiac plexus (A). The Mattox maneuver consists of a left meial rotation of the escening colon (again at the line of Tolt), spleen, an/or kiney towar the miline. Exposure of injuries to the istal IVC an iliac vein bifurcations can be exceeingly ifcult. On occasion, ivision of the right common iliac artery is neee to expose an repair an injury of this area. A primary repair of the iliac artery can then be performe. On rare occasions, with massive bleeing, the junction of the superior mesenteric vein (not artery), splenic, an portal veins may nee to be expose by ivision of the neck of the pancreas (B). References: Asensio JA, Chahwan S, Hanpeter D, et al. Operative management an outcome of 30 abominal vascular injuries. Am J Surg. 000;180(6):58–533. Hoyt DB, Coimbra R, Potenza BM, Rappol JF. Anatomic exposures for vascular injuries. Surg Clin North Am. 001;81(6):199–1330.

40. B. Most veins can be safely ligate in the setting of trau-

matic injury. However, certain veins are less likely to tolerate ligation well. These inclue the superior vena cava (because it may result in an acute superior vena cava synrome), the renal veins close to the renal parenchyma (because there is then inaequate outow for the kiney), the IVC above the renal veins (because it will impair outow to both kineys), or just at the iaphragm (because this will cause an acute Bu-Chiari synrome), an the portal vein (because it supplies 75% of the bloo to the liver) (A, E). An exception to the aforementione is ligation of the left renal vein close to the IVC is well tolerate because rainage can occur via the arenal, gonaal, an iliolumbar veins. This is sometimes performe uring open abominal aortic aneurysm repair. The portal vein has been ligate successfully, provie aequate ui is aministere to compensate for the ramatic but transient eema that occurs in the bowel, but ligation seems to be associate with a higher mortality rate than repair.

241

Ligation of the IVC below the renal veins is better tolerate than the suprarenal IVC; however, marke leg swelling may evelop an may require fasciotomies. Ligation of the superior mesenteric vein is also fairly well tolerate an better tolerate than portal vein ligation, although again it is preferable to repair the superior mesenteric vein if the patient is stable an it is technically feasible because there is similarly marke bowel eema an risk of bowel infarction as with portal vein repair. Arteries for which repair shoul always be attempte inclue the innominate, brachial, superior mesenteric, proper hepatic, iliac, femoral, an popliteal arteries an the aorta (C, D). If enitive repair is preclue ue to hemoynamic instability or if a amage control approach is eeme appropriate, perfusion or ow may be maintaine via a temporary intravascular shunt. In the forearm, either the raial or ulnar artery can be ligate, provie the other vessel is palpable. Similarly, in the lower leg, at least one of the two palpable vessels (anterior or posterior tibial artery) shoul be salvage. Because of the excellent collateralization aroun the shouler, ligation of the subclavian artery is well tolerate. In fact, the artery is often occlue uring stent-grafting of thoracic aneurysms or aortic transection. Reference: Rich NM, Mattox KL, Hirshberg A. Vascular trauma. n e. Elsevier Science; 004.

41. A. Extremity compartment synrome can occur any-

where in the extremities, incluing the buttocks, shoulers, an hans (E). The mechanisms of compartment synrome are numerous an can be ivie into extrinsic an intrinsic causes. Extrinsic causes inclue constriction by a cast, tight circumferential ressings, or eschar from a burn. Intrinsic causes are ivie into bleeing, eema, an exogenous ui. Bleeing is usually ue to trauma but can also be seen after relatively minor injuries in patients with an unerlying coagulopathy or those receiving anticoagulants. Eema of the compartment is the largest an broaest category. It is most often seen after reperfusion of an ischemic limb, from either an arterial embolus or thrombosis or trauma. Ischemia/reperfusion is also seen in a person with a rug overose or an alcoholic who falls asleep on the limb, in patients with profoun shock in whom iffuse muscle ischemia with subsequent reperfusion evelops, an after massive iliofemoral eep venous thrombosis. Finally, inavertent infusion of IV ui into the subcutaneous tissue can lea to compartment synrome. Diagnosis of compartment synrome begins by having a high clinical inex of suspicion an knowing the clinical scenarios in which it occurs. The most common features are severe pain in the limb typically out of proportion to the physical exam, pain on passive motion of the limb, an tense eema with tenerness on palpation of the compartment. Distal arterial pulses typically remain palpable with compartment synrome. The anterior compartment of the leg is usually the rst compartment to be involve in the lower extremity (C). The eep peroneal nerve runs within it so numbness in the rst web space of the toe is one of the early nings (D). Once the iagnosis is suspecte, conrmation is sought by oing irect pressure measurements of the iniviual compartments. If the pressures are increase more than 30 mmHg in any of the compartments, then strong consieration shoul be given to performing a four-compartment fasciotomy. The use of an absolute value has been questione because the perfusion pressure necessary for oxygenation

242

PArt i Patient Care

is partly epenent on the patients’ bloo pressure an, therefore, coul lea to unnecessary fasciotomies (B). The use of ifferential pressure (Δp = iastolic bloo pressure— intracompartmental pressure), with a propose threshol of 30 mmHg, has been propose to be of greater iagnostic value. It is also important to remember that there is no absolute pressure level that rules compartment synrome in or out. The measurements shoul be use in conjunction with the patient’s clinical examination. The eep posterior compartment is the one that is most commonly inaequately ecompresse. Because this compartment contains the tibial nerve, missing this compartment can have evastating consequences. The soleus muscle must be etache from the tibia to ecompress the eep posterior compartment. Buttock compartment synrome has been escribe in patients with obesity after prolonge anesthesia as well. Reference: von Keuell AG, Weaver MJ, Appleton PT, et al. Diagnosis an treatment of acute extremity compartment synrome. Lancet. 015;386(10000):199–1310.

42. C. The management of liver injuries has unergone a

major evolution in the past 5 years, from routine laparotomy in the past to the current application of selective nonoperative management in hemoynamically stable patients, liberal use of angiographic embolization, an operative management with selective packing an amage control when the patient is col an coagulopathic. In a patient who has sustaine blunt trauma an is hemoynamically stable, a CT scan with IV contrast shoul be performe. If a contrast blush is seen in the liver, the patient shoul be taken to angiography for embolization, provie there are no other injuries that require operative intervention. Conversely, if the patient is hemoynamically unstable (as in this patient), the patient shoul be taken to the operating room an unergo packing of all four quarants to obtain temporary hemostasis while anesthesia attempts to “catch up” or aequately resuscitate the patient. Strong consieration shoul be given to activating the institutional massive transfusion protocol in aition to aministering tranexamic aci. Given that this patient ha continue bleeing espite application of a Pringle maneuver, he has likely sustaine an injury to the retrohepatic IVC or hepatic veins. If the bleeing is controlle with packing an, in aition, the patient is col (temperature 1.1, but ui creatinine woul be much higher than serum creatinine (B). Patients with abominal compartment synrome have abominal istention, oliguria, ecrease lung compliance, an hypotension. This patient with normal vital signs oes not have abominal compartment synrome (C). Reference: Runyon BA, Montano AA, Akriviais EA, et al. The serum-ascites albumin graient is superior to the exuate-transuate concept in the ifferential iagnosis of ascites. Ann Intern Med. 199;117(3):15–0.

Vascular—Arterial AMANDA C. PURDY AND NINA M. BOWENS

17

ABSITE 99th Percentile High-Yields I. Peripheral Arterial Disease (PAD) A. Normal ankle-brachial inex (ABI) ranges from 1 to 1.; ABI 5.5 cm in males, growth of >0.5 cm in 6 months or >1 cm in 1 year

CHAPtEr 17 Vascular—Arterial

247

E. Surgical options inclue open AAA repair an enovascular repair (EVAR) F. Compare to open AAA repair, EVAR has a lower perioperative (30-ay) mortality rate but similar longterm mortality G. Complications of AAA repair: 1. MI is the most common cause of in-hospital eath after AAA repair . Kiney injury: increase incience if intraoperative hypotension or suprarenal aortic cross-clamp 3. Ischemic colitis: risk factors (coverage of the IMA or internal iliac, rupture AAA, intraoperative hypotension; can present with abominal pain an iarrhea (sometimes blooy); iagnose with exible sigmoioscopy; treat with antibiotics, NPO, an resuscitation, if patient eteriorates or evelops peritonitis will require surgery 4. Aortoenteric stula (after open or enovascular repair): upper GI blee usually >6 months after surgery; ue to infecte graft that eroes into the uoenum; upper enoscopy (rst step in workup) usually negative, CT shows ui/air aroun aortic graft/sac; management is graft excision, close the uoenum, an revascularization either with in situ human aortic homograft, neoaortoiliac proceure (NAIS) or extra anatomic axillobifemoral bypass V. Mesenteric Ischemia A. Acute mesenteric ischemia (AMI) etiologies: embolism (to SMA, most common), thrombosis, low ow state (non occlusive), venous thrombosis B. AMI presentation: writhing aroun complaining of severe abominal pain, but not signicantly tener on exam (pain out of proportion to physical exam); x with CTA an have preop iscussion regaring bowel viability, quality of life, an possible bowel resection C. Surgery for AMI ue to embolism: resect frankly necrotic bowel, open SMA embolectomy 1. Transverse arteriotomy proximal to the mile colic artery, then embolectomy . If any bowel with questionable viability: o not resect, leave abomen open, plan for n look within 4 to 48 hours D. Chronic mesenteric ischemia (CMI) from atherosclerosis of the celiac, superior mesenteric, an/or inferior mesenteric arteries. E. CMI presentation: severe abominal pain about 30 to 60 minutes after eating, often leaing to “foo fear” (intestinal angina) an weight loss F. First-line treatment for CMI is angioplasty an stenting; if bypass is require in high-risk or sick patient, perform retrograe mesenteric bypass from common iliac artery or infrarenal aorta to avoi supraceliac clamping G. Aggressive ui resuscitation shoul be use with caution in non occlusive mesenteric ischemia in the setting of ecompensate congestive heart failure, use irect intraarterial papaverine instea H. Venous thrombosis usually treate with anticoagulation (unless peritonitis); nee hypercoagulable workup

248

PArt i Patient Care

Fig. 17.1 Types of Endoleaks

CHAPtEr 17 Vascular—Arterial

249

QUESTIONS 1. A 5-year-ol woman presents to the trauma bay after a motor vehicle accient. A pan-CT scan is negative for any acute injuries but oes show an inciental focal ilation of the mi-splenic artery to a iameter of .5 cm. The best management of this ning is: A. No further management is necessary B. Elective splenectomy C. Elective coil embolization D. Open repair with vein interposition graft E. Surveillance with repeat imaging in 6 months 2. A 40-year-ol woman with refractory hypertension unergoes further workup. Her plasma an urine metanephrines are normal, alosterone to renin ratio is 3 cm in patients with low surgical risk, an any size in women of chilbearing age. Coil embolization is appropriate for proximal an mi-portion aneurysms as the spleen continues to be perfuse by the short-gastric arteries avoiing splenic infarction. For istal-thir aneurysms, resection with splenectomy is usually performe (B). Open repair with vein interposition graft has largely fallen out of favor (D). Because this patient is a woman of chilbearing age, it woul be inappropriate to procee with conservative management an surveillance (A, E). Patients foun to have a splenic artery aneurysm without inication for repair shoul be followe with annual CT angiogram or ultrasoun. Women are at highest risk for splenic artery aneurysm rupture uring the 3r trimester of pregnancy. Reference: Chaer RA, Abularrage CJ, Coleman DM, et al. The Society for Vascular Surgery clinical practice guielines on the management of visceral aneurysms. J Vasc Surg. 00;7(1 S):3 S–39 S.

2. C. This patient has bromuscular ysplasia (FMD). Renal

artery stenosis is a cause of seconary hypertension an can be ue to atherosclerosis or FMD. FMD is an iiopathic isease of the musculature of the arterial walls leaing to stenosis of small an meium-size arteries an is most common in women from 30 to 60 years ol. The most commonly involve arteries are the renal, caroti, an vertebral arteries. The “string-of-beas” appearance is a classic imaging ning seen in FMD. FMD is noninammatory an there is no role for sterois (B). Patients with renal artery stenosis ue to atherosclerosis shoul receive aspirin an statin; most are manage meically. A renal stent may be consiere in the case of refractory hypertension or ash pulmonary eema (A, D). On the other han, patients with renal artery stenosis ue to FMD are most appropriately treate with percutaneous angioplasty, as stents have a high rate of fracture when use for FMD renal isease. Open bypass is more invasive an has similar success as angioplasty (E). Reference: Gornik HL, Persu A, Alam D, et al. First international consensus on the iagnosis an management of bromuscular ysplasia. J Hypertens. 019;37():9–5.

3. D. Early failure (within 30 ays) after surgery generally

inicates a technical error. Technical errors inclue anastomotic stenosis, a kink or twist within the graft, poor choice of proximal or istal target, an inaequate-caliber saphenous vein. Intermeiate failures, from 30 ays to  years after

bypass, are generally cause by myointimal hyperplasia (A). Late graft failures (beyon  years) are cause by progression of atherosclerotic occlusive isease, either within the inow or outow vessels (B). A persistent valve woul be a potential problem with an in situ vein bypass (not with a reverse vein), in which case valves are intentionally cut with a valvulotome (E). Young patients may have a more aggressive form of atherosclerotic isease (virulent isease), an some have postulate that this may be seconary to an unerlying hypercoagulable state (C). Reference: McCreay RA, Vincent AE, Schwartz RW, Hye GL, Mattingly SS, Griffen WO Jr. Atherosclerosis in the young: a virulent isease. Surgery. 1984;96(5):863–869.

4. C. In an acutely ischemic limb, in aition to the neu-

rovascular exam of the ischemic limb, the most important aspects of the physical examination are the cariac exam an the neurovascular examination of the nonischemic limb. If the nonischemic limb has normal pulses an no other evience of chronic ischemia (e.g., hair loss, thin ry skin), then the ischemia is most likely embolic in nature. Fining an irregularly irregular rhythm woul further conrm that the heart is the most likely source of the clot ue to atrial brillation. With an absent femoral pulse, the embolus has likely loge in the common femoral artery. Because the patient escribe has class b ischemia (immeiately threatene), heparin shoul be starte, an revascularization shoul be performe without elay (E). In class 1 ischemia (not threatene; no sensory or motor loss), there is no immeiate urgency to going to the operating room. Heparin shoul be starte. It is then useful to obtain imaging to conrm the iagnosis. This can be achieve via an arterial uplex scan or CT angiogram, which has replace iagnostic arteriography as the gol stanar (A). An avantage of CT over angiogram is that it may etect etiologies of acute ischemia that woul otherwise be unsuspecte, such as an aortic issection or aneurysm, an one can image the chest an abomen for possible pathology. Following iagnosis, if the patient is not immeiately threatene, they may unergo enitive treatment via thrombolytic therapy or open embolectomy (B, C). Native arterial occlusions ue to cariac embolization ten to respon less favorably to thrombolytic therapy. Thus, open embolectomy is preferre by some. For the patient in the vignette, a transfemoral approach is optimal because it can be one with the patient uner local anesthesia an allows selective embolectomy own the supercial femoral an profuna femoral arteries. The below-knee popliteal artery approach to embolectomy is reserve for situations in which the patient has normal femoral an popliteal pulses an the embolus is loge in the tibial vessels (E). However, such an approach is technically more ifcult. If the limb is not immeiately threatene, istal clots are better manage by lytic therapy as the tPA can be irecte via catheter irectly into the involve vessel. Echocariogram woul eventually be useful to look for a cariac source of thrombus, but it woul not be of immeiate help in the management (D). With the avent of hybri operating rooms, patients

CHAPtEr 17 Vascular—Arterial with more avance ischemia (class ) can be taken irectly to the operating room where a iagnostic angiography followe by immeiate intervention can be achieve. Reference: Results of a prospective ranomize trial evaluating surgery versus thrombolysis for ischemia of the lower extremity. The STILE trial: The STILE investigators (appenix A). Ann Surg. 1994;0(3):51–68.

5. C. This patient’s history an CT scan nings are most

consistent with acute mesenteric ischemia. Acute mesenteric ischemia can be ivie into four major causes. Embolization from a cariac source is the most common cause (30%–50% of cases), is seen most often in the setting of atrial brillation an is the likely etiology in the patient presente. The ning of an irregularly irregular heart rhythm suggests an arterial embolism from atrial brillation. The most common site of mesenteric embolization is the superior mesenteric artery (SMA) (ue to its angle from the aorta). The embolus typically occlues the SMA just istal to the mile colic artery. These patients often have sparing of the proximal jejunum an transverse colon because the mile colic artery remains patent. Celiac artery embolization is rare, given its take-off at a right angle to the aorta. The inferior mesenteric artery orice is so small that a cariac thrombus rarely loges insie. Mesenteric arterial thrombosis is usually ue to unerlying mesenteric artery atherosclerosis. In this situation, the patient will typically have a long-staning history of pain after eating, fear of eating, an weight loss, an the physical examination will reveal evience of iffuse atherosclerosis an bruits. Mesenteric venous thrombosis is a thir etiology an is most often seen in patients with hypercoagulable states. The acute venous occlusion leas to massive bowel eema with seconary arterial insufciency from bowel wall istention. Patients with mesenteric venous thrombosis ten to present in a less ramatic fashion, often with ays or weeks of abominal pain. Finally, nonocclusive mesenteric ischemia results from shock that creates hypoperfusion of the bowel, such as with cariac failure or severe hypovolemia. The classic nings in acute mesenteric ischemia are the suen onset of severe pain out of proportion to the physical examination nings. Elevate serum lactate levels shoul raise the suspicion of ischemic bowel, but they are not sensitive enough to etect early bowel ischemia. A plain abominal raiograph is often unremarkable, although it may emonstrate evience of eema in the small bowel wall. If the patient has peritoneal signs on abominal examination, this will inicate that the bowel has alreay been infarcte. In the absence of peritonitis an because the ifferential iagnosis is extensive, CT provies the greatest iagnostic yiel initially (E). However, CT scan may not be iagnostic because it may not necessarily emonstrate opacication in the mesenteric veins or arteries (epening on the timing of contrast). The rst step in the management is the aministration of IV heparin. Following heparin, for an embolus, immeiate surgery offers the best chance of treatment an woul involve an SMA embolectomy (D). If the history were suggestive of unerlying mesenteric atherosclerosis (longstaning postpranial abominal pain an weight loss) with thrombosis, arteriography woul be helpful because the management woul involve an arterial bypass or stenting (B). If the CT scan reveale a thrombus in a mesenteric vein, enitive treatment woul be heparin alone, provie there is no peritonitis. For nonocclusive

255

ischemia, correcting the unerlying shock is the initial management. Catheter irecte papaverine may also be useful. There are some case reports in which mesenteric emboli have been successfully manage with lytic therapy, but this is not the stanar approach an is not the best option for elevate lactate suggesting a compromise bowel (A).

6. C. Mesenteric venous thrombosis accounts for approxi-

mately 10% to 15% of cases of mesenteric ischemia. It tens to have a slow, insiious onset, as in this case. Risk factors for mesenteric venous occlusion inclue hypercoagulable states such as factor V Leien, antithrombin III eciency, an protein C an S eciency, as well as liver isease with portal hypertension, pancreatitis, an any intraperitoneal inammatory conitions. Venous thrombosis is less ramatic than arterial occlusion. Abominal pain is vague, an tenerness is mil or equivocal. CT may emonstrate a thickene bowel wall with elaye passage of IV contrast agent into the portal system an a lack of opacication of the portal or superior mesenteric vein. If the iagnosis is establishe from the CT scan, further iagnostic tests are unnecessary. Another useful iagnostic moality is uplex ultrasoun scanning. Arteriography may emonstrate venous congestion an a lack of prompt lling of the portal system (D). If the patient is manifesting peritoneal signs, operative exploration is inicate (E). However, in the absence of peritonitis, therapy shoul consist of ui hyration, hemoynamic support, anticoagulation with heparin, an serial examination. If peritonitis subsequently evelops, exploratory laparotomy is appropriate to assess bowel viability with segmental bowel resection. Surgical thrombectomy of the venous system is not likely to be successful. Fibrinolytic therapy has been use increasingly, but is not yet the stanar treatment of choice, an is ieal when symptoms are of short uration (A). Following heparin, warfarin or a novel oral anticoagulant, such as apixaban or rivaroxaban, is recommene for 3 to 6 months if the hypercoagulable state is provoke or temporary (B). Lifelong warfarin or NOAC is recommene if the venous thrombosis is unprovoke or associate with a permanent thrombophilic state. The family history of venous thrombosis in this patient is highly suggestive of an inherite hypercoagulability an woul warrant lifelong anticoagulation. Aitionally, any mesenteric arterial embolism requires lifelong anticoagulation. Reference: Kumar S, Sarr MG, Kamath PS. Mesenteric venous thrombosis. N Engl J Med. 001;345(3):1683–1688.

7. C. Cilostazol has a number of functions incluing inhib-

iting platelet aggregation an smooth muscle proliferation, increasing vasoilation, an lowering high-ensity lipoprotein an triglycerie levels. Cilostazol has been shown to signicantly increase walking istance by 50% to 67% in patients with clauication in several ranomize trials an results in improvement in physical functioning an quality of life. This rug is contrainicate in patients with congestive heart failure. This rug is more effective than pentoxifylline in the treatment of clauication (A). Pentoxifylline is a methylxanthine erivative that has hemorrheologic properties. Two meta analyses showe that it improves walking istance, but in some more recent ranomize stuies, it prove to be no better than placebo. Pentoxifylline improves symptoms of clauication by increasing re bloo cell exibility

256

PArt i Patient Care

an reucing bloo viscosity. Antiplatelet meications such as aspirin are use in the treatment of peripheral vascular isease an for cariac an stroke prevention but o not appear to improve walking istance (B). Aspirin has been foun to reuce the vascular eath rate by approximately 5% in patients with any manifestation of atherosclerotic isease (e.g., coronary, peripheral). Clopiogrel is effective in reucing overall acute cariovascular events, especially in patients with lower extremity occlusive isease, but is much more expensive (D). It oes not seem to irectly improve walking istance. Pure vasoilators have not been efcacious in the treatment of peripheral vascular isease because most patients with such occlusive isease alreay exhibit marke vasoilation. Anticoagulants also have not been shown to alter the course of peripheral atherosclerosis (E). Reference: Money SR, Her JA, Isaacsohn JL, et al. Effect of cilostazol on walking istances in patients with intermittent clauication cause by peripheral vascular isease. J Vasc Surg. 1998;7():67–74.

8. E. Leg eema after femoral-to-popliteal arterial bypass is

common. In most instances, it is ue to lymphatic isruption. This isruption occurs at both the groin an popliteal incisions as well as from harvesting of the saphenous vein. Deep venous thrombosis can occur after this proceure but is relatively uncommon (A). Reperfusion eema may be associate with compartment synrome an can present with the Ps (pain, pallor, paralysis, paresthesia, an poikilothermia) (B). It is more likely to present after revascularization ue to acute limb ischemia. The saphenous veins are part of the supercial venous system, which contributes a minority of the venous rainage in the leg, so swelling seconary to venous congestion is not expecte after a saphenous vein harvest (C). Cellulitis woul present with erythema, pain, warmth, an possible systemic signs such as fever or leukocytosis (D). Reference: AbuRahma AF, Wooruff BA, Lucente FC. Eema after femoropopliteal bypass surgery: lymphatic an venous theories of causation. J Vasc Surg. 1990;11(3):461–467.

9. D. The patient’s history an examination are most consis-

tent with symptoms of coronary-subclavian steal synrome. Most patients with a coronary artery bypass graft have unergone a left internal mammary artery-to-left anterior escening graft. In the setting of subclavian artery stenosis or occlusion proximal to the take-off of the internal mammary artery, arm exercise leas to vasoilation of the arm vessels an lower resistance. Bloo will travel through the path of least resistance an ow in a reverse fashion from the left anterior escening artery into the left internal mammary artery an towar the arm, leaing to the evelopment of angina. The ifferential bloo pressure in the arms is the clue, as is the left arm clauication. Treatment involves relieving the subclavian artery obstruction. This can be one by subclavian artery stenting but on occasion requires a caroti-to-subclavian artery bypass (A). Since the problem is not relate to unerlying cariac isease, caroti stenting, increasing betablocker ose, or increasing ose of nitrates will not resolve the patient’s chest pain with exercise (B, C, E). Reference: Bryan F, Allen R, Lumsen A. Coronary subclavian steal synrome: report of 5 cases. Ann Vasc Surg. 1995;9(1):115–1.

10. B. Initial management of patients with acute mesenteric

ischemia inclues ui resuscitation an systemic anticoagulation with heparin sulfate to prevent further thrombus propagation. Signicant metabolic aciosis shoul be correcte with soium bicarbonate. A central venous catheter, peripheral arterial catheter, an Foley catheter shoul be place for ui resuscitation an hemoynamic status monitoring. Appropriate antibiotics are given before surgical exploration. The operative management of acute mesenteric ischemia is ictate by the cause of the occlusion. For an SMA embolus, exposure of the SMA is obtaine via rotation of the small bowel to the right an by sharply issecting the ligament of Treitz. The SMA will be foun at the root of the mesentery. The primary goal in the surgical treatment of embolic mesenteric ischemia is to restore arterial perfusion with removal of the embolus from the vessel. This is one by performing a Fogarty embolectomy using a transverse arteriotomy (longituinal arteriotomy will cause stenosis upon closure) (D). It is important to avoi resecting bowel until perfusion has been restore; that way, bowel viability can be better establishe. After restoration of SMA ow, an assessment of the intestinal viability is mae, an nonviable bowel is resecte. Because the amount of bowel resecte can be extensive an this places the patient at risk of short bowel synrome, bowel that is of borerline viability shoul be left in place with a planne secon-look proceure performe 4 to 48 hours later to reassess whether aitional bowel resection is neee (E). Low-ose opamine leas to vasoilatation of mesenteric arteries; however, its benets are unclear (C). Intraoperative angiography will not provie any aitional information that woul assist in the surgical management of SMA embolus (A).

11. B. The most common cause of mesenteric ischemia is

a cariac embolus to the SMA. The SMA provies bloo to the bowel from the ligament of Treitz to the mi transverse colon. Cariac embolus tens to loge just past the SMA origin at a point where the artery begins to narrow, which is just beyon the rst jejunal branches. These patients often have sparing of the proximal jejunum an transverse colon because the mile colic artery remains patent. Thrombosis of the SMA, conversely, is usually cause by unerlying atherosclerotic isease that occurs at the SMA origin an woul thus not spare the proximal jejunum (A). Mesenteric venous thrombosis an nonocclusive mesenteric ischemia woul more likely cause patchy areas of ischemia (C–E). Reference: Elrup-Jorgensen J, Hawkins RE, Breenberg CE. Abominal vascular catastrophes. Surg Clin North Am. 1997;77(6):1305–130.

12. E. The timing of CEA after a stroke is controversial. A elay in surgery increases the risk of recurrent stroke. The risk is highest within the rst month. Conversely, operating too early (within 4 hours) creates a potential risk of a reperfusion injury, particularly if a large infarction is present on compute tomography (CT) an if hypertension cannot be controlle postoperatively. Intracranial bleeing is thought to occur because of altere autoregulation an hyperperfusion of ischemic tissue. In the North American Symptomatic Caroti Enarterectomy Trial (NASCET), however, postoperative intracranial hemorrhage occurre in only 0.% of patients. Until recently, CEA was routinely elaye for 4 to

CHAPtEr 17 Vascular—Arterial 6 weeks after a stroke. Subsequent analysis of the NASCET showe that patients with a stable, nonisabling acute stroke, a normal CT scan, an a normal level of consciousness can safely unergo CEA shortly after the iagnosis is mae, the symptoms have stabilize, an preoperative risk assessment is complete. Thus, the operation is not urgent (D). Delaying the surgery for 6 weeks or more eliminates much of the benet of CEA because the risk of recurrent stroke is greatest early on (B, C). Current treatment guielines from the American Acaemy of Neurology an from the American Stroke Association/American Heart Association recommen that CEA for patients with nonisabling strokes shoul preferably be performe within  weeks of the primary stroke. Patients with a large stroke on CT scan or those with a miline shift may be at higher risk of reperfusion injury, particularly if they have a epresse level of consciousness. Operation shoul be elaye until these patients improve an plateau in their clinical recovery, which is usually in the range of 4 to 6 weeks. If the stroke is completely isabling (A), there remains little if any motor cortex to protect from future stroke, so CEA is not inicate. Thus, patients with severe neurologic ecits, without meaningful recovery or with marke alteration of consciousness, are not caniates for CEA because the goal of CEA is to prevent further amage to the ipsilateral motor cortex. References: Henerson RD, Eliasziw M, Fox AJ, Rothwell PM, Barnett HJ. Angiographically ene collateral circulation an risk of stroke in patients with severe caroti artery stenosis. North American Symptomatic Caroti Enarterectomy Trial (NASCET) Group. Stroke. 000;31(1):18–13. North American Symptomatic Caroti Enarterectomy Trial Collaborators, Barnett HJM, Taylor DW, et al. Benecial effect of caroti enarterectomy in symptomatic patients with high-grae caroti stenosis. N Engl J Med. 1991;35(7):445–453. Sacco RL, Aams R, Albers G, et al. Guielines for prevention of stroke in patients with ischemic stroke or transient ischemic attack: a statement for healthcare professionals from the American Heart Association/American Stroke Association Council on Stroke: co-sponsore by the Council on Cariovascular Raiology an Intervention: the American Acaemy of Neurology afrms the value of this guieline. Circ. 006;113(10):e409–e449.

13. E. The ACAS ranomize patients with asymptomatic

caroti artery stenosis of 60% to 99% to either CEA an aspirin or aspirin alone (C). The stuy was interrupte because of a signicant benet ientie in patients unergoing CEA. A relative reuction in stroke rate by 50%, from 11% to 5% at 5 years, was observe in patients unergoing CEA (A). The Asymptomatic Caroti Surgery Trial conrme the ACAS nings that in patients with 60% to 99% stenosis, the net 5-year risk was 6.4% for all strokes or eath in patients unergoing CEA, versus 11.8% in those not unergoing surgery. This was a net absolute gain of 5.4% (relative risk reuction, 46%). The trial also showe that patients who unerwent CEA were much less likely to have a fatal or isabling stroke (3.5% in the surgery group versus 6.1% in the no-surgery group). The stuies have foun that there is less or no benet in women (E). The greatest benet was in men younger than 75 years of age. CEA for asymptomatic stenosis will only benet the group as a whole if the combine stroke an eath rate is less than 3% (B). Keeping this combine enpoint low is epenent on both patient risk an surgeon skill (C). There is no benet to CEA once the ICA

257

is completely occlue (100%) (D). There is no further ow in the artery, thus the embolic risk is eliminate. The benet of aggressive meical management (incluing antiplatelet agents) is that it can also be protective from coronary events. The biggest limitation of ACAS is that it i not inclue the use of a statin, which, in aition to its lipi-lowering response, also has pleiotropic effects such as plaque stability, which may prove to be a more important contributor in preventing the progression to stroke in caroti isease. The Aggressive Meical Treatment Evaluation for Asymptomatic Caroti Artery Stenosis (AMTEC) trial attempte to compare moern meical management with CEA, but the stuy was prematurely terminate an the results are not yet available. Newer stuies are neee to etermine if moern meical therapy continues to be inferior to surgical intervention in patients with caroti isease. Some authors have suggeste that we shift away from using ecrease luminal caliber as our primary eterminant of choosing which asymptomatic patients to offer surgery. Newer methos of ientifying high-risk patients such as those with plaque ulceration an instability shoul be stuie to either replace or supplement existing societal guielines. References: Enarterectomy for asymptomatic caroti artery stenosis. Executive Committee for the Asymptomatic Caroti Atherosclerosis Stuy. JAMA. 1995;73(18):141–148. Halliay A, Mansel A, Marro J, et al. Prevention of isabling an fatal strokes by successful caroti enarterectomy in patients without recent neurological symptoms: ranomise controlle trial. Lancet. 004;363(940):1491–150. Kolos I, Loukianov M, Dupik N, Boytsov S, Deev A. Optimal meical treatment versus caroti enarterectomy: the rationale an esign of the Aggressive Meical Treatment Evaluation for Asymptomatic Caroti Artery Stenosis (AMTEC) stuy. Int J Stroke. 015;10():69–74. Weyer GW, Davis AM. Screening for asymptomatic caroti artery stenosis. JAMA. 015;313():19–193.

14. D. The rst NASCET stuy foun that CEA was of

benet for symptomatic severe ICA stenosis (70%–99%). A symptomatic caroti artery stenosis was ene as a nonisabling stroke, a hemispheric transient ischemic attack, or a retinal symptom (amaurosis fugax). Life-table estimates of the cumulative risk of any ipsilateral stroke at  years were 6% in the aspirin group an 9% in the aspirin an CEA group. In the secon NASCET stuy, there was no benet for symptomatic patients with less than 50% stenosis (E). For symptomatic patients with stenosis from 50% to 69%, there was a very moest benet: 5-year risk of ipsilateral stroke was 15.7% in the CEA group an .% in the meical group (P = 0.04). The benet was greatest in men, in those with hemispheric symptoms (as oppose to retinal ones), an with recent stroke. Women appeare to have less risk of stroke an also ha higher perioperative mortality than men. ACAS emonstrate the benet of CEA compare with aspirin for asymptomatic ICA stenosis of 60% to 99%. However, the benet is much less than for symptomatic high-grae stenosis. Thus in this question, choice A woul be benecial but of less benet than choice D (symptomatic). Choice B woul be of no benet because the stenosis is moerate, an the symptoms are on the wrong sie (retinal is ipsilateral). In choice C, the symptoms are also on the wrong sie with respect to the stenosis.

258

PArt i Patient Care

15. A. New neurologic ecits that present within the

rst 1 hours of operation are almost always the result of thromboembolic phenomena stemming from the CEA site. Possibilities inclue the evelopment of thrombus on the enarterectomize arterial surface, a resiual intimal ap in the ICA leaing to occlusion, or a resiual ap in the external caroti artery (ECA) leaing to ECA thrombosis an retrograe embolization of the clot into the ICA. Immeiate heparinization an exploration are inicate without the nee for conrmatory arteriography or noninvasive tests. On reexploring the woun, the ECA an ICA shoul be palpate for the presence of a pulse. If there is no pulse, this inicates thrombosis, an initial on-table arteriography is not necessary. The artery shoul be reopene an inspecte to look for a cause of the thrombosis. Before closing the arteriotomy, care shoul be taken to ensure that there is goo back-bleeing from the ICA. Fogarty balloon embolectomy of the cephala ICA shoul be avoie because this can lea to a caroti-cavernous sinus stula. The arteriotomy shoul then be reclose with a patch. On-table arteriography shoul then be performe to ensure that the istal ICA is patent an to etermine whether there is an embolus in the mile cerebral artery. If an embolus is present in the intracranial caroti or mile cerebral artery, local infusion of a lytic agent shoul be consiere (B). If on reopening the woun, an excellent pulse is present in the ICA an ECA, with normal signals on han-hel Doppler ultrasonography, on-table arteriography is performe (C, D). If arteriography reveals an intimal ap or irregular mural thrombus at the enarterectomy site, then reopening of the vessel is inicate. Neurologic ecits that evelop 1 to 4 hours after the operation are usually ue to thromboembolic phenomena stemming from the CEA site but may also be cause by a postoperative hyperperfusion synrome. These latter conitions may be worsene by immeiate heparinization an reexploration. Therefore, ecits occurring 1 to 4 hours after the operation shoul be promptly investigate with hea CT an CT arteriography (E).

16. C. The incience of hyperperfusion synrome after a

CEA is reportely 0.3% to 1%. It is thought to occur as a result of impaire autoregulation of cerebral bloo ow an oes not nee to be taken back to the OR (E). The thought is that longstaning, severe caroti stenosis leas to hypoperfusion, leaing to a compensatory ilation of cerebral vessels istal to the stenosis as part of the normal autoregulatory response to maintain aequate cerebral bloo ow. After CEA restores normal pressure, however, autoregulation is impaire an oes not immeiately ajust to the suen increase in bloo ow. Risk factors associate with cerebral hyperperfusion inclue recent stroke, surgery for very tight ICA stenosis, concomitant contralateral ICA occlusion, evience of chronic ipsilateral hypoperfusion, stage bilateral CEA performe within  months of each other, an poorly controlle pre- an postoperative hypertension. Pathologic changes range from mil cerebral eema an petechial hemorrhage to severe intracerebral hemorrhage an eath, particularly if not promptly treate (B). The synrome is herale by an ipsilateral frontal heaache, most commonly occurring at a meian of the fth postoperative ay (A). By that time, the patient is alreay at home. Thus, it is imperative to warn patients of this rare synrome an ieally have the patient check his or

her bloo pressure aily for the rst week postoperatively. The heaache may be followe by focal motor seizures that are often ifcult to control. Management consists of controlling bloo pressure, ieally with a beta-blocker, with the avoiance of vasoilators (as these may increase cerebral bloo ow), an use of antiseizure meications (D). Reference: Schroeer T, Sillesen H, Sørensen O, Engell HC. Cerebral hyperperfusion following caroti enarterectomy. J Neurosurg. 1987;66(6):84–89.

17. D. This patient has Takayasu arteritis, an inammatory

isease of the aorta an its branches, as well as the coronary an pulmonary arteries (A–C, E). It occurs most commonly in young women, with a meian age of 5 years. The clinical course has been escribe as beginning with constitutional symptoms such as fever an malaise. However, a National Institutes of Health stuy showe that only one-thir of patients recall such symptoms. Characteristic clinical features inclue hypertension, retinopathy, aortic regurgitation, cerebrovascular symptoms, angina, congestive heart failure, abominal pain or gastrointestinal bleeing, pulmonary hypertension, an extremity clauication. The gol stanar for iagnosis is arterial imaging, with the emonstration of occlusive isease in the subclavian arteries. Unlike atherosclerosis, which tens to affect the origin of these vessels, Takayasu arteritis affects the miportions of these arteries. Characteristic signs an symptoms inclue pulselessness or bloo pressure ifferential in the arms, upper or lower extremity clauication, syncope, amaurosis fugax, blurre vision, an palpitations. Treatment initially consists of steroi therapy with the aition of cytotoxic agents use in patients who o not achieve remission. Carotiynia, which is pain along iname arteries, is pathognomonic for Takayasu arteritis. Surgical treatment with arterial bypass is only performe in avance states an in situations in which the patient oes not respon to meical therapy. It shoul ieally be performe when the isease is not active. Because the isease causes transmural arterial inammation with concentric brosis, there is no role for enarterectomy, an angioplasty has not been met with goo results.

18. A. The most common mechanisms of blunt caroti

injury inclue motor vehicle accients, st ghts, hanging, an intraoral trauma. However, it has also been reporte with relatively minor trauma, such as after chiropractic manipulation of the neck an forceful sneezing. Bif et al. have grae blunt caroti injury as follows: grae I: luminal irregularity or issection with less than 5% luminal narrowing; grae II: issection or intramural hematoma with greater than or equal 5% luminal narrowing; grae III: pseuoaneurysm; grae IV: occlusion; grae V: transection with free extravasation. Horner synrome (oculosympathetic paresis) is common with this injury an is thought to be relate to the involvement of the internal part of the pericaroti sympathetic plexus (B). The ecision to perform surgery is base on (1) injury severity, () presence or absence of symptoms, an (3) surgical accessibility of the lesion (C). In general, there is little role for surgical intervention in patients with grae I or II blunt caroti injury as in this patient (E). Antiplatelet therapy with aspirin is the best treatment option. However, some trauma centers chose to use subtherapeutic heparin initially in case patients may require a surgery. Minor (intimal)

CHAPtEr 17 Vascular—Arterial injuries ten to heal themselves (D). Pseuoaneurysms typically o not an are a relative inication for surgery if accessible in the neck. References: Bif WL, Moore EE, Offner PJ, Brega KE, Franciose RJ, Burch JM. Blunt caroti arterial injuries: implications of a new graing scale. J Trauma. 1999;47(5):845–853. Bromberg WJ, Collier BC, Diebel LN, et al. Blunt cerebrovascular injury practice management guielines: the Eastern Association for the Surgery of Trauma. J Trauma. 010;68():471–477.

19. E. The patient has a symptomatic high-grae caroti

stenosis, an, as such, an intervention is inicate. With the history of raiation therapy an neck issection, the patient has what is terme a “hostile neck.” This increases the risk of caroti enarterectomy, in terms of cranial nerve injury an woun healing. The previous neck issection results in a paucity of tissue coverage between the skin an the caroti artery. This can lea to the catastrophic complication of caroti blow out. The best alternative in this patient woul be to perform caroti stenting with a cerebral protection evice (A–D). Patients with asymptomatic ICA stenosis in the 50% to 69% range shoul be starte on meical therapy with an antiplatelet agent (for all patients), antihypertensive agent (if they have hypertension), an the use of a high-intensity statin irrespective of lipi levels (ue to the pleiotropic effect of plaque stabalization). Reference: Harro-Kim P, Kakhoayan Y, Dereyn CP, Cross DT 3r, Moran CJ. Outcomes of caroti angioplasty an stenting for raiation-associate stenosis. AJNR Am J Neuroradiol. 005;6(7):1781–1788.

20. C. Recurrent caroti stenosis can occur after CEA. The

risk of more than 50% restenosis is 5.8%, 9.9%, 13.9%, an 3.4% at 1, 3, 5, an 10 years, respectively. However, severe (>80%) stenosis evelops in only .1% of patients. Early (within 4 weeks) restenosis is usually ue to a technical error. Recurrent caroti stenosis occurring beyon 1 month but within the rst  years after CEA is usually seconary to myointimal hyperplasia. This type of stenosis tens to have a benign course (the lesion is smooth an less prone to embolization), with a low risk of recurrent stroke. In aition, reoperative CEA carries a higher risk of cranial nerve injury (7.3% rate of permanent injury in one series) (A). The patient is asymptomatic. If the patient ha a symptomatic recurrence, the best option woul be caroti stenting (B, D–E). When the recurrent stenosis evelops  or more years after CEA, recurrent atherosclerosis is the usual cause.

21. D. Suen occlusion of the ICA in a young patient is

highly suggestive of a spontaneous issection. This is further supporte by the tapere occlusion seen on imaging (escribe as “ame-shape”). On the other han, occlusion ue to atherosclerosis typically occurs ush with the common caroti, an in oler patients ICA issection may occur either spontaneously or after trauma. Cervical artery issection is a signicant cause of stroke in patients younger than 40 years. Common presenting symptoms of ICA issection are heaache, transient ischemic attack an/or stroke, an Horner synrome (ptosis, miosis, anhyrosis). Risk factors for issection inclue history of infection (syphilis), smoking, Ehlers-Danlos synrome type IV, cystic meial necrosis, Marfan synrome, family history, oral contraceptives, an atherosclerosis. In a young female, bromuscular ysplasia

259

woul be high on the ifferential. The iagnosis is mae by uplex scan an/or CT angiography. Duplex scan may be iagnostic, if it emonstrates a membrane within the lumen, consistent with a issection. The most likely mechanism of acute issection is an intimal tear followe by an acute intimal issection, which prouces luminal occlusion ue to seconary thrombosis. The occlusion angiographically is typically  to 3 cm beyon the bifurcation. Autopsy stuies have shown a sharply emarcate transition between the normal caroti artery an the issecte segment. Treatment is with anticoagulation an, in most cases, results in complete resolution within a few months. Stenting may be an option in symptomatic patients in the absence of occlusion (C). CEA, Fogarty embolectomy, or lytic therapy is not appropriate for a spontaneous issection (A, B, E).

22. E. Thromboangiitis obliterans (Buerger isease) is a

progressive nonatherosclerotic segmental inammatory isease that most often affects small- to meium-size arteries, veins, an nerves of the upper an lower extremities (C). The typical age at onset is 0 to 50 years, an the isorer is more common in men who smoke. The isease also affects the veins, an specically the upper extremities may be affecte by a migratory supercial thrombophlebitis. Patients initially present with foot, leg, arm, or han clauication. Progression of the isease leas to ischemic rest pain an ulcerations of the toes, feet, an ngers. Characteristic angiographic nings may show isease connement to the istal circulation, usually infrapopliteal an istal to the brachial artery. The occlusions are segmental an show skip lesions with extensive collateralization, the so-calle corkscrew collaterals. The iagnosis is ifcult to establish an is a iagnosis of exclusion because there are no pathognomonic features. As such, the isease can be confuse with chronic embolization an other iseases. Several criteria have been establishe to conrm the iagnosis: age younger than 45 years; current (or recent) smoker; istal extremity ischemia (clauication, pain at rest, ischemic ulcers, gangrene); exclusion of autoimmune iseases, hypercoagulable states, an iabetes mellitus; exclusion of a proximal source of emboli by echocariography an arteriography; an characteristic arteriographic nings in the involve limbs. The aortoiliac segments are typically spare, as are the coronary arteries (A, B). The mainstay of treatment revolves aroun smoking cessation. In patients who are able to abstain, isease remission is impressive an amputation avoiance is increase. The role of surgical intervention is minimal because there is usually no acceptable target vessel for bypass (D). Sympathectomy may result in mil improvement of symptoms. Reference: Olin JW. Thromboangiitis obliterans (Buerger’s isease). N Engl J Med. 000;343(1):864–869.

23. C. The ABI normally varies between 1 an 1. because

the ankle pressure in the supine position can be as much as 0% higher than in the arm (A). Peripheral arterial isease has been ene as a value less than 0.9 an inicates some egree of stenosis. Patients with clauication typically have an ABI between 0.5 an 0.7, an those with rest pain have an ABI less than 0.4 (B). Patients with iabetes an enstage renal isease are at risk of eveloping calcication of the arterial meial layer, known as meial calcinosis, or

260

PArt i Patient Care

Mönckeberg arteriosclerosis. This process makes bloo vessels rigi an ifcult to compress, causing falsely increase pressure reaings. The process tens to affect tibial vessels primarily an spares igital vessels in the toes. As such, toe pressures are more reliable, as are other measures of istal perfusion such as transmetatarsal pulse volume recorings an transcutaneous oximetry (D, E). Reference: Belkin M, Whittemore A, Donalson M, et al. Peripheral arterial occlusive isease. In: Townsen CM, Jr, Beauchamp RD, Evers BM, Mattox KL, es. Sabiston textbook of surgery: the biological basis of modern surgical practice. 17th e. Philaelphia: W.B. Sauners; 004:199.

24. B. Absolute contrainications to thrombolytic therapy

inclue recent stroke or transient ischemic attack, active or recent bleeing, an signicant coagulopathy. Relative contrainications inclue patients with recent major surgery (within  weeks, an greatest with recent neurosurgery or eye surgery), recent trauma, uncontrolle hypertension, intracranial tumors, an pregnancy (A). Thrombolytic therapy is most effective in patients with ischemia of less than  weeks’ uration (D). The risk of bleeing with thrombolytic therapy is increase with the longer uration of therapy an with ecreasing brinogen levels. In most series, thrombolytic therapy is use for as long as 48 hours, at which point the bleeing risk increases signicantly (E). The causes of acute limb ischemia can be ivie into embolic an thrombotic. The heart is the most common source of emboli leaing to acute ischemia, most often in the setting of atrial brillation. Other cariac sources inclue mural thrombus after an acute myocarial infarction, valvular isease, an atrial myxoma. Other sources of emboli inclue arterial aneurysms an atherosclerotic plaques. Thrombosis is most often cause by unerlying atherosclerosis in the peripheral arteries, an these patients typically have a history of clauication. The severity of acute limb ischemia is base primarily on the motor an sensory examination. Patients shoul be place in four categories: class 1 (nonthreatene) has normal motor an sensory function; class  (threatene) inclues  a—sensory ecit only an b—(immeiately threatene) both motor an sensory ecit; an class 3 inicates irreversible complete motor an sensory loss. In aition, consieration shoul be given to the uration of ischemia. As a general rule, patients with class 1 ischemia can be treate with multiple options, a trial of heparin alone, thrombolytic therapy, or operative embolectomy/ bypass. Patients with class  ischemia nee prompt restoration of bloo ow, so heparin alone is not acceptable. With class b ischemia, the threat of limb loss is more immeiate. Since thrombolytic therapy may require more than 4 to 48 hours to restore ow, class b ischemia (motor an sensory ecit) is a relative contrainication to thrombolysis (C). Such a patient shoul be taken to the operating room. Category 3 ischemia is consiere irreversible an requires amputation. Irreversible ischemia is conrme by an absence of arterial or venous Doppler signals, uration of ischemia of more than 6 to 8 hours, presence of mottling of the skin, absence of capillary rell, an complete anesthesia an paralysis. References: Norgren L, Hiatt WR, Dormany JA, et al. Inter-society consensus for the management of peripheral arterial isease (TASC II). J Vasc Surg. 007;45 Suppl S:S5–S67. Semba CP, Murphy TP, Bakal CW, Calis KA, Matalon TA. Thrombolytic therapy with use of alteplase (rt-PA) in peripheral arterial

occlusive isease: review of the clinical literature. The Avisory Panel. J Vasc Interv Radiol. 000;11():149–161. Results of a prospective ranomize trial evaluating surgery versus thrombolysis for ischemia of the lower extremity. The STILE trial: The STILE investigators (appenix A). Ann Surg. 1994;0(3):51–68.

25. C. The ipsilateral greater saphenous vein is the conuit

of choice for lower extremity istal bypass for peripheral arterial isease (contralateral vein for trauma). An ieal conuit shoul be a minimum of 3 mm (but ieally 4 mm). When the greater saphenous vein is not available, options inclue the lesser saphenous an cephalic veins. Ectopic veins (i.e., lesser saphenous, arm veins) are generally inferior to a single-segment saphenous vein, although they are still superior to the performance of synthetic grafts. A composite graft, which is a vein graft sewn to a polytetrauoroethylene graft, has a patency rate similar to that of a prosthetic graft an tens to evelop neointimal hyperplasia. The bypass shoul be as short as possible (proximal inow from the most istal normal artery (in this case, popliteal), an istal outow to where the artery reconstitutes most proximally (in this case, above the ankle). Options A an B are suboptimal because it involves a longer bypass than is necessary given the patent femoral artery an normal popliteal pulse an harvesting contralateral vein. Enovascular approaches (such as angioplasty) are options but are less urable, particularly in the presence of a long segment of occlusion. However, in a relatively healthy patient, with a goo saphenous vein an goo runoff into the foot, a bypass is likely the better option (D). Amputation of the toe is unlikely to heal in the absence of a palpable peal pulse an such a low ABI (E). Reference: Gentile AT, Lee RW, Moneta GL, Taylor LM, Ewars JM, Porter JM. Results of bypass to the popliteal an tibial arteries with alternative sources of autogenous vein. J Vasc Surg. 1996;3():7–79.

26. E. This is Raynau isease. First escribe in 186 by

Maurice Raynau, the term Raynau isease applies to a heterogeneous symptom array associate with peripheral vasospasm, more commonly occurring in the upper extremities. The characteristically intermittent vasospasm classically follows exposure to various stimuli, incluing col temperatures, tobacco, or emotional stress. Formerly, a istinction was mae between Raynau isease an the Raynau phenomenon for escribing a benign isease occurring in isolation or a more severe isease seconary to another unerlying isorer, respectively. However, collagen vascular isorers evelop in many patients at some point after the onset of vasospastic symptoms; the rate of progression to a connective tissue isorer ranges from 11% to 65% in reporte series. Characteristic color changes occur in response to the arteriolar vasospasm, ranging from intense pallor to cyanosis to reness as the vasospasm occurs. The igital vessels then relax, eventually leaing to reactive hyperemia. The majority of patients are women younger than 40 years of age. As many as 70% to 90% of reporte patients are women, although many patients with only mil symptoms may never present for treatment. Geographic regions locate in cooler, amp climates such as the Pacic Northwest an Scaninavian countries have a higher reporte prevalence of the isease. Certain occupational groups, such as those that use vibrating tools, may be more preispose to Raynau isease or

CHAPtEr 17 Vascular—Arterial igital ischemia. The exact pathophysiologic mechanism behin the evelopment of such severe vasospasm remains elusive, an much attention has focuse on increase levels of α-arenergic receptors an their hypersensitivity in patients with Raynau isease, as well as abnormalities in the thermoregulatory response, which is governe by the sympathetic nervous system. There is no cure for Raynau isease; thus, all treatments mainly palliate symptoms an ecrease the severity an perhaps frequency of attacks. Conservative measures preominate, incluing the wearing of gloves, use of electric or chemically activate han warmers, avoiing occupational exposure to vibratory tools, abstinence from tobacco, an relocating to a warmer, rier climate. The majority (90%) of patients will respon to avoiance of col an other stimuli. The remaining 10% of patients with more persistent or severe synromes can be treate with a variety of vasoilatory rugs, albeit with only a 30% to 60% response rate. Calcium channel blocking agents such as iltiazem an nifeipine are the rugs of choice. The selective serotonin reuptake inhibitor uoxetine has been shown to reuce the frequency an uration of vasospastic episoes but is not the rst-line treatment (C). Intravenous infusions of prostaglanins have been reserve for nonresponers with severe symptoms (B). Upper extremity sympathectomy may provie relief in 60% to 70% of patients; however, the results are short-live, with a graual recurrence of symptoms in 60% within 10 years (A). Cervical sympathectomy has fallen out of favor an has been replace by localize igital sympathectomy using microsurgery. This involves stripping the aventitia of igital arteries an thus removing sympathetic bers. A col stimulation test or nail fol capillaroscopy may be use to conrm the iagnosis of Raynau isease, but there is no role for arteriography (D).

27. D. Pseuoaneurysms can manifest with pain, a pulsa-

tile mass, an/or compression of ajacent structures. Large, expaning, painful pseuoaneurysms are at signicant risk of rupture an shoul be repaire urgently. Smaller, stable pseuoaneurysms may be observe (E). Duplex ultrasonography has been the iagnostic proceure of choice because it helps ene size, morphology, an location. Pseuoaneurysms less than  cm in iameter have a higher likelihoo of spontaneous thrombosis with compression therapy, whereas larger ones an those in patients receiving anticoagulation therapy are likely to persist. However, given the reporte high failure rates with ultrasoun compression (A, B), ultrasonography-guie thrombin injection is the best treatment option an is the treatment of choice. Surgery is reserve for infecte or rapily expaning pseuoaneurysms (C). Reference: Wixon CL, Philpott JM, Bogey WM Jr, Powell CS. Duplex-irecte thrombin injection as a metho to treat femoral artery pseuoaneurysms. J Am Coll Surg. 1998;187(4):464–466.

28. A. Colonic ischemia is a recognize complication after

AAA repair, whether open or enovascular. It occurs in approximately 1% to 3% of cases. It is thought to be ue to either ligation of the inferior mesenteric artery (IMA) or ligation or exclusion of internal iliac arteries. The most common presentations inclue an unexpectely early return of bowel function manifeste by iarrhea, left lower quarant pain, abominal istention, persistent leukocytosis, elevate white

261

bloo cell count, an lactic aciosis. Diagnosis is conrme by exible proctosigmoioscopy, which reveals a friable mucosa. Proctosigmoioscopy may not be able to accurately istinguish partial ischemia from full-thickness necrosis. Initial management is meical an consists of nasogastric tube ecompression, IV hyration, placing the patient on NPO, an broa-spectrum antibiotics. Full-thickness necrosis of the colon shoul be suspecte in patients with evience of peritonitis or unremitting aciosis. In such cases, laparotomy with colonic resection an colostomy is inicate (C, D). The mortality rate after emergent colectomy approaches 50%. Arteriography woul not typically be helpful because the usual cause is an intene ligation or exclusion of an internal iliac artery or IMA (B, E). Reference: Becquemin JP, Majewski M, Fermani N, et al. Colon ischemia following abominal aortic aneurysm repair in the era of enovascular abominal aortic repair. J Vasc Surg. 008;47():58–63.

29. B. The presentation is consistent with a rupture AAA.

If the patient was hemoynamically unstable, he shoul be taken irectly to the operating room (D). If the patient is relatively stable (as in this case), a CT scan is preferre to conrm the presence of a rupture AAA an etermine feasibility of enovascular repair, provie there is a coorinate multiisciplinary rupture aneurysm team that has immeiate enovascular capabilities. Although most surgeons woul approach a rupture AAA via the enovascular approach, recent Cochrane analysis emonstrate no ifference in 30-ay mortality for patients with rupture AAA that were treate with an enovascular approach compare to an open approach. Although ultrasonography is useful for etermining the presence of an AAA, it is not accurate for etermining the presence of a retroperitoneal rupture (C). Ultrasonography woul be reasonable to perform in this patient was unstable, an no pulsatile mass coul be felt on physical examination, so as to conrm that an aneurysm was present. Once in the operating room, the patient shoul be preppe an rape before anesthesia inuction because the anesthesia may inuce a precipitous ecrease in bloo pressure (E). Because of the large retroperitoneal hematoma that is typically foun, proximal control is best achieve by clamping the aorta at the iaphragm. Most surgeons woul recommen a policy of “permissive hypotension” en route to the operating room. Excessive ui aministration an elevation of the bloo pressure may further exacerbate bleeing (A). References: Bager SA, Harkin DW, Blair PH, Ellis PK, Kee F, Forster R. Enovascular repair or open repair for rupture abominal aortic aneurysm: a Cochrane systematic review. BMJ Open. 016;6():e008391. Lee WA, Hirneise CM, Tayyarah M, Huber TS, Seeger JM. Impact of enovascular repair on early outcomes of rupture abominal aortic aneurysms. J Vasc Surg. 004;40():11–15. Van Der Vliet JA, Van Aalst DL, Schultze Kool LJ. Hypotensive hemostasis (permissive hypotension) for rupture abominal aortic aneurysm: are we really in control? Vascular. 007;15(4):197–00.

30. C. Popliteal aneurysms are the most common periph-

eral artery aneurysms (overall, aortic an iliac aneurysms are more common). They can be suspecte on physical examination. They are bilateral in 50% of patients. Patients who are foun to have a popliteal aneurysm shoul unergo

262

PArt i Patient Care

screening for an AAA because 30% will have a concomitant AAA. The most frequent complication of popliteal aneurysms is leg ischemia ue to thrombosis an embolization from the aneurysm. Guielines for repair are controversial. Some authors recommen repair for all popliteal aneurysms. Most woul agree that inications for repair are (1) all aneurysms larger than  cm, () aneurysms with intraluminal thrombus, regarless of size, or (3) those that are symptomatic or have evience of previous embolization (A, B). Diagnosis is mae by uplex ultrasonography, which can measure the aneurysm size an etect the presence of thrombus. Arteriography assists in operative planning but shoul not be use for iagnosis because it oes not etect the thrombus nor accurately measure the size. The surgical approach to the popliteal artery is either via the meial approach or the posterior approach (E). The posterior approach is ieal if the aneurysm is just behin the knee joint. Magnetic resonance imaging an CT angiography can be use as alternatives for operative planning. The stanar operative approach involves bypassing the aneurysm with saphenous vein an interval ligation of the popliteal artery. With this approach, the aneurysm sac is not opene, an as such, there is a small risk of continue aneurysm expansion an compression of ajacent structures. Formal enoaneurysmorrhaphy, as is one with an open AAA repair, is another alternative. In the setting of acute thrombosis, lytic therapy is the initial treatment of choice. Enovascular stent grafting is another option, especially if no suitable vein is available in a high risk patient, but this may have a lower primary patency (D). References: Ascher E, Markevich N, Schutzer RW, Kallakuri S, Jacob T, Hingorani AP. Small popliteal artery aneurysms: are they clinically signicant? J Vasc Surg. 003;37(4):755–760. Lowell RC, Gloviczki P, Hallett JW Jr, et al. Popliteal artery aneurysms: the risk of nonoperative management. Ann Vasc Surg. 1994;8(1):14–3.

31. E. A patient with an upper gastrointestinal blee an a

history of aortic surgery shoul be presume to have an aortoenteric stula until proven otherwise. The treatment algorithm epens on the hemoynamic stability of the patient. If the patient is unable to be stabilize ue to massive hemorrhage, the patient shoul be taken emergently to the operating room, even if a iagnosis has not yet been establishe. Oftentimes, the patient will have a so-calle heral blee, after which the bleeing may temporarily stop, allowing a workup for an aortoenteric stula. The iagnosis can be ifcult to establish. Upper enoscopy is negative surprisingly often an has a low sensitivity but shoul be the rst step in the workup. Duoenal graft erosion typically occurs at the fourth portion of the uoenum, an nings may be subtle, such as mil mucosal erosion. CT scan is highly useful, as in the presence of an aortoenteric stula will likely emonstrate perigraft ui, air, or inammation, inicative of a graft infection (though less likely contrast extravasation). Flui an inammatory changes aroun a graft woul be abnormal nings beyon 6 weeks after surgery (A). If the CT scan nings are negative, a nuclear-tagge white bloo cell scan may be useful for establishing a graft infection (C). Arteriography is of limite benet for the iagnosis of vascular graft infections but can be useful in preoperative planning (B). In some instances, no source of an upper gastrointestinal

blee is foun, an thus one must empirically procee to graft excision (D). The classic operative management consiste of obtaining proximal aortic control of the aorta at the iaphragm, graft excision, closure of the aortic stump in two layers, closure of the uoenum, placing omentum in the area of the aortic stump closure, followe by an extra anatomic axillobifemoral bypass. Recently, the more accepte treatment is excision of the aortic graft an in situ placement of a human aortic homograft. Reference: Berger P, Moll FL. Aortic graft infections: is there still a role for axillobifemoral reconstruction? Semin Vasc Surg. 011;4(4):05–10.

32. C. Common iliac aneurysms are usually iagnose inci-

entally. In most cases, they are foun in association with an aortic aneurysm. Rare presentation inclues the evelopment of a stula with the ajacent iliac vein or compression of the iliac vein. The natural history of common iliac aneurysms is less well ene. In a recent stuy, the expansion rate of common iliac aneurysms was 0.9 cm per year, an hypertension preicte faster expansion. Because no rupture of a common iliac aneurysm smaller than 3.8 cm was observe, the recommene threshol for elective repair of asymptomatic patients was larger than 3.5 cm (A, B, D, E). Treatment options inclue open surgical replacement with prosthetic graft or enovascular stent grafting. In patients with suitable anatomy, namely, the presence of proximal an istal laning zones, stent grafting has become the treatment of choice. Enovascular repair is associate with fewer complications overall but poses a higher risk of creating buttock clauication ue to occlusion of the internal iliac artery. Reference: Huang Y, Gloviczki P, Duncan AA, et al. Common iliac artery aneurysm: expansion rate an results of open surgical an enovascular repair. J Vasc Surg. 008;47(6):103–110.

33. B. Popliteal aneurysms rarely rupture (A). Most com-

monly, they cause acute or chronic ischemia. In most series, the most common symptom is thrombosis, in as many as 49%, followe by istal embolization. As the aneurysm continues to grow, less commonly, it can compress ajacent structures, such as the popliteal vein (D, E). Chronic embolization can lea to occlusions of the infrapopliteal vessels an can complicate revascularization (C). If they present with acute ischemia, thrombolysis is the intervention of choice, followe by operative repair. Recently, enovascular stent grafting has been use, although long-term patency ata are still lacking. References: Dorigo W, Pulli R, Turini F. Acute leg ischemia from thrombose popliteal artery aneurysms: role of preoperative thrombolysis. Eur J Vasc Endovasc Surg. 00;3(3):51–54. Shortell CK, DeWeese JA, Ouriel K, Green RM. Popliteal artery aneurysms: a 5-year surgical experience. J Vasc Surg. 1991;14(6):771–776.

34. C. Recent stuies have shown that AAAs as large as

5.5 cm in iameter can be safely observe (D). Another recent ranomize stuy inicate that although the perioperative mortality rate of EVAR is lower than that of open repair, long-term mortality is the same (C). Women have been shown to have higher perioperative mortality rates than men with either EVAR or open repair. EVAR shoul not lower the size threshol for repair in a high-cariac risk patient if the AAA has not yet reache the 5.5-cm threshol (B). Following are the guielines for treatment of AAAs as reporte by a

CHAPtEr 17 Vascular—Arterial subcommittee of the Joint Council of the American Association for Vascular Surgery an Society for Vascular Surgery: 1. The arbitrary setting of a single-threshol iameter for elective AAA repair that is applicable to all patients is not appropriate because the ecision for repair must be iniviualize in each case. 2. Ranomize trials have shown that the risk of rupture of small AAAs is quite low an that a policy of careful surveillance of those with a iameter of as large as 5.5 cm is safe, unless there is rapi expansion (>1 cm/yr) or symptoms evelop. However, early surgery is comparable to surveillance with later surgery, so patient preference is important, especially for AAAs 4.5 to 5.5 cm in iameter. 3. Base on the best available current evience, a iameter of 5.5 cm appears to be an appropriate threshol for repair in an average patient. However, subsets of younger, low-risk patients with a long projecte life expectancy may prefer early repair. If the surgeon’s personal ocumente operative mortality rate is low, repair may be inicate at smaller sizes if that is the patient’s preference. 4. For women or for AAAs with a greater than average rupture risk, 4.5 to 5 cm is an appropriate threshol for elective repair (A). 5. For high-risk patients, elay in repair until the iameter is larger is warrante, especially if enovascular aortic repair is not possible (E). 6. In view of its uncertain long-term urability an effectiveness as well as the increase surveillance buren, EVAR is most appropriate for patients at increase risk of conventional open aneurysm repair. EVAR may be the preferre treatment metho if anatomy is appropriate for oler high-risk patients, those with a hostile abomen, or other clinical circumstances likely to increase the risk of conventional open repair. 7. Use of EVAR in patients with unsuitable anatomy markely increases the risk of averse outcomes, the nee for conversion to open repair, or AAA rupture. 8. At present, there oes not seem to be any justication that EVAR shoul change the accepte size threshol for intervention in most patients. 9. In choosing between open repair an EVAR, patient preference is of great importance. It is essential that the patients be well informe to make such choices. References: Brewster DC, Cronenwett JL, Hallett JW Jr, et al. Guielines for the treatment of abominal aortic aneurysms. Report of a subcommittee of the Joint Council of the American Association for Vascular Surgery an Society for Vascular Surgery. J Vasc Surg. 003;37(5):1106–1117. Mureebe L, Egorova N, McKinsey JF, Kent KC. Gener trens in the repair of rupture abominal aortic aneurysms an outcomes. J Vasc Surg. 010;51(4 Suppl):9 S–13 S.

35. B. Enoleak is a common complication after EVAR

that can lea to aneurysm enlargement an even rupture. Enoleaks occur in as many as 40% of patients after EVAR. Most enoleaks are foun in the immeiate postoperative perio, but late enoleaks also evelop. For this reason, routine lifelong postoperative surveillance with CT scanning is recommene. New enoleaks have been ientie as

263

late as 7 years after EVAR. Enoleaks are classie into ve major types (types I–V) base on the source of communication between the circulation an the aneurysm sac. The most common type of leak after enovascular repair is a type II leak, which results from retrograe lling of the aneurysm sac from the lumbar arteries or the IMA. Management of type II leaks is controversial an is base on whether the aneurysm is enlarging or stable. Options inclue coil embolization of the vessel, laparoscopic ligation, or observation. Type I leaks occur at the stent–graft attachment sites (either at the aorta or at the iliac arteries) (A); type III leaks occur at a stent–stent interface an are also known as moular isassociations (C); type IV leaks are irectly through the graft an are ue to graft material porosity (D). They usually heal spontaneously. The most angerous type of leak is a proximal type I leak because there is a failure to achieve a proximal seal, leaing to continue lling of the aneurysm sac at systemic pressures. Type I leaks require immeiate treatment when iscovere, typically by eploying another stent or, if unsuccessful, by open surgical conversion. Type III enoleaks represent a true mechanical failure of the enograft an require repair with an aitional enograft to eliminate systemic ow an pressure in the aneurysm. Type V leak is also referre to as enotension. This can be consiere iiopathic because the aneurysmal sac may appear to be enlarging without any evience of a leak site on imaging (E). Reference: Corriere MA, Feurer ID, Becker SY, et al. Enoleak following enovascular abominal aortic aneurysm repair: implications for uration of screening. Ann Surg. 004;39(6):800–807.

36. E. This patient most likely has an arterial thrombus seconary to heparin-inuce thrombocytopenia thrombosis (HITT). The classic laboratory ning is a ecrease in the platelet count of more than 50%. Although thrombocytopenia usually increases the risk of bleeing, HITT is paraoxically known to cause a hypercoagulable state; it is the secon most common acquire hypercoagulable state (smoking is the most common). There are two types of HITT with type II being more common an responsible for the clinical synrome. HITT type II is cause by antiboies to platelet-factor 4 an heparin sulfate resulting in a prothrombotic state (will appear as a white clot). It typically occurs 3 to 5 ays after starting heparin. If this is suspecte, heparin shoul be iscontinue, an the patient shoul be starte on a irect thrombin inhibitor. Argatroban is the recommene agent for patients with HITT an renal impairment. Lepiruin an bivaliruin both unergo renal excretion an shoul be avoie in patients with ESRD (A, D). The patient initially ha acute limb ischemia seconary to cariac emboli from atrial brillation. His symptoms resolve with initiation of heparin so it is unlikely that he has unerlying antithrombin III eciency (C). More stuies are neee to evaluate the role of tPA in HITT (B). References: Guzzi LM, McCollum DA, Hursting MJ. Effect of renal function on argatroban therapy in heparin-inuce thrombocytopenia. J Thromb Thrombolysis. 006;(3):169–176. Visentin GP, For SE, Scott JP, Aster RH. Antiboies from patients with heparin-inuce thrombocytopenia/thrombosis are specic for platelet factor 4 complexe with heparin or boun to enothelial cells. J Clin Invest. 1994;93(1):81–88.

Vascular—Venous AMANDA C. PURDY AND JOHN McCALLUM

18

ABSITE 99th Percentile High-Yields I. Deep Vein Thrombosis (DVT) an Pulmonary Embolism (PE) A. Can be provoke (known inciting event, such as recent surgery, malignancy) or unprovoke B. Most common inherite prothrombotic isorer: Factor-V Leien mutation (unable to breakown Factor-V); higher incience on left sie (May-Thurner synrome) C. Catheter-associate upper extremity DVTs—etermine if catheter is require, if it is, can keep catheter an start therapeutic anticoagulation; if not require, start anticoagulation an remove catheter in 3 to 5 ays; in both cases, continue anticoagulation for 3 to 6 months D. Malignancy-associate DVT/PE best treate with low-molecular-weight heparin (not warfarin)

Situation

Duration of anticoagulation

Provoked DVT/PE without associated malignancy

3 months

Provoked DVT/PE in association with malignancy

Indeęnite

Unprovoked DVT/PE in patient with low-moderate bleeding risk

Indeęnite

Unprovoked DVT/PE in patient with high bleeding risk

3 months

DVT/PE pregnant woman

3 months OR 6 weeks after delivery (whichever is longer)

Superęcial thrombophlebitis of legs

4–6 weeks of fondaparinux

E. Phlegmasia alba olens 1. Cause by massive DVT, eep venous channels are affecte while sparing collateral veins an therefore maintaining some egree of venous return; patients have pale blanching of extremity (milky color), eema, an iscomfort . Treatment is immeiate anticoagulation an leg elevation; may progress to cerulea olens F. Phlegmasia cerulea olens 1. Venous outow of the entire lower extremity is affecte; persistent venous obstruction eventually inhibits arterial inow . Patients present with pain, swelling, an a blue iscoloration (cyanosis) of the extremity; peal pulses may be iminishe 3. In aition to anticoagulation, catheter-irecte thrombolysis is also recommene (unless > weeks uration) G. Paget-Schroetter synrome (effort thrombosis of axillary or subclavian vein) 1. Thrombosis of the axillary an/or subclavian veins . Often seen in athletes who overuse the affecte extremity, leaing to recurrent vein compression

265

266

PArt i Patient Care

3. Often relate to thoracic outlet synrome, with compression of the subclavian vein in the costoclavicular space (between the 1st rib an the clavicle) 4. Diagnose with uplex ultrasoun or venogram 5. Treat with anticoagulation; if moerate-severe symptoms of 1 secon in the femoral or popliteal veins, or >0.5 secon in the saphenous, tibial, eep femoral, or perforator veins; also, uplex ultrasoun scan to rule out DVT E. Mainstay of treatment of venous insufciency is compression therapy F. If compression fails, can consier surgical intervention if incompetence/reux is in supercial or perforator veins: 1. For supercial veins (greater or small saphenous veins) or perforator veins—treat with enovenous ablation . If have concomitant symptomatic varicose veins: treat unerlying venous incompetence AND can also perform sclerotherapy or stab phlebectomy of supercial varicose veins 3. Complication of saphenous vein ablation: thrombus at saphenofemoral junction (potential risk for DVT), start ASA 4. Telangiectasias (spier veins): not always associate with reux—if no reux compression an ablation will not help; treatment is cosmetic: inclues injection sclerotherapy or transermal laser therapy

CHAPtEr 18 Vascular—Venous

Fig. 18.1 May-Thurner Synrome

267

268

PArt i Patient Care

Questions 1. Which of the following is the most common risk factor for spontaneous venous thromboembolism? A. Antithrombin III eciency B. Factor V Leien C. Protein C eciency D. Protein S eciency E. Antiphospholipi synrome 2. A 60-year-ol male presents with pain over his left mi-meial thigh. He ha a similar event in his other thigh a month earlier. He has note a ecrease appetite. On exam the skin over the meial thigh is re, warm, an tener. He has no varicose veins, nor evience of skin hyperpigmentation or leg swelling. Duplex scan shows an 8-cm segment of thrombosis of the mi saphenous vein, but no DVT. Which of the following is recommene? A. IV heparin followe by warfarin B. Warm compresses, nonsteroial antiinammatory rugs (NSAIDs), an a CT of the abomen C. Ligation of the sapheno-femoral junction D. Fonaparinux an a CT of the abomen E. Warm compresses an NSAIDs 3. A 5-year-ol male college swimmer presents with suen onset of right arm swelling an pain. A uplex ultrasoun scan emonstrates thrombosis of the axillary-subclavian vein. The patient is starte on IV heparin. The most important aitional ajunctive therapy for this patient is: A. First rib resection B. Catheter-guie thrombolysis C. Lifelong anticoagulation D. Venous stenting E. Physical therapy 4. Which of the following is true regaring venous circulation? A. The perforating veins in the leg irect bloo ow from eep to the supercial system B. The common iliac veins have valves C. In a healthy person, venous pressure increases with walking D. The greater saphenous vein joins the femoral vein to become the common femoral vein E. Muscle contraction plays no role in venous return

5. Which of the following is true regaring the initiation of heparin in a 100-kg patient with a newly iagnose DVT? A. A bolus of 10,000 units of heparin shoul be given before starting the rip B. Following a bolus a rip shoul be starte at 18 units/kg per hour C. Dosing shoul be ajuste using the international normalize ratio (INR) D. Activate partial thromboplastin time (aPTT) shoul be titrate to 100 to 10 secons after starting the rip E. Heparin shoul be stoppe if the platelet count ecreases below 00,000 6. A 35-year-ol female presents with left leg swelling. There are no precipitating factors. Ultrasoun conrms a left iliofemoral DVT, an the patient is starte on heparin. Workup reveals no evience of risk factors for DVT, such as recent surgery, prolonge immobilization, nor any evience of malignancy. Which of the following is most likely to be of long-term benet? A. Low-molecular-weight heparin (LMWH) B. Long-term (>1 months) anticoagulation C. Lifelong compression stocking D. Right-to-left femoral vein bypass E. Venous thrombectomy 7. Which of the following is true regar ing the management of DVT? A. For patients with proximal DVT of the leg an no cancer history, irect Xa inhibitor is recommene over warfarin B. For a leg DVT in association with malignancy, warfarin is preferre over LMWH C. For incientally iscovere DVT, anticoagulation is unnecessary D. In patients with isolate istal (calf) DVT of the leg, anticoagulation therapy is superior to serial imaging E. In patients with a secon episoe of DVT, three months of anticoagulation is recommene

CHAPtEr 18 Vascular—Venous

8. A 58-year-ol male with newly iagnose metastatic colon cancer presents to the ED with a swollen right leg an severe pain that starte 1 ay earlier. On exam, he has massive eema of the right leg that is tener to palpation. His foot appears blue. Duplex scan conrms a DVT. Which of the following is true about this conition? A. The risk of limb loss is low B. This occurs more commonly on the right sie C. A pale, white foot carries a worse prognosis than a blue foot D. Associate hypotension is usually the result of sepsis E. Catheter-irecte thrombolysis shoul be performe 9. Which of the following is the best inication for placement of an inferior vena cava (IVC) lter? A. A pregnant patient in the thir trimester iagnose with a new DVT B. A patient with severe pelvic fractures C. A patient with a large free-oating vena cava thrombus D. A recurrent DVT in a patient who is alreay therapeutic on warfarin E. Before planne thrombolysis of a new DVT 10. The most common electrocariographic change after pulmonary embolism (PE) is: A. Atrial brillation B. Right bunle branch block C. Nonspecic ST an T wave changes D. S1, Q3, T3 pattern E. Sinus tachycaria 11. Trauma patients sustaining what type of injury are at highest risk of venous thromboembolism? A. Hea trauma B. Femur fracture C. Pelvic fracture D. Splenectomy E. Spinal cor injury 12. A 45-year-ol woman presents with a nonhealing ulcer at the meial malleolus associate with leg eema an hyperpigmentation but no signs of infection. First-line management consists of: A. Wet-to-ry ressings B. Split-thickness skin grafting C. Subfascial perforator ligation D. Local woun ebriement followe by intravenous antibiotics E. Compression ressings

269

13. A 50-year-ol male presents with a meial malleolar ulcer that has faile to heal with 4 weeks of compression ressings. He has large varicose veins in the lower leg, eema, an hyperpigmentation. There is no eep vein thrombosis (DVT) ientie on uplex ultrasoun. However, there is incompetence of the supercial, eep, an perforator systems. Which of the following is the best next step? A. Vein stripping of the greater saphenous vein B. Raiofrequency ablation (RFA) of the greater saphenous vein an ultrasoun-guie perforator sclerotherapy C. RFA of the greater saphenous vein an compression stockings D. Continue with a 3-month course of compression ressing treatment E. Ultrasoun-guie perforator vein sclerotherapy 14. A 44-year-ol male presents to the emergency epartment (ED) with a temperature of 103°F. He is hypotensive espite a -L ui bolus. He is preppe for a right internal jugular 9-French central venous line to start pressors while being worke up for an unerlying cause. Following placement of the catheter, pulsatile bleeing is note from the catheter. What is the best next step? A. Downsize to a smaller catheter in the ED, transfer the patient to the intensive care unit (ICU), an remove it in several hours B. Immeiately remove the catheter an hol pressure for 10 to 15 minutes C. Immeiately remove the catheter an get a uplex ultrasoun stuy of the neck D. Remove the catheter uner irect surgical exposure E. Transfer patient to the ICU, then remove, hol pressure, an place a suture in the skin 15. A 40-year-ol woman presents with pain an tenerness at the site of a longstaning varicose vein in her calf. There is a palpable cor with surrouning erythema. Duplex scan shows localize thrombus within the varicose vein, an no DVT. Management consists of: A. Intravenous (IV) heparin soium B. Subcutaneous low-molecular-weight heparin C. Warm compresses an nonsteroial antiinammatory rugs D. Ligation of saphenous vein at saphenofemoral junction E. IV antibiotics

270

PArt i Patient Care

Answers 1. B. The primary risk factors for spontaneous venous

thromboembolism (VTE) as escribe by Virchow inclue stasis of bloo ow, enothelial injury, an hypercoagulability. In cases of spontaneous VTE, hypercoagulability is the most important factor. Factors that contribute to hypercoagulability inclue factor V Leien, prothrombin gene mutation, protein C an S eciency, antithrombin III eciency, elevate homocysteine levels, an antiphospholipi synrome. In aition, nonacquire causes of VTE inclue smoking (most common), obesity, pregnancy, malignancy, an use of oral contraceptives. In surgical patients, the cause of VTE is multifactorial because postoperative stasis from prolonge be rest an enothelial injury from trauma or recent surgery are signicant factors. In trauma patients, spinal cor injury has the highest risk of VTE. Other risk factors for VTE inclue history of VTE, avance age, an varicose veins. Factor V Leien is the most common genetic efect associate with thrombophilia (A, C–E). Factor V Leien is a single-point mutation in the gene that coes for coagulation factor V. It makes factor V resistant to inactivation by activate protein C (which is a natural anticoagulant protein). The mutation is transmitte in an autosomal ominant fashion an accounts for 9% of cases of anticoagulant protein resistance. The mutation is present in 4% to 6% of the general population an is associate with a sixfol increase risk of VTE in heterozygotes. In homozygotes, the risk is 80-fol. In patients with their rst VTE, factor V Leien was present in 15% to 0%. There is no stanar guieline for the uration of anticoagulation therapy in patients with an acquire hypercoagulable state. It is believe an iniviualize approach shoul be taken to access each person's risk of a recurrent VTE an compare this to their relative risk of a bleeing event. Interestingly, in one stuy, the risk of recurrent VTE was similar among carriers of the factor V Leien gene compare with those without this mutation, suggesting that they o not nee longer anticoagulation than the stanar recommenation for a rst-time event. References: Bauer KA. Duration of anticoagulation: applying the guielines an beyon. Hematology Am Soc Hematol Educ Program. 010;010(1):10–15. Mazza JJ. Hypercoagulability an venous thromboembolism: a review. WMJ. 004;103():41–49.

2. D. Unprovoke SVT, an in particular, recurrent unpro-

voke SVT, an more specically recurrent, unprovoke SVT in ifferent limbs (supercial migratory thrombophlebitis) shoul prompt concern for hypercoagulability an, in particular, malignancy. Supercial migratory thrombophlebitis is particularly associate with pancreatic cancer (Trousseau sign) an, to a lesser egree, stomach an lung cancer. Thus, treatment shoul inclue a targete workup for malignancy (that shoul be tailore to nings on history, review of systems, an physical exam) (B, C–E). Given the ecrease appetite, suspicion for GI cancer shoul be high an a CT scan appropriate. SVT within the saphenous vein in the upper thigh, within 3 cm of the saphenofemoral junction, an those with long segments (>5 cm) have an increase risk

of propagating into the eep system an thus benet from anticoagulation (A). A recent stuy comparing fonaparinux with placebo emonstrate a ecrease in DVT, recurrent thrombophlebitis, an clot progression with fonaparinux. References: Chengelis DL, Benick PJ, Glover JL, Brown OW, Ranval TJ. Progression of supercial venous thrombosis to eep vein thrombosis. J Vasc Surg. 1996;4(5):745–749. Decousus H, Pranoni P, Mismetti P, et al. Fonaparinux for the treatment of supercial-vein thrombosis in the legs. N Engl J Med. 010;363(13):1–13.

3. A. Paget-Schroetter synrome, also known as effort-in-

uce thrombosis, is a spontaneous thrombosis of the axillary-subclavian vein. It is thought to be, in most instances, a manifestation of thoracic outlet synrome, whereby a hypertrophie or aberrant muscle compresses the axillary-subclavian vein as it passes between the rst rib an the clavicle. It tens to evelop in young, active patients after vigorous activity (swimming, pitching, weightlifting), although it can also occur spontaneously. It usually presents in men more often than women. Seconary axillary/subclavian vein thrombosis can also present in those with meiastinal tumors, congestive heart failure (CHF), an nephrotic synrome. Diagnosis is best establishe via uplex ultrasonography. The patient shoul be promptly starte on IV heparin. The most important ajunctive measure to prevent recurrence an long-term swelling is thoracic outlet ecompression via rst rib resection. The timing is controversial but is not time sensitive. Systemic thrombolysis is not inicate. However, the journal CHEST recommens catheter-irecte thrombolysis for this conition if the patient has moerate-severe symptoms an presents with less than  weeks of symptoms. The benet of catheter-irecte thrombolysis in this situation is a ecrease risk of postthrombotic synrome (B). A follow-up venogram is frequently obtaine to ientify any correctable anatomic abnormalities. Stenting a resiual stenosis in this area without ecompressing the thoracic outlet is contrainicate because the ongoing compression will invariably crush the stent an cause further venous amage, making any further intervention even more ifcult. Resiual venous stenoses can be treate with angioplasty, although some authors recommen oing this after the rst rib resection. A recent metaanalysis emonstrate a signicant improvement in symptoms in those that receive a rst rib resection compare to those that i not. More than 40% of patients in the control group neee to have a rib resection ue to recurrent symptoms. In an active athlete, an in particular one who performs repetitive movements with the arm overhea (which by itself can compress the vein), rst rib resection is the best option (C–E). References: Angle N, Gelabert HA, Farooq MM, et al. Safety an efcacy of early surgical ecompression of the thoracic outlet for Paget-Schroetter synrome. Ann Vasc Surg. 001;15(1):37–4. Lee WA, Hill BB, Harris EJ Jr, Semba CP, Olcott C IV. Surgical intervention is not require for all patients with subclavian vein thrombosis. J Vasc Surg. 000;3(1):57–67. Machleer HI. Evaluation of a new treatment strategy for Paget-Schroetter synrome: spontaneous thrombosis of the axillary-subclavian vein. J Vasc Surg. 1993;17():305–315.

CHAPtEr 18 Vascular—Venous Urschel HC Jr, Razzuk MA. Paget-Schroetter synrome: what is the best management? Ann Thorac Surg. 000;69(6):1663–1668. Lugo J, Tanious A, Armstrong P, et al. Acute Paget-Schroetter synrome: oes the rst rib routinely nee to be remove after thrombolysis? Ann Vasc Surg. 015;9(6):1073–1077.

4. D. The lower extremity veins are ivie into supercial,

perforating, an eep veins. The supercial venous system consists of the greater saphenous an lesser saphenous veins. The eep veins follow the course of major arteries. Paire veins parallel the anterior an posterior tibial an peroneal arteries an join to form the popliteal vein. The popliteal vein becomes the femoral vein as it passes through the auctor hiatus. In the proximal thigh, the greater saphenous vein joins with the femoral vein to become the common femoral vein. Multiple perforating veins traverse the eep fascia to connect the supercial an eep venous systems. The most important perforators are the Cockett an Boy perforators. The Cockett perforators rain the lower part of the leg meially, whereas the Boy perforators connect the greater saphenous vein to the eep vein higher up in the meial lower leg, approximately 10 cm below the knee. Bloo ows from the supercial to the eep venous system (A). Incompetence of these perforators is a major contributor to the evelopment of venous stasis an ulceration. There are no valves in the portal vein, superior vena cava, inferior vena cava (IVC), or common iliac vein (B). The calf muscles serve an important function in augmenting venous return by acting as a pump to return bloo to the heart (E). For this reason, patients who are berien are prone to venous stasis. Venous pressure rops ramatically with walking because of the action of the calf muscles but increases in patients with venous obstruction because this leas to persistent stasis that muscle contraction cannot overcome (C). This is why compression stockings are recommene for these patients as an ajunct to normal venous return.

5. B. If a heparin rip is starte, a bolus of 80 units/kg (8000

units for the above patient) shoul rst be given followe by the continue infusion of heparin at 18 units/kg per hour (A). In patients with DVT, the aPTT nees to be rawn every 6 to 1 hours with a goal rate of 60 to 90 secons (D). INR is checke in patients on warfarin (C). Heparin can potentially lea to heparin-inuce thrombocytopenia (HIT). This usually happens 5 ays or more after the initiation of heparin an will present as a 50% rop in platelet count (E). Reference: Hirsh J, Bauer KA, Donati MB, Goul M, Samama MM, Weitz JI. Parenteral anticoagulants: American college of chest physicians evience-base clinical practice guielines (8th eition). Chest. 008;133(6 Suppl):141S–159S.

6. B. DVT an pulmonary embolism (PE) affect up to

900,000 people per year in the Unite States, an their incience increases with age. When a patient presents with a DVT, always try to etermine which part of the Virchow tria (stasis, vascular injury, an hypercoagulability) can explain the event. This will serve as a reminer to perform a careful history an physical examination to assess risk factors for DVT. In most cases, the causes are multifactorial. The uration an type of anticoagulation epen on whether the DVT is provoke (i.e., malignancy, recent surgery, prolonge immobilization) or unprovoke, what the provoking factor is, an on the location (proximal or istal leg) of the

271

DVT. Proximal (iliofemoral) DVTs are more likely to lea to massive swelling an long-term sequelae of postphlebitic synrome. As such, more consieration shoul be given to the type of anticoagulant, the uration, use of thrombolytics, an mechanical thromboembolectomy as compare with istal DVT. LMWH is not as efcacious for proximal DVT (A). For a proximal (iliofemoral) DVT, that is unprovoke (no clear contributing factors), the recommenation is for long-term (>1 months) anticoagulation. The benet of compression stockings to prevent postphlebitic synrome is controversial (C). Most authors recommen  years of compression; lifelong compression has no benet, has poor patient compliance, an is associate with signicant costs of renewing expensive stockings every 6 months. A rightto-left femoral vein bypass (with right leg saphenous vein) is rarely performe an woul be a last resort for chronic venous stasis that is unresponsive to enovascular options (D). Thrombolytic therapy is an option for select patients with severe iliofemoral DVT, particularly if they present with phlegmasia. Venous thrombectomy is reserve for patients with phlegmasia who have faile thrombolytic therapy (E). Reference: Heffner JE. Upate of antithrombotic guielines: meical professionalism an the funnel of knowlege. Chest. 016;149():93–94.

7. A. The American College of Chest Physicians release

upate guielines in 016 for the management of DVTs. One major change is that patients with proximal DVT of the leg an no cancer history shoul now be treate with a irect Xa inhibitor (abigatran, rivaroxaban, apixaban, or eoxaban) over warfarin. Aitionally, initial parenteral anticoagulation with a heparin rip is not require when using rivaroxaban an apixaban. However, a heparin rip shoul be starte before aministering abigatran or eoxaban an overlappe with warfarin therapy. Several reports have shown the superiority of these novel oral anticoagulants (NOACS). Aitionally, iarucizumab is now available as a reversal agent for abigatran allowing NOACs to be more commonly prescribe. In patients with a cancer history an proximal DVT of the leg, LMWH is recommene over warfarin an irect Xa inhibitors (B). This is unchange from the prior guielines. In patients with a proximal DVT of the leg provoke by surgery, 3 months of anticoagulation therapy is recommene over a longer time-limite perio (6, 9, 1, or 4 months). This recommenation applies to patients with both low an high bleeing risks. The management of isolate calf DVT remains controversial. Anticoagulation is recommene for those with severe leg symptoms or those with risk factors for propagation. In patients with an isolate istal DVT of the leg an without severe symptoms or risk factors for extension, serial imaging of the eep veins for  weeks is recommene over anticoagulation therapy (D). There is no consensus on the uration of therapy for patients with a secon episoe of DVT because this epens on the presence of reversible risk factors, unerlying cause, malignancy, life expectancy, an the buren of therapy. However, most surgeons woul recommen at least 1 year of anticoagulation therapy for patients with a secon episoe of DVT an lifelong anticoagulation for patients with more than two episoes of DVT (E). Incientally iscovere DVTs shoul be treate with anticoagulation (C).

272

PArt i Patient Care

References: Connors JM. Antiote for factor Xa anticoagulants. N Engl J Me. 015;373(5):471–47. Heffner JE. Upate of antithrombotic guielines: meical professionalism an the funnel of knowlege. Chest. 016;149():93–94.

8. E. Massive iliofemoral DVT can lea to impaire arterial

bloo ow ue to massive swelling. Early on, the limb turns pale an is referre to as phlegmasia alba (white) olens. In a subgroup of patients, this may progress to impening gangrene phlegmasia cerulea (blue) olens as in the patient escribe. When the majority of the eep venous channels are burene with clots, the relatively smaller supercial venous channels are taske with raining the entire leg. Patients evelop a tener, pale, an eematous extremity. This is known as “milk-leg” since the pale extremity appears whitish (alba). As the isease progresses an the supercial venous channels are also affecte, the entire venous rainage of the leg is compromise, causing massive eema in the leg. As the swelling continues, arterial malperfusion ensues, leaing to severe ischemia (blue extremity), risking limb loss (C). DVT an as an extension, phlegmasia, both occur more commonly on the left. This is a result of the left iliac vein frequently being compresse by the right iliac artery (known as May-Thurner synrome) (B). Unerlying malignancy is the most common risk factor ientie for phlegmasia. The fastest an safest metho of conrming the iagnosis is with uplex ultrasoun. CT angiography is not require unless history, exam, an ultrasoun nings are equivocal. Initial treatment is similar to that for an acute DVT, with some qualiers. More emphasis shoul be place on leg elevation. Due to ui sequestration, patients may present with hypovolemic shock an thus may nee massive volume resuscitation (D). The risks of limb loss, pulmonary embolism, postphlebitic synrome, an mortality are all high (A). As such, thrombolytic therapy has emerge as the treatment of choice. Reference: Chinsakchai K, Ten Duis K, Moll FL, e Borst GJ. Trens in management of phlegmasia cerulea olens. Vasc Endovascular Surg. 011;45(1):5–14.

9. D. Enthusiasm for the aggressive use of IVC lters is

iminishing. Filters left in place for long perios of time can lea to complications, incluing migration of the lter, fracturing of the legs of the lter, vena cava perforation, an the increase risk of a recurrent DVT. In a prospective ranomize stuy of patients with DVT, the routine aition of an IVC lter i not improve mortality compare with heparin an warfarin alone. Aitionally, PREPIC  trial has also emonstrate an increase number of recurrent PEs in the lter group compare to the anticoagulation only group (3% versus 1.5%). Thus, the majority of IVC lters are now retrievable an shoul optimally be remove within 9 to 1 weeks. Consensus opinion from most societies is that the strongest inication for an IVC lter placement is a patient who evelops a venous thromboembolic event (VTE [DVT or PE]) who has a contrainication to anticoagulation (such as active gastrointestinal bleeing). Other inications are a new VTE that evelops in a patient who is alreay receiving therapeutic anticoagulation, or a patient with a VTE who is alreay receiving anticoagulation an in whom a major hemorrhage evelops (B, C–E). Relative inications inclue prophylaxis in high-risk populations (severe hea, pelvic, or spinal cor trauma), massive PE treate with thrombolysis or

thrombectomy (to prevent further ecompensation), an the presence of a large free-oating thrombus in the IVC. Pregnant patients iagnose with a new DVT shoul be starte on anticoagulation with low-molecular-weight heparin for the remainer of the pregnancy an up to 6 weeks postpartum (A). Warfarin shoul be avoie since it is teratogenic. References: Decousus H, Leizorovicz A, Parent F, etal. A clinical trial of vena caval lters in the prevention of pulmonary embolism in patients with proximal eep-vein thrombosis. N Engl J Med. 1998;338(7):409-416. Millwar SF, Oliva VL, Bell SD, et al. Günther tulip retrievable vena cava lter: Results from the Registry of the Canaian Interventional Raiology Association. J Vasc Interv Radiol. 001;1(9):1053–1058. Rajasekhar A. Inferior vena cava lters: current best practices. J Thromb Thrombolysis. 015;39(3):315–37.

10. E. The most common ning on electrocariogra-

phy after a PE is sinus tachycaria (present in almost half of patients) (A–D). A heart rate greater than 100 beats per minute with associate tachypnea in the setting of suspecte PE shoul further raise concern. The classic ning on an electrocariogram is the S1, Q3, T3 pattern, which consists of a prominent S wave in lea I an a Q wave an inverte T wave in lea III. This electrocariographic ning inicates right ventricular strain from a large PE, but it is not commonly present. A large PE will lea to an enlargement of the right ventricle causing the interventricular septum to eviate to the left. The right bunle branch stretches, leaing to a right bunle branch block.

11. E. The increase risk of the evelopment of VTE in sur-

gical patients is multifactorial. Patients will have a perio of activate coagulation, transient epression of brinolysis, an temporary immobilization. In aition, many patients may have a central venous catheter in place an have concomitant cariac isease, malignancy, or intrinsic hypercoagulable states, all of which increase a patient's chance of a VTE. Trauma patients, in particular, have a high risk of VTE. In trauma patients, spinal cor injury (os ratio, 8.33) an fracture of the femur or tibia (os ratio, 4.8) were the injuries with the greatest risk of VTE (A–D). In one large prospective stuy, other risk factors in trauma patients on multivariate analysis inclue oler age, bloo transfusion, an nee for surgery. Reference: Geerts WH, Coe KI, Jay RM, Chen E, Szalai JP. A prospective stuy of venous thromboembolism after major trauma. N Engl J Med. 1994;331(4):1601–1606.

12. E. This patient has classic signs of chronic venous insuf-

ciency. Venous stasis ulcers are classically locate at the meial malleolus. The precise cause of venous stasis ulcers is unclear but seems to be multifactorial. The increase venous pressure from incompetent valves results in an impeance of capillary ow, which leas to leukocyte trapping. These leukocytes release oxygen free raicals an proteolytic enzymes that lea to local inammation. The increase venous pressure also leas to the leakage of proteins such as brinogen, which act as a barricae to oxygen an growth factors necessary for woun healing. First-line therapy for the treatment of venous stasis ulcers is compression therapy (A–D). The workup for this patient shoul inclue a uplex ultrasoun scan of the venous system, specically looking for valvular incompetence of the eep, supercial, an perforating veins.

CHAPtEr 18 Vascular—Venous A popular an effective compression banage is the Unna boot, which contains zinc oxie, glycerin, gelatin, an calamine lotion. The boot shoul be wrappe starting at the foot, up to just below the knee. It can remain in place for as long as a week. It shoul not be use in the setting of an active infection of the ulcer. In this situation, ebriement an antibiotics will be neee rst.

13. C. A spectrum of chronic venous isorers, from vari-

cose veins to venous stasis ulcers, aficts 0% to 5% of the population. The unerlying etiology is incompetence of the venous valves in either the eep, supercial (saphenous), or perforator veins. Patients with chronic venous isease are classie an treate base on the severity of their isease. The CEAP (clinical, etiologic, anatomic, an pathophysiologic) classication is use worlwie to stanarize this evaluation. It is important when iscussing treatment with a patient that he or she unerstans that this is an incurable isease an that the goal of intervention is to minimize symptoms an prevent recurrence. In general, supercial incompetence is ealt with rst. In a patient with a nonhealing woun that has incompetent valves in all three venous systems, (supercial, perforator, an eep) an is unresponsive to compression therapy, the supercial venous incompetence is aresse rst by obliterating the saphenous vein along with compression therapy. This can be one via saphenous vein stripping, foam sclerotherapy, or RFA. A recent ranomize stuy emonstrate equal results with all three approaches. That being sai, RFA is generally preferre an is the current recommenation of the American Venous Forum, ue to its less invasive nature as compare with stripping (A–E). Freeom of reux has been seen in 93% of patients at  years after ablation therapy. Primary venous insufciency is a recognize risk factor for the evelopment of DVT, an it is important to rule this out before intervention to minimize treatment failure. If treating the supercial system is not successful, the next step is to treat the perforator incompetence. This is one via ultrasoun-guie sclerotherapy. There is no surgical treatment that is reliably effective for eep system incompetence. A recent ranomize stuy conrme the benet of early ablation of the saphenous vein to promote woun healing, as oppose to a longer (6 month) trial of compression therapy. References: Gohel MS, Heatley F, Liu X, et al. A ranomize trial of early enovenous ablation in venous ulceration. N Engl J Med. 018;378():105–114.

273

Meissner MH. What is effective care for varicose veins? Phlebology. 016;31(1 Suppl):80–87. O’Donnell TF Jr, Passman MA, Marston WA, et al. Management of venous leg ulcers: clinical practice guielines of the Society for Vascular Surgery an the American Venous Forum. J Vasc Surg. 014;60( Suppl):3S–59S.

14. D. The right internal jugular vein is the preferre option

for central line placement because it is easily accessible an has a lower risk of pneumothorax compare to a subclavian line. It also has a straight course into the right atrium. In 70% of iniviuals, the internal jugular vein lies anterolateral to the caroti artery. However, in some cases, it may lie irectly anterior or posterior to the caroti artery, increasing the risk of a caroti artery cannulation. If the caroti artery is entere with the probe neele (as evience by pulsatile bleeing), the neele shoul be immeiately remove an pressure shoul be hel for 10 minutes. If the artery is cannulate with a ilator or catheter, then the catheter shoul not be remove blinly. This coul lea to a potential airway-threatening hemorrhage. It is safer to remove the catheter in the operating room via irect surgical exposure, followe by suture repair of the artery (A–C, E). Reference: Kron I, Ailawai G. Cariovascular monitoring an support. In: Fischer JE, e. Fischer's mastery of surgery. 6th e. Lippincott Williams & Wilkins; 011:45–66.

15. C. The patient has supercial venous thrombosis (SVT)

or thrombophlebitis. This entity is essentially a clotte surface vein. A palpable cor is suggestive of the iagnosis, as are accompanying pain an erythema. There are a few pitfalls in the iagnosis an management of SVT. Patients with SVT may have a concomitant DVT (5%–40%); thus, a uplex ultrasoun scan of the venous system is essential. Secon, SVT can easily be misiagnose as cellulitis, in which case antibiotics may be inappropriately prescribe an a uplex ultrasoun scan not obtaine. SVT is generally best manage with warm compresses an NSAIDs. IV antibiotics are reserve for septic thrombophlebitis, which is typically associate with an intravenous line (E). Systemic anticoagulation is reserve for a SVT that is near the eep system (A, B). If anticoagulation is contrainicate, ligation of the saphenous vein at the saphenofemoral junction is inicate for a saphenous vein SVT (D). Varicose veins cause stasis an thus preispose to SVT.

Vascular—Access LUIS FELIPE CABRERA VARGAS, MARK ARCHIE, AND CHRISTIAN DE VIRGILIO

19

ABSITE 99th Percentile High-Yields I. Vascular access for hemoialysis (HD) in Patients with En-Stage Renal Disease (ESRD) A. Catheters 1. Temporary noncuffe, nontunnele ialysis catheter . Permanent (cuffe) or tunnele-ialysis catheter (TDC) B. Autogenous surgical access (arteriovenous stula [AVF]) 1. Preferre locations a) Upper > lower extremities; istal > proximal b) Nonominant > ominant arm c) Cephalic > basilic > brachial veins > femoral vein of the thigh . En-to-sie preferre to sie-to-sie anastomosis C. Nonautogenous surgical access (arteriovenous graft [AVG]) 1. Material: prosthetic (PTFE), biologic (bovine, human cryopreserve veins) . Locations: upper arm, forearm, thigh II. Comparing Vascular Access for HD

Catheters

AVF

AVG

1. Ideal for short-term hemodialysis, or as a bridging tool (AVF, graft or kidney transplantation) 2. Best avoided for long-term use 3. To reduce infection, often coated with antibiotics, silver, or heparin (reduces bacterial trapping) 4. Advantages: immediate use, easily removed if infected 5. Disadvantages: highest incidence of 1 year mortality, thrombotic complications, central vein stenosis, and bacterial infections

1. Best overall option for long-term hemodialysis 2. Advantages: lower risk of infection and thrombosis, highest long-term patency (70%–80% at 1 year), greatest blood Ěow volume during dialysis, lowest need for reinterventions 3. Disadvantages: require at least 6 weeks to mature, some never work

1. Ideal for patients who are not candidates for AVF 2. Advantages: widely available, short wait time to access (does not need to mature), if thrombose relatively easy to declot. 3. Disadvantages: lower long-term patency (about 50% at 1 year), higher thrombosis rates (due to intimal hyperplasia), higher infection rate (vs AVF)

III. Surgical Planning A. Age (elerly may be less suitable for AVF), han-ominance (prefer nonominant arm), comorbi conitions (iabetics may evelop severe raial/ulnar artery atherosclerosis), presence of pacemaker (leas to stenoses of proximal veins) B. Duplex ultrasoun vein an artery mapping 1. Veins (with soft tourniquet in place) a) Cephalic, basilic, brachial; assess patency an size (ieally > 3 mm) . Arteries a) Brachial, raial, ulnar; assess size (ieally >  mm); severe meical calcication may be unclampable, look for ow limiting plaque 275

276

PArt i Patient Care

C. Fistula rst initiative (orer of preference): raio-cephalic (Cimino) > brachio-cephalic > brachio-basilic (may require  stages, rst construction, secon supercialization) > AVG D. For istal stulas from raial or ulnar arteries, assess arterial ominance of the han with Allen test IV. Complications A. Excessive bleeing from ialysis puncture site 1. Nee to rule out stenosis of outow vein or central venous system (high venous pressure) . Rule out use of anticoagulants B. Aneurysm of AVF (may be ue to repeat neele trauma or central stenosis) 1. Most are benign an can be observe . Repair if rapily expaning, overlying skin thinning, skin ulceration, or excessive bleeing 3. Prior to repair, rule out stenosis of outow vein or central venous system as cause C. Pseuoaneurysm of AVG (ue to repeat neele trauma or infection) 1. Small or uninfecte ones can be observe a) Repair if large . Resect AVG if infecte D. Thrombosis E. Steal 1. Stage I: asymptomatic retrograe iastolic ow (US ning alone) . Stage II: pain on exertion an/or uring HD 3. Stage III: pain at rest 4. Stage IV: ulceration/necrosis/gangrene F. Ischemic monomelic neuropathy (IMN) G. High-output cariac failure

Fig. 19.1

Banding Procedure

Reduction of the access Ěow for high Ěowassociated steal syndrome Best results if intraoperative Ěow measurements are used Con: banding a low-Ěow vascular access will lead to ineĜcient dialysis and thrombosis

Access Ligation

Complete resolution of steal syndrome Loss of the vascular access Con: need new AVF with new risk of developing steal syndrome

V.Surgical Treatments of Steal Syndrome

Banding of the vascular access with a nonresorbable suture guided with a 4–5 mm dilatation balloon Controlled reduction in the vessel diameter Only for high Ěowassociated steal syndrome

MILLER Procedure (Minimally Invasive Limited Ligation EndoluminalAssisted Revision) Enhances access Ěow Ideal for low Ěowassociated steal syndrome

PAI Procedure (Proximalization of the Arterial InĚow) Cons: complex surgery Longer operative time Need to harvest suitable vein Hand perfusion reliant on a bypass graft

Ideal for brachial AV access patients with high Ěow induced steal or cardiac failure Creates bypass from distal artery to ęstula (while ligating original anastomosis)

DRIL Procedure (Distal Revascularization RUDI Procedure with Interval (Revision Using Ligation) Distal InĚow)

Uses smaller caliber graft to increase resistance

RUPI Procedure (Revision Using Proximal InĚow)

CHAPtEr 19 Vascular—Access

277

278

PArt i Patient Care

Questions 1. A 63-year-ol man with en-stage renal isease on ialysis via a left upper arm arteriovenous graft (AVG) presents to the emergency epartment with what he escribes as “pulsatile bleeing” from the area of where ialysis was performe the ay before. He states the bleeing stoppe after wrapping his arm. On physical exam, his temperature is 100.5°C an his heart rate is 100. The AVG has a thrill. Overlying the AVG, at the site of the blee, there is a small black eschar with a small aneurysmal swelling. His white bloo cell (WBC) count is 1,000 × 103/mm3 an a uplex ultrasoun emonstrates a pseuoaneurysm at the site of eschar. What is the next best step in management? A. Discharge the patient with oral antibiotics B. Amit the patient for intravenous (IV) antibiotics C. Obtain a stulogram D. Take patient to the operating room for graft excision E. Place a covere stent across the pseuoaneurysm 2. A 45-year-ol female returns to the emergency epartment 8 hours after having unergone a left upper arm arteriovenous graft (AVG) complaining of severe forearm pain. On examination, the forearm is soft an supple. The han appears to be pink, with a normal temperature, an a 1+ raial pulse. There is a goo thrill in the graft. She has both motor an sensory ecits in the han in both the meian an ulnar nerve istributions. The next step in the management is: A. Distal revascularization with interval ligation B. Observation C. Ligation of the graft D. Obtain nerve conuction stuies E. Forearm fasciotomy 3. A 65-year-ol female with en-stage renal isease presents with recurrent episoes of congestive heart failure. She is currently ialyze via a left arm brachiocephalic arteriovenous stula (AVF). Upon compression of the stula, her heart rate ecreases from 80 to 60 beats per minute, an bloo pressure increases from 10/70 to 140/80 mm-Hg. Which of the following is true about this conition? A. The stula shoul be ligate B. It is unlikely that the stula is contributing to the patient’s heart failure

C. Plicating the stula may help prevent another episoe of heart failure D. She shoul unergo a istal revascularization an interval ligation E. The stula shoul be converte to a graft 4. A 65-year-ol woman unergoes creation of an upper arm arteriovenous (AV) graft for hemoialysis in the left arm using a 6-mm polytetrauoroethylene graft. Three weeks later, the patient reports marke coolness, pallor, an numbness in the han as well as pain in the han at rest. Motor exam is normal. On examination, there is no palpable pulse at the raial artery an only a monophasic Doppler signal. Upon graft compression, a pulse becomes palpable an the signal becomes biphasic. Which of the following is the best management option? A. Distal revascularization an interval ligation of the brachial artery B. Ligation of the AV graft an placing the upper arm graft in same arm C. Ligation of the AV graft an placing the stula in the ominant arm D. Baning of the AV graft ajacent to the arterial anastomosis E. Baning of the AV graft ajacent to the venous anastomosis 5. A 45-year-ol male evelops progressive en-stage renal isease seconary to severe hypertension. His glomerular ltration rate is 19 mL/min, an ialysis is anticipate within the next 6 months. He is right-hane an appears to have goo veins in both arms an normal pulses. Which of the following is the best management plan for ialysis access? A. Delay access until about a month before anticipate ialysis B. Procee with left raiocephalic AVF (Cimino stula) C. Procee with left brachiocephalic AVF D. Procee with left brachiobasilic AVF E. Procee with right raiocephalic AVF

CHAPtEr 19 Vascular—Access

6. A 45-year-ol female with en-stage renal isease presents with recent onset of heaaches, hoarseness of her voice, an bilateral arm swelling for  ays. She has a history of multiple proceures in both arms an legs for hemoialysis access. Most recently, she unerwent an arteriovenous graft (AVG) in her right upper arm  weeks earlier. On examination her neck appears to be engorge an her face swollen. There are numerous visible veins on her chest wall. Which of the following is the best management option? A. Ligation of the AVG B. Plication of the AVG C. Attempt venoplasty of superior vena cava (SVC) D. Place stent in SVC E. Move AVG to right arm 7. A 45-year-ol male with long-staning iabetes an progressive en-stage renal failure presents to the emergency epartment (ED) with progressive shortness of breath, vague abominal pain, an marke leg eema. Laboratory values are remarkable for metabolic aciosis an azotemia but a normal white bloo cell count. Dialysis is urgently neee. Dialysis access woul be best institute via: A. Right internal jugular vein tunnele, cuffe catheter B. Right internal jugular vein nontunnele, uncuffe catheter C. Left internal jugular vein, tunnele, cuffe catheter D. Right subclavian vein uncuffe, nontunnele catheter E. Right femoral vein cuffe tunnele catheter 8. A left internal jugular vein central line is place. Fifteen minutes later, the patient is hypotensive. Distene neck veins are note. Breath souns are clear bilaterally. What is the most likely cause of the patient’s hypotension? A. Perforate right atrium B. Perforate subclavian vein C. Perforate subclavian artery D. Tension pneumothorax E. Perforate right ventricle

279

9. A 50-year-ol male with longstaning history of hemoialysis via a left brachiocephalic arteriovenous stula (AVF) presents with an aneurysm within the miportion of the AVF. He reports that there has recently been excessive bleeing when the neeles have been pulle out. On physical examination, the aneurysm is about 3 cm in size. The overlying skin appears supple, without ulceration. The next step in the management consists of: A. Fistulogram B. Resection/plication of the aneurysm C. Replacement of stula with an AV graft D. Ligation of the stula E. Observation 10. In comparing the three moalities use for hemoialysis (central venous catheter [permacath], arteriovenous [AV] graft, an AV stula), which of the following is true? A. They are equal in terms of 1-year patient mortality B. The primary patency for AV stula an AV graft is similar C. The seconary patency for AV stula an AV graft is similar D. Time to maturation for AV stulas an grafts is similar E. A permacath is the best ialysis option in the elerly 11. An intubate patient in the OR evelops an air embolism after central venous catheter insertion. Which of the following murmurs are associate with this conition? A. Austin-Flint murmur B. Carey Coombs murmur C. Means-Lerman scratch murmur D. Still murmur E. Millwheel murmur

280

PArt i Patient Care

Answers 1. D. This patient has a pseuoaneurysm of his AVG that

appears to be infecte, given his fever an elevate WBC. This requires excision of his graft. Though the bleeing has stoppe, an infecte pseuoaneurysm is inherently unstable an will likely blee again, which coul be catastrophic. Thus, antibiotics alone woul not be aequate (A, B). A stulogram is also not inicate as the graft is infecte (C). For a large, noninfecte pseuoaneurysm, a covere stent is a potential treatment option (E). Reference: Muoni A, Cornacchiari M, Gallieni M, et al. Aneurysms an pseuoaneurysms in ialysis access. Clin Kidney J. 015;8(4):363–367.

2. C. This patient has nings of ischemic monomelic neu-

ropathy (IMN), a rare complication after vascular access surgery. The incience of this complication is less than 1% an is more common in female an iabetic patients. IMN results in pain, numbness (in ngers), paresthesia, an motor weakness (intrinsic han muscles) usually shortly after surgery. It can be istinguishe from steal synrome by its faster onset an mil or absent signs of clinical ischemia. The pathophysiology of IMN is poorly unerstoo but is thought to be cause by a loss of bloo ow from istal nerve tissue leaing to istal neuropathies. IMN is a clinical iagnosis an electromyography an nerve conuction stuies are only neee when the clinical neurologic exam is equivocal (D). The recommene treatment for IMN is ligation of the newly create access, which may lea to resolution of neuropathy in some patients (B,C). Distal revascularization an interval ligation(DRIL) can be utilize in steal synrome, but not in IMN (A). This patient shoul not unergo forearm fasciotomy, as she has a soft an supple forearm, making compartment synrome very unlikely (E). References: Datta S, Mahal S, Govinarajan R. Ischemic monomelic neuropathy after arteriovenous stula surgery: clinical features, electroiagnostic nings, an treatment. Cureus. 019;11(7):e5191. Thimmisetty RK, Peavally S, Rossi NF, Fernanes JAM, Fixley J. Ischemic monomelic neuropathy: iagnosis, pathophysiology, an management. Kidney Int Rep. 017;(1):76–79.

3. C. Bloo ow through an AVF is essentially a left-to-right

shunt, an a portion of the cariac output is stolen by the stula (B). Although there is no change in peripheral oxygen consumption after stula placement, there is a rop in peripheral vascular resistance (PVR). Consequently, a compensatory increase in cariac output occurs. The increase in venous return increases cariac preloa an causes rises in atrial natriuretic peptie (ANP) an brain natriuretic peptie (BNP). The ecrease in afterloa results in a ecrease in alosterone an renin levels. This subsequently leas to a ecrease in afterloa as well as suppression of the renin-alosterone-angiotensin system, which promotes natriuresis. Compressing the stula increases PVR an afterloa, leaing to a ecrease in pulse rate an an increase in bloo pressure (Nicolaoni-Branham sign). Patients with higher stula ow will exhibit greater hemoynamic changes with stula occlusion. Objectively, the minimum stula ow rate require

to support hemoialysis is greater than 400 to 500 cc/min. However, when the ow rate excees 000 cc/min or 30% of the cariac output, there is a risk of high-output cariac failure. These patients, an those with clinically evient episoes of cariac failure, shoul unergo intervention aime at reucing ow rates. Surgical plication (narrowing the vein just beyon the anastomosis to the artery by suturing or baning) reuces the ow rate an can partially reverse the hemoynamic changes an prevent future episoes of heart failure. If heart failure continues to occur after an appropriate reuction in ow rates, eventual ligation of the stula is inicate (A). Distal revascularization an interval ligation (DRIL) is use to treat steal synrome, causing ischemic steal synrome istal to the stula. The proceure increases resistance to the stula an ecreases resistance to the istal extremity but may not effectively reuce stula ow in the setting of cariac failure (D). Converting a native stula to a graft woul not help because the large iameter of a graft woul maintain high ow rates (E). References: MacRae JM, Levin A, Belenkie I. The cariovascular effects of arteriovenous stulas in chronic kiney isease: a cause for concern?: cariovascular effects of arteriovenous stulas. Semin Dial. 006;19(5):349–35. High arteriovenous (AV) access ow an cariac complications. NKF Task Force on Cariovascular Disease, America Journal of Kidney Disease, 3(5).

4. A. A patient presenting with marke coolness, pallor,

pain at rest, an han numbness following an AV graft shoul be suspecte of having steal synrome. Ischemic steal synrome occurs in approximately % to 4% of patients unergoing AV access for hemoialysis. Risk factors for steal synrome inclue females, iabetes, age >60, an use of the brachial artery. Proximal stulas have a higher risk of eveloping steal synrome, while istal wrist stulas (Cimino stulas) have a very low risk. AV grafts also have a greater risk of steal compare with native AV stulas (B). This is likely ue to the fact that the large iameter of the graft creates a low-resistance be. In aition, steal seconary to grafts tens to occur early after the access placement, whereas steal after native AV stulas has a bimoal istribution, with some presenting early an others late after the native vein has unergone ilation with lowere resistance. Some egree of physiologic steal occurs in every patient with an AV stula, but only a small minority manifests severe symptoms. The steal synrome is cause by a iversion of bloo ow from the anastomose artery to the low-resistance vein. In aition, the low-resistance venous anastomosis leas to bloo owing in a retrograe fashion from the istal circulation into the stula. Mil steal can be manage conservatively with exercise. More severe symptoms require intervention. Although ligation of the AV graft woul have a great chance of resolving the steal synrome, the patient will require a new access an will again be at risk of eveloping steal (C). Several options exist for the management. The most effective treatment that maintains stula function is istal revascularization an interval ligation. The isavantage of this

CHAPtEr 19 Vascular—Access proceure is that it requires creating a new bypass, usually with a saphenous vein, from the native artery proximal to the AV graft to the artery istal to it, with interval ligation of the native artery just proximal to the istal anastomosis. Baning or plicating of the AV graft, ajacent to the arterial anastomosis, serves to increase the resistance in the graft an reuce steal. The primary isavantage of this approach (for grafts) is that inaequate baning leas to persistent steal, an excessive baning causes graft thrombosis (stulas less likely to thrombose) (D, E). Baning or plication is a more attractive option for steal in an autologous AV stula, such as a brachial artery cephalic vein stula, because the vein is more resistant to thrombosis. This is not yet the stanar approach, however. References: Walz P, Laowski JS, Hines A. Distal revascularization an interval ligation (DRIL) proceure for the treatment of ischemic steal synrome after arm arteriovenous stula. Ann Vasc Surg. 007;1(4):468–473. Yaghoubian A, e Virgilio C. Plication as primary treatment of steal synrome in arteriovenous stulas. Ann Vasc Surg. 009;3(1):103–107. Yu SH, Cook PR, Canty TG, McGinn RF, Taft PM, Hye RJ. Hemoialysis-relate steal synrome: preictive factors an response to treatment with the istal revascularization-interval ligation proceure. Ann Vasc Surg. 008;():10–14.

5. B. When permanent hemoialysis access is neee, the

nonominant arm (E) shoul be consiere rst in orer to mitigate the effects of potentially evastating complications, incluing severe steal synrome, limb ischemia, ischemic monomelic neuropathy, an nerve injury. Once the sie is etermine, the type of AVF must be consiere. Raiocephalic stulas shoul generally be place rst (assuming aequate artery an vein) because subsequent thrombosis will not preclue the placement of a brachiocephalic or brachiobasilic stula more proximally in the arm. Aitionally, raiocephalic stulas may cause ilation of the proximal arm veins, allowing higher success rates of more proximal stulas in the future. Raiocephalic stulas also rarely require a secon-stage supercialization or transposition proceure because the forearm cephalic vein is close enough to the skin to be use upon maturation. If raiocephalic is not possible or has faile, a brachiocephalic shoul be consiere next (C). Brachiocephalic stulas allow stulas to form on the orsal surface of the upper arm an allow easier cannulation an use uring hemoialysis. Further, epening on boy habitus, brachiocephalic stulas may also not require a secon stage to supercialize the stula close to the skin. The thir choice for autogenous stula is the brachiobasilic stula. Since the basilic vein is eep, it requires supercialization of the vein. Many surgeons perform this in two stages so as to allow the vein to mature before supercialization (D). Maturation of a stula typically requires at least 6 weeks an may require aitional interventions. Waiting until 1 month before ialysis will result in placement of a temporary ialysis catheter, which carries high mortality risks (A). Despite the avantages, the raiocephalic stula has a higher early failure or nonmaturation rate an may not be a goo option in iabetics ue to meial calcinosis within the raial artery. Further, when a patient is alreay hemoialysis epenent via tunnele catheter, there is ongoing ebate about whether the ability to rapily cannulate a graft (∼ weeks) shifts the preferences towar initial graft placement rather than stula rst. A forearm loop graft also has the avantage of ilating the basilic an upper cephalic veins for future stula creation.

281

References: Disbrow DE, Cull DL, Carsten CG 3r, Yang SK, Johnson BL, Keahey GP. Comparison of arteriovenous stulas an arteriovenous grafts in patients with favorable vascular anatomy an equivalent access to health care: is a reappraisal of the Fistula First Initiative inicate? J Am Coll Surg. 013;16(4):679–685; iscussion 685–686. Hakaim AG, Nalbanian M, Scott T. Superior maturation an patency of primary brachiocephalic an transpose basilic vein arteriovenous stulae in patients with iabetes. J Vasc Surg. 1998;7(1):154–157. [No authors liste]. NKF-K/DOQI clinical practice guielines for vascular access. Am J Kidney Dis. 006;48(Suppl. 1):S7–S409.

6. D. The patient is presenting with superior vena cava

(SVC) synrome with bilateral arm, neck, an face swelling an hoarseness of the voice. The patient likely has a preexisting central vein stenosis (in the SVC). A high proportion of patients with en-stage renal isease have central vein stenosis (5%–40%) ue to prior central venous access. These stenoses are often asymptomatic, an if SVC synrome oes evelop, it is usually insiious in onset. However, placement of an upper arm AVG access creates a suen, massive increase in venous return that cannot be accommoate by the stenosis, leaing to abrupt venous congestion (E). Central venous stenosis complicates hemoialysis access because it impairs venous stula outow an can reuce ow rates an reuce the likelihoo of maturation in stulas. Further, when access is place ipsilateral to a stenotic lesion, there is a high likelihoo of symptoms ue to the increase venous congestion combine with high venous resistance. Arteriovenous grafts are more likely to cause symptoms than stulas, an upper arm access is more likely to cause symptoms than forearm access. When central stenosis is suspecte, either from history or symptoms, a central venogram shoul be performe to iagnose the lesion. Concomitant enovascular venoplasty is a reasonable option an has a high rate of success. However, rst-line treatment is now enovascular stenting of the SVC (C). This is appropriate for both benign an malignant cases of SVC synrome. Ligation or plication of the graft is not inicate because this estroys the access an oes not aress the unerlying pathology (A, B). Open SVC repair via sternotomy for a benign lesion is overly invasive an unnecessary given the high initial success rates of enovascular treatment. References: Jones RG, Willis AP, Jones C, McCafferty IJ, Riley PL. Long-term results of stent-graft placement to treat central venous stenosis an occlusion in hemoialysis patients with arteriovenous stulas. J Vasc Interv Radiol. 011;(9):140–145. Rizvi AZ, Kalra M, Bjarnason H, Bower TC, Schleck C, Gloviczki P. Benign superior vena cava synrome: stenting is now the rst line of treatment. J Vasc Surg. 008;47():37–380. Trerotola SO, Kothari S, Sammarco TE, Chittams JL. Central venous stenosis is more often symptomatic in hemoialysis patients with grafts compare with stulas. J Vasc Interv Radiol. 015;6():40–46.

7. A. When hemoialysis is urgently neee, temporary

rapi vascular access must be establishe with a catheter that will support high ow (generally >400 cc/min) via  lumens. If long-term ialysis is anticipate, as in this patient, a tunnele, cuffe hemoialysis catheter, or permacath, is preferre (B) an place into a central vein an exits the skin at least 10 cm away via a subcutaneous tract. Tunnele catheters are reay to use immeiately an are less prone to infection than a nontunnele, noncuffe catheter (Quinton catheter). Quinton catheters are preferre in patients neeing urgent ialysis for a short term, or for those with sepsis

282

PArt i Patient Care

(as they are remove rapily). The right internal jugular vein is the rst choice because it is the most irect route to the right atrium. Left-sie placement is less preferable because it jeoparizes venous patency for future permanent access in the left arm (as most patients are right-han ominant). Left-sie catheters also result in lower catheter bloo ow rates an increase the risk of stenosis/thrombosis ue to the longer an more tortuous length of contact with central vein sie-wall (C). The subclavian position is associate with higher rates of complications (D), namely central vein stenosis an pneumothorax, an in some stuies has a higher risk of infection when compare with internal jugular catheters. The femoral position carries the highest risk of infection, which is a signicant cause of mortality in patients with temporary access catheters (E). Femoral lines may compromise a future kiney transplant because it may lea to proximal iliac vein stenosis/thrombosis. Reference: [No authors liste]. NKF-K/DOQI clinical practice guielines for vascular access. Am J Kidney Dis. 006;48(Suppl. 1): S7–S409.

8. A. Clinical signs of cariac tamponae inclue hypo-

tension, istene neck veins, an mufe or istant heart souns (Beck tria). This patient exhibits two of these signs after an invasive proceure of the chest an likely evelope cariac tamponae as a result of perforation of the right atrium. Tamponae cause by central venous catheter placement is a known complication resulting from puncture by the wire, introucer, or the catheter itself. Perforation of the right atrium more often occurs because it has a thinner wall compare to the right ventricle (E). Placing the catheter tip at the right tracheobronchial angle helps avoi placing the catheter tip in the right atrium. A perforate subclavian artery or vein woul likely lea to hemothorax rather than pericarial tamponae (B, C). A tension pneumothorax is a known complication of line placement an may result in hypotension an istene neck veins, but breath souns woul not be clear bilaterally (D). References: Barton JJ, Vanecko R, Gross M. Perforation of right atrium an resultant cariac tamponae: a complication of catheterization to measure central venous pressure. Obstet Gynecol. 1968;3(4):556–560. Darling JC, Newell SJ, Mohamee O, Uzun O, Cullinane CJ, Dear PR. Central venous catheter tip in the right atrium: a risk factor for neonatal cariac tamponae. J Perinatol. 001;1(7):461–464. Hunt R, Hunter TB. Cariac tamponae an eath from perforation of the right atrium by a central venous catheter. AJR Am J Roentgenol. 1988;151(6):150.

9. A. AVF can eventually unergo aneurysmal egeneration

over time, an intervention is require to prevent rupture an exsanguination (E). High outow resistance is a common cause of aneurysm formation an must be rule out by a stulogram. Repeate neele cannulation can cause stenosis, resulting in higher pressures istal to the lesion an subsequent aneurysm formation. Alternatively, repeate neele cannulation can also lea to aneurysmal egeneration of the vein at the stick site. Therefore, cannulation must be avoie in areas unergoing aneurysmal change. A stulogram is iagnostic of the stenotic lesion an potentially therapeutic via venoplasty with or without stent placement. Further, the stulogram will also help istinguish between a true an

pseuo aneurysm. If no lesion is seen on the stulogram, a central venogram shoul be performe to rule out a central stenosis as a cause of high outow pressures. After treatment of the venous stenoses, bleeing may resolve because the abnormally high pressures within the stula return to normal. Thinne/atrophic skin, translucent skin, ulceration, suspecte infection, intraluminal thrombus, high-output cariac failure, steal synrome, or spontaneous bleeing from the stula prompts consieration for revision by resection an plication or reanastomosis with a healthy vein (B). The size of the aneurysm is not an inicator for revision. If no healthy vein is available, graft implantation is an option (C). If outow cannot be salvage, the access may require ligation (D). Reference: Cronenwett JL, Wayne Johnston K. Rutherford's vascular surgery. 7th e. New York, NY: Sauners/Elsevier; 010.

10. C. Fistulas are superior to grafts, which are superior to

catheters in terms of patient survival, mainly because of the infection risks of prosthetic material (A–E). Diabetics have an exaggerate increase in mortality ue to their epresse immune systems. Interestingly, espite the risk of high-output cariac failure associate with stula an graft, patients with tunnele catheters also have the highest risk of cariac-relate mortality. When comparing patency, stulas are known to have higher primary patency (intervention-free patency of 85% at 1 year, 50% at 5 years) compare to grafts (60% at 1 year, 10% at 5 years) (B). However, stulas have a higher rate of primary failure (nonmaturation or early thrombosis) of up to 40%. Furthermore, when comparing seconary patency (patency with interventions to maintain or reestablish ow), stulas an grafts are similar. Grafts o not require maturation because their lumen iameter oes not change (D). However, healing time of at least 10 ays must be observe after graft placement before cannulation to avoi massive pseuoaneurysm formation. Fistulas require at least 6 weeks for maturation, uring which time the outow vein unergoes remoeling seconary to increase ow resulting in an increase in iameter an further increase in ow. Fistulas are eeme mature if they meet the rule of sixes: at 6 weeks, they must be 6 mm in iameter, less than 6 mm from skin surface, support 600 mL/min ow (although a minimum of 400 mL/ min is aequate), an have a 6-inch straight segment for use. References: [No authors liste]. NKF-K/DOQI clinical practice guielines for vascular access. Am J Kidney Dis. 006;48(Suppl. 1): S7–S409. Lok CE, Sontrop JM, Tomlinson G, et al. Cumulative patency of contemporary stulas versus grafts (000–010). Clin J Am Soc Nephrol. 013;8(5):810–818.

11. E. Intubate patients with an air embolus may have

an abrupt increase in en-tial CO followe by a ecrease in en-tial CO an hypotension, an auscultation may reveal a “millwheel” murmur. This is often escribe as a lou churning soun. An Austin-Flint murmur is associate with aortic insufciency an is a mi-iastolic rumble hear best at the apex (A). Carey Coombs murmur is also a mi-iastolic rumble that is associate with rheumatic fever (B). Means-Lerman scratch murmur souns similar to a pericarial rub an may be hear in patients with hyperthyroiism (C). Still murmur is associate with a small ventral septal efect an is escribe as a vibratory systolic ejection rumble (D).

Transplant JOSEPH HADAYA, AREG GRIGORIAN, AND CHRISTIAN DE VIRGILIO

20

ABSITE 99th Percentile High-Yields I. Type of Transplant Rejection A. Hyperacute: occurs in minutes to hours after transplantation (type- hypersensitivity) 1. Due to the presence of preforme or natural antiboies against major bloo group (ABO) or HLA antigen (sensitization from prior transplants, pregnancy, transfusions) . Complement an coagulation cascae is activate causing graft thrombosis an ischemia 3. Requires prompt removal of transplante organ 4. Kiney, heart, pancreas, an lung allografts all are susceptible to hyperacute rejection; however, liver grafts resist this process, so ABO compatibility is not essential for liver transplantation B. Acute: occurs in ays to months 1. Cause by cellular (macrophages an T-lymphocytes) or humoral (antiboy-meiate) response an typically requires biopsy for iagnosis . Treate with immunosuppressants, sterois, antithymocyte globulin C. Chronic: occurs in months to years (major cause of graft failure an mortality) 1. Cause by cellular (cytotoxic T-lymphocyte, helper T cell) an antiboy-meiate reactions . Graual process resulting in brosis an progressive graft ysfunction 3. Treate by increasing immunosuppression, though usually requires retransplantation II. Liver Transplantation A. Inications: en-stage liver isease seconary to alcohol (most common, replace hepatitis C), nonalcoholic steatohepatitis, primary hepatic malignancy, cholangiocarcinoma, fulminant hepatic failure; most common inication in peiatrics: biliary atresia 1. Milan criteria for hepatocellular carcinoma: single tumor less than 5 cm OR 3 tumors less than 3 cm each B. Moel for en-stage liver isease (MELD): utilizes creatinine, INR, bilirubin, an soium 1. Originally evelope for risk assessment for TIPS, now accepte as the score to prioritize organ allocation for liver transplantation . Score is 0–40; MELD > 15 is inication for liver transplant 3. Consiere a superior scoring system to Chils-Pugh because all parameters are objective C. Operative consierations: for living onors, right lobe is utilize in aults, left lateral lobe utilize in chilren (size comparable) D. Postoperative complications 1. Infection is the most common cause of eath, accounting for over 50% of mortality . Bile leak: most common complication, 10% an 30% incience, may present with abominal pain, bilious rainage, fever; early an large-volume leaks manage surgically, late or small volume leaks, or strictures, can be manage by biliary stenting or rainage 3. Hepatic artery thrombosis: most common vascular complication, high mortality an graft loss rate (>50%) 283

284

PArt i Patient Care

a) Early thrombosis presents with transaminitis, hepatic failure, bile leak (ue to breakown of biliary anastomosis), or primary nonfunction b) Late thrombosis presents as biliary stricture an/or abscesses c) Doppler ultrasoun rst-line, may be conrme with angiography, CT scan, or surgical exploration ) If ientie early (15 mm long) E. Management: 1. Start with resuscitation! Normal saline bolus, maintenance IV uis at 1.5× maintenance rate, a KCl to maintenance uis once patient urinates . Goal is for patient to be resuscitate with correction of electrolyte abnormalities prior to surgery; preop lab goals: pH 100, glucose >7; increase risk of postoperative apnea if uncorrecte alkalosis prior to surgery 3. Surgery is Freet-Ramstet pyloromyotomy: partial-thickness longituinal incision in the pylorus 1 to  mm proximal to the uoenum, exten proximally to normal antrum; recurrence most commonly a result of not extening myotomy far enough proximally to antrum

305

306

PArt i Patient Care

III. Malrotation an Migut Volvulus A. Presentation is typically bilious emesis ± abominal istention B. Diagnosis is via UGI series (uoenum oes not cross miline) C. If peritonitis is present, then avoi UGI an immeiately go to OR D. La proceure to minimize chance of future volvulus (create complete nonrotation anatomy) 1. De-torse the bowel in a counterclockwise fashion (“turn back the clock”) . Divie La bans (peritoneal attachments of right colon to paracolic gutter) 3. Straighten the uoenum an x the uoenum in the right upper quarant 4. Mobilize colon to patient’s left an x the cecum in the left lower quarant 5. Wien the base of the mesentery (key component) 6. ± Appenectomy IV. Esophageal Atresia an Tracheoesophageal Fistula A. Echocariogram to rule out associate cariac anomalies (right arch in 1%–%) B. Types:

Type

Esophageal atresia

Tracheoesophageal ęstula

Type A

Present

Absent

Type B

Present

Present; ęstula with proximal esophagus

Type C

Present

Present; ęstula with distal esophagus

Type D

Present

Present; ęstula with both proximal and distal esophagus

Type E

Absent

Present, also called H-type

C. Type C most common; treat with Immeiate repair D. Stage repair for Type A 1. Initial g-tube placement an allow patient to grow until two ens are close enough for repair V. Abominal Wall Defects

Factor

Omphalocele

Gastroschisis

Sac

Present

Absent

Location of defect

Central (through umbilicus)

To the right of the umbilicus

Umbilical cord

Inserts into sac

Normal

Defect Size

Large

Small

Contents

Bowel, liver

Bowel, gonads

Bowel

Normal

MaĴed

Malrotation

Present

Present

Small abdomen

Present

Present

GI function

Normal

Prolonged ileus

Associated anomalies

Common (30%–70%)

Unusual (atresia 15%)

Associated syndromes

Beckwith Wiedemann, Trisomy, Cantrell

Not observed

CHAPtEr 22 Pediatric Surgery

Fig. 22.1 Gastroschisis vs. Omphalocele.

Fig. 22.2 The La Proceure.

307

308

PArt i Patient Care

Questions 1. A 9-year-ol boy is seen in the emergency room with a 1-ay history of right lower quarant abominal pain an low-grae fever. On exam, he is focally tener in the right lower quarant. WBC count is 15,000/mcL an US shows a 9-mm noncompressible appenix an an appenicolith. Which of the following is true about this conition? A. If nonoperative management with antibiotics is to be consiere, a CT scan shoul be rst obtaine B. Success of nonoperative management in this patient is anticipate to be very high C. Nonoperative management tens to result in shorter hospital stay as compare to appenectomy D. Appenectomy is preferre in this patient E. Failure of nonoperative management is likely to manifest as peritonitis 2. A 10-year-ol boy is a restraine passenger in a high-spee motor vehicle collision. On arrival to the emergency epartment, his heart rate is 140 beats per minute an his systolic bloo pressure is 80 mmHg. There is an obvious eformity of his left thigh. GCS is 13. Pupils are equal an reactive. Abomen is milly tener to palpation. Focuse assessment with sonography in trauma (FAST) is positive for peritoneal ui. He is aministere 0 mL/kg of crystalloi, an BP remains 80 mmHg. Which of the following is the most appropriate next step? A. CT scan of hea/abomen/pelvis B. Start bloo prouct transfusion an transport to the OR for exploratory laparotomy C. Infuse aitional bolus of isotonic crystalloi D. Infuse lactate ringers E. Infuse 3% hypertonic saline 3. A 6-month-ol girl is brought to the trauma center for evaluation of a hea injury. Parents report that the patient rolle off a be. Which of the following injuries suggest abusive hea trauma? A. Isolate skull fracture B. Hea an neck bruising C. Subural hematoma D. Epiural hematoma E. Cortical contusion

4. A 1-ay-ol ex-7-week premature boy was previously avancing well on enteral fees. He becomes acutely istene. Initial abominal raiographs reveal moerate pneumatosis intestinalis an enteral feeings were hel. Three hours later, a repeat abominal raiograph reveals pneumoperitoneum. The patient is brought emergently to the operating room for laparotomy where three areas of necrotic bowel are encountere along with numerous other areas of patchy ischemia. What is the next best step in management? A. Resection of necrotic bowel only, with primary anastomoses B. Resection of both necrotic an patchy ischemic bowel with primary anastomosis C. Place rains without bowel resection D. Resection of all necrotic an ischemic bowel with primary anastomosis an proximal iverting stoma E. Resection of necrotic bowel only, leave in iscontinuity, secon look in about 48 hours 5. A previously healthy -month-ol girl is brought to the emergency epartment ue to a -hour history of intermittent inconsolable crying, vomiting, an apparent pain. She is not eating. Her parents brought her to the hospital after she passe a loose, maroon-colore stool. There are no signs of peritonitis on exam. WBC count is normal. Which of the following is recommene? A. CT scan of the abomen B. Laparoscopy C. Colonoscopy D. Nuclear scan E. Abominal ultrasoun 6. A 13-year-ol female presents with severe right lower quarant pain an emesis. At laparoscopy an ovarian torsion is foun. The ovary appears swollen with a blueish-black iscoloration. It remains unchange after etorsion. The next step in management is: A. Biopsy B. Oophoropexy C. Oophorectomy D. Salpingo-oophorectomy E. Close an obtain serial ultrasoun

CHAPtEr 22 Pediatric Surgery

7. A full-term baby girl has a iagnosis of a rightsie congenital lung malformation ientie on prenatal imaging. Chest raiograph in the newborn nursery shows a cystic lesion in the right lower lobe with no meiastinal shift. She is asymptomatic an on room air. What is the next step in management? A. CT scan of the chest prior to ischarge B. Discharge with CT angiogram of the chest within 6 months C. Right lower lobectomy D. Right tube thoracostomy E. Inpatient MRI of the chest 8. During laparoscopy for early acute appenicitis in a 5-year-ol boy, you n a large, right-sie renal mass. You perform an appenectomy an: A. Close, then obtain further workup B. Biopsy the mass C. Right nephroureterectomy D. Right nephroureterectomy with ipsilateral lymph noe sampling E. Right nephroureterectomy with ipsilateral lymph noe sampling an contralateral renal biopsy 9. A -week-ol boy presents with constipation an abominal bloating. He faile to pass meconium on the rst  ays of life. Contrast enema emonstrates a slightly ilate sigmoi colon with a constricte rectum. What is the next most appropriate step in management? A. Rectal irrigations an IV antibiotics B. Creation of a leveling ostomy C. Suction rectal biopsy D. Change to an elemental formula E. Obtain a UGI contrast series with small bowel follow-through 10. A newborn is in severe respiratory istress an has a markely scaphoi abomen. Which of the following is true regaring this conition? A. A chest tube shoul be promptly place B. The patient shoul be ventilate with bag-mask ventilation C. Severe cases may benet from extracorporeal membrane oxygenation D. Ventilation with high-frequency oscillation is contrainicate E. Urgent thoracotomy is require

309

11. A full-term, healthy newborn boy is note to have imperforate anus. After 4 hours, no meconium is visualize in the perineal area. The most appropriate management shoul be: A. Observation for another 4 hours B. Diverting ileostomy C. Sigmoi colostomy D. Primary repair through the perineum E. Laparoscopic primary repair 12. A -month-ol infant has persistent jaunice. Ultrasonography fails to emonstrate a gallblaer. Technetium-99m hepatobiliary iminoiacetic aci (HIDA) scanning with phenobarbital pretreatment reveals uptake in the liver but not in the intestine. α1-Antitrypsin an cystic brosis etermination is normal. The most appropriate surgical management woul be: A. Kasai operation (hepatoportoenterostomy) B. Liver transplantation C. Percutaneous transhepatic liver rainage D. Enoscopic biliary stent placement E. Choleochojejunostomy 13. A 1-ay-ol full-term infant presents with bilious emesis. Abominal x-rays show multiple loops of ilate bowel. A contrast enema shows a microcolon. What is the pathophysiology behin this obstruction? A. A fetal mesenteric vascular accient B. Failure of recanalization of the bowel C. Lack of proper rotation of the bowel D. Lack of ganglion cells in the bowel E. A uplication of a segment of bowel 14. A newborn baby is born with an abominal wall efect. The efect involves the umbilicus an has a membrane associate with it. Which of the following is true regaring this type of efect? A. This patient requires immeiate surgical closure B. Mortality is most often the result of persistent sepsis C. The etiology is ue to an umbilical vein vascular accient D. The efect is usually associate with intestinal atresia E. These patients commonly have associate cariac an genetic abnormalities 15. The most common inication for extracorporeal membranous oxygenation (ECMO) in neonates is: A. Congenital iaphragmatic hernia B. Respiratory istress synrome C. Meconium aspiration D. Persistent pulmonary hypertension E. Congenital cariac abnormalities

310

PArt i Patient Care

16. Which of the following is true regaring Bochalek type of congenital iaphragmatic hernia (CDH)? A. Urgent surgical repair is inicate upon iagnosis B. Associate pulmonary hypoplasia leas to hypocarbia C. Most efects are on the right D. Pulmonary hypertension is a prominent feature E. The iaphragmatic efect is anteromeial

21. Operative management for a patient with malrotation an migut volvulus typically inclues reuction of the volvulus, ivision of La bans, an which of the following? A. Placement of the small intestine in the left lower quarant B. Cecopexy an gastropexy C. Broaen base of the small bowel mesentery D. Placement of the cecum in the right upper quarant E. Reconstruction of the ligament of Treitz

17. A full-term baby is born with rooling, coughing, an cyanosis after the rst feeing, but these resolve quickly an spontaneously. The next step in management shoul be: A. Immeiate intubation B. Placement of orogastric tube C. Two-view abominal x-ray D. Two-view chest x-ray E. Upper gastrointestinal (UGI) contrast series

22. A full-term baby boy is note to have facial features of trisomy 1 an bilious emesis. The rest of his exam is normal. Abominal x-rays show a ouble-bubble sign with no istal gas. Which of the following is the best next step in management? A. Serial abominal x-rays B. UGI contrast stuy C. Contrast enema D. Operative exploration E. Echocariogram

18. A patient is iagnose with pyloric stenosis after 3 ays of nonbilious emesis. This patient’s electrolyte an aci/base balance will result in: A. Respiratory alkalosis B. Hyperkalemia C. Aciuria D. Hyperchloremia E. Hyponatremia 19. A 900-g premature infant evelops formula intolerance with vomiting, abominal istention, an blooy stools. Labs show an elevate white bloo cell (WBC) count an platelets of 100,000/ mcL. Abominal x-rays show ilate loops of bowel with pneumatosis intestinalis. The most appropriate treatment woul be: A. Bloo an platelet transfusions B. Antibiotics an bowel rest/ecompression C. Ultrasoun an paracentesis D. Placement of a besie peritoneal rain E. Exploratory laparotomy 20. A healthy -week-ol girl evelops bilious emesis. On exam, her abomen is nontener an nonistene. What is the most appropriate stuy to make the iagnosis? A. -view abominal x-ray B. Ultrasoun C. UGI series D. Contrast enema E. Compute tomography (CT) scan of abomen/pelvis

23. A -year-ol chil presents with an abominal mass, “raccoon eyes,” an “blueberry mufn” skin lesions. These most likely represent: A. Rhabomyosarcoma B. Neuroblastoma C. Wilms tumor D. Hepatoblastoma E. Teratoma 24. The most common anomaly associate with gastroschisis is: A. Cariac B. Renal C. Limb D. Malrotation E. Down synrome 25. A newborn baby is born with a istene abomen an bilious emesis. Both parents are carriers for cystic brosis. On examination, the patient has a istene but soft abomen. Abominal x-rays show ilate loops of bowel with a groun-glass appearance. The most appropriate initial management is: A. Water-soluble contrast enemas B. Resection of terminal ileum with stoma C. Resection of terminal ileum with primary anastomosis D. UGI with small bowel follow-through E. Small bowel enterotomy with evacuation of meconium

CHAPtEr 22 Pediatric Surgery

311

26. A 6-month-ol boy presents to the ED crying in pain an has bilious emesis. On exam, he has a istene abomen, an there is a tener mass in the right groin. Appropriate management woul be: A. Ultrasoun of right groin B. Besie incision an rainage (I&D) of right groin C. IV antibiotics D. Attempt reuction E. Operative exploration

30. A newborn baby with a prenatal iagnosis of gastroschisis is born with the entire small intestine outsie of the abomen. The bowel appears ischemic an the abominal wall efect is small an tight. The most appropriate next step in management is: A. Place a besie silo B. Primary reuction an closure C. Open the abominal wall efect D. Resect the ischemic bowel E. Create a iverting ileostomy

27. A 4-year-ol girl presents with recurrent jaunice. Ultrasoun shows a 5-cm fusiform ilation of the common bile uct. During surgery, the posterior aspect of the cystic mass is rmly aherent to the portal vein. The most appropriate management is: A. Abort surgery, IV antibiotics, an reoperate in 3 months B. Place a rain into the cyst, IV antibiotics, an reoperate in 3 months C. Resect the anterior cyst, mucosectomy of the posterior cyst with reconstruction D. Internal rainage of the cyst with a Roux-en-Y cystojejunostomy E. Resect the cyst an portal vein with reconstruction of the portal vein an common bile uct (CBD)

31. A 1-week-ol full-term baby with abominal istention, fever, tachycaria, an low urine output is transferre to the NICU. The patient has not passe meconium. He ha a suction rectal biopsy showing aganglionosis. Digital rectal examination shows explosive, foul-smelling liqui stools. Despite broa-spectrum IV antibiotics an rectal irrigation, he is clinically eteriorating. The next step in management is to: A. Perform contrast enema B. Perform loop colostomy C. Perform subtotal colectomy an ileostomy D. Perform abominal ecompression for abominal compartment synrome E. A aitional antifungal coverage

28. A -week-ol, ex-5-week premature boy is in the neonatal ICU (NICU) an is iagnose with a left inguinal hernia. His current weight is 1 kg an he requires supplemental oxygen. The hernia is easily reucible. The next appropriate step in management is: A. Ultrasoun evaluation B. Immeiate open operative repair C. Immeiate laparoscopic repair D. Repair just prior to ischarge E. Delay repair until 1 year of age 29. The pathogenesis of necrotizing enterocolitis (NEC) is thought to be relate to: A. A genetic preisposition B. An enzyme eciency C. A perio of intestinal hypoperfusion D. Preexisting intestinal atresia E. An antibiotic reaction

32. A 4-week-ol infant presents with bilious vomiting, irritability, abominal wall eema, an erythema. Plain lms reveal proximal ilate bowel, with a paucity of istal bowel gas. Which is true regaring this patient? A. An urgent UGI series is inicate B. A trial of nasogastric tube ecompression is often helpful C. Enoscopic ecompression is often benecial D. A CT scan of the abomen an pelvis shoul be obtaine E. Delay in management may lea to a nee for intestinal transplantation 33. A neonate is foun to have bilateral unescene testes that are not palpable in the inguinal canal. Which of the following is true regaring this conition? A. A bilateral orchiopexy shoul be performe by 1 year of age B. Orchiopexy oes not improve fertility potential C. It is not associate with prune belly synrome D. Chorionic gonaotropin oes not ai in testicular escent E. The testicular arteries must be preserve uring operation

312

PArt i Patient Care

Answers 1. D. Appenectomy is the preferre treatment strategy

for this patient because of the presence of a fecolith. While appenectomy has been the gol stanar for the treatment of uncomplicate appenicitis, multiple stuies have emonstrate that nonoperative management of uncomplicate appenicitis is safe an effective. Nonoperative management consists of initial broa-spectrum IV antibiotics an IV uis. Patients can be transitione to oral antibiotics an ischarge when their pain improves, fever resolves, an they are able to tolerate a iet. The total antibiotic course shoul be 7 ays. Nonoperative management is initially successful in 85% to 9% of patients. Patients manage nonoperatively have a higher reamission rate within 1 year, primarily ue to recurrent appenicitis. Due to recurrence of appenicitis, the 1-year success rate of avoiing appenectomy is 67%. Most woul recommen only offering nonoperative management in chilren age 5 to 17 years meeting the following criteria: uncomplicate appenicitis conrme on imaging (US, CT, or MRI) (A), WBC between 5000 an 18,000, pain for 40-years, systolic bloo pressure < 90 mmHg, abnormal motor posturing . External ventricular rain (EVD) is preferre as it is iagnostic an therapeutic D. Approach to management of elevate ICP 1. In orer of intervention to be attempte: hea of be to 30 egrees, seation, hypertonic saline or mannitol (contrainicate if systemic hypotension), short-term mil hyperventilation (PaCO2 30–35 mmHg), ventricular rainage, barbiturates, paralysis, an ecompressive craniectomy . Hypertonic saline an/or mannitol shoul both be given as boluses an not continuous infusions as they will equilibrate an thus become ineffective; the goal of these interventions is to create an acute osmotic isequilibrium, which can only be achieve with a bolus E. Nutrition: start enteric feeing within 4 to 48 hours, postpyloric preferre F. Venous thromboembolism (VTE) prophylaxis: very high risk of VTE in TBI 1. Brain Trauma Founation guielines (016) leave the timing an choice of agent to the clinician’s jugment; however, most start low-molecular weight heparin (LMWH) 48 hours after the last stable CT G. Anticoagulation reversal agents

Drug

Reversal

Half-life (hrs)

Warfarin

Prothrombin complex concentrate (preferred), vitamin-K

20–60

Dabigatran

Idarucizumab

10–20

Edoxaban

Andexanet alpfa

10–15

Rivaroxaban

Andexanet alpfa

5–10

Apixaban

Andexanet alpfa

10–12

357

358

PArt i Patient Care

II. Spinal Cor Injuries

Syndrome

Epidemiology

Aěected spinal tracts

Clinical presentation

Prognosis

Central cord

Most common incomplete spinal cord injury syndrome, commonly in elderly with cervical spondylosis and spinal stenosis, hyperextension injury

Bilateral central corticospinal and lateral spinothalamic tracts

Motor deęcits in upper extremities more than lower extremities

Good prognosis although full functional recovery is rare

Anterior cord

Infarction of anterior spinal artery or trauma (e.g., penetrating trauma, burst fracture of vertebra, Ěexion injury)

Corticospinal and spinothalamic tracts

Motor loss, pain, and temperature loss (proprioception and vibratory sense preserved)

Worst prognosis of incomplete syndrome; low chance (10%–20%) of motor recovery

Posterior cord

Very rare, caused by infarction of posterior spinal artery, trauma (e.g., penetrating trauma), multiple sclerosis

Posterior columns

Loss of proprioception, light touch, vibratory sense (motor, pain, and temperature sensation preserved)

Recovery variable and related to completeness of lesion

BrownSéquard

Most commonly due to penetrating injury

Hemisection of the cord

Ipsilateral motor and proprioception loss, contralateral pain and temperature loss

Best prognosis for functional motor activity recovery (99% ambulate)

Questions 1. Which of the following is true regaring the management of severe traumatic brain injury (TBI) in aults? A. A CT scan is require prior to placement of an intracranial monitoring evice B. External ventricular rains (EVD) are preferre over intraparenchymal intracranial pressure monitors if both are available C. The goal cerebral perfusion pressure (CPP) is greater than 40 mmHg D. Decompressive craniectomy oes not lower mortality in cases of refractory intracranial hypertension as compare to meical management E. Heparin is the preferre agent for VTE chemoprophylaxis

2. Which of the following is true regaring gunshot wouns to the hea? A. Suicie attempts have the same mortality rate as assaults or accients B. The incience of vascular injury is low C. Extene antibiotic prophylaxis is recommene D. Bihemispheric injuries are a signicant risk factor for mortality E. GCS on arrival is not a signicant preictor of mortality 3. Which of the following is true regaring primary brain tumors? A. Meulloblastomas are the most common malignant tumors in aults B. Aults with glioblastoma have a 5-year survival rate of aroun 30% C. Corticosterois are use for symptomatic peritumoral vasogenic eema D. Brain tumors in infants typically present with focal neurologic ecits E. In chilren over the age of 10, infratentorial tumors are more common than supratentorial

CHAPtEr 27 Nervous System

4. A 6-year-ol intubate male is opening his eyes to voice an attempts to open his mouth. His only consistent motor movement is to occasionally withraw from painful stimuli. What is his current GCS score? A. GCS 4T B. GCS 8T C. GCS 9T D. GCS 11T E. GCS 13T 5. An 88-year-ol female is brought by ambulance to the ED after being struck by a vehicle while crossing the street. She is only responsive to painful stimuli an is promptly intubate for airway protection. Her seconary exam reveals only a small abrasion to the left forehea. Her systolic bloo pressure suenly increases to the 00s, an her left pupil becomes ilate an unresponsive to light. What is the next best course of action? A. Hypertension control with nicaripine continuous infusion B. Placement of intraparenchymal intracranial pressure monitor C. Immeiate mannitol bolus D. Rectal lorazepam an initiation of levetiracetam E. Raise hea of be 6. A 17-year-ol boy presents to the ED via ambulance after new-onset seizure activity that starte 30 minutes ago. He is unable to provie a goo history because of wor ning issues but is able to convey that his hea hurts. His parents state that he felt completely normal until about 4 weeks ago when he began to complain of left ear pain. Vital signs reveal a mil tachycaria an high fever. Physical exam shows absent light reex in the left eye an papilleema. Which of the following is contrainicate in the workup an subsequent treatment of his conition? A. Lumbar puncture B. Compute tomography with intravenous contrast C. Stereotactic neele aspiration D. Surgical ebriement E. Corticosterois

359

7. Which of the following is true regaring Cushing tria? A. The pulse pressure narrows B. The heart rate increases C. It oes not lea to changes on electrocariogram D. It is associate with hypocarbia E. It is a late manifestation of increase intracranial pressure 8. Which of the following is true regaring rupture intracranial aneurysms? A. Following repair, ui restriction is recommene B. Most arise from the posterior circulation C. The initial stuy of choice is a contrastenhance hea CT D. Following repair, the risk of cerebral vasospasm causing stroke persists for 3 weeks E. Outcomes are overall quite favorable 9. An 85-year-ol female presents to the ED after falling an striking her chin on the kitchen counter. She is unable to lift her arms or hans off the be an oes not respon to painful stimuli. However, she is able to wiggle her toes an seems to feel pain at her feet. She has a history of cervical raiculopathy. A igital rectal exam reveals goo sphincter tone an squeeze pressure. What is the most likely incomplete spinal cor injury that she has sustaine? A. Posterior cor synrome B. Anterior cor synrome C. Caua equina synrome D. Brown-Séquar synrome E. Central cor synrome 10. Which of the following is true regaring hea trauma an/or intracranial hemorrhage? A. The most common cause of subarachnoi hemorrhage is rupture of a berry aneurysm B. Epiural hematoma is typically associate with acceleration-eceleration injuries C. A single episoe of systolic bloo pressure (BP) less than 90 mmHg oubles the mortality rate in patients with hea trauma D. Xanthochromia is virtually pathognomonic for acute subural hemorrhage E. In the absence of other nings, reimaging for cerebral contusion is generally unnecessary

360

PArt i Patient Care

11. A 5-year-ol male is being evaluate in the emergency epartment (ED) after sustaining a blow to the hea with an unknown object uring an assault. He has a 6 cm, stellate laceration with an unerlying scalp hematoma. Compute tomography (CT) scan shows evience of a skull fracture. In which of the following situations can this patient be manage nonoperatively? A. Fracture penetrates ura but not brain B. 0.5 cm of skull epression C. Involvement of the frontal sinus only D. Pneumocephalus E. Gross woun contamination 12. A 45-year-ol female arrives at the ED after iving hea-rst into a half-empty swimming pool. She is combative an appears intoxicate. She is not able to move her lower extremities or trunk. You observe her lifting her arms an bening at the elbows but are unable to assess any movement in her hans. It has been 30minutes since she rst sustaine her injury. Which of the following is true regaring this patient? A. The likely site of her injury is C3-C4 B. In the absence of other injuries, methylprenisolone shoul be aministere immeiately C. This is a rare spinal cor injury after a iving accient D. Anticoagulation shoul be starte within  to 3 ays an continue for  to 3 months E. Mean arterial pressure shoul be maintaine between 65 an 75 mmHg for the rst 7 ays

13. Neurogenic thoracic outlet synrome most commonly affects which nerve? A. Raial B. Ulnar C. Meian D. Musculocutaneous E. Axillary 14. A 4-ay-ol female infant weighing 1400 g born at 8 weeks’ gestation is being monitore in the neonatal critical care unit because of multiple episoes of apnea an ifculty with feeing. Supplemental oxygen has been sufcient to maintain saturations. Over the last several hours, she has ha waxing an waning alertness an ecrease spontaneous eye movements. Her fontanelle appears to be full. Which of the following is the most appropriate next step? A. Immeiate aministration of furosemie an acetazolamie B. Besie intracranial ultrasoun C. Lumbar puncture D. Noncontrast CT of hea E. Aminister IV steroi bolus

Answers 1. B. One of the rst ecision points in managing a

patient with severe TBI is the placement of an intracranial pressure (ICP) monitor. ICP monitors are inicate in patients with a CT scan showing intracranial hemorrhage an who have a GCS of less than 8 (or higher than 8 but with a high risk of progression). Aitionally, ICP monitors are also inicate in patients with a low GCS who are having emergent extracranial surgery (A). A CT scan is not neee in this scenario. Though EVDs an intraparenchymal pressure monitors can both be use to measure ICP, EVDs are preferre as they are both iagnostic an therapeutic (B). Once an ICP monitor is place, CPP can be calculate with CPP = mean arterial pressure (MAP)−ICP. The goal CPP is greater than 60 mmHg in aults (C). However, the goal CPP is >40 mmHg for peiatrics patients.

All efforts shoul be mae to maintain an aequate CPP with techniques incluing seation, ventricular rainage, mannitol, hypertonic saline, an paralytics. If intracranial hypertension persists espite these measures, ecompressive craniectomy is often utilize, though there is still some controversy regaring its outcomes. In a ranomize controlle trial in 016, ecompressive craniectomy for refractory intracranial hypertension resulte in lower mortality compare to meical treatment alone (D). Although Brain Trauma Founation guielines leave the choice of VTE chemoprophylaxis to the clinician’s jugement, a national atabase stuy incluing over 10,000 patients emonstrate LMWH to be associate with reuce mortality an thromboembolic complications, regarless of timing of prophylaxis initiation in severe TBI patients (E).

CHAPtEr 27 Nervous System References: ACS Trauma Quality Improvement Program. Best Practices in the Management of Traumatic Brain Injury. American College of Surgeons, Committee on Trauma; January 015. https:// www.facs.org/-/meia/files/quality-programs/trauma/tqip/ tbi_guielines.ashx. Kolias PJ, Timofeev AG, IS, et al. Trial of ecompressive craniectomy for traumatic intracranial hypertension. N Engl J Med. 016;375(1):1119–1130. Benjamin E, Recinos G, Aiol A, Inaba K, Demetriaes D. Pharmacological thromboembolic prophylaxis in traumatic brain injuries: low molecular weight heparin is superior to unfractionate heparin. Ann Surg. 017;66(3):463–469.

2. D. Gunshot wouns to the hea have a high morbiity

an mortality. In a large meta-analysis, factors preictive of mortality inclue age greater than 40 years, GCS less than 9 on arrival, xe an ilate pupils, ural penetration, bihemispheric injuries, multilobar injuries, tranventricular injuries, an suicie attempts (A, D, E). In fact, suicies ha a six times higher rate of mortality compare to assaults or accients. Another interesting ning in this stuy was that vascular injuries were very common (38%–50%) with intracranial aneurysm, arterial issection, arterial occlusion, an arteriovenous stulas being the most common types in escening orer of incience (B). There is a lack of high-quality evience regaring the management of this type of injury. Though surgery is associate with lower mortality, it is unclear whether this is a result of surgery itself or ue to patient selection. The rate of CNS infection after penetrating TBI is less than 10% an there is no reuction in the risk of infection with prophylactic antibiotics (C). However, surgical intervention an ICP monitoring appear to be risk factors for infection, regarless of prophylactic use. References: Maragkos GA, Papavassiliou E, Stippler M, Filippiis AS. Civilian gunshot wouns to the hea: prognostic factors affecting mortality: meta-analysis of 1774 patients. J Neurotrauma. 018;35():605–614. Harmon LA, Haase DJ, Kufera JA, et al. Infection after penetrating brain injury-An Eastern Association for the Surgery of Trauma multicenter stuy oral presentation at the 3n annual meeting of the Eastern Association for the Surgery of Trauma, January 15–19, 019, in Austin, Texas. J Trauma Acute Care Surg. 019;87(1):61–67.

3. C. The types an presentations of brain tumors are signi-

cantly ifferent in chilren an aults. In aults, the majority of tumors are benign, with meningiomas being the most common. The most common malignant tumor is glioblastoma, which carries a 5-year survival rate of 5% (A, B). The management is typically focuse on maximal resection an is sometimes followe by raiation. Other consierations are seizure management an corticosteroi use for symptomatic peritumoral vasogenic eema (C). In chilren, brain tumors are relatively more common an are the most common cause of eath among chilhoo cancers. In chilren up to 14 years ol the most common brain tumor is a glioma, but pituitary tumors are the most common in chilren 15 years an oler. The most common malignant brain tumor in chilren is a meulloblastoma. The location of brain tumors in chilren also varies by age, with chilren age 4 to 10 years ol being more likely to have infratentorial tumors. All other ages are more likely to have supratentorial tumors (E). Supratentorial tumors ten to present with focal neurologic ecits epening on the exact location, an infratentorial tumors ten to

361

have cranial nerve palsies or cerebellar ysfunction. The caveat to this rule is in infants (who will not noticeably isplay these ecits) who more commonly present with macrocephaly, irritability, failure to thrive, loss of evelopmental milestones, an vomiting (D). References: Lapointe S, Perry A, Butowski NA. Primary brain tumours in aults. Lancet. 018;39(10145):43–446. Uaka YT, Packer RJ. Peiatric brain tumors. Neurol Clin. 018;36(3):533–556.

4. B. The Glasgow Coma Scale uses the combine scores

from the motor, verbal, an speech sections to give an estimate of a patient’s level of functional status. The scoring is as follows. For eye opening: 4: Spontaneously, 3: To verbal comman, : To pain, 1: No response. Best motor response scores: 6: Obeys comman, 5: Localizes pain, 4: Flexion withrawal, 3: Flexion abnormal (ecorticate), : Extension (ecerebrate), 1: No response, an for Best verbal response: 5: Oriente an converses, 4: Disoriente an converses, 3: Inappropriate wors; cries, : Incomprehensible souns, 1: No response. If the patient is intubate, the maximum score that he or she can get in the verbal category is 1T (the letter T inicating intubate) an maximum overall score of 11T. This patient opens his eyes to voice commans but not spontaneously, which correlates with an eye score of 3. The best calculate motor score is a 4 for withrawing from pain. This places his total GCS at 1T (verbal) + 3 (eye opening) + 4 (motor) = 8T.

5. C. Without a CT scan, one cannot be sure of the exact eti-

ology of these neurologic nings, but, base on the history an physical exam nings, this likely represents a close hea injury with an elevate intracranial pressure (ICP). A “blown” pupil in the setting of hea trauma is consistent with uncal herniation, which is often fatal an will cause permanent neurologic ecits if not treate promptly. Systolic bloo pressure greater than 180 mmHg can aggravate vasogenic brain eema an intracranial hypertension. However, systemic hypertension may be a physiologic response to reuce cerebral perfusion. Thus, early an aggressive treatment of hypertension shoul be avoie until ICP monitoring has been establishe (A). While this patient likely nees an ICP monitor, a iagnosis still nees to be mae before surgical treatment or invasive monitoring (B). Aitionally, an external ventricular rain is a better choice in this patient because it allows therapeutic rainage of cerebrospinal ui. Current inications for a mannitol bolus are for situations just like the above—a quick bailout maneuver to be use as a brige to more enitive therapies. Mannitol immeiately improves cerebral perfusion ue to the fact that it ecreases bloo viscosity an therefore increases cerebral bloo ow an cerebral oxygen elivery. Its osmotic properties take 15 to 30 minutes to work. There is some evience that prolonge or scheule use will rener it ineffective at best an potentially harmful. Immeiately following mannitol, the patient nees a CT scan an shoul be evaluate for possible surgical rainage of an intracranial hematoma. Lorazepam an levetiracetam (Keppra) are both meications use for the treatment of seizures, which is not consistent with her exam at this time (D). Raising the hea of the be can lower ICP, but with a blown pupil, the patient nees more aggressive treatment (E).

362

PArt i Patient Care

Reference: Brain Trauma Founation, American Association of Neurological Surgeons, Congress of Neurological Surgeons. Guielines for the management of severe traumatic brain injury. J Neurotrauma. 007;4 Suppl 1:S91–S95.

6. A. The tria of heaache, focal neurologic ecits, an

fevers shoul raise concern for brain abscess; however, this classic presentation is present in less than half of all patients. The most common presenting symptom is a heaache, which is present in approximately 70% of patients. They arise primarily by two forms of sprea: hematogenously from istant sites an irect sprea from contiguous sites of infection (otitis meia being most common). This leas to a wie array of potential pathogens, though the most common are Streptococcus spp. an Staphylococcus spp. Initial iagnosis shoul be obtaine by CT scan with contrast, which will show a rim-enhancing collection (B). Lumbar puncture is generally not iagnostic an contrainicate in the setting of elevate ICP. Changes in cerebrospinal ui volume in this setting can precipitate herniation. All patients shoul be starte on broa-spectrum antibiotics, which can be tailore once cultures are obtaine. Total uration of treatment is typically 4 to 6 weeks. Traitional management inclue surgical rainage an excision of the abscess cavity (D). However, serial neele aspiration has now become the treatment of choice unless the abscess is traumatic in origin (potentially has foreign ebris), fungal, multiloculate, or oes not improve with neele aspiration (C). Corticosterois are controversial in this setting but may be consiere when there is substantial mass effect from the abscess (E). References: Brouwer MC, Coutinho JM, van e Beek D. Clinical characteristics an outcome of brain abscess: systematic review an meta-analysis. Neurology. 014;8(9):806–813. Muzumar D, Jhawar S, Goel A. Brain abscess: an overview. Int J Surg. 011;9():136–144.

7. E. Cushing tria is a vasomotor an respiratory response

to an elevate ICP that inclues braycaria, irregular breathing, an elevation in systolic bloo pressure with a wiene pulse pressure (A). The increase ICP leas to impaire respiration, which worsens hypercarbia (D). Typically, Cushing tria is a late sign of elevate ICP an suggests imminent herniation. In aition to braycaria on ECG, Mayer waves can be seen with elevate ICP (B). The waves are cyclic changes in arterial bloo pressure brought about by oscillations in baroreceptor an chemoreceptor reex control systems an are note on ECG (C).

8. D. Intracranial aneurysms affect 4% of the population

but are asymptomatic in the majority of cases, an most patients are unaware of the iagnosis. Risk factors inclue female gener, polycystic kiney isease, an Marfan synrome. The majority of the aneurysms occur in the circle of Willis with the anterior communicating artery being the most frequent site (B). When the aneurysm ruptures, it can result in intraparenchymal an subarachnoi hemorrhage, which is a catastrophic event with a mortality rate up to 50% (E). Noncontrast CT hea is the stuy of choice to conrm the iagnosis (C). Bleeing on brain parenchyma elicits a vasospasm response, which can result in stroke an patients are at increase risk for 1 ays; thus, most neurosurgeons will start calcium channel blockers. Because

cerebral autoregulation is compromise, these patients shoul be given volume to maintain aequate cerebral perfusion pressure (A). Reference: Keey A. An overview of intracranial aneurysms. McGill J Med. 006;9():141–146.

9. E. Central cor synrome is the most common type of

incomplete spinal cor injury an is primarily foun in patients that suffere a hyperextension injury in the setting of previous cervical spine abnormalities. Symptoms inclue muscle weakness of the upper extremities with relative sparing of the lower extremities. Sensory function is variable. Posterior cor synrome is a relatively rare entity typically cause by infarction of the posterior spinal artery. Classic presentation inclues sparing of muscles with the loss of proprioception an vibration sensation below the level of the lesion with preservation of most motor function (A). Anterior cor synrome can be cause by either infarction of the anterior spinal artery or, less frequently, by fracture or islocation of vertebrae. It is characterize by loss of motor function, pain sensation, an temperature sensation but preservation of touch an proprioception (B). Caua equina synrome can be cause by trauma, mass lesions, or lumbar spinal stenosis an occurs at the level that the spinal cor has split into nerve roots. Symptoms can be variable but generally inclue paresthesia of the perineum, anus, an external genitalia (“sale anesthesia”), bilateral or unilateral paralysis, an incontinence of bowel an blaer (C). Brown-Séquar synrome is hemisection of the spinal cor from a mass lesion or more commonly trauma. It causes an ipsilateral loss of motor, proprioception, an vibration sensation with contralateral loss of pain an temperature sensation (D).

10. C. Traumatic brain injuries are among the most com-

mon presenting symptoms in emergency epartments in the Unite States, with over 1.7 million amissions each year. The early recognition an management of brain injury is critical in this patient population because it is consiere the most common cause of trauma-relate eath in patients reaching the hospital alive. Preventing seconary injury is an important part of management, an this involves maintaining cerebral perfusion pressure greater than 60 mmHg. One prospective trial foun that a single episoe of hypotension with a systolic bloo pressure of less than 90 mmHg ouble mortality in patients with brain injury. Trauma is consiere the most common etiology of subarachnoi hemorrhage, followe by rupture of berry aneurysms (A). In nontraumatic cases, patients may report mil “sentinel” heaaches in the prior weeks leaing up to a severe, unrelenting, “thunerclap” heaache. Noncontrast compute tomography (CT) scan is the iagnostic tool of choice to look for hyperensities suggestive of acute bleeing. Aitionally, xanthochromia of cerebrospinal ui is consiere pathognomonic for subarachnoi hemorrhage (D). Epiural hematoma is generally the result of irect trauma to the skull causing isruption of arterial vessels, particularly the mile meningeal artery. It initially presents with unconsciousness from the concussive effects of the injury, followe by a “luci” interval that progresses to somnolence, lethargy, an eventually a coma as the hematoma grows. Noncontrast CT scan will emonstrate a lentiform (biconvex), hyperense clot that oes not

CHAPtEr 27 Nervous System cross suture lines. Acute subural hematoma is generally the result of acceleration-eceleration injuries that tear the briging veins as the brain shifts in relation to the ura (B). Patients are often unconscious from the moment of impact. Noncontrast CT scan will emonstrate a hyperense, lunar (crescent-shape) lesion that oes not cross the miline. Cerebral contusion is ue to the brain irectly striking the skull in either a coup or countercoup mechanism after a close hea injury. Lesions on noncontrast CT scans are typically scattere, hyperense, an intraparenchymal, though they can also present as hypoense lesions. There is a signicant propensity for these lesions to worsen, an repeat imaging is typically recommene in the rst 4 hours (E). References: Chesnut RM, Marshall LF, Klauber MR, et al. The role of seconary brain injury in etermining outcome from severe hea injury. J Trauma. 1993;34():16–. Faul M, Xu L, Wal MM, Coronao VG. Traumatic brain injury in the United States: emergency department visits, hospitalizations and deaths 2002–2006. Centers for Disease Control an Prevention, National Center for Injury Prevention an Control; 010. https://www.cc. gov/traumaticbraininjury/pf/blue_book.pf

11. B. Any skull fracture with an overlying laceration is

consiere an open fracture. Traitional teaching is that all of these patients shoul be taken to the operating room to prevent infection. However, there seems to be a subset of patients that can be treate expectantly without signicant increases in morbiity. Nonoperative management of open skull fracture can be consiere in patients without evience of ural penetration, signicant intracranial hematoma, frontal sinus involvement, woun infection, pneumocephalus, or gross woun contamination (A, C–E). Aitionally, patients with less than 1 cm of skull epression can be manage nonoperatively. Reference: Bullock MR, Chesnut R, Ghajar J, et al. Surgical management of epresse cranial fractures. Neurosurgery. 006;58 (3 Suppl):S56–60.

12. D. Although it is ifcult to ascertain the exact level

of spine injury in a noncooperative patient, complete paralysis of the lower extremities an the trunk with preservation of her shoulers an elbows most likely inicates an injury at C5 or below (A). The most common spinal injury after a iving accient is C5 followe by C6 (C). The use of sterois in spinal cor injury has been controversial. However, recent level 1 evience recommens against the use of sterois in the management of acute spinal cor injury (B). Among trauma victims, patients with spinal cor injury an hea injury have the highest risk of venous thromboembolic events (VTEs). Without prophylaxis, the risk of VTE is about 40% after complete spinal cor injury. Mechanical prophylaxis with compression evices shoul be starte immeiately. Anticoagulation shoul be starte within 7 hours an continue for  to 3 months. Low-molecular-weight heparin is preferre over heparin. Mean arterial pressure shoul be maintaine between 85 an 90 mmHg for the rst 7 ays (E).

363

References: Bailes JE, Herman JM, Quigley MR, et al. Diving injuries of the cervical spine. Surg Neurol. 1990;34(3):155–158. Theoore N, et al. Guielines for the management of acute cervical spine an spinal cor injuries: 013 upate. Neurosurgery. 013;7():1–59.

13. B. Neurologic symptoms occur in 95% of cases of tho-

racic outlet synrome. The lower  nerve roots of the brachial plexus, C8 an T1, are most commonly (90%) involve, proucing pain an paresthesias in the ulnar nerve istribution (A, C–E). The secon most common anatomic pattern involves the upper three nerve roots of the brachial plexus, C5, C6, an C7, with symptoms referre to the neck, ear, upper chest, upper back, an outer arm in the raial nerve istribution.

14. B. Intraventricular hemorrhage (IVH) occurs in approx-

imately 15% to 0% of infants born with a birth weight of less than 1500 g. Because of the frequency of this conition, serial ultrasoun screening is recommene for all premature infants an any infants that show signs of IVH. In premature infants, the relative fragility of the germinal matrix makes them sensitive to changes in cerebral bloo ow with subsequent hemorrhage into the ventricles. Preisposing factors in aition to prematurity inclue maternal chorioamnionitis or preeclampsia, an neonatal respiratory istress, hypotension, or anemia. While 5% to 50% of infants can have clinically silent IVH, symptoms range from nonspecic changes in alertness to stupor or coma. Once it has been iagnose, management is largely supportive to prevent long-term complications such as posthemorrhagic hyrocephalus (PHH). Prior to the avent of intracranial ultrasoun, CT scan was utilize to make the iagnosis, but has now been largely abanone (D). Once the iagnosis is establishe, treatment is supportive, incluing correction of anemia (patients can suffer major bleeing), hypotension, aciosis, an ventilatory support. Treatments to try to prevent hyrocephalus have been largely ineffective. Though furosemie an acetazolamie have been use in oler chilren with PHH, they o not seem to alter the course in premature infants an coul potentially be eleterious (A). Serial lumbar puncture has been trie with no signicant change in eterioration or progression to permanent ventricular rainage proceures (C, E). Temporary ventricular rainage with transition to permanent rainage proceures if necessary is currently the treatment of choice for PHH with elevate intracranial pressures. Ultimately, if signicant hyrocephalus persists, the infant may nee a ventriculoperitoneal shunt. References: Mazzola CA, Chouhri AF, Auguste KI, et al. Peiatric hyrocephalus: systematic literature review an evience-base guielines. Part : management of posthemorrhagic hyrocephalus in premature infants. J Neurosurg Pediatr. 014;14 Suppl 1:8–3. Robinson S. Neonatal posthemorrhagic hyrocephalus from prematurity: pathophysiology an current treatment concepts: a review. J Neurosurg Pediatr. 01;9(3):4–58.

PART II

MEDICAL KNOWLEDGE

Anesthesia ERIC O. YEATES AND CATHERINE M. KUZA

28

ABSITE 99th Percentile High-Yields I. American Society of Anesthesiologists Physical Status (ASA PS)

ASA PS

Deęnition

Examples

I

Normal healthy patient

Young with no comorbidities

II

Mild systemic disease

Well-controlled hypertension, current smoker

III

Severe systemic disease

Poorly controlled hypertension, morbid obesity, stable angina, prior myocardial infarction, controlled congestive heart failure with no symptoms, end-stage renal disease on scheduled dialysis

IV

Severe systemic disease that is a constant threat to life

Unstable angina, congestive heart failure with symptoms, sepsis, end-stage renal disease not on scheduled dialysis

V

Moribund patient not expected to survive without the operation

Ischemic bowel, intracranial hemorrhage with midline shift, ruptured abdominal aortic aneurysm

VI

Brain-dead patient undergoing organ donation

Traumatic brain-injured patient with no brain stem reĚexes

II. Opioi Equivalents

Opioid

Oral

Parenteral

Morphine

30 mg

10 mg

Hydromorphone

7.5 mg

1.5 mg

Hydrocodone

30 mg

N/A

Fentanyl

N/A

0.1 mg

Codeine

200 mg

N/A

Oxycodone

20 mg

N/A

Tramadol

120 mg

N/A

365

366

PArt ii Medical Knowledge

III. Local Anesthetic Maximum Dose

Medication

Without epinephrine

With epinephrine

Lidocaine

5 mg/kg

7 mg/kg

Bupivacaine/Ropivacaine

2.5 mg/kg

3 mg/kg

0.5% = 5 mg/ml, 1% = 10 mg/ml, 2% = 20 mg/mg

IV. Common Intravenous Inuction Agents

Medication

Mechanism of action

Side eěects

Propofol

GABA agonist

Hypotension, propofol infusion syndrome

Etomidate

Mechanism unclear, modulates or activates GABA

Adrenal suppression, myoclonus

Ketamine

NMDA receptor antagonist

Emergence delirium, hypertension, arrhythmias, increased intracranial pressure

Midazolam

GABA agonist

Nausea, vomiting, delirium

V. Neuromuscular Blocking Drugs for Rapi Sequence Intubation

Medication

Mechanism of action

Onset (min)

Histamine release

Succinylcholine

Depolarizing

1–1.5

Rocuronium

Nondepolarizing

Vecuronium Cisatracurium/ atracurium

Reversal agent

Metabolism

Side eěects

Yes

Plasma cholinesterase

Hyperkalemia, malignant hyperthermia, bradycardia

N/A

1.5–3

Yes

Hepatic, metabolites excreted renally

Allergy

Sugammadex

Nondepolarizing

3–4

No

Hepatic

Allergy

Sugammadex

Nondepolarizing

5–7

No

Hofmann elimination

Bronchospasm

Neostigmine

VI. Steroi Potency: hyrocortisone < prenisone < methylprenisolone < examethasone VII. Malignant Hyperthermia: rare, severe reaction to meications use uring general anesthesia A. Genetics: rare, autosomal ominant isorer cause by a mutation in the ryanoine receptor, locate on the sarcoplasmic reticulum (in skeletal muscle) B. Triggering meications: volatile anesthetics (halothane, sevourane, esurane, isourane, enurane) or epolarizing muscle relaxants (succinylcholine, ecamethonium) C. Signs/symptoms: can occur immeiately an as late as 4 hours postoperatively 1. Hyperthermia, tachycaria, increase en-tial CO, muscle rigiity, rhabomyolysis, lactic aciosis D. Diagnosis: acutely, the iagnosis is clinical 1. Conrmatory testing or testing of close relatives who have suffere from malignant hyperthermia; this inclues a skeletal muscle biopsy followe by a caffeine-halothane contracture test (CHCT); the muscle is expose to halothane an caffeine with a positive test causing signicant muscle contraction; testing must take place in centers specialize in iagnosing malignant hyperthermia E. Treatment: stop all anesthetics, aminister .5 mg/kg of IV antrolene which inhibits calcium ion release from the sarcoplasmic reticulum (can aminister aitional 1–.5 mg/kg boluses, max cumulative ose of 10 mg/kg), cooling, correction of hyperkalemia, an ui resuscitation. F. Outcomes: mortality approximately 5%

CHAPtEr 28 Anesthesia

VIII. Propofol Infusion Synrome: rare synrome triggere by high ose (>4 mg/kg/hr) infusion >48 hours A. Mechanism: unknown, but possibly ue to the impairment of fatty aci metabolism B. Risk factors: chilren, concomitant catecholamine or steroi infusion, severe critical illness C. Signs/symptoms: metabolic aciosis, arrhythmias (most often braycaria), rhabomyolysis, hyperlipiemia, hepatomegaly (not splenomegaly), renal failure, cariovascular collapse D. Treatment: immeiate cessation of propofol, early hemoialysis, supportive care E. Screening tool: aily CPK an lactate levels

IX. Differential Diagnosis of Intraoperative Changes in En-Tial CO

Rising end-tidal CO 2

Dropping end-tidal CO 2

Hypoventilation

Hyperventilation

Rebreathing

Circuit disconnected

Malignant hyperthermia

Inadvertent extubation

Sepsis

Endotracheal tube obstruction

Hyperthyroidism (thyroid storm)

Hypothermia

Pheochromocytoma

Pulmonary embolism

Rhabdomyolysis

Cardiac arrest Reduced cardiac output Tension pneumothorax

367

368

PArt ii Medical Knowledge

QUESTIONS 1. A 35-year-ol man involve in a motorcycle cycle collision sustains a large laceration to his right thigh. The ecision is mae to washout an close the woun at besie with the assistance of proceural seation. The patient has an oral airway in place, is breathing spontaneously, an is maintaining aequate oxygen saturation with a simple face mask. With painful stimulation, he awakens briey an is able to follow simple commans. What level of seation is this patient currently uner? A. Minimal seation B. Moerate seation C. Conscious seation D. Deep seation E. General anesthesia

4. Which of the following is most likely associate with opioi abuse an postsurgical prescribing patterns? A. The majority of opiois abuse in the US originate from international rug cartels B. Heroin users rarely report previously abusing prescription opiois C. 30 pills of 5 mg oxycoone are the recommene amount to be prescribe after laparoscopic cholecystectomy D. New persistent opioi use after surgery is more common after major proceures compare to minor proceures E. Preoperative tobacco use is a signicant risk factor for new persistent opioi use after surgery

2. Which of the following is associate with opioi tolerance? A. Characterize by pronounce cravings an compulsive rug taking B. Decrease analgesic effect of opiois evelops before ecrease effects on respiratory epression C. Increase sleeping an eating, epression, an pupillary constriction D. Constipation resolves over time with longterm opioi use E. Genetic components associate with opioi use have not been ientie

5. A 75-year-ol woman is brought to the operating room for laparoscopic cholecystectomy. She has a history of progressive ementia an is unable to provie a meical history. Fifteen minutes into the operation performe uner general anesthesia, the anesthesiologist reports ifculty ventilating the patient, an she evelops a iffuse maculopapular rash with urticaria. Which of the following is the most likely offening agent? A. Rocuronium B. Latex C. Cefazolin D. Sevourane E. Propofol

3. A 68-year-ol woman is unergoing a laparoscopic liver resection. An arterial line an central line are place prior to surgical incision. As the hepatic parenchyma is being ivie, the anesthesiologist reports suen hypotension an a rop in en-tial CO. There is no break in the ventilatory circuit. There is only minimal bleeing at this time. There are ST changes note on the EKG. Which if the following is the next best step in management of this conition? A. Transthoracic echocariography (TTE) examination of the heart B. Aminister epinephrine C. Aminister ui bolus D. Emergently place a pulmonary artery catheter line E. Release (esufate) pneumoperitoneum

6. A 9-year-ol man unergoes a laparoscopic cholecystectomy for symptomatic cholelithiasis. Shortly after inuction, the anesthesiologist notes an increase in core boy temperature an ential CO. After aministration of antrolene an aborting the operation, his status improves. Which of the following is most likely associate with this iagnosis? A. It is an autosomal recessive isorer B. Genetic analysis is require for iagnostic conrmation C. It is more common in elerly patients D. It may present as late as 4 hours after anesthesia E. Mortality rate is less than 1%

CHAPtEr 28 Anesthesia

7. After excision of multiple subcutaneous lipomas uner local anesthesia, a 4-year-ol woman seizes violently. What is the maximum safe ose of a local anesthetic agent in a 70-kg woman? A. 10 to 0 mL 1% liocaine B. 40 to 50 mL % liocaine with epinephrine C. 40 to 50 mL 1% liocaine with epinephrine D. 40 to 50 mL 0.5% liocaine E. 40 to 50 mL 1% liocaine without epinephrine 8. A 0-year-ol man is about to unergo arthroscopic surgery on his left shouler. During anesthetic inuction with succinylcholine, the anesthesiologist note trismus that persiste for > minutes, an the mouth coul not be opene to perform irect laryngoscopy or place an enotracheal tube. The anesthesiologist was able to bag mask ventilate the patient. The ential CO, heart rate, an temperature remaine normal. Which of the following is the next best step in management? A. Aminister an aitional ose of succinylcholine B. Procee with surgery if the patient can be intubate C. Cancel surgery an sen the patient home D. Cancel surgery, aminister antrolene, an amit for 4-hour observation E. Cancel surgery, amit for 4-hour observation, an refer for muscle biopsy 9. Which of the following is the best immeiate way to conrm placement of an enotracheal tube in the airway after intubation? A. Direct visualization of tube passing through the vocal cors B. Auscultation of lungs C. Observation of conensation within tube D. Pulse oximetry E. Capnography 10. A 65-year-ol man is unergoing urgent surgery for gangrenous cholecystitis. The patient has a history of moerate aortic valve stenosis that was recently iagnose on echocariography but he enies any symptoms. Which of the following woul be most important goal in the anesthetic management? A. Preloa reuction B. Afterloa reuction C. Avoiance of hypotension D. Heart rate goal of >90 beats per minute E. Use of epherine for hypotension

369

11. A 9-year-ol boy has been in the peiatric intensive care unit for the last 7 ays after presenting to the hospital with inuenza infection leaing to respiratory failure requiring mechanical ventilation. He is receiving continuous fentanyl an propofol infusions for pain control an seation, respectively. This morning he evelope braycaria, an his urinary output ecrease. He is note to have hepatomegaly on physical examination. Laboratory values show an elevate creatinine, hyperlipiemia, hyperkalemia, an lactic aciosis. Which of the following is the best next step in management? A. Start bicarbonate infusion B. Perform liver biopsy C. Initiate hemoialysis D. Initiate treatment with low-ose epinephrine E. Discontinue propofol an start exmeetomiine infusion 12. A 55-year-ol iabetic man unerwent a rightsie vieo-assiste thoracoscopic surgery (VATS) for an empyema yesteray. This morning he is complaining of pain along his meial left forearm an has paresthesia of his fourth an fth igits. Which of the following risk factors are most likely associate with this complication? A. Male sex B. Emergency surgery C. Supine positioning uring surgery D. Hyperthermia uring surgery E. Diabetic neuropathy 13. Which of the following is true regaring invasive lines use for the monitoring of surgical patients? A. Trauma patients show improve mortality with placement of a pulmonary artery catheter (PAC) B. A normal Allen Test before raial artery cannulation will reuce incience of han ischemia C. PAC will provie irect measurement of systemic vascular resistance D. Systolic bloo pressure measure on a raial artery catheter will typically be higher than the aortic pressure E. A right bunle branch block seen on electrocariogram is consiere a contrainication for PAC placement

370

PArt ii Medical Knowledge

14. A 47-year-ol woman is recovering from pneumonia complicate by multiorgan system ysfunction. She is currently receiving hemoialysis after eveloping renal failure seconary to sepsis. This morning, a rapi response was calle for respiratory epression an confusion, which improve after the aministration of naloxone. Which of the following meications most likely contribute to her respiratory compromise? A. Fentanyl B. Hyromorphone C. Morphine D. Methaone E. Oxycoone 15. A 37-year-ol woman unerwent a percutaneous besie tracheostomy tube placement. On postoperative ay 1, she evelope signicant subcutaneous emphysema of the neck over the course of an hour, an her current oxygen saturation is 80%. A respiratory therapist attempte irectional suctioning, but they were unable to pass the catheter. What is the most appropriate next step in management? A. Remove the tracheostomy tube an attempt recannulation with a smaller caliber cannula B. Remove the tracheostomy tube an recannulate over a suction catheter C. Remove the tracheostomy tube an recannulate over a beroptic bronchoscope D. Replace the tracheostomy tube using a percutaneous tracheostomy kit E. Bag mask ventilation an prepare for orotracheal intubation 16. Which of the following correctly pairs the invasive mechanical ventilation moe with its mechanism of action? A. Synchronize intermittent mechanical ventilation (SIMV): every breath has a manate volume B. Airway pressure release ventilation (APRV): maintains continuous positive airway pressure (CPAP) with an intermittent release phase C. Assist-control (AC) ventilation: patient etermines the rate an volume of breaths D. CPAP: two ifferent pressure settings for inhalation an exhalation E. High-frequency oscillatory ventilation (HFOV): high respiratory rate with large tial volumes

17. Which of the following parameters is most likely to preict successful iscontinuation of mechanical ventilation? A. Rapi shallow breathing inex (RSBI) (f/VT) less than 105 B. Negative inspiratory force (NIF) −0 to −30 cm HO C. Successful spontaneous breathing trial (SBT) D. Respiratory rate less than 30 breaths per minute E. Tial volume greater than 5 mL/kg 18. A 66-year-ol woman presents in septic shock ue to a perforate uoenal ulcer. She is taken urgently to the operating room for an exploratory laparotomy. Due to persistent hypotension, opamine is infuse by the anesthesiologist an is eventually titrate to a rate of 15 mcg/kg per minute. At that rate, which of the following receptors is exerting the preominant effect? A. α1-Arenergic B. α-Arenergic C. β1-Arenergic D. β-Arenergic E. Dopaminergic 19. A 55-year-ol man with a history of chronic obstructive pulmonary isease (COPD) unergoes an interscalene regional block with bupivacaine for surgery of a left humerus fracture. Soon after placement of the block, the patient evelops signicant yspnea. Breath souns are equal to auscultation an clear. Which of the following factors is the most likely cause of his shortness of breath? A. Pneumothorax B. COPD exacerbation C. Inavertent intravascular injection of bupivacaine D. Air embolism E. An elevate left hemiiaphragm 20. At the en of a surgery, an anticholinesterase is aministere to a patient to reverse the neuromuscular blockae. Which of the following muscles woul be expecte to recover rst? A. Diaphragm B. Auctor pollicis C. Ocular muscles D. Pharyngeal E. Quariceps femoris

CHAPtEr 28 Anesthesia

21. A 40-year-ol man with obesity, hypertension, cirrhosis, iabetes mellitus, an chronic kiney isease (CKD) stage 1 unergoes general anesthesia for repair of an incarcerate inguinal hernia. He takes insulin, echothiopate, amloipine, an simvastatin at home. Propofol an pancuronium are use for inuction. At the en of the proceure, a peripheral nerve stimulator emonstrates no recovery of muscle twitches espite 60 minutes of time elapsing. Which of the following unerlying factors is most likely responsible for this conition? A. Diabetes mellitus B. Obesity C. Stage 1 CKD D. Pancuronium E. Simvastatin

371

23. A patient is given benzocaine spray in anticipation of a besie exible laryngoscopy. After several minutes, he evelops a heaache an shortness of breath. Pulse oximetry shows an SpO of 85%, while an arterial bloo gas shows an SaO of 80% with a PaO of 150 mmHg. Which of the following is the most appropriate treatment? A. Intubation B. Intravenous methylene blue C. Discontinue benzocaine an aminister prilocaine D. Metoclopramie E. Thiosulfate

22. Which of the following is associate with barbiturate toxicity? A. Myocarial epression B. Acute tubular necrosis C. Hepatotoxicity D. Peripheral neuropathy E. Seizures

ANSWERS 1. D. Level of seation is a continuum ene by the

patient's response to the meications aministere. During minimal seation, patients have a normal response to verbal stimulation (A). During moerate seation, patients have purposeful responses to verbal or tactile stimulation (B). While uner eep seation, repeate verbal or painful stimulation is neee to achieve purposeful movements. Intervention on the airway may be require at this level of seation. Uner general anesthesia, the patient is unarousable even with painful stimulus (E). Moerate seation an conscious seation are terms that are often use interchangeably (C). Reference: Practice guielines for moerate proceural seation an analgesia 018: a report by the American Society of Anesthesiologists Task Force on Moerate Proceural Seation an Analgesia, the American Association of Oral an Maxillofacial Surgeons, American College of Raiology, American Dental Association, American Society of Dentist Anesthesiologists, an Society of Interventional Raiology. Anesthesiology. 018;18(3):437–479.

2. B. Long-term opioi use commonly results in tolerance

an physical epenence. Tolerance escribes a ecrease in opioi potency with repeate aministration. Tolerance to analgesic effects of opiois evelops more quickly than

tolerance to respiratory epression, which partially explains the high overose rates. Tolerance within the colon typically oes not evelop an results in chronic constipation (D). Depenance is characterize by the unpleasant response to stopping or reucing intake of the rug, also referre to as withrawal symptoms. Opioi withrawal symptoms inclue lacrimation, piloerection, muscle aches, nausea, vomiting, iarrhea, pupillary ilation, insomnia, tachycaria, hyperreexia, an hypertension (C). Aiction is much less preictable an less common than both tolerance an epenance an is characterize by pronounce cravings, obsessive thinking, compulsive rug taking, an an inability to refrain from use (A). It is also now believe that opioi aiction has a fairly strong genetic component with heritability rates similar to iabetes an hypertension (E). References: Volkow ND, McLellan AT. Opioi abuse in chronic pain–misconceptions an mitigation strategies. N Engl J Med. 016;374(13):153–163. Akbarali HI, Inkisar A, Dewey WL. Site an mechanism of morphine tolerance in the gastrointestinal tract. Neurogastroenterol Mot. 014;6(10):1361–1367.

3. E. Given the unexplaine hypotension an ecrease in en-tial CO, this patient most likely has a CO embolism.

372

PArt ii Medical Knowledge

Clinically signicant CO embolism is very rare uring laparoscopic surgery but has a mortality rate of approximately 8%. CO embolism is thought to be cause by either intravascular injection of CO into a vessel with either a Veress neele or trocar uring initial insufation, or by gas entering an injure vessel later uring the operation. Signs of a CO embolism are unexplaine hypotension, hypoxia, or a suen ecrease in en-tial CO. Transesophageal echocariography (TEE) is the most sensitive metho for etecting CO embolism, though often not necessary when clinical suspicion is high (A). Precorial oppler is the most sensitive noninvasive test. If CO embolism is suspecte, insufation shoul be stoppe an the abomen esufate immeiately. Though historically it has been recommene to place the patient in the left lateral an Trenelenburg position to move the air bubble out of the pulmonary artery, new evience suggests that neither of the above positions results in signicant hemoynamic improvements. Rather, for proceures below the level of the heart, the patient shoul be place in the reverse Trenelenburg position to reuce further air entrainment. Vasopressor aministration an a ui bolus are reasonable interventions for persistent hypotension, but shoul be one after reucing the risk of further air entrapment (B, C). Pulmonary artery catheters have been shown to be ineffective at aspirating air with a success rate between 6% an 16% an shoul not be the next step in management (D). A "mill-wheel" murmur is present in less than half of patients. References: Cottin V, Delafosse B, Viale JP. Gas embolism uring laparoscopy: a report of seven cases in patients with previous abominal surgical history. Surg Endosc. 1996;10():166–169. Mirski M, Lele AV, Fitzsimmons L, et al. Diagnosis an treatment of vascular air embolism. Anesthesiology. 007;106:164–177.

4. E. Opioi abuse has risen substantially in the US in

recent years, prompting research investigating the causes of this new epiemic. Though the majority of opiois abuse in the US originate from legitimate prescriptions, only 0% of opioi users were the intene recipients of the initial prescription (A). The majority of opioi abusers receive pills for free from family members or friens with excessive pills or from other methos of iversion. Opioi abuse can also lea to further illicit rug use, as 50% to 85% of heroin users report having previously abuse prescription opiois (B). As excessive opioi prescriptions appear to be one of the inciting factors in opioi abuse, aitional attention has been place on prescribing patterns after surgery. A large retrospective stuy showe that new persistent opioi use was fairly common after both major an minor surgical proceures, with an incience of aroun 6%. The incience was not signicantly ifferent between major an minor surgeries inicating that pain is not the riving factor for this postsurgical complication (D). Risk factors inepenently associate with new persistent opioi use inclue preoperative tobacco use, alcohol an substance abuse isorers, moo isorers, anxiety, an preoperative pain isorers (E). To aress the overprescribing of opiois after surgery, one stuy ientie the number of pills (equivalent to 5 mg oxycoone) that woul fully supply the nees of 80% of patients unergoing a number of ifferent operations. Examples of these nees inclue 5 pills after a partial mastectomy, 15 pills after a laparoscopic cholecystectomy, an 15 pills after an open inguinal hernia repair (C).

References: Brummett CM, Waljee JF, Goesling J, et al. New persistent opioi use after minor an major surgical proceures in US aults. JAMA Sur. 017;15(6):e170504. Hill MV, McMahon ML, Stucke RS, et al. Wie variation an excessive osage of opioi prescriptions for common general surgical proceures. Ann Surg. 017;65(4):709–714.

5. A. A stuy one in France from 1997 to 004 looke at all patients who ha immeiate hypersensitivity reaction presume to be from allergic reaction. Of the 1816 patients that met criteria for the stuy, the top three offening agents for immeiate hypersensitivity reaction were neuromuscular blocking agents (58%), latex (0%), an antibiotics (13%) (B, C). Allergy to inhale anesthetics an hypnotics was much less common (D, E). In chilren, latex was more common than neuromuscular blocking agents, but the sample size for this population was much lower. References: Butterworth J, Mackey D, Wasnick J, etal., es. Inhalation anesthetics. In: Morgan & Mikhail's clinical anesthesiology. 5th e. McGraw-Hill; 013;44–88. Butterworth J, Mackey D, Wasnick J, etal., es. Intravenous anesthetics. In: Morgan & Mikhail's clinical anesthesiology. 5th e. McGrawHill; 013;141–156. Di Leo E, Delle Donne P, Calogiuri GF, Macchia L, Nettis E. Focus on the agents most frequently responsible for perioperative anaphylaxis. Clin Mol Allergy. 018;16:16. Mertes PM, Alla F, Tréchot P, Auroy Y, Jougla E, Groupe ’Etues es Réactions Anaphylactoïes Peranesthésiques. Anaphylaxis uring anesthesia in France: an 8-year national survey. J Allergy Clin Immunol. 011;18():366–373.

6. D. This patient likely has malignant hyperthermia, a rare

autosomal ominant isorer of skeletal muscle (A). The conition is characterize by a hypermetabolic state triggere by exposure to inhalation anesthetics (sevourane, esurane, isourane) an/or succinylcholine. The oler anesthetic agents associate with this reaction inclue halothane an enurane. It is not cause by nitrous oxie, intravenous anesthetic agents, or other neuromuscular blockers (except for succinylcholine). Malignant hyperthermia occurs when uncontrolle amounts of intracellular calcium accumulate in skeletal muscle. Symptoms may evelop as early as 30 minutes after anesthetic aministration an as late as 4 hours postoperatively. Even after treatment with antrolene, patients nee to be monitore because they can have a refractory response an go back into a malignant hyperthermic crisis. The initial clues occur in the operating room after inuction. Rather than achieving complete paralysis, the anesthesiologist may notice rigiity in the masseter muscle. Other nings inclue an increase in ential CO, tachycaria, an an increase in temperature. It is imperative that all anesthetics are immeiately stoppe an antrolene given (.5 mg/kg every 5 minutes) until resolution of symptoms. Dantrolene stabilizes muscle channels in the sarcoplasmic reticulum. The mortality rate was previously 30%, but recent evience suggests the mortality rate is now approximately 5% (E). A functional test on skeletal muscle biopsy (caffeine halothane contracture test) is use for iagnosis (B). More than 50% of the families show linkage of the invitro contracture test phenotype to the gene encoing the skeletal muscle ryanoine receptor. The test requires a muscle biopsy with exposure of the muscle to halothane an caffeine. A positive test will cause signicant muscle contraction. The majority of cases occur in chilren or young aults (C).

CHAPtEr 28 Anesthesia References: Jurkat-Rott K, McCarthy T, Lehmann-Horn F. Genetics an pathogenesis of malignant hyperthermia. Muscle Nerve. 000;23(1):4–17. Ellinas H, Albrecht MA. Malignant hyperthermia upate. Anesthesiol Clin. 00;38(1):165–181.

7. C. There are relatively few sie effects of local anes-

thetic agents such as liocaine, unless they are inavertently injecte intravenously or aministere in oses higher than recommene. Toxicity begins with neurologic signs an symptoms such as light-heaeness, facial paresthesias, blurre vision, an tinnitus. It can progress to lethargy, tremors, an tonic-clonic seizures. Neurologic symptoms precee the more severe cariovascular symptoms, which inclue hypertension an tachycaria (early symptoms) an later hypotension, cariovascular collapse, braycaria or conuction abnormalities, an even cariac arrest. The maximum oses for local injection of liocaine are 5 mg/kg without epinephrine an 7 mg/kg with epinephrine because the vasoconstriction elays the systemic release of liocaine. Because a 1% solution of liocaine contains 10 mg/mL, an easy way to remember this is to multiply the patient's weight by either 5 (no epinephrine) or 7 (with epinephrine) an then ivie by 10. Therefore, for this patient: 70 kg × 5 mg/kg = 350 mg an ivie by 10 mg/mL = 35 mL of 1% liocaine. For liocaine with epinephrine, 70 kg × 7 mg/kg = 490 mg an ivie by 10 = 49 mL of 1% liocaine. For a % liocaine solution, one woul ivie by 0 (4.5 mL an 17.5 mL, respectively, with an without epinephrine), an for a 0.5% solution, one woul ivie by 5 (70 mL an 98 mL, respectively, with an without epinephrine). Patients who experience local anesthetic systemic toxicity (LAST) shoul be treate by iscontinuing the local anesthetic, aministering uis, support with 100% FiO, hyperventilation, an aministering 0% intralipi with a bolus of 1 to 1.5 mL/kg over one minute. The bolus can be repeate every 3 minutes up to a total ose of 3 mL/kg, followe by an infusion of 0.5 mL/kg/min which is continue until the patient is hemoynamically stable for at least 10 minutes. CPR an epinephrine shoul be use in cariac arrest, an bicarbonate shoul be use in aciosis. Benzoiazepines are preferre over propofol to manage seizures. References: Warren JA, Thoma RB, Georgescu A, Shah SJ. Intravenous lipi infusion in the successful resuscitation of local anesthetic-inuce cariovascular collapse after supraclavicular brachial plexus block. Anesth Analg. 008;106(5):1578–1580. Neal JM, Mulroy MF, Weinberg GL, American Society of Regional Anesthesia an Pain Meicine. American Society of Regional Anesthesia an Pain Meicine checklist for managing local anesthetic systemic toxicity: 01 version. Reg Anesth Pain Med. 01;37(1):16–18. Cao D, Hear K, Foran M, Koyfman A. Intravenous lipi emulsion in the emergency epartment: a systematic review of recent literature. J Emerg Med. 015;48(3):387–97.

8. E. Masseter muscle rigiity, or trismus, is consiere a

normal reaction to the aministration of neuromuscular blocking agents. However, if this conition persists for more than 0 to 30 secons, it is consiere an abnormal response, an the clinician nees to have a high level of concern for malignant hyperthermia, an nonemergent surgeries shoul be cancele (B). Persistent trismus is not a sign of inaequate neuromuscular blockae, an thus aitional neuromuscular blocker aministration is not inicate (A). Masseter spasm is an early inicator of susceptibility to malignant hyperthermia. Other markers for malignant hyperthermia

373

inclue fevers, increase en-tial CO, generalize muscle rigiity, autonomic instability, an rhabomyolysis. The incience of patients who evelop masseter spasms an go on to evelop malignant hyperthermia is unknown. It shoul be note that isolate masseter spasm is not pathognomonic for malignant hyperthermia. The surgery shoul be cancele an the patient amitte for at least 4 hours of observation to watch for the evelopment of rhabomyolysis or malignant hyperthermia; the patient shoul not be sent home prior to 4 hours of observation an monitoring in the hospital (C). In the absence of hemoynamic instability, elevate CO, or fever, it is unnecessary to aminister antrolene (D). However, these patients shoul be referre to a center that can perform the necessary testing, incluing genetic testing an a caffeine halothane contracture test (muscle biopsy test). After muscle biopsy, the tissue is only viable for several hours, so testing must take place in centers specialize in iagnosing malignant hyperthermia (E). References: Schneierbanger D, Johannsen S, Roewer N, Schuster F. Management of malignant hyperthermia: iagnosis an treatment. Ther Clin Risk Manag. 014;10:355–36. Bauer SJ, Orio K, Aams BD. Succinylcholine inuce masseter spasm uring rapi sequence intubation may require a surgical airway: case report. Emerg Med J. 005;:456–458. Sheikh MM, Riaz A, Umair HM, Waqar M, Muneeb A. Succinylcholine-inuce masseter muscle rigiity successfully manage with propofol an laryngeal mask airway: a case report an brief review. Cureus. 00;1(7):e9376.

9. E. Although irect visualization of the tube passing

through the vocal cors, auscultation of the lungs, visualization of conensation within the tube, an pulse oximetry are goo ajuncts to conrm initial placement of the enotracheal tube, interpretation is subjective an not as accurate as more objective methos for conrming the position of the enotracheal tube within the trachea (A–C). Both the American College of Emergency Physicians an the American Society of Anesthesiologists recommen capnography or en-tial CO etection evices as the preferre conrmatory test for tracheal intubation (E). Patients shoul have a continuous uniform waveform of en-tial CO with similar amplitues to conrm tracheal intubation; however, this oes not ifferentiate a tracheal from a bronchial intubation. A capnographic waveform that shows en-tial CO etection but oes not have a continuous waveform or the amplitues get smaller an smaller until no aitional en-tial CO can be etecte is inicative of an esophageal intubation. Direct visualization of the tube passing through the cors is not always reliable, as the cors can be misientie or the tube can be isloge from the trachea before it is secure (A). Auscultation of the lungs is not always reliable because it is possible to get referre souns from the stomach (B). Conensation within the tube can occur even with esophageal intubation (C). Pulse oximetry is also not reliable, as hypoxia with esophageal intubation can be very elaye if the patient is preoxygenate well (D). References: American Society of Anesthesiologists Task Force on Management of the Difcult Airway. Practice guielines for management of the ifcult airway: an upate report by the American Society of Anesthesiologists Task Force on Management of the Difcult Airway. Anesthesiology. 003;98(5):169–77. Grmec S. Comparison of three ifferent methos to conrm tracheal tube placement in emergency intubation. Intensive Care Med. 00;8(6):701–704.

374

PArt ii Medical Knowledge

10. C. While asymptomatic aortic stenosis is not a contra-

inication to surgery, it requires careful intraoperative monitoring. The increase pressures require to overcome the stenosis cause concentric hypertrophy of the left ventricle, which in turn reuces the compliance of the ventricle. This makes these patients heavily preloa epenent for ventricular lling, an careful attention shoul be pai to maintaining aequate intravascular volume (A). In aition, up to 40% of the left ventricular en-iastolic volume (LVEDV) is provie by the atrial kick. Atrial arrhythmias can quickly lea to heart failure an shoul be aggressively treate, preferably with ebrillation. Braycaria (8 French) are require for aequate rainage D. Treatment of the organizing phase requires open rainage (e.g., Eloesser ap) E. If it progresses to an empyema, vieo-assiste thoracoscopic surgery shoul be performe if it oes not respon to chest tube rainage 13. Which of the following is true regaring antibiotic mechanisms? A. Penicillin-erivative antibiotics bin to the bacterial cell membrane an increase its permeability B. Piperacillin-tazobactam works partly by bining β-lactamases C. Metroniazole, though limite, has some effect against aerobic bacteria D. Linezoli competitively inhibits the 30S ribosome E. Clinamycin, like the macrolies, reversibly bins the 50S ribosome 14. Which of the following shoul be use as part of a screening tool to ientify non-ICU patients that are at increase risk of organ failure from infection? A. Temperature higher than 38°C B. Heart rate greater than 90/min C. Altere mentation D. White bloo cell (WBC) count greater than 1,000/mm3 E. PaCO less than 3 mmHg

CHAPtEr 31 Infection and Antimicrobial Therapy

15. A 56-year-ol HIV-positive (with a low CD4 count) patient presente to the ED with a spontaneous pneumothorax an unerwent a tube thoracostomy proceure. While trying to re-cap the 0-gauge neele use for anesthetizing the skin, the resient who performe the proceure was inavertently stuck resulting in visible bleeing from the skin. Which of the following is true regaring this exposure? A. Postexposure prophylaxis with a -rug regimen shoul be aministere for 8 weeks B. Postexposure prophylaxis with a 3-rug regimen shoul be aministere for 4 weeks C. Potential HIV infection shoul be isclose to future patients D. At least 6 months of postexposure treatment is recommene E. The hollow bore neele use for this proceure lowers the risk of HIV transmission 16. A 45-year-ol HIV-positive male presents to the ED with perianal pain for the past two ays. Physical exam reveals a small area of tenerness in the right posterolateral position istal to the external sphincter that is extremely tener. His CD4 count is 550 cells/mL, an he is currently on highly active antiretroviral therapy (HAART). Which of the following is the most correct management of this patient? A. Intravenous (IV) antibiotics B. Incision an rainage uner local anesthesia in the ED C. Oral antibiotics an incision an rainage uner local anesthesia in the ED D. IV antibiotics, exam uner anesthesia (EUA), an if an area of uctuance is ientie, then incision an rainage an biopsy E. IV antibiotics, EUA, incision an rainage, an biopsy of the area of tenerness even if no uctuance is ientie

401

17. A 6-year-ol man is postoperative ay 6 from an elective laparoscopic sigmoi colectomy for recurrent iverticulitis. He ha return of bowel function  ays ago an was getting reay to be ischarge home. Throughout his hospital course, he has been having low-grae fevers. He is now complaining of tenesmus an urinary retention. Which of the following represents the most appropriate next step in management? A. Transition to nonnarcotic pain meications B. Blaer scan an in-an-out catheterization as neee C. Abominal raiography D. Compute tomography (CT) E. Diagnostic laparoscopy 18. A 60-year-ol man presents with gas gangrene of his left leg requiring below-knee amputation. Woun cultures were positive for Clostridium septicum. Aitional workup shoul inclue: A. Hea CT scan B. Bronchoscopy C. Colonoscopy D. HIV serology E. Chest CT scan 19. Which of the following is true regaring tetanus? A. It is highly contagious B. Trismus is usually the rst sign C. It is cause by a gram-negative anaerobic ro D. A prior history of surviving tetanus provies immunity E. The iagnosis is establishe by emonstrating the organisms in a woun 20. Which of the following is associate with an enotoxin? A. Streptococcus pyogenes B. Bacteroides fragilis C. Clostridium tetani D. S. aureus E. C. perfringens

402

PArt ii Medical Knowledge

Answers 1. B. The Stuy to Optimize Peritoneal Infection Therapy

infections. Their most recent recommenations aress timing of antibiotics, choice of antibiotics, hair removal techniques, normothermia, euglycemia, an inwelling urinary catheter use. Prophylactic antibiotics shoul be given within 1 hour prior to the incision (A). Aitionally, the chosen antibiotic shoul cover the most likely pathogen to be encountere uring the operation. Cefazolin oes not have appropriate anaerobic coverage for a colectomy (B). Surgical site hair shoul be remove with electrical clippers, rather than shaving, on the ay of or ay prior to surgery. Shaving has been shown to have a higher rate of meiastinitis in patients unergoing open-heart surgery (C). Inwelling urinary catheter use longer than  ays after an operation is associate with higher rates of urinary tract infection an it is recommene to remove catheters prior to this point when possible (D). Maintaining euglycemia for the rst 48 hours postoperatively has been shown to ecrease surgical site infection. Aitionally, intravenous insulin infusion postoperatively was associate with ecrease eep sternal woun infections in open cariac surgery compare to sliing-scale insulin injections. References: Furnary AP, Zerr KJ, Grunkemeier GL, Starr A. Con-

short-course antimicrobial therapy for intraabominal infection. N Engl J Med. 015;37(1):1996–005.

tinuous intravenous insulin infusion reuces the incience of eep sternal woun infection in iabetic patients after cariac surgical proceures. Ann Thorac Surg. 1999;67():35–360. Rosenberger LH, Politano AD, Sawyer RG. The surgical care improvement project an prevention of post-operative infection, incluing surgical site infection. Surg Infect (Larchmt). 011;1(3):163–168.

(STOP-IT) trial was a ranomize controlle trial esigne to etermine the optimal length of antibiotic treatment after source control in patients with intraabominal infections. Patients were ranomize to receive antibiotics for 4 ays postoperatively versus  ays after the resolution of fever, leukocytosis, an ileus. The meian uration of antibiotics was 4 ays versus 8 ays postoperatively, an there was no ifference in surgical-site infections, recurrent intraabominal infections, or mortality. Therefore, antibiotics shoul continue only for 4 ays postoperatively in most cases (A, C–E). Procalcitonin may be useful in helping etermine the uration of antibiotic therapy. A single absolute value is less useful than the tren over several ays. It is best use in cases where systemic inammation is present without an obvious infectious etiology. If the procalcitonin level eclines with antibiotic therapy, it is reasonable to complete a 5 to 10-ay course epening on the suspecte source(s). However, if the level is normal an/or oes not change with antibiotics, it woul be reasonable to stop antibiotic therapy. Reference: Sawyer RG, Clarige JA, Nathens AB, et al. Trial of

2. E. This patient is showing multiple signs of sepsis with

the most likely source being a catheter-associate urinary tract infection. The Surviving Sepsis Campaign is a joint collaboration committe to reucing morbiity an mortality relate to sepsis an septic shock. Upate recommenations etailing the ieal management of sepsis an septic shock are provie perioically. Flui resuscitation shoul begin as soon as possible with 30 cc/kg of crystalloi (C). Albumin (colloi) bolus has not been consistently emonstrate to be associate with improve outcomes in patients with septic shock an is signicantly more costly than crystalloi. As such, crystallois are preferre over collois (B). Empiric broa-spectrum intravenous antibiotics shoul also be aministere within one hour. However, appropriate cultures shoul be obtaine prior to starting antibiotics if this incurs no substantial elay. If hypotension persists espite ui resuscitation, vasopressors shoul be initiate with a MAP goal of >65 mmHg (A). Aitionally, intravenous hyrocortisone at 00 mg/ay shoul be consiere if shock is refractory to both ui resuscitation an vasopressors (C). This patient has an elevate lactate which shoul be measure at regular intervals an utilize as an enpoint for aequate resuscitation. Though he oes have a signicant aciemia, aministration of soium bicarbonate is not recommene for correction as long as his pH is greater than 7.15 an certainly shoul not occur prior to aequate ui resuscitation (D). Reference: Rhoes A, Evans LE, Alhazzani W. Surviving Sepsis Campaign: international guielines for management of sepsis an septic shock. Crit Care Med. 017;45(3):486–55.

3. E. The Surgical Care Improvement Project (SCIP) is a pro-

gram esigne to reuce the rates of postoperative surgical

4. E. Aspergillus species are wiely isperse in the envi-

ronment an, when implicate as a pathogen, primarily affect the lungs. It typically presents as one of four synromes: aspergilloma, allergic bronchopulmonary aspergillosis (ABPA), chronic necrotizing Aspergillus pneumonia, an invasive aspergillosis. Aspergilloma typically presents as an asymptomatic raiographic ning in patients with a preexisting cavitary lung isease such as sarcoiosis. A soft-tissue mass within a cavity that is surroune by a crescent of air (Mona sign) is iagnostic, an because the aspergilloma is not aherent to the cavity walls, the air will remain in a nonepenent position. Biopsy or bronchoscopy is not inicate or necessary for iagnosis (A, B). As long as the patient is asymptomatic, no further workup or treatment is necessary. The most common symptom associate with aspergilloma is hemoptysis, which can occasionally be life threatening. In this setting, an emergency bronchial artery embolization shoul be performe followe by surgical resection (D). ABPA is a noninvasive hypersensitivity isease that, if left untreate, can lea to brotic lung isease. Therapy is aime at the treatment of acute exacerbations either with inhale bronchoilators/sterois (mil isease) or systemic corticosterois (severe isease) to prevent long-term sequelae. Serial chest raiographs, pulmonary function tests, an IgE levels shoul be monitore because permanent pulmonary amage can take place even in asymptomatic patients. Invasive aspergillosis an chronic necrotizing Aspergillus pneumonia are both treate with intravenous antifungals (C). Invasive

CHAPtEr 31 Infection and Antimicrobial Therapy isease can be rapily fatal an is typically only foun in immunocompromise hosts. High-risk transplant patients, such as bone marrow recipients, receive prophylactic agents to prevent invasive infection. Reference: Limper AH, Knox KS, Sarosi GA, et al. An ofcial American Thoracic Society statement: treatment of fungal infections in ault pulmonary an critical care patients. Am J Respir Crit Care Med. 011;183(1):96–18.

5. E. Hepatitis B surface antigen is foun on the surface

of the hepatitis B virus an is foun in high quantities in the serum of iniviuals with acute or chronic infection. Antiboies against this antigen (anti-HBs) are consiere to represent an immunity to the virus either from previous infection or vaccination. All patients with chronic hepatitis B infection will be anti-HBs negative. Antiboies against hepatitis core antigen (anti-HBc) appear at the onset of symptoms an persist for life, though they o not confer immunity to the isease. Vaccination will not prouce antiboies to hepatitis B core antigen. Presence of these antiboies inicates either active or previous infection with hepatitis B but oes not confer a timeline associate with that infection. However, IgM against hepatitis B core antigen is only present for the rst 6 months of infection, so its presence inicates a recent exposure to the virus. The aforementione serologic proles represent: A, susceptible to infection; B, immunity from previous infection; C, immunity from vaccination; D, acute infection; E, chronic infection.

6. E. The risk of eveloping hepatitis B from a neelestick

injury is far greater than that of hepatitis C, particularly when the patient is hepatitis Be surface antigen (HBesAg) positive (A). If the patient's bloo is both HBeAg an HBsAg positive, the risk of eveloping clinical hepatitis is very high (%–31%). If the bloo is HBsAg positive but HBeAg negative, the risk rops to 1% to 6% (although seroconversion is still high at 3%–37%). Hepatitis B is highly infectious, an the virus can survive on rie bloo an on environmental surfaces for at least a week. The majority of health-care workers infecte with hepatitis B o not recall a neelestick exposure, though they were in contact with a hepatitis B–positive patient (D). For health-care workers who have never been vaccinate for hepatitis B, or are seronegative, treatment with both HBIG (immunoglobulin prepare from human plasma known to contain a high titer of antiboy to HBsAg) an the hepatitis B vaccine is recommene (B). Data on clinical hepatitis C following exposure is lacking. However, the average incience of anti-HCV seroconversion from an HCV-positive source is very low (only 1.8%), suggesting that the risk of transmission from a neelestick injury is very low. In fact, some stuies suggest that the risk of hepatitis C transmission from a soli surgical neele is negligible. No effective prophylaxis for HCV has been ientie. Immunoglobulin an antiviral agents are not recommene for HCV postexposure prophylaxis (C). Reference: Kuhar DT, Henerson DK, Struble KA, et al. Updated U.S. public health service guidelines for the management of occupational exposures to HIV and recommendations for postexposure prophylaxis. Division of Healthcare Quality Promotion, National Center for Emerging an Zoonotic Infectious Diseases, Center for Disease Control an Prevention (CDC); 013. https://stacks.cc.gov/view/ cc/0711.

403

7. B. NSTI is a broa term that encompasses infections lim-

ite to skin an subcutaneous tissue (necrotizing cellulitis) an those involving the fascia (necrotizing fasciitis) an muscle (myonecrosis). They can be extremely ifcult to accurately iagnose early on because fewer than half present with obvious har signs of NSTI, such as bullae, skin necrosis, gas on raiograph, an crepitus. Other signs inclue tense eema, violaceous skin color, severe pain, an neurologic ecit. Several laboratory values have been shown to be useful in istinguishing NSTI from simple cellulitis. The LRINEC (Laboratory Risk Inicator for Necrotizing Fasciitis) score uses the total WBC count, hemoglobin, soium, glucose, serum creatinine, an C-reactive protein levels. A simpler moel uses an amission WBC count greater than 15.4 × 109/L an/or a serum soium level less than 135 mEq/L. This latter moel is more useful for its negative preictive value (99%). A low serum soium level is theorize to be the result of either a sepsis-inuce synrome of inappropriate antiiuretic hormone or arenal insufciency, but this has not been conrme. Risk factors for NSTI inclue iabetes, illicit IV rug abuse, immunosuppression, an liver isease. Seventy percent to 80% of NSTIs are ue to polymicrobial infection. Of those that are cause by a single organism, Klebsiella, S. pyogenes, an C. perfringens are the most common. The NSTI is subivie into two categories; type I infections are cause by polymicrobial infection with aerobic an anaerobic bacteria (e.g., Clostridium an Bacteroides spp.), which work synergistically to prouce infection. Type II infections are cause by group A Streptococcus with or without Staphylococcus. Treatment inclues rapi aministration of broa-spectrum antimicrobial agents, aggressive ui resuscitation, an aggressive surgical ebriement. The mortality rate remains at 0% to 40% an is higher with surgical elays, particularly beyon 4 hours. A rising WBC count an lactate after ebriement are highly suggestive of progression of the NSTI. A secon-look operation is often require an shoul be performe for this patient in orer to ensure that no aitional tissues have become involve since the initial ebriement. Amputation may be necessary, but only a secon-look operation will inicate whether this is the case (A). CT scan in the postoperative setting may not be useful because interpretation can be ifcult seconary to postsurgical changes (D). With septic shock, pressors may be necessary, but this woul not be the enitive treatment (E). Aitionally, no hemoynamic parameters (bloo pressure, central venous pressure) are provie that woul inicate that pressors are neee. Similarly, aing antifungal coverage can be consiere, but this is not a enitive intervention (C). References: Anaya DA, Dellinger EP. Surgical infections an choice of antibiotics. In: Townsen CM, Jr, Beauchamp RD, Evers BM, Mattox KL, es. Sabiston textbook of surgery: the biological basis of modern surgical practice. 17th e. W.B. Sauners; 004:57–8. Dunn DL, Beilman GJ. Surgical infections. In: Brunicari FC, Anersen DK, Billiar TR, etal., es. Schwartz's principles of surgery. 8th e. McGraw-Hill; 005:109–18. Wall DB, Klein SR, Black S, e Virgilio C. A simple moel to help istinguish necrotizing fasciitis from nonnecrotizing soft tissue infection. J Am Coll Surg. 000;191(3):7–31. Wong CH, Khin LW, Heng KS, Tan KC, Low CO. The LRINEC (Laboratory Risk Inicator for Necrotizing Fasciitis) score: a tool for istinguishing necrotizing fasciitis from other soft tissue infections. Crit Care Med. 004;3(7):1535–1541.

404

PArt ii Medical Knowledge

Yaghoubian A, e Virgilio C, Dauphine C, Lewis RJ, Lin M. Use of amission serum lactate an soium levels to preict mortality in necrotizing soft-tissue infections. Arch Surg. 007;14(9):840–846.

8. E. Risk for surgical site infections is relate to several

factors, incluing microbial contamination uring surgery, length of operation, an patient factors such as iabetes, nutritional state, obesity, an immunosuppression (cancer, renal failure, immunosuppressive rugs) (B–D). The National Nosocomial Infection Surveillance Risk Inex is a useful tool to assess the risk of woun infection. This inex inclues (1) American Society of Anesthesiologists physical status score higher than , () class III or IV wouns, an (3) uration of an operation greater than the 75th percentile for that particular proceure (A). Wouns are classie as clean (class I) (e.g., hernia repair, breast biopsy), clean/ contaminate (class II) (e.g., cholecystectomy, elective gastrointestinal surgery), contaminate (class III) (e.g., bowel injury from trauma or inavertent enterotomy), an irty (class IV) (e.g., perforate appenicitis, iverticulitis, necrotizing soft-tissue infections [NSTIs]). Hemoglobin levels have not been shown to increase the risk of woun infection. In a ranomize stuy of patients unergoing colorectal surgery, surgical woun infections were foun in 19% who were permitte to become hypothermic but in only 6% who were actively kept normothermic. In a ranomize stuy of clean surgery (breast, varicose vein, hernia), those who were actively warme 30 minutes before surgery ha only a 5% woun infection rate versus 14% in nonwarme patients. Active control of glucose via continuous infusion was shown to ecrease sternal woun infection in iabetic patients unergoing cariac surgery. The main concern with aggressive glucose control, however, is that it may incite episoes of hypoglycemia. A recent stuy also highlighte the risk of bloo transfusion in woun infection, likely the result of its immunosuppressive effects. References: Campbell DA Jr, Henerson WG, Englesbe MJ, et al. Surgical site infection prevention: the importance of operative uration an bloo transfusion–results of the rst American College of Surgeons-National Surgical Quality Improvement Program Best Practices Initiative. J Am Coll Surg. 008;07(6):810–80. Furnary AP, Zerr KJ, Grunkemeier GL, Starr A. Continuous intravenous insulin infusion reuces the incience of eep sternal woun infection in iabetic patients after cariac surgical proceures. Ann Thorac Surg. 1999;67():35–360. Kurz A, Sessler DI, Lenhart R. Perioperative normothermia to reuce the incience of surgical-woun infection an shorten hospitalization: stuy of Woun Infection an Temperature Group. N Engl J Med. 1996;334(19):109–115. Melling AC, Ali B, Scott EM, Leaper DJ. Effects of preoperative warming on the incience of woun infection after clean surgery: a ranomise controlle trial. Lancet. 001;358(985):876–880.

9. D. Acute mesenteric aenitis presents most commonly in

chilren an young aults. It can frequently be confuse with appenicitis in chilren. Usually, an upper respiratory infection is present or has recently resolve. The abominal pain is usually iffuse, but involuntary guaring on exam is rare. Laboratory values are of little help in establishing the iagnosis. More than 50% have an elevate WBC count. Although infection with the other answer choices can lea to mesenteric lymphaenitis, Y. enterocolitica is the most commonly associate organism in chilren (A–C, E). If the iagnosis

is clear preoperatively (which is usually not the case), treatment is supportive because it is a self-limite isease. Ultrasoun has emerge as a useful tool in chilren to suggest this iagnosis. Finings inclue enlarge, hypoechoic mesenteric lymph noes (at least one more than 8 mm in iameter) an the absence of an iname (ilate) appenix. The iagnosis can also be mae with CT by the emonstration of enlarge, clustere mesenteric lymph noes in the right lower quarant in the absence of acute appenicitis, but there is increasing reluctance to expose chilren to the raiation associate with CT scanning. The iagnosis is sometimes mae uring laparoscopy.

10. D. A rare cause of infection in the rst 48 hours after an operation is woun toxic shock synrome. Toxic shock synrome is an acute onset, multiorgan illness that resembles severe scarlet fever. It was originally escribe in menstruating women in association with tampon use, but it has been increasingly recognize in postsurgical wouns. In the majority of cases, the illness is cause by S. aureus strains that express toxic shock synrome toxin-1, enterotoxin B, or enterotoxin C. It has rarely been escribe in association with S. pyogenes (group A streptococci) (C). The remaining answer choices are not associate with toxic shock synrome (A, B, E). Half of the postsurgical toxic shock synrome cases present early, within 48 hours of operation. Symptoms inclue fever, iarrhea, vomiting, iffuse reness of the skin, an hypotension. This is followe a ay or two later by iffuse esquamation. Physical examination nings of woun infection are often unremarkable. Woun rainage an antibiotics are recommene. Aministration of clinamycin may be helpful because it inhibits exotoxin prouction. Reference: Reingol AL, Dan BB, Shans KN, Broome CV. Toxic-shock synrome not associate with menstruation. A review of 54 cases. Lancet. 198;1(86):1–4.

11. E. Lung abscesses typically present with an inolent

course over several weeks. Patients often complain of fevers, purulent sputum, an cough. Single lung abscesses are frequently monomicrobial an are usually associate with aspiration pneumonia. As such, they are typically foun in segments of the lung that are epenent in the supine position (i.e., the posterior segment of the upper lobes or the superior segments of the lower lobes). An air-ui level on a chest raiograph an purulent sputum are virtually iagnostic of an anaerobic lung infection. However, coinfection with antibiotic-resistant gram-positive organisms is possible in patients with frequent hospitalizations. Most lung abscesses will resolve with antibiotics alone, but aptomycin cannot be use to treat lung infections because it is inhibite by pulmonary surfactant (A). In aition to intravenous (IV) antibiotics, a patient with risk factors for lung cancer (e.g., smoking, recent weight loss) shoul unergo bronchoscopy to rule out an unerlying neoplasm (obstruction leaing to infectious process). Surgical treatment may be necessary for infections that fail to respon to meical management, abscesses greater than 6 cm in size, an abscesses seconary to an obstructe bronchus from a foreign boy or neoplasm. This typically involves either lobectomy or pneumonectomy (C). Percutaneous rain placement can be consiere in patients who are poor surgical caniates (D). Thoracotomy

CHAPtEr 31 Infection and Antimicrobial Therapy an ecortication are treatment options for empyema, not lung abscess (B). References: Manal K. Thoracic infections. In: Yuh DD, Vricella LA, Yang SC, Doty JR. es. Johns Hopkins textbook of cardiothoracic surgery. n e. McGraw-Hill; 014.

12. E. Parapneumonic effusion refers to the accumulation

of pleural ui in response to a respiratory infection. It is generally ivie into three stages: exuative, brinopurulent, an organizing. The rst (exuative) stage is characterize by the evelopment of sterile pleural ui in response to increase capillary permeability. After 5 ays, bacteria begin to enter the ui an inammatory cells follow. This marks the beginning of the brinopurulent phase. In general, new effusions shoul unergo iagnostic thoracentesis to rule out an empyema. If transuative, antibiotic treatment of the pneumonia is all that is require (A). Urgent rainage via tube thoracostomy is recommene for frankly purulent effusions or those with bacteria on Gram stain or culture. The iameter of the chest tube oes not seem to be important so long as smaller caliber tubes are routinely ushe to prevent blockage of the catheter (C). As the brinopurulent phase progresses, loculations begin to form within the collection, making rainage with a single catheter or tube thoracostomy ifcult. Several stuies have been one evaluating the use of intrapleural brinolytics, such as alteplase, to prevent progression to surgery. However, the results are controversial at best, an a 008 Cochrane Review of the practice foun no consistent benet (B). At this stage, vieo-assiste thoracoscopic ebriement an ahesiolysis are viable options, though a certain number of patients will still nee to be converte to thoracotomy (E). After  to 3 weeks of untreate infection, broblasts begin to form a pleural peel an the nal (organization) stage is reache. Once this membrane has forme, formal ecortication via thoracotomy is generally necessary. In patients that are unt for surgery, open rainage (e.g., Eloesser ap) may be consiere. However, this subjects patients to months of ressing changes an signicant morbiity (D). References: Cameron R, Davies HR. Intra-pleural brinolytic therapy versus conservative management in the treatment of ault parapneumonic effusions an empyema. Cochrane Database Syst Rev. 008;:CD0031. Davies HE, Davies RJO, Davies CWH, BTS Pleural Disease Guieline Group. Management of pleural infection in aults: British Thoracic Society Pleural Disease Guieline 010. Thorax. 010;65(Suppl ):ii41–ii53. Light RW. Parapneumonic effusions an empyema. Proc Am Thorac Soc. 006;3(1):75–80.

13. B. All

penicillin-erivative antibiotics (β-lactams) inhibit the nal step of bacterial cell wall synthesis by bining transpeptiases or penicillin-bining proteins (A). Cephalosporins work by the same mechanism but are more resistant to egraation by β-lactamases. Tazobactam, sulbactam, an clavulanic aci bin β-lactamases an are frequently combine with penicillin-erivative antibiotics to increase their effectiveness. Examples of this inclue piperacillin-tazobactam an amoxicillin-clavulanic aci. Metroniazole is an antibiotic that only has action against anaerobic bacteria by inhibiting nucleic aci synthesis. It is not effective in aerobic cells because it requires reuction to its active state, which only takes place in anaerobic cells (C). Aminoglycosies an

405

tetracyclines inhibit the 30S ribosome. Linezoli, on the other han, inhibits the 50S ribosome subunit. Several other antibiotics (macrolies, linezoli, chloramphenicol) also inhibit the 50S ribosome; however, it is a slightly ifferent process (D). Clinamycin is a lincosamie antibiotic, which interferes with the amino acyl-tRNA complex (E). Aminoglycosies an tetracycline antibiotics inhibit the 30S ribosome.

14. C. The Third International Consensus Denitions for Sepsis

and Septic Shock, publishe in JAMA in 016, reene the current enition use for sepsis an septic shock. The panel came to the conclusion that the previously use enition of sepsis (+ SIRS criteria an a source of infection) was too nonspecic an generally unhelpful in the ientication of patients at increase risk of mortality from infection (A, B, D, E). Instea, the committee recommene a besie screening tool calle the quick Sequential Organ System Failure score (qSOFA) for ientication of patients that are likely to have a poor outcome as the result of an infection. If a patient meets two of the three criteria (respiratory rate >/min, altere mental status, an systolic bloo pressure 40 shoul receive .5g protein/kg/ay as part of an overall strategy of hypocaloric feeing E. The presence of a 40% burn requires nutritional support with .5 g protein/kg/ay F. Critically ill patients with renal failure shoul receive 1.5 to 1.75 g protein/kg/ay, whereas patients with renal failure on continuous renal replacement therapy require .5 g protein/kg/ay G. Respiratory quotient (RQ): estimates basal metabolic rate; ratio of carbon ioxie prouce by the boy to oxygen consume by the boy 1. RQ > 1.0: overfeeing, can lea to ifculty weaning from ventilator ue to hypercarbia . RQ = 1: carbohyrate utilization 3. RQ = 0.8 to 0.9: protein utilization (average 0.85; mixture of fat, protein, carb metabolization) 4. RQ = 0.7: fat utilization 5. RQ < 0.7: starvation II. Amino Acis: Builing Blocks for Proteins A. Essential: not mae by the boy, must be ingeste; phenylalanine, valine, threonine, tryptophan, isoleucine, methionine, histiine, leucine, lysine B. Nonessential: boy can prouce these: alanine, asparagine, aspartic aci, glutamic aci, serine C. Conitional amino acis: boy can prouce these, but are essential in times of stress: Arginine, cysteine, glutamine, glycine, ornithine, proline, tyrosine III. Nitrogen Balance: To Stuy Protein Metabolism A. Nitrogen balance = protein intake/6.5 – (4-hour urine nitrogen + 4g) B. 4g approximates losses via sweat an feces C. Positive = anabolism; negative = catabolism IV. Starvation A. Certain cells (brain, re bloo cells) primarily use glucose for energy (except when starving) B. During starvation, insulin ecreases an glucagon increases, leaing to an increase in glycogenolysis, lipolysis, an ketogenesis

409

410

PArt ii Medical Knowledge

C. Glycogen stores (glucose supply) are eplete after 1 to  ays D. Next, fatty acis (can’t cross bloo-brain barrier) are oxiize to make ketones (for brain) E. With further starvation, muscle (protein) breaks own to provie alanine for gluconeogenesis V. Vitamin an Mineral Deciencies

Deęciency

Manifestation/disease

Vitamin A

Night blindness

Vitamin B1 (Thiamine)

Wernicke’s encephalopathy, Beriberi

Vitamin B3 (Niacin)

Pellagra (diarrhea, dermatitis, dementia)

Vitamin B6 (Pyridoxine)

Anemia, peripheral neuropathy

Vitamin B12 (Cyanocobalamin)

Megaloblastic anemia, peripheral neuropathy

Vitamin C

Impaired collagen cross-linking, scurvy

Vitamin D

Rickets, osteomalacia

Vitamin E

Neuropathy

Vitamin K

Coagulopathy

Chromium

Hyperglycemia, neuropathy

Copper

Anemia, leukopenia, muscle weakness

Iodine

Goiter

Phosphate

Diaphragm muscle weakness, arrhythmia, confusion

Selenium

Cardiomyopathy, weakness

Zinc

Delayed wound healing, hair loss, acne

Essential faĴy acids

Dermatitis, hair loss, easy bruising, delayed wound healing

Essential amino acids

Decreased immune function

VI. Nutritional Deciencies Associate With Surgeries/Surgical Diseases

Surgery/surgical diseases

Related nutritional deęciency

Gastric bypass

Deęciency of vitamin B12, folate, zinc, iron, copper, calcium, vitamin D

Gastric sleeve resection

Deęciency of vitamin B12, folate, zinc, iron

Carcinoid syndrome

Tryptophan deęciency (due to conversion to serotonin) causes pellagra

Blind loop syndrome

Deęciency of vitamin B12, folate, iron, vitamin E

Refeeding syndrome

Hypophosphatemia, hypomagnesemia, hypokalemia

CHAPtEr 32 Nutrition and Metabolism

411

Questions 1. A 45-year-ol male is iagnose with severe gallstone pancreatitis. It is currently hospital ay  an he is not requiring vasopressor support or invasive mechanical ventilation. He still reports mil epigastric pain. Which of the following is true regaring the ieal management of his nutrition? A. He shoul be starte on an oral iet as tolerate B. A nasoenteric tube shoul be place with tube fees starte at a trophic rate an avance to goal as tolerate C. Nasojejunal feeing is preferre over nasogastric feeing D. Total parenteral nutrition (TPN) is preferre over enteral nutrition E. Enteric feeing shoul not be consiere until abominal pain has resolve 2. Which of the following is true regaring immunonutrition? A. Alanine is a substrate in the prouction of nitric oxie (NO) B. Glutamine has been shown to reuce raiation injury to the small bowel C. For cancer patients unergoing surgery, immunonutrition reuces postoperative infectious complications D. For patients unergoing major abominal surgery, immunonutrition reuces mortality E. Glutamine has been shown to ecrease ventilator time in critically ill patients 3. A 55-year-ol alcoholic female is amitte to the hospital for a small bowel obstruction. Her serum albumin is 1.8 g/L (normal range is 3.4–5.4 g/L) an prealbumin is 8 mg/L (normal range is 15–36 mg/L). On hospital ay 5, she fails nonoperative management an unergoes an exploratory laparotomy with

lysis of ahesions. She subsequently evelops a postoperative ileus an is starte on total parenteral nutrition on postoperative ay 3. A ay later, she rapily evelops weakness, altere mental status, an hypoxic respiratory failure requiring intubation. Which of the following is true regaring her conition? A. Thiamine eciency is the most likely cause of her symptoms B. Alcoholism is not a risk factor for eveloping this conition C. This conition rarely occurs with enteral nutrition D. This conition coul have potentially been avoie by starting TPN at a slower rate E. She shoul be given a calcium infusion 4. Which of the following amino acis can be synthesize e novo in humans in any physiologic state? A. Tryptophan B. Tyrosine C. Glycine D. Serine E. Any branche-chain amino aci 5. Which of the following is true regaring the use of preoperative TPN to prevent postoperative complications? A. It is useful even if use for as little as 3 ays B. It is efcacious if the patient has lost more than 15% weight before surgery C. There is no evience that it lowers the complication rate D. Slightly overfeeing for 7 ays is recommene as a means to maximize replacement of caloric ecits E. TPN is efcacious even in mil to moerate malnutrition

412

PArt ii Medical Knowledge

6. Which of the following is true regaring nutritional eciencies after a partial gastrectomy with a Billroth II (gastrojejunostomy) reconstruction? A. Calcium absorption will be minimally affecte B. Iron eciency anemia is more common with a Billroth I (gastrouoenostomy) than a Billroth II C. Vitamin B1 eciency will present with a low mean corpuscular volume D. The stomach has no intrinsic absorptive ability E. Carbohyrate absorption is not impaire after surgery 7. Which of the following is true regaring nutrition nees an requirements? A. Preterm infants may nee up to  g/kg per ay of protein B. 1 g of fat provies 4 kcals of energy C. A respiratory quotient (RQ) greater than 1.0 suggests overfeeing D. Ventilate critically ill patients require more aily caloric intake than nonventilate critically ill patients E. Obese patients require less aily protein intake compare to nonobese patients 8. A 3-year-ol male was amitte 7 ays ago for multisystem trauma incluing multiple long-bone fractures, subural hematoma, an pulmonary contusions an is still on the ventilator. Which of the following is true regaring tools for assessing nutritional status? A. Use of serial measurements of albumin an prealbumin is the “gol stanar” for trauma patients B. Measurement of nitrogen balance unerestimates nitrogen input C. The Mini Nutritional Assessment is esigne specically for hospitalize patients D. Creatinine height inex may overestimate lean boy mass in trauma patients E. Transferrin is the serum protein that correlates the closest to nitrogen balance

9. Which of the following is true regaring the risk of hypoglycemia following cessation of total parenteral nutrition (TPN)? A. It commonly occurs in patients with liver isease B. Tapering of TPN is recommene so as to avoi this complication C. This complication is relatively common D. It is more likely to occur in a iabetic patient E. It is more likely to occur in patients with renal isease 10. Which of the following is true about the pharmacologic treatment of cancer cachexia? A. There is no evience that ghrelin mimetics are of benet B. Cannabinois are superior to megestrol acetate in stimulating weight gain C. When initiate early, megestrol acetate has been emonstrate to improve survival D. Megestrol is a progesterone erivative E. Anabolic sterois lea to improve long-term weight gain 11. Which of the following is true regaring energy homeostasis uring perios of starvation? A. The largest source of energy after glycogen is eplete is free fatty acis B. Skeletal muscle has the largest store of glycogen available systemically C. Glucose is converte to lactate in the liver D. Re bloo cells metabolize glucose aerobically E. The brain is unable to utilize ketones 12. The most important amino aci use for gluconeogenesis by the liver is: A. Glutamine B. Serine C. Alanine D. Tyrosine E. Asparagine 13. Poor glucose control is a manifestation of eciency of: A. Zinc B. Copper C. Chromium D. Molybenum E. Selenium

CHAPtEr 32 Nutrition and Metabolism

14. Which of the following is true regaring longterm TPN? A. Fat is consiere the nutritional basis of TPN B. It may lea to a mucin gel matrix of cholesterol crystals an calcium bilirubinate in the gallblaer C. Hepatic ysfunction relate to TPN is less likely to be lethal in infants than in aults D. It has not been shown to lea to hepatic brosis E. Carnitine supplementation has been shown to reverse TPN-relate liver amage

413

15. Which of the following amino acis has shown potential for increasing the absorptive capability of the intestine in patients that have unergone large segment small bowel resection? A. Glutamine B. Serine C. Alanine D. Tyrosine E. Arginine

Answers 1. B. Though avancing to an oral iet as tolerate is rec-

ommene in mil acute pancreatitis, this is not the recommene management in moerate to severe pancreatitis (A). Instea, patients with moerate to severe acute pancreatitis shoul have a nasoenteric/oroenteric tube place an enteral nutrition starte in the rst 1 to  hospital ays (B). Mil epigastric pain is not a contrainication to enteric feeing (E). With regars to the level at which to fee, three ranomize controlle trials showe no ifference in tolerance or clinical outcomes between gastric an jejunal feeing (C). The use of parenteral nutrition rather than enteral nutrition has also been explore in multiple metaanalyses of ten ranomize clinical trials. These have shown that those receiving enteral nutrition ha lower infectious morbiity, shorter length of stay, fewer surgical interventions, an ecrease mortality (D). Guielines from the Society of Critical Care Meicine an the American Society for Parenteral an Enteral Nutrition now recommen consieration of probiotics in severe pancreatitis for those receiving early enteral nutrition, which is base on a 010 metaanalysis that showe a reuction in infection an hospital LOS. Reference: McClave SA, Taylor BE, Martinale RG, et al. Guielines for the Provision an Assessment of Nutrition Support Therapy in the Ault Critically Ill Patient: Society of Critical Care Meicine (SCCM) an American Society for Parenteral an Enteral Nutrition (A.S.P.E.N.). J Parenter Enter Nutr. 016;40():159–11.

2. C. Immunonutrition is the ability to moulate the

immune system using specic nutrients. This strategy has most often been utilize in critically ill an surgical patients who often require exogenous nutrients through enteral or parenteral routes. The nutrients most robustly stuie for immunonutrition are arginine, glutamine, omega-3 fatty acis, branche chain amino acis, an nucleoties. Arginine an glutamine, two amino acis that have been of particular interest in this el, have unique properties that may explain their mechanism of action. Arginine, via the arginine eaminase pathway, is a unique substrate for prouction of NO (A). Glutamine is the most prevalent free amino aci in the

human boy an is also the major metabolic fuel for enterocytes an other cells within the immune system. Aministration of glutamine has been shown to have no effect on reucing raiation injury (B). A number of large systematic reviews have shown benets of immunonutrients in various subsets of surgical patients. For example, in surgical cancer patients an patients unergoing major abominal surgery, immunonutrition reuces infectious complications an shortens length of stay, but oes not ecrease mortality (C, D). In burn patients, initial clinical trial ata suggests that glutamine may reuce mortality, length of stay, an gram-negative bacteremia though the enitive RE-ENERGIZE trial is ongoing. However, there may be subsets of patients, like the critically ill, who may be harme by immunonutrition. Two large multicenter ranomize controlle trials of critically ill ventilate patients showe that supplementation with glutamine an/or antioxiants may increase 6-month mortality (E). References: Caler PC. Immunonutrition. BMJ. 003;37(7407): 117–118. Suchner U, Kuhn KS, Fürst P. The scientic basis of immunonutrition. Proc Nutr Soc. 000;59(4):553–563. Probst P, Ohmann S, Klaiber U, et al. Meta-analysis of immunonutrition in major abominal surgery. BJS. 017;104(1):1594–1608. Wischmeyer PE. Glutamine in burn injury. Nutr Clin Pract. 019;34(5):681–687. Yu K, Zheng X, Wang G, et al. Immunonutrition vs stanar nutrition for cancer patients: a systematic review an meta-analysis (part 1). J Parenter Enter Nutr. 00;44(5):74–767. van Zanten AR, Hofman Z, Heylan DK. Consequences of the REDOXS an METAPLUS Trials: the en of an era of glutamine an antioxiant supplementation for critically ill patients? J Parenter Enter Nutr. 015;39(8):890–89.

3. D. This patient evelope refeeing synrome, which

is a potentially fatal metabolic isturbance after the reinstitution of nutrition in a malnourishe patient (low serum albumin an prealbumin). Prolonge starvation leas to the severe epletion of a number of minerals, though serum concentrations remain relatively normal ue to compensatory

414

PArt ii Medical Knowledge

intra/extracellular shifts. During refeeing, insulin is release, leaing to stimulation of glycogen, fat, an protein synthesis, which requires phosphate, magnesium, an other cofactors. Phosphate, magnesium, an potassium (through the ATPase symporter) are all taken up into cells, leaing to a suen ecrease in serum levels. Refeeing synrome is cause by these epletions with hypophosphatemia being the most common an most severe isturbance (A). Some common clinical manifestations of severe hypophosphatemia are arrhythmias, metabolic aciosis, seizures, elirium, hyperglycemia, an profoun weakness, sometimes manifesting as iaphragm insufciency requiring mechanical ventilatory support. Risk factors for refeeing synrome inclue anorexia nervosa, malnutrition, chronic alcoholism, cancer, recent surgery, elerly patients with comorbiities, BMI 00 ug/L before resection = poor prognosis B. CA 19-9: Pancreatic cancer 1. >1000U/mL = likely metastatic isease C. CA-15: Epithelial ovarian cancer D. Inhibin-A: epithelial stroma tumors such as mucinous an enometrioi carcinoma an sex cor-stromal tumors such as granulosa cell tumor an Sertoli-Leyig cell tumor E. AFP: hepatocellular carcinoma (HCC), nonseminomatous germ cell tumors F. β-hCG: germ cell tumors G. Calcitonin: meullary thyroi cancer H. Thyroglobulin: papillary thyroi cancer; particularly for etecting recurrence after surgery an raioactive ioine therapy I. Chromogranin A: carcinoi tumors Chemotherapeutic Sie Effects

Agent

Side eěects

Bleomycin

Pulmonary ębrosis

Carboplatin

Myelosuppression

Cisplatin

Nephrotoxicity, neurotoxicity, ototoxicity

Cyclophosphamide

Hemorrhagic cystitis (treat with Mesna), gonadal dysfunction, SIADH

Doxorubicin

Cardiac toxicity (irreversible)

Methotrexate

Nephrotoxicity, nausea (treat with folinic acid)

Oxaliplatin

Peripheral neuropathy

Taxols

Neuropathy

Trastuzumab

Cardiomyopathy (reversible)

Vinblastine

Myelosuppression

Vincristine

Peripheral neuropathy

Hereitary Cancer Synromes

Gene

Syndrome

Associated cancers

APC

Familial adenomatous polyposis (FAP)

Colon, duodenum, gastric, thyroid, desmoid tumors

TP53

Li Fraumeni

Breast (90%), colon, lung, sarcomas, brain, adrenal

DNA Mismatch Repair (MLH1, MSH2, MSH6, PMS2, EPCAM)

Hereditary nonpolyposis colorectal cancer (HNPCC)

Colorectal, endometrial, gastric, ovarian, GU tract, hepatobiliary, small bowel, and CNS

SMAD4, BMPR1A

Juvenile polyposis

Colon, gastric

STK11

Peuĵ-Jeghers

Hamartomas—GI and mucocutaneous. Malignancies—breast, colon, pancreatic, gastric, ovarian, lung, small intestine, endometrial, testicular, esophageal

PTEN

Cowden

Breast, facial lesions, GI hamartomas, thyroid, endometrial

BRCA1

Breast (87% risk), ovarian (40%–60%)

BRCA2

Breast (80%), ovarian (15%–20%), prostate, pancreatic

CHAPtEr 33 Oncology and Tumor Biology

419

Multiple Enocrine Neoplasia (MEN) Synromes

MC Clinical presentation

Organs/tumors involved

What to treat ęrst

Autosomal Dominant

Hypercalcemia

Pancreatic NET Pituitary Hyperparathyroidism

Hyperparathyroidism

RET

Autosomal Dominant

Medullary Thyroid Cancer

Medullary Thyroid CA Pheochromocytoma Hyperparathyroidism

Pheochromocytoma

RET

Autosomal Dominant

Medullary Thyroid Cancer

Medullary Thyroid CA Pheochromocytoma Mucosal Neuromas Marfanoid Habitus

Pheochromocytoma

Syndrome Gene

Protein

Inheritance

MEN 1

11q13

Menin

MEN 2A

10q11.21

MEN2B

10q11.21

420

PArt ii Medical Knowledge

Questions 1. A 44-year-ol male with a history of hypertension well controlle on meications is foun to be anemic an with a positive fecal occult bloo test uring his yearly physical. He notes a family history that inclues eaths ue to colon cancer in of both his mother at age 46 an his maternal granfather at age 51. He unergoes colonoscopy which emonstrates four aenomatous polyps in the ascening colon as well as an aenocarcinoma of the ileocecal junction. This patient is most likely to have which of the following? A. A mutation in the TP53 gene B. A mutation in the PMS gene C. A mutation in the PTEN gene D. A mutation in the STK11 gene E. A mutation in the aenomatous polyposis coli (APC) gene 2. An otherwise healthy 68-year-ol woman is iagnose with locally avance gastric aenocarcinoma. There is no evience of istant metastases. Her tumor is biopsie an note to have HER2/neu overexpression. She is starte on an appropriate chemotherapy regimen. After her secon cycle, she presents with new-onset yspnea on exertion an orthopnea. Which of the following chemotherapeutic agents is likely responsible for her symptoms? A. Bleomycin B. 5-Fluorouracil C. Vinblastine D. Trastuzumab E. Cisplatin 3. A 70-year-ol otherwise healthy male with a history of colon aenocarcinoma that was treate with a formal resection returns two years later with a 3-cm lesion on his liver. Workup conrms a colorectal metastasis with no evience of sprea elsewhere. Which of the following is the most appropriate next step? A. Chemotherapy only B. Surgical resection only C. Surgical resection followe by chemotherapy D. Chemotherapy followe by surgical resection E. Surgical resection followe by raiation

4. An 87-year-ol female presents to the emergency epartment (ED) with weight loss, vomiting, obstipation, an a istene abomen. She has not ha a bowel movement in 3 ays. Past history is signicant for a non-ST segment elevation myocarial infarction (NSTEMI) 6 weeks earlier. A compute tomography (CT) scan with oral contrast shows evience of an obstructing mass in the sigmoi colon. However, the lumen oes appear to be patent. Her vitals are stable. Which of the following is the best recommenation? A. Diverting ileostomy B. Diverting transverse colostomy C. Open sigmoi resection with proximal colostomy D. Colonoscopy with placement of a temporizing stent followe by elective surgery E. Laparoscopic sigmoi resection with proximal colostomy 5. A patient with metastatic sigmoi colon cancer is about to unergo chemotherapy, an the oncologist recommens the use of an anti-EGFR monoclonal antiboy. Which of the following genetic proles is most likely to benet from the aition of this agent? A. K-ras wiltype gene B. BRAF mutation C. NRAS D. PIK3CA mutation E. K-ras mutant gene 6. Which of the following patients shoul be referre to a genetic counselor for BRCA testing? A. Family history of breast cancer in mother at the age of 55 B. Both parents are Sepharic Jews C. Aopte an unknown family history, evelope breast cancer at 55 D. 55-year-ol female with breast cancer in bilateral breasts E. 55-year-ol female with an inammatory breast cancer

CHAPtEr 33 Oncology and Tumor Biology

7. A 55-year-ol male presents to the ED with vomiting an an inability to tolerate oral intake for the last week. CT scan shows a signicantly istene stomach, with a thickene mass near the pylorus. Upper enoscopy shows a large mass in the stomach that partly occlues the istal lumen. Biopsy is consistent with low-grae mucosa-associate lymphoi tissue (MALT) lymphoma. He takes proton-pump inhibitors for aci reux. Which of the following is true regaring his conition? A. Triple antibiotic therapy for eraication of Helicobacter pylori shoul be starte regarless of whether the patient is H. pylori positive or negative B. The patient shoul be given chemotherapy along with triple antibiotic therapy C. Gastrectomy has no role in the treatment of gastric MALT lymphoma D. Raiotherapy has no role in the treatment of gastric MALT lymphoma E. Surgery is recommene for patients who o not respon to triple antibiotic therapy 8. Which of the following is true regaring the interaction between raiation therapy an tumor cells? A. Raiation therapy leas to cancer cell eath by irectly inhibiting aenosine triphosphate (ATP) prouction in the mitochonria B. Larger tumors are more sensitive to raiation therapy C. As the energy use in raiation therapy increases, collateral amage to overlying skin also increases D. The S phase of the cell cycle is most sensitive to raiation effects E. Correcting anemia can increase the efcacy of raiotherapy 9. A 60-year-ol male with cirrhosis presents to clinic with a newly iagnose 4-cm hepatocellular carcinoma (HCC) in segment 6. There is no evience of gross vascular invasion an no regional noal or extrahepatic istant metastases. His international normalize ratio (INR) is 1.8, creatinine is 1.0 mg/L, bilirubin is 3.1 mg/L, an albumin is .6 mg/L, an his compute tomography (CT) scan shows no evience of ascites. Which of the following woul be the best treatment option? A. Transarterial chemoembolization (TACE) B. Liver resection C. Raiofrequency ablation (RFA) D. Irreversible electroporation E. Liver transplantation

421

10. Which of the following is true regaring the evelopment of skin cancers? A. Ultraviolet (UV) raiation both initiates an promotes DNA amage B. UVA is the ultraviolet frequency most responsible for chronic skin amage C. An increase level of skin melanin increases the risk of eveloping basal cell carcinoma D. UV raiation amages the DNA mismatch repair gene E. Mutations in the BCL- gene are a known mechanism for the evelopment of skin cancer 11. A 43-year-ol male is iagnose with a highgrae right lower extremity osteosarcoma an unergoes surgical resection an ajuvant chemotherapy with MAP (methotrexate, oxorubicin, an cisplatin). After the thir treatment cycle, the patient evelops severe nausea, vomiting, an altere mental status. Workup reveals increase liver transaminases, a reuction in glomerular ltration rate (GFR), as well as leukopenia an thrombocytopenia. What meication can potentially reverse these effects? A. Cobalamin B. Folinic aci C. Folic aci D. Folate E. Omeprazole 12. Which of the following statements is true regaring patterns of metastatic sprea? A. The most common metastatic location for breast cancer is the arenal glan B. The most common metastatic location for melanoma is the small bowel C. Metastases to the arenal glan most commonly originate in the lungs D. The most common metastatic location for colon cancer is the lungs E. The transverse colon is frequently the rst location of metastatic sprea of pancreatic cancer

422

PArt ii Medical Knowledge

13. Which of the following statements is true regaring the human protein p53? A. Germline mutations of the p53 gene result in Cowen Synrome B. The unregulate growth seen with human papillomavirus (HPV) is partly ue to bining an inactivation of the p53 protein C. The p53 gene suppresses the translation process in DNA sequencing an cell growth D. Overexpression of this gene leas to uncontrolle cell growth E. Mutations frequently result in benign neoplastic growth rather than malignancy

14. A 77-year-ol male who resies in a subacute care facility has just nishe ajuvant chemotherapy (FOLFOX an Bevacizumab) for metastatic colon cancer. Despite a normal albumin, minimal weight loss, an meticulous local woun care, his nurses have been unable to aequately treat a nonhealing sacral ecubitus ulcer. The woun base looks clean, an he has no signs of systemic infection. Which of the following is true? A. The sacral woun shoul be preemptively ebrie to avoi infection an facilitate woun healing B. Supplemental enteral nutrition will facilitate faster woun healing C. Rescue therapy can be attempte with leucovorin D. The patient shoul be converte to Cetuximab E. Barriers to healing will likely resolve in 6 months

Answers 1. B. This patient has Lynch synrome (hereitary nonpol-

yposis colorectal cancer—HNPCC). The Amsteram criteria ene the criteria necessary for iagnosis. They inclue three or more family members who have been iagnose with an HNPCC-associate cancer (colorectal, enometrial, gastric, ovarian, GU tract, hepatobiliary, small bowel, an CNS—but most commonly colorectal cancer), one of whom is a rst-egree relative of the other two; at least two generations of family members involve; an at least one member iagnose with colorectal cancer prior to the age of 50. In aition, no family members may have been iagnose with FAP. HNPCC is characterize by mutations in mismatch repair genes (MLH1, MSH, MSH6, PMS, EPCAM), resulting in microsatellite instability (MSI), an is inherite in an autosomal ominant fashion. Aitionally, there is a high frequency of cancers in HNPCC arising in the proximal colon when compare to other hereitary colorectal cancer synromes (B). A mutation in the TP53 gene results in Li Fraumeni synrome which is characterize by tumors of the breast (90%), colon, lung, brain, an arenal, as well as sarcomas (A). PTEN mutations result in Cowen synrome, in which patients evelop tumors of the breast, thyroi, an enometrium as well as facial lesions an GI hamartomas (C). Peutz-Jeghers synrome is ue to a mutation in the STK11 gene an is characterize by hamartomas (both GI an mucocutaneous) as well as malignancies of the breast, colon, pancreas, stomach, ovaries, lung, small intestine, enometrium, testicles, an esophagus (D). Familial aenomatous polyposis (FAP) is ue to a mutation in the APC gene an is typically characterize by the appearance of thousans of aenomatous polyps throughout the colon early in life (E). References: Greenel LJ, Mulhollan MW, es. Greeneld’s surgery: scientic principles & practice. 5th e. Lippincott Williams an Wilkins; 010.

Mayer RJ. Lower gastrointestinal cancers. In: Jameson J, Fauci AS, Kasper DL, Hauser SL, Longo DL, Loscalzo J. es. Harrison’s principles of internal medicine. 0th e. McGraw-Hill; 018. Morris A. Epiemiology—clinical risk factors—familial cancer synromes. In: Greenel LJ, Mulhollan MW, es. Greeneld’s surgery: scientic principles & practice. 5th e. Lippincott Williams an Wilkins; 010.

2. D. Commonly utilize chemotherapy regimens in gas-

tric cancer inclue FLOT (5-FU, leucovorin, oxaliplatin, an ocetaxel) as well as capecitabine, cisplatin, an epirubicin. In cases of HER/neu overexpression, trastuzumab may be ae. Trastuzumab is a monoclonal antiboy therapy use in the treatment of HER/neu overexpressing cancers (most commonly breast an GI origins). Trastuzumab can cause reversible cariomyopathy (D). 5-uorouracil (5-FU) is a component of the FLOT regimen for gastric cancer, but cariomyopathy is not a common sie effect (B). Bleomycin is utilize in the treatment of lymphoma, testicular, ovarian, an cervical cancers an can cause pulmonary brosis (A). Vinblastine is not a typical therapeutic agent in the treatment of gastric cancer an can cause myelosuppression (C). Cisplatin is frequently utilize in the treatment of gastric cancer, but its sie effects inclue nephrotoxicity, neurotoxicity, an ototoxicity (E). References: Sah BK, Zhang B, Zhang H, et al. Neoajuvant FLOT versus SOX phase II ranomize clinical trial for patients with locally avance gastric cancer. Nat Commun. 00;11(1):6093. Wagner AD, Syn NL, Moehler M, et al. Chemotherapy for avance gastric cancer. Cochrane Database Syst Rev. 017;8:CD004064.

3. C. Recent literature shows a conferre survival benet

for the resection of hepatic metastases in colorectal cancer. Multiple high-volume centers have emonstrate the 5-year survival for patients with metastatic colorectal cancer to the liver to be 5% to 58% with resection of the metastatic lesion.

CHAPtEr 33 Oncology and Tumor Biology Over the last two ecaes, the perioperative mortality associate with hepatic resection has fallen signicantly, with most high-volume centers reporting a 30-ay perioperative mortality of less than %. The presence of any of the following risk factors ha a negative, an aitive, effect on survival in patients with hepatic metastases from colorectal cancer: (1) noe-positive primary tumor, () isease-free interval less than 1 months, (3) multiple liver metastases, (4) largest hepatic metastasis greater than 5 cm, an (5) serum carcinoembryonic antigen (CEA) level greater than 00 ng/mL. Those with none of these risk factors have the greatest 5-year survival at 60%. Treatment will vary epening on whether it is a synchronous or metachronous lesion. Synchronous lesions can be safely treate with combine colon an liver resection, provie the hepatic resection is limite (75 ( points), previous TIA/stroke ( points), iabetes (1 point), previous vascular isease (1 point), hypertension (1point), CHF (1 point), female (1 point) ) Novel oral anticoagulant (NOAC) rugs a) Direct-thrombin inhibitors (1) Bivaliruin, argatroban, an esiruin: parenteral aministration () Dabigatran: only available oral agent; half-life is 1 to 17 hours; stop  ays before surgery; renally metabolize so use with caution in patients with renal isease (a) Reverse with iarucizumab (a monoclonal antiboy that bins abigatran) b) Factor-Xa inhibitors (all factor-Xa inhibitors en in -xaban) (1) Rivaroxaban: half-life is 6 to 9hours, an its therapeutic activity wears off after 4 to 5half-lives; rivaroxaban shoul be iscontinue 1 to  ays before a surgical proceure; in patients with a reuce creatinine clearance, it shoul be iscontinue 3 to 5 ays before surgery () Apixaban: factor-Xa inhibitor, half-life of 1 hours; iscontinue  ays prior to surgery; metabolize by liver so safe for patients with renal isease (a) Reverse with 4-factor prothrombin complex concentrate (b) Anexanet alfa is a recombinant analog of factor Xa an may be consiere an antiote for factor-Xa inhibitors c) Can restart anticoagulation 6 to 4 hours after a minor proceure, an after  to 3 ays for major surgery (barring any perioperative bleeing)

437

438

PArt ii Medical Knowledge

C. Steroi therapy: chronic steroi therapy can affect the hypothalamic–pituitary–arenal axis, leaing to arenal atrophy an a ecrease capability to prouce cortisol leaing to a theoretical risk of hypotension in the perioperative perio; however, perioperative “stress-ose” sterois are not supporte by recent evience 1) “Stress-ose” sterois shoul not be routinely aministere; instea, the patient shoul continue their home ose of sterois perioperatively ) Shoul consier aitional sterois only if the patient evelops refractory hypotension suggestive of arenal insufciency in the perioperative perio

CHAPtEr 35 Preoperative Evaluation and Perioperative Care

439

Questions 1. A 58-year-ol man presents with a reucible inguinal hernia. He is not limite in his aily activities but is bothere by the appearance. He unerwent percutaneous coronary intervention (PCI) with placement of a rug-eluting stent (DES)  months ago an is currently taking aspirin an clopiogrel. What is the most appropriate management of this patient? A. Scheule surgery an continue aspirin an clopiogrel B. Scheule surgery an Continue aspirin an stop clopiogrel 5 ays prior to the operation C. Scheule surgery an stop aspirin an clopiogrel 5 ays prior to the operation D. Delay surgery for an aitional 4 months E. Delay surgery for a year post-DES 2. A 38-year-ol woman evelops fever, abominal pain, an multiple loose nonblooy bowel movements following amission for perforate appenicitis. Her WBC count is 1,000 an has normal kiney function. On imaging, there is no evience of eep space abscess or ileus an her stool tests positive for Clostridium difcile. This is her rst episoe. What is the most appropriate treatment? A. Fecal transplant B. Oral an rectal vancomycin C. Oral vancomycin an intravenous metroniazole D. Intravenous metroniazole E. Oral vancomycin 3. Five ays after a laparoscopic Roux-en-Y gastric bypass, a patient evelops fever with rigors, hypotension, tachycaria, an pain in the left shouler. This most likely represents: A. Gas bloat synrome B. Internal hernia C. Woun ehiscence D. Gastric volvulus E. Disruption of the gastric pouch–jejunal anastomosis

4. A 8-year-ol woman unergoes ahesiolysis for an acute small bowel obstruction. During the course of the surgery, she requires a segmental ileal resection with primary anastomosis. On postoperative ay 6, she is note to have thick bile-colore ui emanating from the miline woun. After IV hyration, the next step in the management shoul be: A. CT scan of the abomen B. Water-soluble upper gastrointestinal series with small bowel follow-through C. Fistulogram D. Operative reexploration E. Octreotie 5. The most important preictor of colonic ischemia after repair of a rupture abominal aortic aneurysm is: A. Age B. Presence of preoperative shock C. Time to operation D. Presence of associate cariac isease E. Preoperative patency of inferior mesenteric artery 6. Five ays after surgery for perforate appenicitis, liqui stool emanates from the right lower quarant woun. Which of the following is true about this conition? A. It is most commonly ue to an unrecognize malignancy B. The majority will close spontaneously C. The patient shoul be place immeiately on TPN D. Flui an electrolyte erangements are common E. The patient shoul be returne immeiately to the operating room for surgical repair 7. Five ays after a Billroth II gastric resection for a bleeing ulcer, high fever, hypotension, tachycaria, an generalize peritonitis evelop in the patient. This most likely represents: A. Postoperative pancreatitis B. Acalculous cholecystitis C. Duoenal stump blowout D. Woun ehiscence E. Intraabominal hemorrhage

440

PArt ii Medical Knowledge

8. Which of the following moalities is LEAST likely to assist in the prevention of postoperative pulmonary complications in a 65-year-ol male smoker? A. Postoperative use of an incentive spirometer B. Postoperative eep-breathing exercises C. Postoperative use of continuous positive airway pressure D. Smoking cessation 1 week before surgery E. Placement of a nasogastric tube 9. Which of the following preoperative stuies is most strongly associate with an increase risk of pulmonary-relate postoperative complications? A. Bloo urea nitrogen B. Incentive spirometry C. Chest raiograph D. Serum albumin E. Room air arterial bloo gas 10. A 67-year-ol male recovering from a pelvic exenteration seconary to locally avance rectal cancer is starte on total parenteral nutrition for prolonge ileus via a right-sie peripherally inserte central catheter (PICC) line. Several ays later his arm becomes swollen. Ultrasoun conrms clot in the basilic an axillary veins. What is the appropriate management of his conition? A. Warm compress an nonsteroial antiinammatory rugs (NSAIDs) B. Immeiately remove the line C. Immeiately remove line an then start heparin D. Start heparin an move the line to an alternate site E. Start heparin, keep the line in place, an therapeutic anticoagulation for 3 to 6 months 11. A 76-year-ol iabetic male is amitte to the surgical intensive care unit after a fall. His injuries inclue a right femoral neck fracture an subarachnoi hemorrhage. He continues to have intermittent elevation in his intracranial pressure an is still requiring respiratory support after  ays. Which of the following is true regaring nutritional supplementation in this patient? A. Postpyloric feeing may reuce his risk of eveloping pneumonia B. Gastric feeing is associate with a longer length of ICU stay C. Diabetic patients have better outcomes with gastric versus postpyloric feeings D. Postpyloric feeing more closely simulates normal physiologic feeing E. Gastric feeing is associate with increase total nutrition

12. A 55-year-ol obese female with chronic obstructive pulmonary isease (COPD) is unergoing preoperative evaluation for ventral hernia repair. She has a 30 pack/year smoking history, though she quit 1 year ago. Her COPD symptoms are well controlle with her current meication regimen, an her last amission for COPD exacerbation was over  years ago. Which of the following is true regaring risk assessment for postoperative pulmonary complications in this patient? A. Higher ASA class is a signicant risk factor B. Preoperative pulmonary function tests (PFTs) shoul be obtaine C. A nasogastric tube shoul be use postoperatively to ecrease pulmonary complications D. Upper miline an lower miline laparotomy confer similar risk for pulmonary complications E. A PaCO of more than 45 mmHg is an absolute contrainication to major abominal surgery 13. A 65-year-ol woman is amitte to the hospital with a large bowel obstruction. Workup reveals a sigmoi cancer, an on hospital ay 4, she unergoes laparoscopy with a plan to perform a resection with a proximal colostomy. During the operation, her en-tial carbon ioxie suenly rops, an she evelops tachycaria to the 10s with occasional premature atrial contractions. Her systolic bloo pressure is 80 mmHg. Which of the following woul be most helpful in establishing the presumptive iagnosis? A. Electrocariogram B. Cariac enzymes C. Transesophageal echocariogram (TEE) D. Arterial bloo gas E. Flexible bronchoscopy

CHAPtEr 35 Preoperative Evaluation and Perioperative Care

14. A 59-year-ol male with a coronary artery bypass grafting 1 year prior for multivessel isease unergoes a right hip replacement surgery. His postoperative course is complicate by pneumonia requiring mechanical ventilation. Electrocariogram shows a stable Q wave in lea II. Heart rate is 80 beats per minute an bloo pressure is 116/8 mmHg. Chest raiograph shows bilateral patchy inltrates. Laboratory exam emonstrates PaO of 70 mmHg, a white bloo cell count of 17,000 cells/μL, an hemoglobin of 7.4 g/L. Which of the following is true regaring the management of his anemia? A. Bloo transfusion will lower his risk of eveloping an acute coronary synrome B. He shoul be transfuse to a hemoglobin goal of 10 g/L C. Re bloo cell transfusion is inepenently associate with lower mortality D. Bloo transfusion is not necessary at this time E. Hemoglobin-base oxygen carriers offer a goo alternative to transfusion in this patient 15. Four ays after a pancreaticouoenectomy for pancreatic aenocarcinoma, a 65-year-ol man evelops a fever an tachycaria. Exam reveals tenerness, eema, an erythema over the angle of the jaw. Which of the following is true regaring this conition? A. It is usually ue to Staphylococcus B. Massage of the area is benecial C. It can be prevente with antibiotics D. The incience has been increasing E. It can be avoie with the use of anticholinergics 16. Which of the following is true regaring venous thromboembolism (VTE) prophylaxis in surgical patients? A. Intermittent pneumatic compression (IPC) prevents DVT by increasing circulating tissue plasminogen activator (tPA) B. Thigh-high IPC is superior to knee-high IPC C. IPC is equivalent to pharmacologic prophylaxis in the majority of patients D. Unfractionate heparin (UFH) is superior to lower-molecular-weight heparin (LMWH) E. LMWH is superior to IPC

441

17. A 69-year-ol patient with a tumor at the rectosigmoi junction unergoes laparoscopic sigmoi colectomy. Postoperative pain is well controlle with patient-controlle thoracic epiural anesthesia. On postoperative ay 1, prophylactic anticoagulation is starte with lowmolecular-weight heparin (LMWH). The blaer is unergoing rainage with an inwelling Foley catheter. Which of the following is true regaring epiural anesthesia? A. Blaer catheterization shoul continue while the thoracic epiural is in place B. LMWH shoul be hel for 4 hours before removal of the thoracic epiural C. The risk of urinary tract infection is the same regarless of whether the urinary catheter is remove on postoperative ay 1 versus postoperative ay 3 D. Risk of urinary retention is not signicantly higher with early removal of the Foley catheter E. Unfractionate heparin shoul not be restarte for at least 4 hours after removal of an epiural catheter 18. A 5-year-ol woman evelops a fever of 104°F 1 hours after an open cholecystectomy. On examination, she has foul-smelling, purulent rainage from her woun. She unergoes the appropriate treatment, an culture of the woun grows gram-positive ros. Which of the following is true regaring this patient an her conition? A. The causative organism is an aerobe B. Diabetes is not consiere to be a risk factor C. Broa-spectrum antibiotics an ui resuscitation resolve the majority of cases D. The organism prouces an enotoxin E. Clinamycin shoul be inclue in the management 19. A 34-year-ol woman unergoes a subtotal thyroiectomy for Graves isease. In the recovery room, she evelops anxiety an progressive respiratory istress with strior. Her incision is bulging an tense on exam. The most important initial step woul be: A. Nebulize racemic epinephrine B. Rapi-sequence intubation C. Neele aspiration of the neck woun D. Ultrasoun examination of the neck E. Rapily opening the incision at the besie

442

PArt ii Medical Knowledge

20. One ay after a left colectomy for recurrent iverticulitis, a patient is note to have an elevation of his serum creatinine. Other laboratories are unremarkable. He has a urine output of 30 to 50 mL/hour. A renal ultrasoun shows no evience of abnormalities with the exception of ascites. Compute tomography (CT) scan emonstrates iscontinuity of the left ureter with contrast extravasation at the level of the pelvic brim. Which of the following about this injury is true? A. Immeiate reoperation shoul not be performe B. Placement of ureteral stents woul have prevente this complication C. A percutaneous nephrostomy shoul be place D. A retrograe stent shoul be place E. A ureteroneocystostomy will likely be the best option 21. Two ays after sustaining signicant crush injury to her bilateral lower extremities from a motor vehicle collision, a 3-year-ol female becomes oliguric an is only proucing scant ark urine. Urine ipstick reveals 4+ bloo, an follow-up urinalysis shows 5 to 10 re bloo cells per high power el. Prevention of acute kiney injury is best achieve by which of the following? A. Urgent 4-compartment fasciotomies B. Loop iuretics C. Vigorous IV ui hyration D. Alkalization of urine with intravenous soium bicarbonate E. Mannitol 22. Which of the following is true regaring PFTs? A. Total lung capacity (TLC) is generally reuce with aging B. A preoperative force expiratory volume in one secon (FEV1) of less than 1.5 L is a contrainication for pulmonary lobectomy C. Diffusion capacity of the lungs for carbon monoxie (DLCO) will stay relatively constant with age so long as there is no intrinsic lung isease D. Percent-preicte postoperative FEV1 of >40% is acceptable for a lobectomy but not for a pneumonectomy E. Chest wall compliance ecreases with age

23. A 45-year-ol male with en-stage renal isease is unergoing placement of a tunnele hemoialysis catheter. During the operation, the anesthesiologist notices a sharp ecline in the continuous capnography an the calculate physiologic ea space is increase. This is followe by massive myocarial infarction an cariac arrest. Which of the following is true regaring this conition? A. Electrocariogram (ECG) will most commonly emonstrate right heart strain B. A congenital heart efect likely contribute to the cariac arrest C. The patient shoul be positione left sie up D. Besie transesophageal echocariography is generally not sensitive enough to etect this complication E. Aspiration from the central line is usually helpful 24. A 65-year-ol man with Barrett esophagus an new-onset ysphagia is being evaluate for iagnostic esophagogastrouoenoscopy (EGD), enoscopic ultrasoun (EUS), an mucosal biopsy. He is on warfarin for mechanical mitral valve an has a history of embolic stroke 10 years ago. What is recommene for his anticoagulation regimen before this proceure? A. Hol warfarin for 3 to 5 ays an brige with low-molecular-weight heparin B. Hol warfarin for 48 to 7 hours, brige with unfractionate heparin, an hol heparin 4 to 6 hours before the proceure C. Perform EGD an EUS while therapeutic on warfarin; if inicate, the mucosal biopsy can be performe at a later ate after holing warfarin D. Continue warfarin without interruption E. Hol warfarin 3 to 5 ays before proceure an restart within 4 hours after the proceure

CHAPtEr 35 Preoperative Evaluation and Perioperative Care

25. A 4-year-ol female with long-staning systemic lupus erythematosus (SLE) complicate by lupus nephritis an ebilitating arthritis is in the ICU following an emergency bowel resection 4 ays earlier. Over the next several hours, she becomes febrile, hypotensive, an complains of abominal pain. She is given ui boluses, but the bloo pressure oes not respon. Her abominal exam is unremarkable. Laboratory values reveal a white bloo cell count of 1,000 cells/L with eosinophilia, serum Na of 133 mEq/L, serum bicarbonate of 0 mEq/L, an serum K of 5.3 mEq/L. Which of the following represents the best management of this conition? A. Two liters of normal saline followe by 4 mg of examethasone B. Exploratory laparotomy C. Vasopressin D. Immeiate aministration of broa-spectrum antibiotics an 100 mg of hyrocortisone E. Flui resuscitation, vasopressor support, an AM cosyntropin test

443

26. Which of the following is true regaring the use of beta-blockers in the perioperative perio for patients unergoing noncariac surgery? A. Starting a beta-blocker within 4 hours of surgery may increase the incience of perioperative stroke B. Beta-blockers shoul be stoppe at least 1 week before surgery C. In low- an intermeiate cariac risk patients, beta-blockers shoul be initiate  to 3 weeks before surgery D. Beta-blockers shoul be avoie even in the high cariac risk group E. Perioperative initiation of beta-blocker ecreases the 30-ay mortality

Answers 1. D. The management of antiplatelet meications in

patients unergoing noncariac surgery after PCI poses a common surgical ilemma. In general, patients shoul be stratie by thrombotic risk base on the type an timing of PCI as well as hemorrhagic risk base on the type of surgery. Patients who unerwent plain ol balloon angioplasty within the past  weeks, bare metal stent within the past 1 month, an DES within the past 6 months are at high thrombotic risk. In general, surgery shoul be elaye until after this perio (A–C, E). For most abominal operations, continuing aspirin while holing clopiogrel 5 ays prior to surgery is appropriate. However, in the case of time-sensitive surgery such as a colectomy in a patient with colon cancer, surgery shoul not be elaye an the risks/benets of continuing ouble antiplatelet therapy shoul be iscusse with the patient. Of note, there is recent literature supporting the continuation of aspirin an clopiogrel in patients with colon cancer unergoing resection, even within a couple months of DES placement. Reference: Banerjee S, Angiolillo DJ, Boen WE, et al. Use of antiplatelet therapy/DAPT for post-PCI patients unergoing noncariac surgery. J Am Coll Cardiol. 017;69(14):1861–1870.

2. E. Infection with C. difcile is not an uncommon com-

plication following antibiotic treatment. While it has been classically associate with clinamycin, it can occur following treatment with a wie variety of antibiotics. For initial episoes, infection can either be nonsevere (leukocytosis 1.5), or fulminant (shock, toxic

megacolon). Nonsevere an severe infections are treate with either PO vancomycin or PO axomicin (if available) ×10. In cases of fulminant isease, treatment shoul inclue PO vancomycin an consieration of total abominal colectomy with en ileostomy. In cases of fulminant isease with ileus, rectal vancomycin an intravenous metroniazole shoul be ae (B, C, D). First recurrences can be treate with PO vancomycin (usual osing if metroniazole was use for the initial episoe or a prolonge pulse/tapere regiment if a stanar PO vancomycin regimen was use for the initial episoe). Alternatively, axomicin may be use if a stanar PO vancomycin regimen was use for the initial episoe. Subsequent recurrences can be treate with antibiotics or fecal transplant (A). Reference: McDonal LC, Gering DN, Johnson S, et al. Clinical practice guielines for Clostridium difcile infection in aults an chilren: 017 Upate by the Infectious Diseases Society of America (IDSA) an Society for Healthcare Epiemiology of America (SHEA). Clin Infect Dis. 018;66(7):987–994.

3. E. Fever, chills, tachycaria, hypotension, an peritoneal

irritation occurring together within 1 week of any surgery involving a new bowel anastomosis shoul immeiately raise suspicion for an anastomotic isruption. Left shouler pain is often a consequence of left iaphragm irritation an, in this case, correlates with the gastric pouch–jejunal anastomosis. Water-soluble contrast stuies can ai in the iagnosis an inicate how large the leak is because containe leaks can often be manage nonoperatively. However, in

444

PArt ii Medical Knowledge

this patient, hypotension an signs of peritonitis necessitate operative exploration an repair of the anastomosis. Gasbloat synrome results from the inability to relieve gas from the stomach after a funoplication (A). Gastric volvulus can occur after gastric surgery; however, this is extremely rare (D). Internal hernia is less likely given the timeline (most occur beyon a month after surgery) an left shouler pain inicative of iaphragmatic irritation (B). Woun ehiscence woul be suspecte if the skin is erythematous, warm, raining purulent or serous material, an has fallen apart (C).

4. A. This case represents an enterocutaneous stula, likely

resulting from either an anastomotic leak or an unrecognize intraoperative bowel injury away from the anastomosis. Management of enterocutaneous stulas shoul begin with stabilizing the patient via aggressive ui hyration an control of sepsis (if present). If the patient is manifesting signs of sepsis, prompt aministration of IV antibiotics shoul be institute. Sepsis, ehyration, an electrolyte/ nutrient losses are the most evastating early consequences. Prompt return to the operating room is not recommene because the peritoneal cavity will likely have highly vascular ahesions, making reentry treacherous, an early attempts to reclose stulas typically fail (D). Once the patient has been stabilize, the best initial stuy is a CT scan of the abomen. This will ientify whether any intraabominal abscesses are present that might require percutaneous rainage an rule out whether there is a istal obstruction (B, C). Fistulas are loosely categorize as high an low output. High output is ene as outputs of more than 500 mL/ay an low output as less than 00 mL/ay. High-output stulas are less likely to close an often cause signicant ui, electrolyte, an nutritional challenges. Factors that preict whether a stula will close (mnemonic “FRIEND”) inclue foreign boy, raiation to the bowel, inammation/infection (such as inammatory bowel isease), epithelialization of the stula tract, neoplasia at the stula site, an distal obstruction. The mortality rate of enterocutaneous stulas remains signicant at 10% to 15%. Approximately 50% close spontaneously. Conservative treatment shoul be continue for at least 6 weeks before any reoperation is performe. Operating before 6 weeks results in higher mortality an stula recurrence rates. Octreotie has not been shown in ranomize trials to ai in earlier stula closure but oes not ecrease mortality (E). Reference: Sancho JJ, i Costanzo J, Nubiola P, et al. Ranomize ouble-blin placebo-controlle trial of early octreotie in patients with postoperative enterocutaneous stula. Br J Surg. 1995;8(5):638–641.

5. B. Colonic ischemia after repair of a rupture abominal

aortic aneurysm occurs in 1% to 6% of operations but can occur in up to 5% of cases uner certain circumstances. The greatest risk factor is the presence of prolonge hypotension preoperatively. In a patient with stable bloo pressure, age, time to operation, an the presence of cariac isease have little effect on the incience of colonic ischemia after aortic repair (A–C, D). Patency of a patient’s inferior mesenteric artery is not a goo preictor of colonic ischemia because of signicant avenues of collateral ow (E). Symptoms an signs of ischemia inclue blooy iarrhea, abominal pain/ istention, an elevate white bloo cell count. If the patient has evience of peritonitis, urgent reoperation is inicate.

Otherwise, urgent enoscopy is require to view the colonic mucosa. The majority of cases of colonic ischemia can be manage nonoperatively with bowel rest, hyration, an IV antibiotics. If the patient requires colon resection, mortality rates are as high as 75%.

6. B. This case represents a cecal stula. The most common

causes are slippage of the suture or necrosis of the remaining appeniceal stump, leaing to leakage of the enteric contents into the peritoneal cavity (A). Rarely, the stula results from unrecognize Crohn isease, malignancy, tuberculosis or istal colon obstruction. Cecal stulas are low-output stulas an are not associate with losses of large amounts of ui, electrolytes, or nutrients (D). Therefore, TPN is not necessary to maintain aequate nutrition (C) an mortality rates are low in the absence of other serious complications (A). Spontaneous closure is promote in as many as 75% of patients maintaine on low-resiue iets because absorption is mostly complete by the time the contents reach the cecum (B, E).

7. C. Duoenal stump blowout occurs after Billroth II oper-

ations, where back pressure on the uoenal stump results in breakown of the stump closure, leaing to abominal sepsis an peritonitis. Acute pancreatitis is associate postoperatively with Billroth II gastrectomy an jejunostomy, in which increase intrauoenal pressure can cause backow of activate enzymes into the pancreas but is unlikely to cause peritonitis (A). Woun ehiscence is characterize as suen ramatic rainage of relatively large volumes of a clear, salmon-colore ui an is apparent on physical exam (D). Acalculous cholecystitis can also occur postoperatively; however, the clinical presentation woul mainly consist of right upper quarant pain (B). Intraabominal hemorrhage woul be less likely to present with sepsis (E).

8. E. Smoking is a preictor of postoperative pulmonary

complications. The respiratory epithelium is altere in smokers, an poor ciliary activity combine with the prouction of more viscous mucus leas smokers to be more reliant on coughing to clear secretions from their lungs. Several ays after patients have stoppe smoking, there may be a transient increase in sputum volume. The above reasons have typically prevente health professionals from encouraging smoking cessation in the weeks leaing up to surgery. However, a metaanalysis publishe by the American Meical Association has conclue that the concern that stopping smoking only a few weeks before surgery might worsen clinical outcomes is unfoune an clinicians shoul avise smoking cessation as soon as possible (D). Postoperative lung expansion moalities (A–C) reuce postoperative pulmonary complications, although there is no ae benet from using all three. Routine use of a nasogastric tube may increase aspiration risk because the tube stents open the gastroesophageal junction. However, selective use in patients with nausea, bloating, an/or vomiting is probably protective. References: Bluman LG, Mosca L, Newman N, Simon DG. Preoperative smoking habits an postoperative pulmonary complications. Chest. 1998;113(4):883–889. Lawrence VA, Cornell JE, Smetana GW, American College of Physicians. Strategies to reuce postoperative pulmonary complications after noncariothoracic surgery: systematic review for the American College of Physicians. Ann Intern Med. 006;144(8):596–608.

CHAPtEr 35 Preoperative Evaluation and Perioperative Care Myers K, Hajek P, Hins C, McRobbie H. Stopping smoking shortly before surgery an postoperative complications: a systematic review an meta-analysis. Arch Intern Med. 011;171(11):983–989.

9. D. A serum albumin less than 3.5 g/L is the single most

important laboratory preictor of averse pulmonary events after surgery. Bloo urea nitrogen (>1 mg/L) is also useful, although the correlation is not as strong (A). Routine spirometry for all operations oes not seem to a value beyon a careful history an physical examination (B). An exception for the use of spirometry woul be for lung resection. Chest raiograph an arterial bloo gas are iagnostic stuies that woul only be preictive of postoperative complications if there were abnormal nings (C, E). References: Lawrence VA, Cornell JE, Smetana GW, American College of Physicians. Strategies to reuce postoperative pulmonary complications after noncariothoracic surgery: systematic review for the American College of Physicians. Ann Intern Med. 006;144(8):596–608. Qaseem A, Snow V, Fitterman N, et al. Risk assessment for an strategies to reuce perioperative pulmonary complications for patients unergoing noncariothoracic surgery: a guieline from the American College of Physicians. Ann Intern Med. 006;144(8):575–580.

10. E. Thrombosis of supercial an eep veins of the

upper extremity is cause by intravenous catheters in most cases. Upper extremity DVT oes pose a risk of pulmonary embolus, though less risk than pelvic an lower extremity DVT. Management begins with anticoagulation an etermining the necessity for the line; in the above case, there is a continue nee for TPN via a PICC line as the patient has not yet emonstrate a return of bowel function. Stuies have shown that it is not necessary to remove the PICC line espite the DVT. Therapeutic anticoagulation for 3 to 6 months is recommene. Thus, removal of the catheter without anticoagulation is not acceptable because there is a risk of PE (B). Inications to remove a line inclue infection an a contrainication to anticoagulation. If the line is to be remove, anticoagulation is still recommene; however, the recommenation is to wait 5 to 7 ays after the initiation of heparin before removing it (ue to the theoretic fear that pulling the line with a fresh clot might isloge the thrombus) (C, D). Warm compresses an NSAIDs woul be appropriate for supercial thrombophlebitis (A). Reference: Kucher N. Clinical practice. Deep-vein thrombosis of the upper extremities. N Engl J Med. 011;364(9):861–869.

11. A. While there are many theoretic avantages between

each metho of feeing, a 015 Cochrane review comparing postpyloric an gastric feeings showe only two signicant ifferences: lower rates of pneumonia in the postpyloric group an some evience for increase total nutrition elivere in the postpyloric group (E). There was no signicant ifference in length of ICU stay, mortality, or time on the ventilator (B). There was also no signicant ifference in associate complications with tube placement between the two stuy groups. While postpyloric feeing was associate with a longer time to initiation of tube feeing, this i not seem to affect the time it took to reach nutritional goals. Avantages of gastric feeing inclue a better approximation of normal physiology, ease of placement, an convenience (D). It may be a reasonable choice in patients without risk for aspiration, but patients with elaye gastric emptying

445

(which is common in ICU patients), iabetes, an gastroesophageal reux shoul be consiere for postpyloric fees (C). In terms of timing, there is abunant evience that earlier enteral feeing in critically ill patients results in better outcomes. Reference: Alkhawaja S, Martin C, Butler RJ, Gwary-Srihar F. Post-pyloric versus gastric tube feeing for preventing pneumonia an improving nutritional outcomes in critically ill aults. Cochrane Database Syst Rev. 015;(8):CD008875.

12. A. Patient-relate risk factors for the evelopment

of postoperative pulmonary complications inclue: age more than 50 years, COPD, congestive heart failure, American Society of Anesthesiologists (ASA) class greater than , serum albumin less than 3.5 g/L, obstructive sleep apnea, pulmonary hypertension, an current smoking. While a preoperative PaCO greater than 45 oes increase the surgical risk, there is currently no enitive number that prohibits abominal surgery (E). Current American College of Physicians Guielines recommen against the routine use of preoperative chest raiography or PFT (B). Although it is important to ientify patients with COPD, an some COPD patients may benet from preoperative interventions, patients who require aitional testing can be ientie by history of new symptoms or physical examination nings. Thus, PFT shoul be restricte to those who have current symptoms or signs base on history an physical. Location of the surgical incision is an important risk factor for postoperative pulmonary complications, with incisions closer to the iaphragm inferring more risk (D). When patients have been ientie as high risk for pulmonary complications, current evience supports the use of perioperative eep-breathing exercises an incentive spirometry. Routine use of nasogastric ecompression has been associate with increase rates of pneumonia an atelectasis. Current recommenations are for more selective use in patients with nausea, vomiting, or gastric istention (C). Reference: Qaseem A, Snow V, Fitterman N, et al. Risk assessment for an strategies to reuce perioperative pulmonary complications for patients unergoing noncariothoracic surgery: a guieline from the American College of Physicians. Ann Intern Med. 006;144(8):575–580.

13. C. The ifferential iagnosis for a suen rop in

en-tial CO in the operating room (OR) inclues an obstructe airway, acciental extubation, isconnection of the circuit, cariac arrest, an pulmonary embolism (PE). The patient escribe has at least three risk factors for PE incluing malignancy, a heart rate greater than 100, an more than 3 ays of immobilization (Wells criteria). PE is estimate to occur in 1% to % of surgical patients in the perioperative perio. While yspnea, anxiety, tachycaria, an tachypnea are the most common nings in awake patients, physical signs of PE will be limite in patients uner general anesthesia. In this situation, an astute clinician can recognize PE as presenting with hypotension, tachycaria, ecrease en-tial CO, an hypoxemia. In general, laparoscopic proceures have been associate with a low risk of both fatal an nonfatal PE (E). This complication is associate with 10% to 15% mortality in the perioperative perio (D). Electrocariogram changes have been shown to be present in up to 83% of patients, but they

446

PArt ii Medical Knowledge

are generally nonspecic (A). Uncommonly, PE can present with a prominent S wave in lea 1, Q wave in lea 3, an inverte T wave in lea 3; this is suggestive of right heart strain. Despite potential cariovascular consequences of massive PE, an elevate cariac enzyme level occurs in less than 50% of cases an is not specic for PE (B). The two most sensitive tests that can be one to help iagnose PE are a TEE an calculating the physiologic ea space to look for elevations (though it can be time-consuming). TEE has been shown to yiel a iagnosis in an average of 9.6 minutes with a sensitivity of 80% an specicity of 97% an is ieal in the OR setting. Although TEE is relatively poor at visualizing the PE, it is excellent at emonstrating right heart strain, which provies inirect evience of PE. Arterial bloo gas in awake patients with PE may emonstrate a low CO, but this may not be the case for a ventilate patient uner general anesthesia (D). Flexible bronchoscopy is not helpful in iagnosing PE (E). Reference: Desciak MC, Martin DE. Perioperative pulmonary embolism: iagnosis an anesthetic management. J Clin Anesth. 011;3():153–165

14. D. Re bloo cell transfusion has been inepenently

associate with longer intensive care unit (ICU) an hospital stays, increase complications, an increase mortality (C). It is also an inepenent risk factor for multiorgan system failure an systemic inammatory response synrome (SIRS). Most societal guielines agree that a liberal transfusion strategy (goal of 10 g/L) is no better an likely worse than a more restrictive strategy (goal of 7–9 g/L) in the majority of patients. So transfusion is neee as long as the hemoglobin remains above 7 g/L. The Transfusion Requirements in Critical Care (TRICC) trial emonstrate that critically ill patients without active bleeing fare better with a restrictive transfusion strategy. There is also no evience that transfusion lowers the risk of acute coronary synromes. (A, B). While hemoglobin can improve oxygen elivery to tissues, it has not been shown to lower risk of acute coronary synromes, ecrease time on the mechanical ventilator, improve oxygen consumption, or improve outcomes in patients with ault respiratory istress synrome or acute lung injury (A). Because of the negative effects of bloo transfusion, alternative methos of managing anemia are actively being researche, incluing the use of recombinant human erythropoietin (EPO) an hemoglobin-base oxygen carriers. Trials esigne specically looking at aministering exogenous EPO in trauma patients, the EPO-1 an EPO- trials showe reuctions in require bloo transfusions an improve mortality, respectively. While hemoglobin-base oxygen carriers show some promise, they are not currently approve for use in the Unite States (E).

15. A. This patient has postoperative parotitis. This most

commonly occurs in elerly patients with poor oral hygiene, poor oral intake, prolonge nasogastric tube ecompression, an ehyration, all leaing to a ecrease in saliva prouction. The pathophysiology involves obstruction of the salivary ucts with seconary infection an is more common in the iabetic or immunocompromise patient. Most patients will be iagnose with parotitis 4 to 1 ays postoperatively. Signs an symptoms begin with pain an tenerness over the angle of the jaw that can then progress to high fevers an

leukocytosis, as well as signicant eema involving the oor of the mouth. If left uniagnose an untreate, it can lea to life-threatening sepsis. Initial treatment is with high-ose broa-spectrum antibiotics with Staphylococcus coverage (most common organism) an warm compresses (B). If the patient oes not improve, surgical incision an rainage are inicate. In extreme cases involving progressive airway obstruction, emergent tracheostomy may be inicate. Use of measures to stimulate salivary ow, such as sucking on cany, seems to help prevent this complication, but prophylactic antibiotics are generally not inicate (C). Within improve oral hygiene, the incience of this rare complication is eclining (D). Aitionally, the use of anticholinergics will ecrease salivary ow an increase the risk of eveloping postoperative parotitis (E).

16. E. VTE prophylaxis is generally ivie into two

categories: pharmacologic an mechanical. Mechanical prophylaxis inclues static compression evices (like grauate compression stockings) an IPC evices. While grauate compression stockings work primarily by preventing venous stasis in the legs, IPC combines that with its effects on the intrinsic brinolytic system. It was originally hypothesize that intermittent compression cause the release of agents like tPA from the vascular enothelium. However, when these levels are irectly measure, they seem to be relatively constant espite an increase in tPA activity. The currently propose mechanism is relate to measure ecreases in plasminogen activator inhibitor-1 (PAI-1), which functions as a tPA inhibitor (A). Currently, there is no evience that one IPC evice is superior to another in preventing VTE (B). While there are relatively few contrainications to mechanical prophylaxis, traumatic injury to the extremity an evience of ischemia seconary to peripheral vascular isease are both contrainications. Aitionally, patients with conrme DVT in the lower extremity shoul not be place on IPC. Limitations of its usefulness are primarily relate to interruption in treatment an improper application. Comparison between mechanical an pharmacologic VTE prophylaxis shows that in certain low-risk patients there is an equivalent reuction in the incience of DVT an PE, though combination therapy is superior to mechanical prophylaxis alone (C). It can be consiere as sole therapy in low-risk patients an patients with contrainications to pharmacologic agents. In terms of pharmacologic prophylaxis, UFH an LMWH are the two most commonly use agents. LMWH is generally regare as more effective, especially in certain populations (e.g., trauma patients) (D). References: Comerota AJ, Chouhan V, Haraa RN, et al. The brinolytic effects of intermittent pneumatic compression: mechanism of enhance brinolysis. Ann Surg. 1997;6(3):306–314. Ho K, Tan J. Stratie meta-analysis of intermittent pneumatic compression of the lower limbs to prevent venous thromboembolism in hospitalize patients. Circulation, 013;18(9), 1003–100. Morris R, Woocock J. Evience-base compression. Ann Surg. 004;39(), 16–171.

17. D. Epiural anesthesia is an excellent tool to control

postoperative pain an has been shown to ecrease cariac morbiity, an as such, it has been gaining popularity in clinical practice. Routine use of urinary rainage in the setting of

CHAPtEr 35 Preoperative Evaluation and Perioperative Care epiural anesthesia remains controversial. However, postoperative ay 1 removal of the Foley with thoracic epiurals has been shown to signicantly ecrease the incience of urinary tract infections with minimal change to the rate of urinary retention as measure by rates of recatheterization (A–C). Current recommenations for the placement an removal of epiural catheters in patients receiving prophylactic LMWH is intene to prevent an epiural hematoma an subsequent paralysis. For the placement of epiural catheters, LMWH must be hel at least 4 hours before placement, an it shoul not be remove within 1 hours of the last ose. Prophylactic anticoagulation can be restarte 6 hours after placement an no sooner than 4 hours after removal of the epiural (B). Unfractionate heparin may be restarte after 1 hour (E). References: FDA Safety Information an Averse Event Reporting Program. Low Molecular Weight Heparins: Drug Safety Communication – Upate recommenations to ecrease risk of spinal column bleeing an paralysis in patients on low molecular weight heparins. U.S. Foo an Drug Aministration; 013. https://www. fa.gov/meia/87316/ownloa. Henren S. Urinary catheter management. Clin Colon Rectal Surg. 013;6(3):178–181. Horlocker TT. Regional anaesthesia in the patient receiving antithrombotic an antiplatelet therapy. Br J Anaesth. 011;107 Suppl 1:i96–i106. Townsen CM Jr, Beauchamp RD, Evers BM, Mattox KL. Sabiston textbook of surgery: the biological basis of modern surgical practice. 17th e. WB Sauners; 004. Zaouter C, Kaneva P, Carli F. Less urinary tract infection by earlier removal of blaer catheter in surgical patients receiving thoracic epiural analgesia. Reg Anesth Pain Med. 009;34(6):54–548.

18. E. Postoperative necrotizing soft-tissue infection is

a rare but well-escribe complication. The escription of “ishwater pus” is classic for a postoperative clostriial woun infection. The causative organisms are typically Streptococcus pyogenes or Clostridium perfringens. C. perfringens is an anaerobic gram-positive ro that prouces alphatoxin; this is a virulent exotoxin that leas to extensive tissue necrosis an cariovascular collapse. Immunocompromise patients (iabetes, malignancy, chronic liver isease) are at increase risk (A–D). Clinamycin has been shown to limit toxin prouction, which ecreases the virulence, slows tissue estruction, an can potentially reuce inammatory cytokine release. Effective therapy requires rapi aministration of broa-spectrum antibiotics incluing aerobic coverage (C) an source control via emergent operative excision of necrotic infecte tissue, incluing fascia. Conservative management is not appropriate if a necrotizing soft-tissue infection is suspecte (B). Reference: Hakkarainen TW, Kopari NM, Pham TN, Evans HL. Necrotizing soft tissue infections: review an current concepts in treatment, systems of care, an outcomes. Curr Probl Surg. 014;51(8):344–36.

19. E. Postoperative hematomas after neck surgery (thy-

roi, parathyroi, caroti artery) can have catastrophic consequences. Physical examination nings can be eceptively benign. Attempts at intubation may be hampere by tracheal compression an eviation (B). Furthermore, the recent neck issection, combine with the hematoma, causes venous an lymphatic obstruction, leaing to airway eema,

447

further compromising attempts at intubation. Rapily opening the incision at the besie is necessary because urgent ecompression is the fastest way to restore proper respiratory function. Denitive hemostasis must then be obtaine in the operating room. Although ultrasonography is an important iagnostic ai for hematomas (D), clinical suspicion is sufcient in this emergent situation an the urgency of ecompression oes not permit waiting for an ultrasoun examination. Neele aspiration woul not be sufcient (C). While nebulize racemic epinephrine is use for the treatment of strior in conitions like croup, it is not appropriate when the cause of strior is external compression of the airway (A).

20. E. The ureters rst pass meial to the psoas muscle an

travel alongsie the transverse processes of the lumbar vertebrae an cross anterior to the common iliac arteries near the bifurcation into the internal an external iliac arteries. The anatomic position places the ureters at risk for injury uring pelvic surgery, an the situation is particularly precarious when inammation, abscess, an/or phlegmon are present. The highest risk of ureteral injury is uring an abominoperineal resection. During mobilization of the left colon an ligation of the inferior mesenteric artery, visualization an protection of the ureter from injury are imperative. Placement of ureteral stents before the operation may help to ientify the ureters an assist with ientifying an injury intraoperatively, but this oes not seem to correlate with a reuction in the number of injuries (B). The presence of blue ye in the operative el after intravenous aministration of inigo carmine or methylene blue is iagnostic for injury to the ureter. The ecision to immeiately reoperate is base on the elay associate with injury recognition, the severity of the injury, an whether the patient has evelope urosepsis. If iscovere within a week postop, reoperation is generally recommene (A). Beyon 10 ays, the inammation present will make reoperation hazarous. In this latter case, percutaneous nephrostomy an/or retrograe rainage with a ureteral stent is inicate (C, D). The type of repair epens on the location an extent of the injury. For miureter injuries, a ureteroureterostomy is preferre. For pelvic injuries, ureteroneocystostomy is neee. If this is not possible, a psoas hitch or a Boari ap (from the blaer) may be neee. Reference: Bothwell WN, Bleicher RJ, Dent TL. Prophylactic ureteral catheterization in colon surgery. A ve-year review. Dis Colon Rectum. 1994;37(4):330–334.

21. C. Crush injury to the extremities causing signicant

muscle injury is often complicate by rhabomyolysis, which can lea to acute renal failure. Degraation proucts of both hemoglobin an myoglobin are toxic to the nephron in aciic urine. Elevate serum creatine phosphokinase, hyperkalemia, an the presence of heme without a signicant amount of re bloo cells on urinalysis are inicative of rhabomyolysis. Management consists of aggressive IV hyration to maintain a urine output of more than 100 mL/hour an shoul begin with infusion rates of at least 00 cc/hour. Myoglobin concentrates in the renal tubules precipitates when it comes in contact with Tamm-Horsfall protein. This precipitation is enhance uner aciic conitions. Routine aministration of bicarbonate (D) an mannitol (E) in the prevention of acute

448

PArt ii Medical Knowledge

kiney injury from rhabomyolysis is controversial, but, theoretically, alkalinization of the urine increases the solubility of the myoglobin–Tamm-Horsfall protein P complex an shoul increase myoglobin washout. It also prevents lipi peroxiation an renal vasoconstriction an seems to have relatively few negative sie effects if use in patients without high serum bicarbonate an without alkalosis. However, it reuces the amount of ionize calcium so shoul be use with caution in patients with hypocalcemia. Historical treatment of rhabomyolysis has inclue force iuresis with mannitol, but its routine use is being questione now in the literature an may actually increase the risk of eveloping renal failure. A retrospective stuy publishe in The Journal of Trauma in 004 looking at over 000 patients with elevate creatine kinase showe no ifference in renal failure, nee for hemoialysis, or mortality in patients receiving bicarbonate an mannitol versus volume resuscitation alone. Mannitol may ai in ecreasing muscle swelling an compartment pressures, but the mainstay of treatment remains ecompression of muscle compartments (A). However, in the case of crush injury, normal compartment pressures woul not change your strategy for preventing acute kiney injury because tissue amage alone coul cause the release of myoglobin. Loop iuretics are not use in the prevention of acute kiney injury in the setting of rhabomyolysis (B). While retrograe urethrogram woul assist in the iagnosis of misse urethral injury, the urine is positive on the ipstick from myoglobin, not hemoglobin. References: Brown CVR, Rhee P, Chan L, Evans K, Demetriaes D, Velmahos GC. Preventing renal failure in patients with rhabomyolysis: o bicarbonate an mannitol make a ifference? J Trauma. 004;56(6):1191–1196. Holt SG, Moore KP. Pathogenesis an treatment of renal ysfunction in rhabomyolysis. Intensive Care Med. 001;7(5):803–811.

22. E. Pulmonary function testing generally inclues three

separate tests: spirometry, lung volumes, an the iffusion capacity of the lungs. Expecte changes with aging inclue an increase in the functional resiual capacity an the resiual volume, with a corresponing ecrease in the vital capacity. This reciprocal change generally means the TLC is preserve (A). DLCO will also ecrease with age (C). Compliance of the lung can be misleaing because even though the compliance of the lung tissue itself increases with age, the chest wall compliance is signicantly reuce. In general, this means that the overall compliance of the pulmonary system is reuce. Preoperative pulmonary function tests are manatory for the evaluation of potential pulmonary resection. The preoperative values to remember are FEV1 greater than  L for pneumonectomy, FEV1 greater than 1.5 L for lobectomy, FEV1 greater than 80% preicte, an DLCO greater than 80% preicte. However, these numbers are not absolute inications, an failure to meet them simply necessitates more workup; this inclues getting a ventilation/perfusion scan to etermine the contribution of the preicte segment (B). If the percent-preicte postoperative FEV1 an DLCO are greater than 60%, then the patient is a caniate for resection of the propose segment without further testing (D). If the percent-preictive postoperative FEV1 an DLCO are less than 60%, exercise tolerance shoul be teste. Reference: Brunelli A, Kim AW, Berger KI, Arizzo-Harris DJ. Physiologic evaluation of the patient with lung cancer being consiere for resectional surgery: iagnosis an management of lung

cancer, 3r e: American College of Chest Physicians evience-base clinical practice guielines. Chest. 013;143(5 Suppl):e166S–e190S.

23. B. Venous air embolism is a rare an typically asymp-

tomatic conition. Though it is often associate with the placement of central venous access catheters, it has been associate with other conitions incluing trauma, hea an neck proceures, an neurosurgical proceures. When suspecte, the patient shoul be immeiately place in Trenelenburg position an left lateral ecubitus or right sie up (Durant maneuver) (C). This maneuver is esigne to trap the air embolus in the right ventricle an prevent it from going into the pulmonary arteries. Physical exam nings inclue jugular venous istention, millwheel murmur, an a sucking soun as air enters the venous system through a catheter. The most sensitive besie test for iagnosis is likely transesophageal echocariography, which can etect even small volumes of air (D). ECG suggestive of right heart strain is associate with pulmonary emboli (A). Treatment inclues correct positioning as previously escribe, increasing inspire oxygen, mechanical ventilation, hyperbaric oxygen, an, as a last resort, close-chest cariac massage to try to force the air out of the pulmonary arteries an into the smaller capillaries of the lung. An attempt can be mae to aspirate the air from the ventricle either through an existing central line or irectly through the chest wall, but the return of air with these proceures is generally low (E). Myocarial infarction is uncommon with venous air embolism an is typically the result of the air entering the arterial system via a congenital heart efect, such as a patent foramen ovale, an occluing the coronary arteries. The volume for a fatal venous air embolism is typically estimate at 3 to 5 mL/kg injecte at a rate of 100 mL/s, but these are largely base on animal stuies. The volume is much lower if the air enters the arterial system. Reference: Gory S, Rowell S. Vascular air embolism. Int J Crit Illn Inj Sci. 013;3(1):73–76.

24. D. This patient is scheule for a low-risk enoscopic

proceure an represents a high risk for thromboembolic events (mechanical valves an previous thromboembolic events), so anticoagulation shoul be continue without interruption. When consiering enoscopic proceures for patients on anticoagulation or antiplatelet therapy, three main things nee to be consiere: the urgency of the proceure, the patient’s risk of thromboembolic events (an in this case, the type of valve), an risk of bleeing uring the propose intervention. If the anticoagulation is temporary (e.g., treatment of venous thrombosis) or iscontinuation will be safer at a later ate (e.g., recent myocarial infarction [MI] with stent placement) an the enoscopy is completely elective (such as screening), the enoscopy shoul be elaye. The type of prosthetic valve matters too. Prosthetic mitral valves have a much higher risk of thrombosis than aortic valves (much higher ow) with cessation of anticoagulation. As such, briging is typically not neee for aortic valves. Low-risk enoscopic proceures can safely be performe on therapeutic anticoagulation or antiplatelet therapy an these meications shoul be continue regarless of the intervention. Examples of these proceures are iagnostic enoscopy with mucosal biopsy, ERCP without sphincterotomy, EUS, enteroscopy, an stent eployment. In all

CHAPtEr 35 Preoperative Evaluation and Perioperative Care high-risk enoscopic proceures (polypectomy, sphincterotomy, therapeutic ilation, ne-neele aspiration, enoscopic hemostasis, tumor ablation, cyst gastrostomy, an treatment of varices), anticoagulation shoul be iscontinue with or without briging. However, aspirin an NSAIDs can safely be continue in all enoscopic proceures. For patients on antiplatelet therapy with agents other than aspirin, they shoul be hel 7 to 10 ays before the proceure unless the thromboembolic risks are high, in which case patients may nee to be switche to aspirin or, in the case of ual antiplatelet therapy, aspirin continue an the other agent iscontinue. If the thromboembolic risk is low, anticoagulation can be stoppe an simply restarte after the proceure. For anticoagulation with warfarin in patients with high thromboembolic risk, brige therapy with LMWH or unfractionate heparin shoul be consiere. Use of LMWH an mechanical valves is controversial because of reporte events of fatal thromboembolism on LMWH in these patients. In general, anticoagulation can be restarte within 4 hours after the proceure (A–C, E). Reference: ASGE Stanars of Practice Committee, Anerson MA, Ben-Menachem T, et al. Management of antithrombotic agents for enoscopic proceures. Gastrointest Endosc. 009;70(6):1060–1070.

25. A. Refractory hypotension in the postoperative perio

in patients with conitions such as SLE that are commonly treate with sterois shoul raise concern for acute arenal insufciency. When the iagnosis is suspecte, treatment shoul begin immeiately before conrmatory tests become available (E). Initial treatment consists of: volume resuscitation, laboratory stuies (electrolytes, glucose, arenocorticotropic hormone [ACTH], cortisol), an aministration of either 4 mg of examethasone or 100 mg of hyrocortisone. Dexamethasone is preferre because it will not interfere with cosyntropin stimulation testing, which shoul be one the next morning to conrm the iagnosis. Glucocorticois can then be tapere to regular maintenance oses. Routine aministration of “stress-ose sterois” for patients on longterm corticosterois is not supporte by evience. It is now recommene that patients on long-term sterois shoul not be given “stress-ose” perioperative corticosterois. They shoul be continue on their regular maintenance ose with the consieration of aitional sterois only if they evelop refractory hypotension suggestive of arenal insufciency. While the cosyntropin stimulation test can be instrumental in etecting acute arenal insufciency, its usefulness as a preoperative measure for assessing risk of postoperative arenal crisis is lacking sufcient ata to support its routine use. While septic shock in the early postoperative perio

449

is possible, this vignette provies insufcient ata to point to this iagnosis (C, D). Exploratory laparotomy is not an appropriate option for the above patient (B). References: Brunicari FC, Anersen DK, Billiar TR, Dunn DL, Hunter JG, Matthews JB, Pollock RE. Schwartz’s principles of surgery. 10th e. McGraw-Hill Eucation; 015. Kelly KN, Domajnko B. Perioperative stress-ose sterois. Clin Colon Rectal Surg. 013;6(3):163–167. Marik PE, Varon J. Requirement of perioperative stress oses of corticosterois: a systematic review of the literature. Arch Surg. 008;143(1):1–16.

26. A. The 008 POISE trial was a ranomize controlle

trial to measure the effects of perioperative initiation of beta-blockers. The control group receive a placebo while the stuy arm was starte on metoprolol on the ay of surgery an receive it for 30 ays postoperatively. The stuy foun that patients who receive metoprolol ha a lower incience of myocarial infarction, cariac revascularization, an clinically signicant atrial brillation. However, patients in the stuy arm also ha increase mortality, stroke, hypotension, an braycaria (A, E). This increase in mortality was not seen in the previously publishe DECREASE trials, which also showe a reuction in myocarial infarction. However, several of these stuies were retracte because of falsie ata an questionable ata collection techniques. Without any other large ranomize trials to counter the POISE trial, it has largely become the basis for current guielines regaring perioperative use of beta-blockers. The 014 ACC/AHA guielines for perioperative beta-blocker therapy can be summarize as: (1) Beta-blockers shoul be continue if patients are on them chronically. () Management of beta-blockers after surgery shoul be base on clinical jugment to avoi negative consequences such as hypotension or braycaria (B). (3) Beta-blockers shoul not be starte on the ay of surgery. (4) It is unclear what the risk of starting beta-blockers is in the  to 45 ays before surgery (C). (5) It shoul be consiere in high-risk iniviuals (D). References: POISE Stuy Group, Devereaux PJ, Yang H, et al. Effects of extene-release metoprolol succinate in patients unergoing non-cariac surgery (POISE trial): a ranomise controlle trial. Lancet. 008;371(967):1839–1847. Wijeysunera DN, Duncan D, Nkone-Price C, etal. Perioperative beta blockae in noncariac surgery: a systematic review for the 014 ACC/AHA guieline on perioperative cariovascular evaluation an management of patients unergoing noncariac surgery: a report of the American College of Cariology/American Heart Association Task Force on practice guielines. J Am Coll Cardiol. 014;64():406–45.

Transfusion and Disorders of Coagulation CAITLYN BRASCHI, JOON Y. PARK, AND ERIC R. SIMMS

36

ABSITE 99th Percentile High-Yields I. Coagulation Cascae an Factors A. Factor I = brinogen, factor IA = brin, factor II = prothrombin, factor IIA = thrombin B. Intrinsic pathway of coagulation: initiate by expose subenothelial collagen, prekallikrein, high molecular weight kininogen; also involves factors VIII, IX, XI, XII; if impaire, PTT will be elevate C. Extrinsic pathway of coagulation: involves factor VII; if impaire, PT/INR will be elevate D. Common pathway of coagulation: involves factors I, II, V, X E. Factor VII has the shortest half-life of all coagulation factors F. Protein C an S breakown factors V an VIII G. Factor VIII is the only coagulation factor not mae in the liver (mae in the enothelium); von Willebran factor (vWF) also mae in the enothelium II. Bloo Proucts an Drugs for Coagulation Reversal

Contents/mechanism of action

Indications

Notes

Fresh Frozen Plasma (FFP)

All coagulation factors, vWF, Antithrombin III (ATIII)

Warfarin reversal, DIC, TTP, liver disease, AT III deęciency, Factor V deęciency

INR of FFP is 1.4–1.6; takes 1–2 hours to thaw

Cryoprecipitate

I, VIII, XIII, vWF

DIC, vWD type III, hemophilia A, hypoębrinogenemia

Highest concentration of ębrinogen (Factor I)

Prothrombin complex concentrate (PCC)

3-factor: II, IX, X (not used in clinical practice) 4-factor: II, VII, IX, X, C, S

Warfarin reversal in life-threatening bleed (intracranial hemorrhage), reversal of direct Xa inhibitors (rivaroxaban, apixaban)

Immediate warfarin reversal

Recombinant factor VIII

VIII

Hemophilia A

Transfuse to 100% normal factor VIII levels before major surgery

Recombinant factor IX

IX

Hemophilia B

Transfuse to 100% normal factor IX levels before major surgery

Recombinant factor Xa

X

Reverse direct Xa inhibitor

Not widely available

Vitamin K

Cofactor of carboxylation of coagulation factors II, VII, IX, X, C, S

Nonurgent warfarin reversal

Warfarin reversal begins after 6–10 hours, full eěect after 1–2 days

Tranexamic acid (TXA)

Binds plasmin (inhibits ębrinolysis)

Traumatic hemorrhagic shock with hyperębrinolysis

Must be given within 3 hours of injury for beneęt; if patient does not have hyperębrinolysis, TXA increases risk for thromboembolic events

451

452

PArt ii Medical Knowledge

Contents/mechanism of action

Indications

Notes

Aminocaproic acid

Binds plasmin (inhibits ębrinolysis)

tPA-associated bleed DIC

Protamine sulfate

Binds to and inhibits heparin

Heparin overdose with associated bleed

Only partially eěective against LMWH

Desmopressin (DDAVP)

V2 agonist, causes release of vWF and factor VIII from endothelium and platelets

vWD type I/II, uremia

vWD type III has absent vWF, so DDAVP ineěective

III. Anticoagulants

Mechanism of action

Notes

Heparin

Indirect thrombin and factor X inhibitor via ATIII

Ineěective in ATIII deęciency (give FFP with heparin) Aěects the intrinsic pathway (PTT)

Low-molecular-weight heparin (LMWH)

Indirect factor X inhibitor via ATIII

Superior DVT prophylaxis in cancer patients

Coumadin

Vitamin K antagonist

Contraindicated in pregnancy Extrinsic pathway (PT)

Argatroban, bivalirudin, hirudin, dabigatran

Direct thrombin inhibitors

Used in HIT (argatroban ęrst line; bivalirudin if liver failure); Hirudin is irreversible inhibitor; Dabigatran reversal: idarucizumab

tPA, streptokinase, urokinase

Active plasminogen

Monitor ębrinogen levels

Apixaban, rivaroxaban

Direct factor Xa inhibitor

Rivaroxaban reversed with andexanet alfa

IV. Transfusion Reactions

Reaction

Clinical ęndings

Cause

Febrile, nonhemolytic

Fever, pruritus, shivering as Cytokines from nontransfusion is being given; leukoreduced donor most common transfusion product reaction

Related products

Treatment

All products (rarely plasma)

Stop transfusion (although no long-term eěects, need to evaluate why patient is febrile), control symptoms (antipyretics, antihistamines)

Febrile hemolytic Fevers, chills, hypotension, chest/back pain; DIC, hematuria, renal failure

ABO incompatibility

Usually RBCs

Stop transfusion, give Ěuids, hemodynamic support

Urticarial

Hives

IgE reaction to product component

All products

Symptomatic treatment (antihistamines)

Anaphylactic

Hives, hypotension, wheezing, angioedema, hypoxemia

Recipient anti-IgA All products antibodies aĴack donor IgA antibodies, often in IgA-deęcient patients

Stop transfusion, resuscitation, epinephrine

Sepsis

Fevers, chills, hypotension, leukocytosis

Microorganism in stored Usually platelets product (stored at room temp)

Antibiotics, hemodynamic support

TransfusionRespiratory distress, “Two-hit”: neutrophil related lung hypoxemia, fever, sequestration and injury (TRALI) hypotension, leukopenia, activation by donor bilateral inęltrates on CXR; product within 6 hrs of transfusion

All products

Stop transfusion, ventilatory support

Transfusionassociated circulatory overload (TACO)

All products

Diuresis, ventilatory support

Respiratory distress, hypoxemia, JVD, hypertension, pulmonary edema; 6–12 posĴransfusion

Fluid overload; underlying cardiac or renal dysfunction

CHAPtEr 36 Transfusion and Disorders of Coagulation

V. Thromboelastography (TEG) A. TEG is the best way to etermine which bloo proucts shoul be given to a bleeing patient B. Interpretation: 1. R time—time to initial clot formation—if high, lacking coagulation factors -> give FFP . K time—time to brin cross linking—if high, lacking brinogen -> give cryoprecipitate 3. a angel—rate of clot formation—if low, lacking brinogen -> give cryoprecipitate 4. MA (max amplitue)—maximum clot strength—if low, lacking platelets (contributes most to clot strength) -> give platelet 5. LY30—rate of clot lysis—if high, increase brinolysis -> give TXA an/or aminocaproic aci

453

454

PArt ii Medical Knowledge

Questions 1. A 19-year-ol male is evaluate in the trauma bay following a motorcycle accient. He is foun to be hypotensive with an open book pelvic fracture. CT angiography of the pelvis oes not emonstrate active extravasation. A thrombelastography is performe showing an elevate K time an high LY30. He has receive bloo proucts an is 5 hours post injury. Which of the following is true? A. Cryoprecipitate woul not benet this patient B. K time is a measure of the time to initial clot formation C. This ning is inicative of ecrease brinogen levels D. FFP is inicate E. The patient shoul receive tranexamic aci (TXA) 2. The risk of posttransfusion sepsis is greatest with: A. Packe re bloo cells B. Cryoprecipitate C. Fresh frozen plasma D. Platelets E. Whole bloo 3. Which of the following is correct with regar to unfractionate heparin (UFH) an low-molecularweight heparin (LMWH)? A. LMWH is contrainicate while breastfeeing B. UFH is associate with fewer cases of heparininuce thrombocytopenia (HIT) C. Protamine is more effective in reversing LMWH compare to UFH D. LMWH oes not nee to be ose-ajuste in obese patients E. LMWH is consiere superior in trauma patients with traumatic brain injury (TBI)

4. A 34-year-ol woman with no past meical history presents with 6 weeks of left lower extremity pain an marke swelling an is foun to have a left iliofemoral DVT on CT venogram. She is given a heparin bolus, an a heparin rip is starte. She then unergoes catheter-irecte thrombolysis (CDT). Which of the following is true? A. The half-life of alteplase (tPA) is 4 to 6 hours B. Bleeing from inavertent overose may benet from aministration of aminocaproic aci C. Bleeing risk best is best monitore by following INR D. The rate of intracranial bleeing following CDT is higher than systemic thrombolysis E. The heparin rip shoul be continue uring CDT 5. Which of the following is true regaring the use of intraoperative bloo salvage (autotransfusion)? A. Use of intraoperative bloo salvage may lea to coagulopathy B. Malignancy is an absolute contrainication C. Autotransfusion can still be utilize if sterile water is being use in the el D. Most major abominal surgeries woul benet from its use E. Activate clotting time (ACT) shoul be use intraoperatively to monitor for coagulopathy 6. A 49-year-ol female with a history of von Willebran isease type 3 presents for scheule lobectomy for lung cancer. Which of the following is the correct perioperative management? A. Aminister 1 unit of FFP in the preoperative holing area B. Transfuse recombinant factor IX to 100% normal levels prior to surgery C. DDAVP shoul be given prior to incision D. He can procee without any intervention for von Willebran type 3 E. Preoperative von Willebran factor concentrate shoul be aministere

CHAPtEr 36 Transfusion and Disorders of Coagulation

7. A 75-year-ol woman with a history of atrial brillation on coumain presents to the ED with a painful, enlarging bulge in her abominal wall. She is iagnose with a rectus sheath hematoma. Her INR is supratherapeutic at 5. She enies any recent coughing episoes or trauma. However, she reports starting a new meication. Which of the following meications coul have contribute to her conition? A. Cimetiine B. Carbamazepine C. Rifampin D. Phenobarbital E. Phenytoin 8. Persistent life-threatening bleeing in a patient with Hemophilia A with high titers of inhibitors (factor VIII alloantiboies) is best treate with: A. A higher ose of factor VIII B. Fresh frozen plasma C. Cryoprecipitate D. Recombinant factor VIIa E. DDAVP (esmopressin) 9. A 76-year-ol male is unergoing a laparoscopic colectomy for sigmoi colon cancer. Which of the following is the best prophylaxis for venous thromboembolic events (VTEs)? A. Leg compression evice B. Unfractionate heparin (UFH) until fully ambulatory C. Leg compression evice intraoperatively, UFH until fully ambulatory D. Leg compression evice intraoperatively, LMWH until fully ambulatory E. Leg compression evice intraoperatively, LMWH for 4 weeks after surgery 10. A 50-year-ol male unergoes a resection of a large retroperitoneal leiomyosarcoma. There is an estimate bloo loss of 750 cc. The next ay, the patient is foun to be anemic an is given  units of bloo. Halfway through the rst unit, the patient evelops chills an his temperature increases from 37 to 39°C. Which of the following is true in regar to this patient’s conition? A. The transfusion oes not nee to be stoppe B. This occurs more commonly when given packe re bloo cells versus platelets C. Filtration is more effective than leukocyte washing in preventing this conition D. Aspirin is more effective than acetaminophen in treating this conition E. Pretransfusion aministration of acetaminophen an iphenhyramine is the most effective prevention

455

11. Which of the following oes not affect the bleeing time? A. Aspirin B. von Willebran isease C. Hemophilia A D. Severe thrombocytopenia E. Qualitative platelet isorers 12. A eciency of which of the following factors woul increase INR but not prolong the PTT? A. II B. V C. VII D. IX E. X 13. The most important preoperative assessment to etermine the risk of abnormal intraoperative bleeing is: A. Bleeing time B. Activate partial thromboplastin time (aPTT) C. International normalize ratio (INR) D. History an physical examination E. Platelet count 14. Glanzmann thrombasthenia is characterize by: A. Normal bleeing time B. Treatment response to DDAVP (esmopressin) infusion C. Autosomal ominant inheritance D. Defect in platelet aggregation E. Prolonge INR 15. Cryoprecipitate contains a low concentration of which of following? A. Fibrinogen B. Factor VIII C. von Willebran factor D. Fibronectin E. Factor XI 16. Which of the following is most likely to be useful in the treatment of bleeing in the uremic patient? A. Desmopressin B. Cryoprecipitate C. Fresh frozen plasma D. Recombinant human erythropoietin E. Estrogens

456

PArt ii Medical Knowledge

17. A 60-year-ol man with iabetes presents with right upper quarant pain an leukocytosis. The patient has an elevate INR of .5 an a prolonge PTT of 60 secons, a low brinogen level, an a platelet count of 70,000 cells/μL. An ultrasoun scan reveals gas in the wall of the gallblaer. The most important part in management of this patient woul be: A. Aministration of fresh frozen plasma B. Aministration of cryoprecipitate C. Checking the D-imer assay D. Emergent cholecystectomy E. Aministration of platelets 18. Which of the following is true in regar to von Willebran isease (vWD)? A. It is the secon most common congenital efect in hemostasis B. Type 1 vWD is transmitte in an autosomal recessive fashion C. DDAVP (esmopressin) is helpful in type 3 vWD D. Increase partial thromboplastin time (PTT) rules out vWD E. DDAVP is ineffective for type B vWD 19. A 40-year-ol female presents with a swollen left lower extremity, an ultrasoun conrms a eep venous thrombosis (DVT). The patient is starte on therapeutic heparin but espite progressively increasing the ose, the pharmacy is having ifculty achieving a therapeutic partial thromboplastin time (PTT) after 4 hours. Which of the following is the best option? A. Convert from unfractionate heparin to lowmolecular-weight heparin B. Aminister fresh frozen plasma C. Start a irect thrombin inhibitor D. Place an inferior vena cava lter E. Continue to increase heparin ose as neee 20. Which of the following is true regaring prothrombin complex concentrate (PCC)? A. Three-factor an 4-factor PCC refer to varying concentrations of factor II B. It is thawe more rapily than fresh frozen plasma (FFP) C. PCC reverses warfarin to an international normalize ratio (INR) less than 1.5 within 30 minutes D. PCC lowers INR as profounly as recombinant factor VIIa E. It reverses the anticoagulant effect of abigatran

21. A 31-year-ol woman in her thir trimester of pregnancy presents with fever, heaaches, an myalgia. She is a former intravenous rug user. She enies pruritus, but her skin appears jaunice. Bloo pressure is normal. Her laboratory exam is remarkable for elevate aspartate aminotransferase (AST) an alanine transaminase (ALT), hyperbilirubinemia as well as thrombocytopenia, anemia, an severe hypoglycemia. From which of the following conitions is she most likely suffering? A. HELLP (hemolysis, elevate liver enzymes, low platelet count) synrome B. Acute fatty liver of pregnancy (AFLP) C. Intrahepatic cholestasis of pregnancy (ICP) D. Preeclampsia E. Hepatitis E 22. A 35-year-ol man has been in the intensive care unit sepsis ue to enterocutaneous stulas, ventilator epenence, an pneumonia for  weeks. He is receiving nutrition parenterally. The INR is .0. The aPTT is normal. The total bilirubin level is normal. The platelet count is normal. Which of the following is the most likely etiology? A. Factor VIII eciency B. DIC C. Vitamin K eciency D. Primary brinolysis E. Chronic liver isease 23. Which of the following electrolyte abnormalities are the most likely to occur with massive bloo transfusion? A. Hypocalcemia, hypokalemia, an metabolic aciosis B. Hypercalcemia, hyperkalemia, an metabolic alkalosis C. Hypocalcemia, hyperkalemia, an metabolic alkalosis D. Hyponatremia, hyperkalemia, an metabolic alkalosis E. Hyponatremia, hyperkalemia, an metabolic aciosis

CHAPtEr 36 Transfusion and Disorders of Coagulation

24. A 75-year-ol male with a history of atrial brillation presents to the ED with an acute onset of left lower extremity pain an pulselessness. Heparin is starte. He is foun to have an occlue popliteal artery. The clot is successfully cleare with thrombolytic therapy. He remains on a heparin rip with plans to convert to warfarin. However, on hospital ay 5 his platelet count rops to 160,000 u/L (from an amission level of 370,000 u/L). Which of the following is true with regar to the rop in platelet count an the concern for heparin-inuce thrombocytopenia (HIT)? A. Because the platelet count is above 100,000 u/L, heparin can be continue B. The risk of recurrent thrombosis at this point is low C. Because the platelet count in’t rop until ay 5, the concern for HIT is low D. HIT is less common in men E. Warfarin shoul be starte 25. A 1-month-ol infant with mil skeletal abnormalities suffers a cariac arrest an passes away. On autopsy, he is foun to have extensive thrombosis in his coronary arteries. Which of the following is the most likely unerlying conition? A. Factor V Leien mutation B. Prothrombin gene mutation C. Antithrombin III eciency D. Homocystinuria E. Protein eciency 26. The most common cause of transfusion-relate eath is: A. Infection B. ABO incompatibility C. Acute lung injury D. Delaye transfusion reaction E. Graft-versus-host reaction 27. A 35-year-ol female evelops postpartum hemorrhage an requires a transfusion of packe re bloo cells an platelets. Twelve hours after transfusion, the patient abruptly evelops rigors an chills. Her temperature increases to 39°C, her bloo pressure rops from 110/70 to 70/40 mmHg, an her heart rate increases from 80 to 10 beats per minute. Urine output rops, although the urine is clear. Despite attempts at resuscitation, the patient expires within 4 hours. The eath is most likely ue to: A. Gram-positive sepsis B. ABO incompatibility C. Acute lung injury D. Anaphylaxis E. Gram-negative sepsis

457

28. A 55-year-ol patient unergoes surgery, uring which bloo transfusions were given. One week later, skin lesions evelop that appear to be purpura. The platelet count ecreases from 50,000 cells/μL to 10,000 cells/μL an an upper gastrointestinal blee evelops. The patient has not been receiving any meication that coul affect platelets. Which of the following is true about this conition? A. It is more common in mile-age men B. Severe bleeing is best manage by platelet transfusions C. It can occur without prior antigenic exposure D. It is an antiboy-meiate reaction E. Platelet counts are typically higher than with heparin-inuce thrombocytopenia 29. Which of the following is true in regar to clopiogrel (Plavix)? A. It functionally mimics the pathophysiology of Bernar-Soulier isease B. It has been linke to fatal episoes of pulmonary hypertension C. It is recommene that clopiogrel be stoppe 3 ays before a major operation D. It inhibits platelet aggregation within  hours of oral aministration E. It can inhibit the release of von Willebran factor 30. Which of the following factors has the shortest half-life? A. I B. II C. VII D. IX E. X 31. A 9-year-ol female is unergoing splenectomy for iiopathic thrombocytopenic purpura. Intraoperatively, the surgeon notes a signicant amount of bleeing at the splenic hilum uring mobilization. The surgeon woul like to temporarily stop bleeing with a hemostatic agent. Which of the following woul be the least effective choice for this patient? A. Microbrillar collagen B. Oxiize cellulose C. Thrombin D. Fibrin sealant E. Glutaralehye cross-linke peptie

458

PArt ii Medical Knowledge

Answers 1. C. The use of thromboelastography (TEG) has become

4. B. Alteplase, a tissue plasminogen activator (tPA),

trial: a ranomise controlle trial an economic evaluation of the effects of tranexamic aci on eath, vascular occlusive events an transfusion requirement in bleeing trauma patients. Health Technol Assess. 013;17(10):1–79.

Naiu S, Oklu R. Catheter-irecte thrombolysis of eep vein thrombosis: literature review an practice consierations. Cardiovasc Diagn Ther. 017;7(S3):S8–S37.

more common in the setting of hemorrhagic shock seconary to trauma or cirrhosis. TEG provies real-time information about clotting activity an can guie resuscitation. K time refers to the time to brin cross linking an an elevate K time inicates a eciency of brinogen (B). Therefore, transfusion of cryoprecipitate woul be inicate for this patient (A). Platelets woul be inicate in the event of a low MA. R time on a TEG result refers to the time to initial clot formation, an if this is prolonge, transfusion of FFP is inicate (D). A high LY30 is consistent with hyperbrinolysis an suggests the patient woul benet with TXA aministration. Trauma patients with massive hemorrhage receiving TXA have reuce all-cause mortality. However, this benet is only seen for patients receiving TXA within 3 hours of injury. TXA aministration past the 3-hour mark is associate with worse outcomes (E). Reference: Roberts I, Shakur H, Coats T, et al. The CRASH-

2. D. The risk of posttransfusion sepsis is greatest with

platelet transfusion. The risk is the greatest in transfusion of poole platelet concentrates from multiple onor versus single-onor platelet transfusion. Platelets are store at °C which makes this bloo prouct the most vulnerable to bacterial colonization an growth. If bacteria contamination of aministere bloo proucts is suspecte, the transfusion shoul be stoppe immeiately an bloo cultures obtaine.

3. E. The rate of serious bleeing complications has been

shown to be lower with the use of LMWH compare to UFH. It has also been shown to be associate with improve mortality in trauma patients with TBI. However, LMWH oes not have a completely effective reversal agent available. Only 60% of the anticoagulant effect of LMWH can be reverse with the aministration of protamine (C). Higher rates of major bleeing events have been shows in patients with renal insufciency with the use of both UFH an LMWH. LMWH is renally cleare, however, an therefore shoul be avoie in the setting of reuce creatinine clearance. UFH unergoes excretion via the reticuloenothelial system an enothelial cells (D). Although both UFH an LMWH are associate with the evelopment of HIT, this is more commonly seen after exposure to UFH (B). Either UFH, LMWH, or warfarin can be safely use while breastfeeing (A). Patients with obesity have a larger volume of istribution of lipophilic rugs such as LMWH an as such will require ose ajusting to reach aequate levels for thromboprophylaxis (D). Reference: Crowther MA, Berry LR, Monagle PT, Chan AKC. Mechanisms responsible for the failure of protamine to inactivate low-molecular-weight heparin: inactivation of low-molecular-weight heparin by protamine. Br J Haematol. 00;116(1):178–186.

is the rug most commonly use in CDT an has a very short half-life (5 minutes) (A). tPA triggers the activation of plasminogen into plasmin which then breaks brin cross links to issolve clot. Aminocaproic aci is the treatment of overose or reversal of tPA. Fibrinogen levels shoul be monitore closely following thrombolysis. Low levels of brinogen (usually less than 100 or 150 epening on clinical practice), are inicative of an increase risk of bleeing events (C). Although CDT has lower rates of intracranial hemorrhage than systemic thrombolysis (0%–1% versus 3%–6%, respectively), the patient shoul be monitore closely while unergoing treatment (D). Many of the same absolute contrainications to systemic thrombolysis are true for CDT incluing recent stroke, active bleeing, an intracranial trauma. Systemic heparin shoul be hel uring lytic therapy ue to the risk of bleeing (E). Reference: Fleck D, Albaawi H, Shamoun F, Knuttinen G,

5. A. Intraoperative bloo salvage is recommene for

clean (non-GI, noncontaminate) proceures with an estimate bloo loss of 500 to 1000 mL (e.g., cariac, liver, vascular, orthopeic cases) or more (D). This involves removing the patient’s bloo with a suction catheter uring surgery from the operative el. The bloo is then ltere, washe an returne to the patient. It has been shown to reuce the amount of allogenic transfusion require. It also theoretically increases operating room efciency as there is less time neee to request an prepare allogenic prouct. Absolute contrainications inclue mixture with other uis, particularly sterile water, as this hypotonic solution can lea to hemolysis (C). Malignancy is not an absolute contrainication; however, the risks an benets shoul be assesse on a case-by-case basis (B). Intraoperative bloo salvage only replaces re bloo cells an therefore patients are at risk of coagulopathy an ilution of coagulation factors. ACT monitoring is use in the setting of systemic heparinization, not for the use of re cell salvage. Goal ACT varies by provier an proceure but 150 to 00 secons for routine anticoagulation is commonly use (E). References: American Society of Anesthesiologists Task Force on Perioperative Bloo Management. Practice guielines for perioperative bloo management: an upate report by the American Society of Anesthesiologists Task Force on Perioperative Bloo Management. Anesthesiology. 015;1():41–75. Carless PA, Henry DA, Moxey AJ, O’Connell D, Brown T, Fergusson DA. Cell salvage for minimising perioperative allogeneic bloo transfusion. Cochrane Database Syst Rev. 010;(4):CD001888.

6. E. von Willebran isease (vWD) is the most common

congenital bleeing isorer. Patients with WVD type 3 have the most severe bleeing iathesis among patients with VWD. In this type of VWD, there is an absence of von Willebran factor (vWF). DDAVP causes release of vWF an

CHAPtEr 36 Transfusion and Disorders of Coagulation factor VIII from enothelial stores, an therefore, patients with type 3 VWD are not responsive to DDAVP (C). The perioperative management for patients with VWD unergoing major surgery (e.g., cariothoracic, hepatobiliary, neurologic, open vascular) inclues aministration of vWF (D). For patients with Hemophilia B, recombinant factor IX shoul be aministere to a goal of 100% of normal factor IX levels preoperatively (B). FFP can be use to correct INR in the acute setting (A). Reference: Lavin M, O’Donnell JS. New treatment approaches to von Willebran isease. Hematology Am Soc Hematol Educ Program. 016;016(1):683–689.

7. A. This patient has a supratherapeutic INR while on

coumain. Coumain works by interfering with the gammacarboxylation of vitamin K-epenent coagulation factors (factors II, VII, IX, X, protein C, S), an is metabolize by the cytochrome-P450 in the liver. Several rug interactions can lea to altere coumain metabolism. Meications that inhibit cytochrome-P450 lea to ecrease coumain metabolism an supratherapeutic INR. Inhibitors of cytochromeP450 inclue cimetiine, amioarone, several antibiotics (macrolies, uoroquinolones, metroniazole, isoniazi, sulfonamies), voriconazole, an grapefruit juice. Conversely, inucers of cytochrome-P450 will increase metabolism of warfarin ecreasing its effect. These patients may present with a new venous thromboembolism even though they have been on the same ose of warfarin for years. Examples of cytochrome-P450 inclue carbamazepine, rifampin, phenytoin, an phenobarbital (B–E).

8. D. Hemophilia A is a sex-linke recessive genetic coni-

tion an consiere the most common coagulation isorer, accounting for 80% of all inherite coagulation isorers. With time, as many as 10% to 15% of patients with factor VIII–ecient hemophilia A evelop inhibitors (alloantiboies) against factor VIII. This is usually from previous factor VIII transfusions. In situations in which life-threatening hemorrhage evelops, recombinant factor VIIa is the best option. Another option is porcine factor VIII, but there is approximately a 5% cross-reactivity with inhibitors. Factor VIIa complexes with tissue factor at the site of injury, resulting in an activation of factor X, which then results in clot formation. Factor VIIa bypasses the requirement for factors VIII an IX an thus has been shown to be effective in prevention an treatment of joint hemorrhage, the treatment of life-threatening bleeing, an the prevention of surgical bleeing. Restimulation of antiboies to factors VIII an IX shoul theoretically be less problematic than with the use of plasma-erive proucts. The primary concerns with recombinant factor VIIa are the potential for inucing thrombosis (stroke, eep venous thrombosis) an the high cost. A higher ose of factor VIII woul not efeat prouction of patient antiboies (A). Both fresh frozen plasma an cryoprecipitate contain factor VII but woul be ilute with other factors incluing factor VIII (B, C). DDAVP woul not help a patient with a coagulation efect (E). Other options that have been use but are only a temporary x in patients with signicant bleeing are plasmapheresis an immune absorption. References: DiMichele D. Inhibitors in hemophilia: a primer. Treatment of Hemophilia, 008;(7):1–4.

459

Kenet G, Lubetsky A, Luboshitz J, Martinowitz U. A new approach to treatment of bleeing episoes in young hemophilia patients: a single bolus megaose of recombinant activate factor VII (NovoSeven): recombinant FVIIa (NovoSeven) megaose. J Thromb Haemost. 003;1(3):450–455.

9. E. Patients unergoing surgery shoul be assesse for

VTE risk an categorize as very low, low, moerate, an high-risk patients. The Caprini score can be use to facilitate the estimation. A score of 5 or more places a patient at high risk. Age of 75 years or more = 3 points, cancer =  points, an major open or laparoscopic surgery longer than 45 minutes is also  points. As such this patient woul be consiere high risk. In low-risk patients, mechanical prevention (compression evice) is recommene. In moerate risk, pharmacologic prophylaxis with either UFH or LMWH is recommene. High-risk patients shoul get both mechanical an pharmacologic prophylaxis. The rug shoul be aministere close to surgery an continue until the patient is fully ambulatory. Recent ata in high-risk patients (such as those with cancer) emonstrate enhance VTE prophylaxis with extene LMWH for 4 weeks after surgery (A–D). Interestingly, recent ata inicate that patients unergoing colectomy for inammatory bowel isease (IBD) are also at very high risk for VTE (though IBD is not inclue in the Caprini score). Reference: Veovati MC, Becattini C, Ronelli F, et al. A ranomize stuy on 1-week versus 4-week prophylaxis for venous thromboembolism after laparoscopic surgery for colorectal cancer. Ann Surg. 014;59(4):665–669.

10. C. The patient is likely manifesting a febrile nonhe-

molytic transfusion reaction (FNHTR), the most common bloo transfusion reaction. It occurs in 0.5% to 1.5% of all cases of bloo transfusion (A). It is ene as a rise in temperature of at least 1.8°C from baseline an is not accounte for by the patient’s clinical conition. However, FNHTR is a iagnosis of exclusion. As such, it is generally recommene to at least temporarily stop the transfusion an assess the patient. In particular, attention shoul be pai to aitional symptoms an signs such as respiratory compromise, cyanosis, back pain, an hypotension; these may suggest a hemolytic reaction, TRALI, or sepsis from contaminate bloo. FNHTR is more common in pregnancy an in patients with immunocompromise states (such as leukemia, lymphoma). It occurs more commonly after the transfusion of platelets but can also occur with PRBC or FFP (B). Pretreatment with acetaminophen was thought to reuce the severity of the complication. However, the only ranomize controlle trial to ate emonstrate no ifference in the rate of FNHTR in patients that were pretreate with acetaminophen an iphenhyramine when compare to a placebo (E). The incience of febrile reactions can be greatly reuce by the use of leukocyte-reuce bloo proucts. Filtration removes 99.9% of the white bloo cells an platelets an is more effective than washing. Leukocyte reuction prevents almost all febrile transfusion reactions. There is ebate in the literature as to whether leukocyte reuction leas to a ecrease in postoperative infections or mortality. Aspirin is not avise given its effects on platelets an bleeing (D). References: Hébert PC, Fergusson D, Blajchman MA, et al. Clinical outcomes following institution of the Canaian universal

460

PArt ii Medical Knowledge

leukoreuction program for re bloo cell transfusions. JAMA. 003;89(15):1941–1949. Wang SE, Lara PN Jr, Lee-Ow A, et al. Acetaminophen an iphenhyramine as premeication for platelet transfusions: a prospective ranomize ouble-blin placebo-controlle trial. Am J Hematol. 00;70(3):191–194.

11. C. Bleeing time tests platelet ahesion an aggrega-

tion an will be normal in erangement of the coagulation pathways. Hemophilia A is associate with a factor VIII eciency, which manifests as an abnormality in the coagulation cascae an presents with a prolonge PTT. Drugs that inhibit platelet function, such as aspirin (which works by inhibiting cyclooxygenase), will increase bleeing time (A). von Willebran isease will result in prolonge bleeing time because of the qualitative or quantitative eciency in Willebran factor, which is require for platelet ahesion to other platelets via the IIb/IIIa receptor (B). Severe thrombocytopenia (quantitative) an platelet ysfunction (qualitative) both prolong bleeing time (D, E). Fibrinogen eciency also prolongs bleeing time because brinogen is require for platelet aggregation.

12. C. The INR etects abnormalities in the extrinsic an

common pathways. The extrinsic pathway is triggere by exposure of the injure vessel to tissue factor an starts with factor VII. It then merges with the intrinsic pathway at factor X (E) an is followe by activation of factors V an II an brinogen (factor I) (A, B). Thus, both the prothrombin time an the PTT will be prolonge in factors I, II, V, an X because they are all part of the common pathway between the intrinsic an extrinsic pathways. Factor IX is part of the intrinsic pathway an a eciency woul prolong PTT only (D).

13. D. The most important element in etecting an

increase risk of abnormal bleeing before surgery is a etaile history an physical examination. A systematic review in 008 emonstrate the poor value of using coagulation tests when it came to ientifying the risk of bleeing uring an operation (A–C, E). Other stuies have likewise shown that routine use of laboratory testing is neither sensitive nor specic for etermining increase risk of bleeing. One nees to inquire about a history of prolonge bleeing after minor trauma, tooth extraction, menstruation, an in association with major an minor surgery. In aition, one must make inquiries into meications an over-the-counter supplements that might affect hemostasis. If a careful history is negative an the planne surgical proceure is minor, then further testing is not necessary. A potential pitfall in relying solely on the history is that the history obtaine might not be sufciently thorough or the patients might not recall or recognize that they ha previous abnormal bleeing after an operation. If a major operation is planne that is not a high-bleeing risk, then a platelet count, a bloo smear, an an aPTT are recommene. If the history suggests abnormal bleeing or the operation is either a high bleeing–risk operation or one in which even minor bleeing may have ire consequences (neurosurgery), then a bleeing time an INR shoul be ae an a brin clot to etect abnormal brinolysis. If there is high suspicion for a history of abnormal bleeing, a hematology consult shoul also be obtaine.

References: Chee YL, Crawfor JC, Watson HG, Greaves M. Guielines on the assessment of bleeing risk prior to surgery or invasive proceures. British Committee for Stanars in Haematology: British Committee for Stanars in Haematology. Br J Haematol. 008;140(5):496–504. Chee YL, Greaves M. Role of coagulation testing in preicting bleeing risk. Hematol J. 003;4(6):373–378. Klopfenstein CE. Preoperative clinical assessment of hemostatic function in patients scheule for a cariac operation. Ann Thorac Surg. 1996;6(6):1918–190. Suchman AL, Mushlin AI. How well oes the activate partial thromboplastin time preict postoperative hemorrhage? JAMA. 1986;56(6):750–753.

14. D. Glanzmann thrombasthenia is an autosomal reces-

sive isorer that results in absence of functional glycoprotein IIb/IIIa (C). Glycoprotein IIb/IIIa is a receptor for brinogen an von Willebran factor an causes platelet ahesion an aggregation. Therefore, bleeing time will be prolonge, but aPTT an INR will be normal (A–E). These patients will not respon to DDAVP because there is no quantitative efect in the enothelial release of von Willebran factor or factor VIII (von Willebran isease) (B). The bleeing tenency for patients with Glanzmann’s is variable. Treatment is with platelets. Repeate use of platelet transfusions can inuce antiglycoprotein IIb/IIIa alloimmunization, renering the treatment ineffective. In this circumstance, recombinant factor VIIa may be useful. References: ’Oiron R, Ménart C, Trzeciak MC, et al. Use of recombinant factor VIIa in 3 patients with inherite type I Glanzmann’s thrombasthenia unergoing invasive proceures. Thromb Haemost. 000;83(5):644–647. Nuren AT. Glanzmann thrombasthenia. Orphanet J Rare Dis. 006;1(1):10.

15. E. Cryoprecipitate contains all items liste as well as

factor XIII. However, it contains low concentrations of factor XI (A–D). Cryoprecipitate was originally create as a treatment for hemophilia; however, it is now more often use in patients receiving massive resuscitation in conjunction with fresh frozen plasma to replenish brinogen levels. Factor XI eciency is also known as hemophilia C or Rosenthal synrome, occurs more often in the Ashkenazi Jewish population, an is treate with fresh frozen plasma (uring bleeing episoes).

16. A. The etiology of abnormal bleeing in uremic patients

is multifactorial, but the most important is impairment of platelet function that may be partly ue to a functional efect in von Willebran factor, which leas to impaire platelet aggregation. DDAVP (esmopressin) seems to enhance the release of von Willebran factor by enothelial cells. A single ose of 0.3 to 0.4 mcg/kg is given intravenously or subcutaneously. It has a rapi onset an relatively short uration (4–6 hours). Dialysis is also effective in the treatment of uremic bleeing by removing toxins that cause platelet ysfunction. Cryoprecipitate has high concentrations of von Willebran factor as well as factor VIII an brinogen an may also be effective; however, it shoul not be rst-line therapy (B). Recombinant human erythropoietin (Epogen [epoetin alfa]) has been shown to help uremic bleeing in several stuies as well (D). In aition to stimulating erythropoiesis, Epogen (epoetin alfa) enhances platelet aggregation. The

CHAPtEr 36 Transfusion and Disorders of Coagulation increase re cell mass also seems to isplace platelets from the center of the bloo vessel an places them closer to the enothelium. Estrogens have been shown to help with bleeing in men an women. The exact mechanism is unknown, but it is theorize that they ecrease arginine levels, which ecreases nitric oxie. This may lea to increases in thromboxane A an aenosine iphosphate (E). FFP oes not have high concentrations of von Willebran factor an thus is not effective for uremic bleeing (C). Reference: Heges SJ, Dehoney SB, Hooper JS, Amanzaeh J, Busti AJ. Evience-base treatment recommenations for uremic bleeing. Nat Clin Pract Nephrol. 007;3(3):138–153.

17. D. This is a classic presentation of emphysematous

cholecystitis complicate by sepsis, which then resulte in DIC. Elerly male iabetic patients are at higher risk of emphysematous cholecystitis, an gas in the gallblaer conrms the iagnosis. DIC leas to a ysregulation of the coagulation cascae, leaing to clotting an resultant bleeing. The consumption of brinogen, platelets, an coagulation factors from the overactivation of the coagulation cascae results ultimately in iffuse bleeing. There is no specic test for DIC, but thrombocytopenia, hypobrinogenemia, prolonge PT an PTT, an the presence of increase brin egraation proucts are sufcient to suggest the iagnosis of DIC (C). Fresh frozen plasma, platelets, an cryoprecipitate are all important components of the treatment, especially for an actively bleeing patient, but the most important part in the management of DIC is to ientify an correct the unerlying source, which in this case is by broa-spectrum intravenous (IV) antibiotics an emergent cholecystectomy (A, B, E). Without removal of the source, DIC will continue to consume transfuse proucts. The mortality rate from DIC ranges between 10% an 50%. Reference: Levi M, Toh CH, Thachil J, Watson HG. Guielines for the iagnosis an management of isseminate intravascular coagulation. British Committee for Stanars in Haematology. Br J Haematol. 009;145(1):4–33.

18. E. The most frequent congenital efect in hemostasis is

vWD (A). Laboratory tests will emonstrate increase bleeing time with a normal prothrombin time (PT). Patients may have a normal or increase PTT because von Willebran factor (vWF) is consiere a stabilizing factor for factor VIII (D). There are three types of vWD: Type I is an autosomal ominant isease characterize by a low level of vWF an consiere the most common form of vWD (B). Type I is treate with DDAVP because this increases circulating vWF release from enothelial cells. Type  vWD is also inherite in an autosomal ominant fashion an is characterize by a qualitative efect in which there is an appropriate amount of vWF, but it oes not function properly. Type  has multiple variants, some that can be treate with DDAVP or cryoprecipitate. Type b, in particular, when treate with DDAVP can inuce thrombocytopenia an form platelet complexes leaing to a prothrombotic state. DDAVP is contrainicate in type b but may be useful in other type  variants. Finally, type 3 is the most severe form because there is no vWF prouce by enothelial cells. It is transmitte in an autosomal recessive fashion. For type 3, the recommene treatment is recombinant vWF an factor VIII because these patients

461

o not make any vWF an therefore DDAVP will have no effect (C). References: Holmberg L, Nilsson IM, Borge L, Gunnarsson M, Sjörin E. Platelet aggregation inuce by 1-esamino-8-D-arginine vasopressin (DDAVP) in Type IIB von Willebran’s isease. N Engl J Med. 1983;309(14):816–81. Tosetto A, Castaman G. How I treat type  variant forms of von Willebran isease. Blood. 015;15(6):907–914.

19. B. Heparin resistance is ene as the nee for more

than 35,000 units in 4 hours to prolong the PTT into the therapeutic range or as an activate clotting time (ACT) less than 400 secons espite excessive eman for heparin (>400– 600 IU/kg). Heparin resistance is most commonly the result of antithrombin-III (ATIII) eciency. Heparin bins to ATIII causing a conformational change that results in its activation. Activate ATIII then inactivates thrombin an other proteases involve in bloo clotting, most notably factor Xa. ATIII eciency can be congenital or acquire. Hereitary ATIII eciency is rare (much less common than factor V Leien eciency) an can lea to venous thrombosis. Causes of acquire ATIII eciency inclue pregnancy, liver isease, isseminate intravascular coagulation (DIC), nephrotic synrome, major surgery, acute thrombosis, an treatment with heparin. For this latter reason, measurement of ATIII levels while on heparin is an inaccurate metho of ientifying heparin resistance. Treatment of heparin resistance consists of either aministering FFP or ATIII concentrates. FFP has the highest concentration of ATIII, an therefore patients shoul be initially treate with FFP to replete ATIII in plasma, followe by reaministration of heparin. A irect thrombin inhibitor (argatroban) is a potential alternative; however, it has the isavantage of having no way of being reverse in the case of overosage an bleeing (C). A isavantage of FFP in the cariac surgery setting is that large volumes may be require an it exposes the patient to the risks of transfusions, incluing transfusion-relate lung injury (TRALI). Thus, in the setting of cariac bypass, ATIII concentrate is another alternative (though it is very costly). Low-molecularweight heparin has no effect on ATIII eciency an shoul not be use in this event (A). An inferior vena cava (IVC) lter woul be inicate if the patient began to blee while on heparin but not for heparin resistance. In fact, a lter, though protective against PE, increases the risk for DVT, ue to the stasis it may create (D). Most patients achieve therapeutic PTT within 6 to 18 hours of starting heparin, so simply increasing the heparin ose is not appropriate (E). References: Kearon C, Akl EA, Comerota AJ, et al. Antithrombotic therapy for VTE isease: Antithrombotic Therapy an Prevention of Thrombosis, 9th e: American College of Chest Physicians Evience-Base Clinical Practice Guielines. Chest. 01;141( Suppl):e419S–e496S. Spiess BD. Treating heparin resistance with antithrombin or fresh frozen plasma. Ann Thorac Surg. 008;85(6):153–160.

20. C. PCC is an inactivate concentrate of proteins C an

S, an factors II, IX, an X, with variable amounts of factor VII. PCC with normal amounts of factor VII is known as 4-factor PCC, while PCC with low levels of factor VII is 3-factor PCC (A). Since 3-factor PCC has low levels of factor VII, the aition of fresh frozen plasma is sometimes necessary for full reversal of warfarin an thus, 4-factor PCC is superior. When a nonbleeing patient on warfarin nees

462

PArt ii Medical Knowledge

INR reversal, vitamin K is given, either orally (slower acting) or intramuscularly. If a patient is bleeing with an elevate INR, vitamin K an an exogenous clotting factor formulation are given. The options are FFP, PCC, or recombinant factor VII (less often use). PPC has several avantages over FFP; it oes not nee to be thawe (it is lyophilize [i.e., freeze rie]), it has a more rapi correction of INR, an it can be infuse faster an with less volume (this also makes it ieal for patients with congestive heart failure or chronic kiney isease) (B). Recombinant factor VIIa will lower INR faster than PCC (D). However, the concerns regaring recombinant factor VIIa inclue the potential for inucing thrombosis (stroke, eep venous thrombosis) as well as the high cost. PPC oes not reverse the anticoagulant effect of abigatran; this can be accomplishe with iarucizumab (E).

21. B. AFLP is an uncommon but potentially fatal complica-

tion that occurs in the thir trimester of pregnancy or uring the early postpartum perio. It typically presents with a viral prorome characterize by fever, lethargy, malaise, an nausea an vomiting. It is thought that AFLP may be the result of mitochonrial ysfunction resulting in microvesicular fatty inltration of hepatocytes without signicant inammation or necrosis. The mortality rate previously was very high; however, with prompt iagnosis an treatment, the maternal an perinatal mortality have ecrease to 18% an 3%, respectively. Prompt elivery an intensive supportive care are the cornerstones in management of AFLP. Laboratory abnormalities in AFLP inclue elevations of AST an ALT (usually less than 1000 IU/L), prolongation of PT an PTT, ecrease brinogen, renal failure, profoun hypoglycemia, an hyperbilirubinemia. Laboratory stuies of AFLP are similar to HELLP, but the key ning to help ifferentiate the two is hypoglycemia, which oes not occur commonly in HELLP (A). In aition, patients with HELLP typically have preeclampsia, evience of hemolysis, an thrombocytopenia. Preeclampsia presents with hypertension, proteinuria, an rapi weight gain an can progress to seizures (eclampsia) (D). Patients with ICP report intense pruritus most commonly in the hans an soles of the feet that is unrelieve with antihistamines (C). Hepatitis E is cause by a single-strane RNA virus. In men an nonpregnant women, it tens to be mil. However, it can lea to severe fulminant hepatic failure in pregnant patients in the thir trimester, with a mortality rate of up to 5% (particularly in eveloping countries) (E). References: Ko H, Yoshia EM. Acute fatty liver of pregnancy. Can J Gastroenterol. 006;0(1):5–30. Rahman TM, Wenon J. Severe hepatic ysfunction in pregnancy. QJM. 00;95(6):343–357. Vigil-De Gracia P. Acute fatty liver an HELLP synrome: two istinct pregnancy isorers. Int J Gynaecol Obstet. 001;73(3):15–0.

22. C. Several stuies have emonstrate that patients

in the ICU have a high incience of coagulopathy an that vitamin K eciency is the most common cause (B, D, E). The ifferential iagnosis for an elevate INR with a normal aPTT woul inclue a factor VII eciency, warfarin aministration, the acute phase of liver isease, an vitamin K eciency. Vitamin K is not stable in patients receiving total parenteral nutrition; therefore, in this case, the prolonge PT correlates with vitamin K eciency. Prolonge parenteral nutrition often leas to cholestatic liver isease, which in

turn leas to the liver’s inability to use vitamin K appropriately. Factors II, VII, IX, an X as well as proteins C an S all require vitamin K an will be ecient in these patients (A). Twenty percent of hospitalize patients given intravenous nutrition over a 3-week perio evelope elevations of INR. Vitamin K shoul be given at least 6 to 1 hours before a proceure in patients with aequate liver function. IM route of aministration is preferre because an IV push may result in anaphylaxis. In patients with hepatocellular isease, FFP or whole bloo is require. Platelets an cryoprecipitate are unrelate to prolonge prothrombin time. References: Chakraverty R, Davison S, Peggs K, Stross P, Garrar C, Littlewoo TJ. The incience an cause of coagulopathies in an intensive care population. Br J Haematol. 1996;93():460–463. Crowther MA, McDonal E, Johnston M, Cook D. Vitamin K eciency an D-imer levels in the intensive care unit: a prospective cohort stuy. Blood Coagul Fibrinolysis. 00;13(1):49–5. Duerksen DR, Papineau N. Clinical research: is routine vitamin K supplementation require in hospitalize patients receiving parenteral nutrition? Nutr Clin Pract. 000;15():81–83. Fiore LD, Scola MA, Cantillon CE, Brophy MT. Anaphylactoi reactions to vitamin K. J Thromb Thrombolysis. 001;11():175–183. Shearer MJ. Vitamin K in parenteral nutrition. Gastroenterology. 009;137(Suppl. 5):S105–S118.

23. C. The correct answer is hypocalcemia, hyperkalemia,

an metabolic alkalosis (A, B, D, E). Severe hypocalcemia with massive bloo transfusion is uncommon an oes not typically manifest unless the patient is receiving more than 1 unit of packe re bloo cells (PRBCs) every 5 minutes. The hypocalcemia is the result of citrate toxicity because the citrate in the transfuse bloo bins to circulating calcium in the patient. Because citrate is metabolize in the liver, hypocalcemia can be more severe in patients with hepatic ysfunction. Aitionally, the citrate is metabolize to bicarbonate leaing to metabolic alkalosis. Potassium concentration of store PRBC is higher than human plasma potassium level. This is thought to occur as a result of re bloo cell lysis uring storage, releasing potassium in the supernatant. The concentration of potassium in PRBC increases linearly an is approximately equal to the number of ays of PRBC storage. Reference: Vraets A, Lin Y, Callum JL. Transfusion-associate hyperkalemia. Transfus Med Rev. 011;5(3):184–196.

24. D. HIT occurs in approximately 1% to 1.% of patients

receiving heparin. A scoring system has been evise to assess risk of HIT, known as the 4 “T”s (Thrombocytopenia, Timing, Thrombosis, an other causes for Thrombocytopenia). Variables that shoul heighten suspicion of HIT inclue a platelet count rop greater than 50%, occurrence between ays 5 an 10 (it takes time for antiboies to evelop), nair of platelet count greater than 0,000 (nair below 10,000 is less likely HIT), no other reason for platelet count rop, an new skin necrosis or VTE (C). Thus, more important than the absolute nair is the percentage rop (A). HIT is cause by antiboies that attack the heparin-platelet factor 4 (PF4) complex. Heparin-PF4 antiboies (sometimes calle “HIT antiboies”) in the resultant multimolecular immune complex activate platelets via their FcγIIa receptors, causing the release of prothrombotic platelet-erive microparticles, platelet consumption, an thrombocytopenia. The microparticles in turn promote excessive thrombin generation, frequently resulting in thrombosis. Patients receiving any type

CHAPtEr 36 Transfusion and Disorders of Coagulation of heparin at any ose an by any route of aministration are at risk of eveloping HIT antiboies. It oes occur less commonly in men an occurs more frequently in the elerly. However, not all of those with HIT antiboies will necessarily evelop the clinical synrome. If this is suspecte, heparin shoul be iscontinue, an the patient shoul be starte on a irect thrombin inhibitor (E). If anticoagulation is not initiate, the chance of another thromboembolic event is approximately 5% to 10% per ay (B). Diagnosis is performe by an ELISA antiboy test. If these results are equivocal, then a conrmatory serotonin release assay shoul be performe. References: Ahme I, Majee A, Powell R. Heparin inuce thrombocytopenia: iagnosis an management upate. Postgrad Med J. 007;83(983):575–58. Jang IK, Hursting MJ. When heparins promote thrombosis: review of heparin-inuce thrombocytopenia. Circulation. 005;111(0):671–683. Warkentin TE, Haywar CP, Boshkov LK, et al. Sera from patients with heparin-inuce thrombocytopenia generate platelet-erive microparticles with procoagulant activity: an explanation for the thrombotic complications of heparin-inuce thrombocytopenia. Blood. 1994;84(11):3691–3699. Wheeler HB. Diagnosis of eep vein thrombosis. Review of clinical evaluation an impeance plethysmography. Am J Surg. 1985;150(4A):7–13.

25. D. Although all the answer choices can increase the

risk of venous thromboembolism, homocystinuria is the most common inherite conition preisposing patients to arterial thrombosis an affects 5% to 10% of the population. It is an autosomal recessive isease. Homocystinuria is most commonly cause by a eciency of cystathionine beta-synthase resulting in an elevate level of homocysteine in plasma an urine. The toxic effect of an elevate level of homocysteine in the brain results in mental retaration as well as seizures. Skeletal abnormalities (marfanoi habitus)may occur seconary to the interference of collagen cross-linking. Patients are at increase risk of thrombosis ue to the isruption of vascular enothelium by homocysteine leaing to platelet activation an aggregation. Patients ientie early to have this conition will benet with aministration of pyrioxine (vitamin B6) to inuce cystathionine beta-synthase activity. Factor V Leien mutation is the most common inherite conition increasing the risk of venous thromboembolism followe by prothrombin gene mutation (A, B). Patients that o not have a response to the aministration of unfractionate heparin may have antithrombin III eciency (C). Protein C eciency is a rare cause of venous thromboembolism (E). References: D’Angelo A, Selhub J. Homocysteine an thrombotic isease. Blood. 1997;90(1):1–11. Greico AJ. Homocystinuria: pathogenetic mechanisms. Am J Med Sci. 1977;73():10–13. Rosenaal FR. Risk factors for venous thrombosis: prevalence, risk, an interaction. Semin Hematol. 1997;34(3):171–187.

26. C. The leaing causes of allogeneic bloo transfusion

(ABT)–relate mortality in the Unite States (in the orer of reporte number of eaths) inclue transfusion-relate acute lung injury (TRALI), ABO an non-ABO hemolytic transfusion reactions, an transfusion-associate sepsis (A, B, D). Graft-versus-host reaction is not a common cause of ABT (E). Aitionally, it has been emonstrate that

463

nonleukocyte-reuce bloo transfusions have been associate with increase mortality when compare with leukocytereuce bloo transfusions. Reference: Vamvakas E, Blajchman M. Transfusion-relate mortality: the ongoing risks of allogeneic bloo transfusion an the available strategies for their prevention. Blood. 009;113(15):3406–3417.

27. E. Bacterial contamination of bloo is the most fre-

quent cause of eath from transfusion-transmitte infectious isease an is the thir most common cause of eath overall in a large series (after acute lung injury an ABO incompatibility) (B, C). A key feature of ABO incompatibility (hemolytic reaction) is the evelopment of re urine (hemoglobinuria). Patients also often complain of back pain an a sense of oom. Acute lung injury manifests with rapi onset of yspnea an tachypnea aroun 6 hours after transfusion. Anaphylactic reaction rarely occurs (D). Bacterial contamination now accounts for 1 in every 38,500 cases of bloo transfusion. This increase ha coincie with a ramatic ecrease in viral infections. The highest risk of bacterial infection is from poole platelet transfusions because many microorganisms can live an propagate uner the storage conitions of platelets (0–4°C). Gram-negative sepsis is the most lethal (A), an Yersinia is one of the most common organisms. Gram-negative sepsis can become clinically apparent within 9 to 4 hours after bloo transfusion. Cytomegalovirus is the most common infectious agent transmitte, but because it is so ubiquitous, it is generally not a threat to most patients. The exception to that rule is the transplant recipient. References: Benjamin RJ. Transfusion-relate sepsis: a silent epiemic. Blood. 016;17(4):380–381. Bihl F, Castelli D, Marincola F, Do RY, Braner C. Transfusiontransmitte infections. J Transl Med. 007;5(1):5. Kuehnert M, Roth V, Haley N, et al. Transfusion-transmitte bacterial infection in the Unite States, 1998 through 000. Transfusion. 001;41(1):1493–1499.

28. D. Transfusion purpura is an uncommon cause of

thrombocytopenia an bleeing after transfusion. A small minority of patients lack the HPA-1a antigen on their platelets that is present in almost all humans. Transfusion purpura requires that the patient has been previously sensitize to the HPA-1a antigen; this happens usually by a prior pregnancy or previous bloo transfusion. When these patients later receive bloo proucts that contain a small number of platelets with the ubiquitous HPA-1a, they prouce alloantiboies that attack both the onor’s an the patient’s own platelets (C). This usually presents 5 to 1 ays after a transfusion an leas to profoun thrombocytopenia an bleeing that can last for weeks. Mortality occurs in 10% to 0% ue to hemorrhage. Although sensitization can occur after prior bloo transfusions, it has become less common with leukocyte-reuce re cells an therefore this issue is most common in women who have been pregnant (A). Diagnosis is mae by emonstrating platelet alloantiboies with an absence of the corresponing antigen on the patient’s platelets. Treatment is primarily with intravenous immunoglobulin (IVIG). Plasmapheresis an corticosterois are also potential options. Treatment with platelet transfusions can exacerbate the isease process (B). The presentation can easily be confuse with heparin-inuce thrombocytopenia without appropriate

464

PArt ii Medical Knowledge

testing. A platelet count of fewer than 15,000 cells/μL is more suggestive of transfusion purpura (E). References: Hillyer CD, Hillyer KL, Strobl FJ, Jefferies LC, Silberstein LE, es. Handbook of transfusion medicine. Acaemic Press; 001:38. Lubenow N, Eichler P, Albrecht D. Very low platelet counts in post-transfusion purpura falsely iagnose as heparin-inuce thrombocytopenia: report of four cases an review of literature. Thromb Res. 000;100(3):115–15.

29. D. Clopiogrel (Plavix) irreversibly inhibits platelet

aggregation within  hours of aministration an its effects last 5 to 7 ays (the half-life of platelets is 1 week) (C, D). It works by inirectly inhibiting the activation of the glycoprotein IIb/IIIa complex (E). It oes this by antagonizing the ADP receptor which, when activate, inserts glycoprotein IIb/IIIa receptors on the platelet’s surface. This is functionally similar to Glanzmann thrombasthenia, which is characterize by a GpIIb/IIIa receptor eciency on platelets preventing brin from linking platelets together. Bernar-Soulier isease is characterize by GpIb receptor eciency on platelets which prevents vWF from linking the platelet to expose collagen on amage tissue (A). Clopiogrel has been shown to ecrease the rate of a combine enpoint of cariovascular eath, myocarial infarction, an stroke in patients with acute coronary synromes. Use with aspirin increases the risk of bleeing. Clopiogrel has been associate with the evelopment of thrombotic thrombocytopenic purpura, even with short-term use (00 mmHg heale 90% of the time with HBOT (C). TOT applies 100% oxygen irectly to the woun an has been stuie far less than HBOT. Though there are some clinical trials suggesting efcacy in some patients, it is accepte that TOT oes not penetrate to bone ue to its mechanism of action (B). References: Kranke P, Bennett MH, Martyn-St James M, Schnabel A, Debus SE, Weibel S. Hyperbaric oxygen therapy for chronic wouns. Cochrane Database Syst Rev. 015;(6):CD00413. Moon H, Strauss MB, La SS, Miller SS. The valiity of transcutaneous oxygen measurements in preicting healing of iabetic foot ulcers. Undersea Hyperb Med. 016;43(6):641–648. Mutluoglu M, Cakkalkurt A, Uzun G, Aktas S. Topical oxygen for chronic wouns: a PRO/CON ebate. J Am Coll Clin Wound Spec. 013;5(3):61–65.

2. E. Diabetic foot ulcers are very common, with the lifetime risk of a patient with iabetes approximately 5% (A). Diabetic foot ulcers are mainly cause by peripheral neuropathy leaing to scrapes/cuts of the foot that may go unnotice for several ays (B). There is also autonomic neuropathy leaing to failure of sweating. This manifests as ry skin at risk for mechanical breakown which can initiate ulcer formation. Aitionally, autonomic ysregulation of the microcirculation results in poor ow to istal extremities preventing aequate woun healing. Preventative care is paramount in the prevention of iabetic foot ulcers an inclues maintaining normoglycemia an aily exams for occult scrapes/cuts of the foot along with aily moisturizer use. Diabetic foot ulcers shoul be manage with a combination of ebriement an off-loaing. Sharp ebriement, enzymatic ebriement, biological ebriement, an autolytic ebriement are all acceptable methos of removing ebris an necrotic tissue (C, D). Ofoaing is also critical to woun healing, with nonremovable total-contact casts being the gol stanar. Contrainications to this type of cast are ischemia, ongoing infection, osteomyelitis, an poor skin quality. References: Alexiaou K, Doupis J. Management of iabetic foot ulcers. Diabetes Ther. 01;3(1):4. Boulton AJM, Armstrong DG, Albert SF, et al. Comprehensive foot examination an risk assessment: a report of the task force of the foot care interest group of the American Diabetes Association, with enorsement by the American Association of Clinical Enocrinologists. Diabetes Care. 008;31(8):1679–1685.

3. B. A large boy of evience supports the utilization of

a moist woun environment to promote faster woun healing an less scar formation. Therefore, the woun ressing shoul create an ieal amount of moisture by either aing moisture to ry wouns or absorbing it from highly exuative wouns. Despite the progress mae in woun ressings, many outate strategies like wet-to-ry ressings are overly use. Wet-to-ry ressings, most often gauze, are rst allowe to ry on the woun an then are remove, resulting in nonselective ebriement of both slough an healthy tissue (C). Aitionally, moisture-retentive ressings like hyrocollois an transparent lms have a lower infection rate than gauze ressings (D). Some wouns, espite moistureretentive ressings, remain too ry an require supplementation. Hyrogels are useful in this case as they are hyrating to the woun be, but care shoul be taken not

to use in combination with enzymatic ebriing agents, as they may become inactive (E). Other wouns have a large amount of exuate which nees to be controlle in orer to prevent maceration an improve woun healing. Both foam ressings an alginate ressings, which can absorb 0 times their weight, can be use in this scenario (B). Enzymatic ebriing agents, like collagenase (e.g., Santyl) an meicalgrae honey, are also commonly use to remove slough an necrotic tissues (A). Reference: Niezgoa JA, Baranoski S, Ayello EA, et al. Woun treatment options. In: Baranoski S, Ayello EA, es.Wound care essentials. 5th e. Wolters Kluwer Health; 00:184–41.

4. C. All humans have three main types of cell junctions:

anchoring junctions, communicating (gap) junctions, an tight junctions. The rst group (anchoring junctions) is further subivie into esmosomes, hemiesmosomes, an aherens junctions (D). Hemiesmosomes an esmosomes both connect with intermeiate laments in the cytoskeleton, but the former connects cells to the unerlying extracellular matrix, an esmosomes connect ajacent cells to one another (A, E). Aherens junctions serve the same purpose but use actin laments as their cytoskeletal anchor. Anchoring junctions, as a whole, provie structural integrity to a tissue mae up of iniviual cells. Communicating junctions allow irect chemical communication between ajacent cells. This is facilitate by six iniviual subunits, calle connexins, which form a central pore, calle a connexon. When two connexons from ajacent cells come in contact, a channel is forme allowing communication between the two cells. The nal group, tight junctions, refers to a group of proteins that allow the selective iffusion of molecules base mainly on size, molecular charge, an polarity. These primarily act as selective barriers such as in the ifferent layers of the skin (B).

5. B. Delaye woun healing is a multifactorial problem

with many ientiable risk factors incluing malnutrition, vitamin eciencies, smoking, obesity, iabetes, an hypoxemia. However, few systemic factors have been shown to spee up woun healing. Short perios of starvation can have negative effects on postoperative woun healing (A). This seems to occur primarily by prolongation of the inammatory phase because there are inaequate builing blocks for cell proliferation, protein synthesis, an creation of new DNA. The notion that malnutrition plays a key role in the evelopment of chronic wouns le to multiple stuies aime at etermining if nutritional supplementation can prevent chronic wouns or spee recovery. A Cochrane review one in 014 looking at 3 ranomize controlle trials evaluating the effect of enteral an parenteral nutrition on the prevention an treatment of pressure ulcers foun no clear benet of any intervention (D). By knowing the halflives an current serum measurements of certain proteins, we are able to estimate the synthetic ability of the liver over a given time perio. Albumin (14–0 ays), transferrin (8–9 ays), an prealbumin (–3 ays) all give a snapshot into someone’s nutritional status but nee to be combine with the entire clinical picture (C). Granulation tissue, if present, is preictive of aequate woun healing (E). References: Greenel LJ, Mulhollan MW, es. Greeneld’s surgery: scientic principles & practice. 5th e. Lippincott Williams an Wilkins; 011.

CHAPtEr 37 Wound Healing

469

Langer G, Fink A. Nutritional interventions for preventing an treating pressure ulcers. Cochrane Database Syst Rev. 014;(6):CD00316. Stechmiller JK. Unerstaning the role of nutrition an woun healing. Nutr Clin Pract. 010;5(1):61–68.

Mangram AJ, Horan TC, Pearson ML, et al. Guieline for the prevention of surgical site infection. Infect Control Hosp Epidemiol. 1990;0:47–80.

6. C. A eciency of trace elements an essential fatty acis

hemostasis/inammation (combine in the 3-phase moel), proliferation, an maturation (or remoeling). The hemostasis/inammation phase is initiate with the isruption of capillaries resulting in hemorrhage. This immeiately causes vasoconstriction to assist with the formation of a platelet plug. After 10 to 15 minutes, local tissue factors an platelets begin to facilitate vasoilation an increase vascular permeability. The inltration of ui an cells (mainly neutrophils) causes the woun to become erythematous (A). In aition, the woun is warm an eematous (inuration) (E). At this point, changes in tissue pH an local tissue estruction cause the woun to be painful (B). The rst cells to arrive after formation of a platelet plug are neutrophils, which on’t seem to be critical to healing an mainly help with phagocytosis of bacteria an estruction of ea tissue. Neutrophil preominance persists for 48 hours, at which point they are largely replace by macrophages, which will remain in the woun until the completion of healing (D). Macrophages are arguably the most important cell in healing because of their effects on angiogenesis, matrix eposition, an remoeling via the release of cytokines an growth factors. By ay 4, the proliferative phase begins an enothelial cells an broblasts begin to appear in the woun. By ays 5 to 7, there is no longer a signicant population of inammatory cells. The previously create matrix of type III collagen is slowly replace with type I collagen, angiogenesis takes place, granulation tissue begins to form, an woun contraction commences. This phase persists for 3 to 4 weeks an nally gives way to the remoeling phase. At this point, vascularity ecreases an collagen is continuing to be synthesize, but it is being broken own at the same rate an collagen cross-linking occurs. References: Brunicari FC, Anersen DK, Billiar TR, Dunn DL,

is a relatively rare entity in patients taking foo by mouth. However, it has occurre with increase frequency with the avent an wiesprea use of TPN, particularly in patients with a history of short gut synrome. Copper is primarily associate with anemia resistant to iron supplementation, leukopenia, an neurologic efects (A). Vitamin C eciency, or scurvy, causes elaye woun healing, bleeing gums, loose teeth, an abnormal bone eposition in chilren (B). Selenium eciency is associate with a fatal cariomyopathy (E). Zinc an essential fatty aci eciency (linoleic aci an alpha-linolenic aci) have many similar features incluing elaye woun healing, increase infections, iarrhea, an a rash. However, the essential fatty aci rash tens to be scalier an is associate with ry skin, an the rash from zinc is primarily locate in the perioral area an intertriginous skin of the ngers an toes. While alopecia an thrombocytopenia can be foun with both conitions, it is more closely associate with free fatty aci eciency. Conversely, the impaire taste, night blinness, an loss of appetite are more closely relate with zinc eciency (D). References: Jeppesen PB, Høy CE, Mortensen PB. Essential fatty aci eciency in patients receiving home parenteral nutrition. Am J Clin Nutr. 1998;68(1):16–133. Kumar V, Fausto N, Abbas A, es. Robbins and Cotran pathologic basis of disease. 7th e. WB Sauners; 004. O’Leary JP, Tabuenca A, Capote LR. The physiologic basis of surgery. 4th e. Wolters Kluwer Health/Lippincott Williams & Wilkins; 008.

7. E. Surgical site infections have been shown to increase

the cost of hospitalizations an length of hospital stays prompting the Surgical Care Improvement Project (SCIP) to aress this major economic buren to moern health care. While preoperative bathing with antiseptic solution has been shown to ecrease bacterial colonization of skin, it has not been proven to be associate with ecrease rates of surgical site infections (C). Multiple preparations for preoperative skin antisepsis have been esigne; however, the two most commonly in use are ioine-base an chlorhexiine-base in either an aqueous or alcohol solution. Ioine works primarily by passing through the bacterial cell membrane an replacing intracellular ions with molecular ioine an oxiizing various structures within the bacterium (D). It is also, however, toxic to normal tissues, so it is generally combine with a carrier molecule (e.g., povione) to reuce the systemic availability of molecular ioine an reuce its toxicity. In contrast, chlorhexiine works by its ability to estabilize cellular membranes. A Cochrane review one in 015 comparing ioine-base an chlorhexiine-base preoperative antiseptic techniques foun the latter to be superior in preventing surgical site infections (A). However, it is generally not recommene for use above the chin because of ototoxicity an potential for causing amage to the cornea in higher concentrations (B). References: Dumville JC, McFarlane E, Ewars P, et al. Preoperative skin antiseptics for preventing surgical woun infections after clean surgery. Cochrane Database Syst Rev. 015;(4):CD003949.

8. C. Woun healing is typically ivie into 3 or 4 phases:

Hunter JG, Matthews JB, Pollock RE. es. Schwartz’s principles of surgery. 10th e. McGraw-Hill Eucation; 015. O’Leary JP, Tabuenca A, Capote LR. The physiologic basis of surgery. 4th e. Wolters Kluwer Health/Lippincott Williams & Wilkins; 008.

9. C. Hypertrophic scarring an keloi formation are both

examples of pathologic excessive healing. Both are cause by the increase eposition of collagen (A). Formation of kelois has a large genetic component that is inherite in an autosomal ominant fashion. It is also more prominent in arker-skinne iniviuals. Hypertrophic scarring is generally cause by a elay in woun healing or by excessive tensile forces on a new woun an is at a particularly high risk of forming after burns (D). They o not sprea outsie of the borers of the original woun, unlike kelois (E). Hypertrophic scars ten to recee with time, but if they persist, they ten to respon better to surgical excision as compare to kelois. Excision of kelois shoul be performe with caution, as they ten to reoccur an become bigger. If excision is planne, it shoul be accompanie by an ajunctive treatment such as sterois or low-ose raiation to prevent recurrence (B). Several other ajuncts have also been shown to reuce scarring incluing silicone banages, occlusive ressings, an extremity compression evices.

470

PArt ii Medical Knowledge

References: Gauglitz GG, Korting HC, Pavicic T, Ruzicka T, Jeschke MG. Hypertrophic scarring an kelois: Pathomechanisms an current an emerging treatment strategies. Mol Med. 011;17(1–):113–15. Greenel LJ, Mulhollan MW, es. Greeneld’s surgery: scientic principles & practice. 5th e. Lippincott Williams an Wilkins; 011. O’Leary JP, Tabuenca A, Capote LR. The physiologic basis of surgery. 4th e. Wolters Kluwer Health/Lippincott Williams & Wilkins; 008.

10. B. This patient most likely has scurvy cause by a e-

ciency in vitamin C an is uncommon in the moern age. It is typically seen in patients with severe malnutrition often from unerevelope countries without access to fresh fruits an vegetables. Patients present with loose or missing teeth, open sores, pigmente spots on the extremities, bleeing mucous membranes, vague myalgias, an fatigue. It is a key cofactor in the hyroxylation of lysine an proline uring collagen synthesis; as such, collagen cross-linking is extremely iminishe in patients with vitamin C eciency. It can even cause the involution of previous scars because remoeling continues, but patients are unable to synthesize new collagen. Vitamin C is also involve in iron absorption (C). Vitamin A is another essential vitamin in woun healing an assists with epithelialization, proteoglycan synthesis, an normal immune function (A). It has also been shown to reverse the effects of sterois on woun healing. Vitamin D is consume in the iet an prouce in the skin. It then unergoes activation (hyroxylation) by the liver an kiney to play an essential role in calcium metabolism (E). Exogenous vitamin E has been shown in animal trials to cause elaye woun healing via an inammatory mechanism similar to corticosterois (D). Reference: O’Leary JP, Tabuenca A, Capote LR. The physiologic basis of surgery. 4th e. Wolters Kluwer Health/Lippincott Williams & Wilkins; 008.

11. C. The most common types of collagen locate in the

boy inclue types I to V, though there are many more that are clinically relevant in certain iseases. Type I collagen makes up 90% of the boy’s collagen an is foun to some egree in most tissue, incluing skin, bones, tenons, arterial walls, an scars. It is implicate in isease like osteogenesis imperfecta (D). Type II collagen makes up about 50% of the protein in hyaline cartilage (carTWOlige). Type III collagen is foun in bone, cartilage, an multiple types of connective tissue, an abnormalities have been foun in Dupuytren contracture an the formation of aneurysms. Type IV collagen is foun primarily in the basement membrane (type four

is oor) an has been associate with Alport an Goopasture synrome (A). Type V collagen is closely associate with type I an is in most of the same tissues but with the aition of placental tissue. While there exist clinically signicant collagens outsie of these main ve, such as type VII (epiermolysis bullosa) an type XVII (bullous pemphigoi), they are not nearly as prevalent (E). Ehlers-Danlos is a spectrum of connective tissue isorers that can affect multiple types of collagen (B). However, the most common is type V (seen in classic type Ehlers-Danlos). Reference: De Paepe A, Malfait F. Bleeing an bruising in patients with Ehlers-Danlos synrome an other collagen vascular isorers: review. Br J Haematol. 004;17(5):491–500.

12. D. While healing of the gastrointestinal tract goes

through the same basic steps as healing of the skin, there are several key ifferences an unique features. Skin wouns unergo a relatively steay increase of the tensile strength of the woun over time. In contrast, the increase collagenase activity in the small bowel allows collagen breakown to excee collagen eposition on ays 3 to 5 after an anastomosis (B). This is why anastomotic leaks in the gastrointestinal tract occur with increase frequency in this critical time perio. However, the gastrointestinal tract is quicker to reach maximal tensile strength when compare with the skin. The submucosa provies most of the tensile strength for an anastomosis because of the coarse, interwoven bers that make it up. However, the mucosa an serosa are also important, an both help provie a quick, leakproof barrier over the rst several ays (C). One can appreciate this effect in action by noting the relatively higher leak rates with portions of the GI tract that lack serosa such as the esophagus. Multiple ajuncts an techniques have been trie to ecrease the rate of anastomotic leaks, an while there may be a tren towar fewer leaks with a staple anastomosis in certain circumstances, there still isn’t conclusive evience that one is superior to the other in all cases (A). While omental wrapping has been shown to improve outcomes in certain situations, a evitalize “omental free ap” will necrose an will not help with the anastomosis (E). References: Brunicari FC, Anersen DK, Billiar TR, Dunn DL, Hunter JG, Matthews JB, Pollock RE. es. Schwartz’s principles of surgery. 10th e. McGraw Hill Eucation; 015. Egorov VI, Schastlivtsev V, Turusov RA, Baranov AO. Participation of the intestinal layers in supplying of the mechanical strength of the intact an suture gut. Eur Surg Res. 00;34(6):45–431. Thornton FJ, Barbul A. Healing in the gastrointestinal tract. Neurosurg Clin N Am. 1997;77(3):549–573.

Conf idence is ClinicalKey Evidence-based answers, continually updated

The latest answers, always at your ngertips A subscription to ClinicalKey draws content from countless procedural videos, peer-reviewed journals, patient education materials, and boos authored by the most respected names in medicine

our patients trust you ou can trust ClinicalKey Euip yourself with trusted, current content that provides you with the clinical nowledge to improve patient outcomes

et to now ClinicalKey at storeclinicaleycom 2019v1.0